Vous êtes sur la page 1sur 316

2021-2022

Problèmes de révisions
M.LAAMOUM
Concours National Commun – Session 2018 – MP

L’énoncé de cette épreuve, particulière aux candidats de la filière MP,


comporte 4 pages.
L’usage de tout matériel électronique, y compris la calculatrice, est interdit

Les candidats sont informés que la qualité de la rédaction et de la présentation, la clarté et la précision
des raisonnements constitueront des éléments importants pour l’appréciation des copies. Il convient en
particulier de rappeler avec précision les références des questions abordées.
Si, au cours de l’épreuve, un candidat repère ce qui lui semble être une erreur d’énoncé, il le signale
sur sa copie et poursuit sa composition en expliquant les raisons des initiatives qu’il est amené à prendre.
Le sujet de cette épreuve est composé d’un exercice et d’un problème indépendants
entre eux, à traiter dans l’ordre souhaité.

exercice
Pour tout p ∈ N∗ , on note Mp (R) l’espace vectoriel des matrices carrées d’ordre p à coefficients réels ;
la matrice identité de Mp (R) se notera Ip . Si M ∈ Mp (R), on note Tr (M ) sa trace, tM sa transposée et
πM son polynôme minimal.

1ère Partie
Réduction d’une matrice

Soient a et b deux réels, avec b 6= 0, et soit n un entier naturel > 2. On pose β = a − b, γ = a + (n − 1)b
et on note A, D les matrices de Mn (R) définies par :
   
a b ... b β 0 ... 0
. .
 b a . . . ..   0 . . . . . . .. 
 
A = . .

.  et D =  .. . .
  .
.. . . . . b . . β 0

b ... b a 0 ... 0 γ

1.1. Préciser le rang de la matrice A − βIn .


1.2. En déduire que β est une valeur propre de la matrice A et que le sous-espace propre associé est de
dimension n − 1.
1.3. Montrer qu’il existe une matrice orthogonale P ∈ Mn (R) telle que A = tP DP .
1.4. Calculer le déterminant de la matrice A. À quelle condition, sur a et b, la matrice A est-elle
inversible ?
1.5. Préciser le polynôme minimal de A puis donner l’expression de l’inverse de la matrice A, si elle est
inversible, en fonction des matrices A et In .
1.6. On suppose de plus que les réels β et γ sont positifs ou nuls. Donner une matrice S ∈ Mn (R),
symétrique et ayant toutes ses valeurs propres positives ou nulles, telle que A = S 2 .

2ème Partie
Application à l’étude d’une famille de vecteurs d’un espace euclidien

Soit E un espace vectoriel euclidien de dimension n > 2 muni d’un produit scalaire noté (.|.) ; la norme
euclidienne sur E associée à ce produit scalaire est notée k.k.
2.1. On suppose qu’il existe une famille (u1 , . . . , un+1 ) de n + 1 vecteurs unitaires de E et un réel α,
non nul et distinct de 1, tels que, pour tout i 6= j, (ui |uj ) = α. On note G la matrice de Mn+1 (R) de
terme général (ui |uj ), pour tout (i, j) ∈ {1, . . . , n + 1}2 .
2.1.1. Montrer que α ∈ [−1, 1[. Si (i, j) ∈ {1, . . . , n + 1}2 et i 6= j, la famille (ui , uj ) est-elle liée ?
2.1.2. Justifier que la famille (u1 , . . . , un+1 ) est liée.

Épreuve de Mathématiques II 1/4 Tournez la page S.V.P.


Concours National Commun – Session 2018 – MP

2.1.3. Montrer que les colonnes de G sont liées et en déduire que la matrice G n’est pas inversible.
2.1.4. En appliquant les résultats de la partie précédente à la matrice G, déterminer la valeur de α
en fonction de n.
2.2. Étude de la réciproque
 
1 c ... c
.
 c 1 . . . .. 

1
On pose c = − n et on note M la matrice de Mn+1 (R) définies par : M =  . .

.
 ; on
.. . . . 
. c
c ... c 1
désigne par < , > le produit scalaire canonique de Rn+1 .
2.2.1. Montrer qu’il existe une matrice symétrique B ∈ Mn+1 (R) telle que M = B 2 .
 
Dans la suite, une telle matrice B est choisie et on pose M = mi,j 16i,j6n+1 , B = bi,j 16i,j6n+1 .
2.2.2. Pour tout (i, j) ∈ {1, . . . , n + 1}2 , exprimer mi,j en fonction des coefficients de la matrice B.
2.2.3. Moyennant le résultat de la question précédente, construire une famille (w1 , . . . , wn+1 ) de n+1
vecteurs unitaires de Rn+1 telle que, pour tout (i, j) ∈ {1, . . . , n + 1}2 , < wi , wj >= mi,j .
2.2.4. Montrer que la matrice M n’est pas inversible et en déduire qu’il existe un sous-espace vectoriel
F de Rn+1 , de dimension n et contenant tous les vecteurs w1 , . . . , wn+1 .
2.2.5. Montrer qu’il existe effectivement une famille (v1 , . . . , vn+1 ) de n + 1 vecteurs unitaires de E
tels que, pour tout i 6= j, (vi |vj ) = − n1 .
On pourra construire une isométrie  entre E et l’espace euclidien F , muni de la structure euclidienne
induite par celle de R n+1 ,< , > .

problème
Dans ce problème, C désigne le corps des nombres complexes, E un C -espace vectoriel, non nécessairement
de dimension finie, et L(E) l’algèbre des endomorphismes de E.
Pour tout g ∈ L(E), on pose g 0 = idE et, pour tout k ∈ N∗ , g k désigne le composé de k endomor-
phismes égaux à g. Si u, v ∈ L(E), l’endomorphisme u ◦ v se notera simplement uv. Un endomorphisme
de E de la forme λ idE , avec λ ∈ C, est dit une homothétie.
On note Mn (C) l’algèbre des matrices carrées d’ordre n ∈ N∗ à coefficients dans C ; la matrice identité
de Mn (C) se notera In . Une matrice de Mn (C) est dite scalaire si elle est de la forme λ In avec λ ∈ C.
C[X] désigne l’algèbre des polynômes à coefficients complexes et, pour tout p ∈ N∗ , Cp [X] est le
sous-espace vectoriel de C[X] formé des polynômes de degré ≤ p.
L’objet de ce problème est d’établir le résultat suivant dû à aupetit en 1988 : Si f est un endomor-
phisme d’un espace vectoriel complexe E pour lequel il existe un entier n > 1 tel que, pour tout x ∈ E,
la famille (x, f (x), . . . , f n (x)) est liée, alors la famille (idE , f, . . . , f n ), d’éléments de L(E), est liée.

1ère Partie
Un résultat utile sur les fractions rationnelles
R
On se donne une fraction rationnelle où R et Q sont des polynômes, à coefficients complexes,
Q
R
premiers entre eux. On suppose que la fraction est définie et bornée sur C \ D, où D est un ensemble
Q
|R(z)|
fini, c’est à dire qu’il existe une constante M > 0 telle que, pour tout z ∈ C \ D, 6 M.
|Q(z)|
3.1. Montrer que, pour tout z ∈ C, |R(z)| 6 M |Q(z)|.
3.2. Montrer que cette fraction n’a aucun pôle et qu’il s’agit en fait d’un polynôme qu’on notera P .
Xd
3.3. On pose P = ak X k où d est le degré de P .
k=0

Épreuve de Mathématiques II 2/4 −→


Concours National Commun – Session 2018 – MP
Z 2π
3.3.1. Soit (k, q) ∈ N2 ; calculer l’intégrale ei(k−q)t dt selon les valeurs de k et q.
0 Z 2π
3.3.2. Soient q ∈ {1, . . . , d} et r > 0 ; calculer l’intégrale P (reit ) e−iqt dt et l’exprimer à l’aide de
0
r et des coefficients du polynôme P .
R
3.3.3. Montrer alors que le polynôme P = Q est constant.

2ème Partie
Étude du cas n = 1 et applications

Soit f un endomorphisme de E.
4.1. Étude du cas n = 1
Dans cette section on suppose que, pour tout vecteur x ∈ E, la famille (x, f (x)) est liée.
4.1.1. Démontrer que, pour tout x ∈ E \ {0E }, il existe un unique λx ∈ C tel que f (x) = λx x.
4.1.2. Soit (x, y) ∈ (E \ {0})2 ; démontrer que si la famille (x, y) est liée alors λx = λy .
4.1.3. Soit (x, y) ∈ (E \ {0})2 ; démontrer que si la famille (x, y) est libre alors λx = λy .
4.1.4. En déduire alors que f est une homothétie ; en particulier, la famille (idE , f ) est liée.
4.2. Quelques applications
4.2.1. Montrer que si f laisse stables toutes les droites vectorielles de E, alors f est une homothétie.
4.2.2. Montrer que si E est de dimension finie ≥ 3 et si f laisse stables tous les plans vectoriels de
E, alors f est une homothétie.
4.2.3. On suppose ici que f n’est pas une homothétie et que E est de dimension finie p > 2.
(i) Démontrer qu’il existe x0 ∈ E tel que la famille (x0 , f (x0 )) soit libre.
(ii) Justifier l’existence d’une famille (e3 , . . . , ep ) d’éléments de E telle que la famille (x0 , f (x0 ), e3 , . . . , ep )
soit une base de E.
(iii) On désigne par h la symétrie vectorielle de E par rapport au sous-espace vectoriel C.x0 , engendré
par x0 , parallèlement au sous-espace vectoriel Vect({f (x0 ), e3 , . . . , ep }).
Comparer h(f (x0 )) et f (h(x0 )) puis en déduire que hf 6= f h.
4.2.4. On suppose encore que E est de dimension finie p ≥ 2. Déduire de ce qui précède que si
f g = gf pour tout g ∈ L(E) alors f est une homothétie.
4.2.5. Traduction matricielle
Soit A ∈ Mp (C), p ≥ 2. Montrer que A est une matrice scalaire si, et seulement si, AM = M A pour
tout M ∈ Mp (C).

3ème Partie
Étude du cas général

On se donne ici un endomorphisme f de E pour lequel il existe un entier n > 2 tel que, pour tout
e ∈ E, la famille (e, f (e), . . . , f n (e)) est liée.
5.1. Soit x ∈ E \ {0E }.
nx −1 (x) soit

5.1.1. Montrer qu’il existe un unique n x ∈ {1, . . . , n} tel que la famille x, f (x), . . . , f
libre et la famille x, f (x), . . . , f nx (x) soit liée.

 
5.1.2. Montrer que le sous-espace vectoriel Vect x, f (x), . . . , f nx −1 (x) est stable par f .
5.2. On pose p = max{nx ; x ∈ E \ {0E }}.
5.2.1. Justifier que p est bien défini, que p 6 n et qu’il existex0 ∈ E \ {0E } tel que la famille
x0 , f (x0 ), . . . , f p−1 (x0 ) soit libre et la famille x0 , f (x0 ), . . . , f p (x0 ) soit liée.
5.2.2. Montrer qu’il existe un unique polynôme unitaire P ∈ C[X], de degré p, tel que P (f )(x0 ) = 0E
et justifier que Q(f )(x0 ) 6= 0E , pour tout polynôme non nul de Cp−1 [X].

Épreuve de Mathématiques II 3/4 Tournez la page S.V.P.


Concours National Commun – Session 2018 – MP

Dans la suite, un tel couple (p, x0 ) est choisi.


On va établir que P (f ) = 0 et par suite, la famille (idE , f, . . . , f p ) est liée, donc à fortiori la famille
(idE , f, . . . , f n ) l’est aussi puisque p 6 n.
Pour cela, on considère e ∈ E \ {0E } et on cherche à montrer que P (f )(e) = 0E .
 
5.3. On pose F = Vect x0 , f (x0 ), . . . , f p (x0 ), e, f (e), . . . , f p (e) et, pour tout complexe λ, vλ = x0 + λ e.
5.3.1. Montrer que le sous-espace vectoriel F est stable par f .
5.3.2. Montrer que le sous-espace vectoriel F est de dimension finie comprise entre p et 2p.
5.3.3. Montrer qu’il existe une famille (ϕ0 , . . . , ϕp−1 ) de formes linéaires sur F telle que, pour tout
couple (i, j) d’éléments de {0, . . . , p − 1}, ϕj (f i (x0 )) = δi,j .
 
Dans la suite, on note ∆(λ) le déterminant de la matrice ϕj f i (vλ ) ∈ Mp (C).
06i,j6p−1
5.4. Montrer que ∆(λ) est un polynôme en λ, de degré 6 p et que ∆(0) = 1.

5.5. Montrer que, pour tout λ ∈ C, il existe une famille α0 (λ), . . . , αp−1 (λ) de complexes tels que
p−1
X
p
f (vλ ) = αk (λ)f k (vλ ). (1)
k=0

5.6. On dispose ainsi des p application α0 , . . . , αp−1 qui sont des fonctions complexes de la variable
complexe.
5.6.1. Justifier que, pour tout complexe λ, les scalaires α0 (λ), . . . , αp−1 (λ) vérifient le système
d’équations linéaires :
p−1
p
 X
αk (λ)ϕj f k (vλ ) , 0 6 j 6 p − 1.

ϕj f (vλ ) = (2)
k=0
5.6.2. On note Z l’ensemble des racines complexes du polynôme ∆. Déduire de ce qui précède que
les restrictions à C \ Z des fonctions α0 , . . . , αp−1 sont des fractions rationnelles.
p−1
X
5.7. On considère le polynôme Pλ = Xp − αk (λ)X k et on note β0 (λ), . . . , βp−1 (λ) ses racines dans C,
k=0
p−1
Y 
chacune d’elles étant répétée autant de fois que son ordre de multiplicité. On a donc Pλ = X −βk (λ) .
k=0
5.7.1. Montrer que, pour tout λ ∈ C \ Z, la famille vλ , f (vλ ), . . . , f p−1 (vλ ) est libre.


5.7.2. En déduire que, pour tout j ∈ {0, . . . , p − 1} et tout λ ∈ C \ Z,


p−1
Y 
f − βk (λ) idE (vλ ) 6= 0E .
k=0
k6=j

5.7.3. Montrer alors que, pour tout j ∈ {0, . . . , p − 1} et tout λ ∈ C \ Z, βj (λ) est une valeur propre
de fF , endomorphisme de F induit par f .
5.8. On note k.k une norme quelconque sur F et, pour tout g ∈ L(F ), on pose kgk = sup kg(x)k.
kxk=1
5.8.1. Montrer que g 7→ kgk est une norme sur L(F ).
5.8.2. Montrer que, pour tout couple (g, h) d’éléments de L(F ), kghk ≤ kgkkhk.
5.8.3. Montrer que, pour tout j ∈ {0, . . . , p − 1} et tout λ ∈ C \ Z, |βj (λ)| 6 kfF k.
5.8.4. En utilisant les formules de Viète, donnant les relations entre les coefficients et les racines
d’un polynôme, et dont on demande ici des précisions, montrer que les restrictions à C \ Z des fonctions
α0 , . . . , αp−1 sont bornées.
5.9. Conclure que les fractions rationnelles α0 , . . . , αp−1 sont constantes et en déduire que Pλ = P pour
tout λ ∈ C, puis justifier que P (f )(e) = 0E . On pourra utiliser le résultat de la première partie.

Fin de l’épreuve

Épreuve de Mathématiques II 4/4 Fin


Un corrigé du cnm 2018 Maths-2- Filière MP

Mr : HAMANI Ahmed

EXERCICE

1ère Partie
Réduction d’une matrice
 
b b ... b
.. .
. ..
 
 b b 
1.1. • A − βIn =  ..
, b étant non nul, donc rang(A − βIn ) = 1.
 .. .. 
 . . . b 
b ··· b b
1.2. • Par le théorème du rang dimKer(A − βIn ) = n − 1 ≥ 1, donc β ∈ Sp(A) et dim(Eβ (A)) = n − 1.
1.3. • A étant symétrique réelle, donc d’après le théorème spectral, A est orthogonalement diagonalisable.
• Soit λ l’autre valeur propre de A, alors λ = T r(A) − (n − 1)β = na − (n − 1)(a − b) = a + (n − 1)b = γ,
donc ∃P ∈ On (R) tel que A = t P DP où D = diag(β, ..., β, γ).
1.4. • det(A) = det(D) = β n−1 γ.
• A est inversible si et seulement si, βγ 6= 0 si et seulement si, (b − a)(a + (n − 1)b) 6= 0.
1.5. • A étant diagonalisable, donc le polynôme minimal de A est scindé à racines simples, c’est à dire
ΠA = (X − β)(X − γ).
• ΠA est annulateur de A, donc ΠA (A) = A2 − (β + γ)A + βγIn = A(A − (β + γ)In ) + βγIn = 0, ce qui
1
entraine que A est inversible et que A−1 = − (A − (β + γ)In ).
βγ
√ √ √
1.6. • En posant ∆ = diag( β, ..., β, γ) et S = t P ∆P , on aura S 2 = A et S ∈ Sn (R).

2 ème Partie
Application à l’étude d’une famille de vecteurs d’un espace euclidien

2.1. 2.1.1. • L’inégalité de Cauchy-Schwarz donne |α| ≤ kui k.kuj k = 1, or α ∈ / {0, 1}, donc α ∈ [0, 1[\{0}.
• (ui , uj ) liée si et seulement si, |α| = 1 si et seulement si, α = −1.
Donc si (ui , uj ) est liée, on doit avoir uj = −ui , mais si k ∈ / {i, j}, (ui |uk ) = (uj |uk ) = −1, donc
uk = −ui = −uj ,ce qui aboutit à la contradiction ui = uj . On conclut que si i 6= j (ui , uj ) ne peut être
liée.
2.1.2. • La famille (u1 , ..., un+1 ) est de cardinal > n = dim(E), donc elle est liée.
2.1.3. • La liaison de la famille (u1 , ..., un+1 ) entraine l’existence de α1 , ..., αn+1 non tous nuls tels que
n+1
X n+1
X n+1
X
αk uk = 0, donc ∀i ∈ [[1, n + 1]] 0 = (ui | αk uk ) = αk (ui |uk ).
k=1 k=1 k=1
n+1
X n+1
X
Si on note Ck la k ème colonne de G, alors ∀i ∈ [[1, n]], ( αk Ck )i = αk (ui |uk ) = 0, donc
k=1 k=1
n+1
X
αk Ck = 0 c’est à dire (C1 , ..., Cn+1 ) est liée.
k=1
• Si on pose U = t (α1 , ..., αn+1 ), alors U 6= 0 et l’égalité précédente s’écrit GU = 0, donc Ker(G) 6= {0}
et par suite G n’est pas inversible.
 
1 α ... α
. 
 α 1 . . . .. 

2.1.4. • G =  .
 
 .. . . . . . . α 

α ... α 1
G n’est pas inversible, donc d’après la question 1.4 de la partie précédente α = 1 ou 1 + nα = 0, la
1
première condition étant exclue, ce qui donne α = − .
n
2.2. Étude de la réciproque
1 1
2.2.1. • On ait dans les conditions de la partie 1, avec a = 1 et b = − , donc β = a − b = 1 + et
n n
γ = a + nb = 1 − 1 = 0 sont positifs, ce qui permet d’appliquer la question 1.6 de la partie 1 qui assure
l’existence de B ∈ Sn+1 (R) vérifiant B 2 = M .

1
n+1
X
2.2.2. • L’égalité M = B 2 est équivalente à ∀i, j ∈ {1, ..., n + 1}, mi,j = bi,k bk,j .
k=1
n+1
X
2.2.3. • B est symétrique, donc mi,j = bi,k bj,k =< wi |wj > avec wi = t (bi,1 , ..., bi,n+1 ), en particulier
k=1
1 = mi,i = kwi k2 , donc wi est unitaire.
2.2.4. • γ = 0 est une valeur propre de M , donc M n’est pas inversible.
• Si on pose A la matrice de Mn+1 (R) de colonnes w1 , ..., wn+1 , alors M = t AA, donc 0 = det(M ) =
det2 (A), donc A n’est pas inversible et par suite la famille (w1 , ..., wn+1 ) est liée, ce qui entraine que
dimV ect(w1 , ..., wn+1 ) ≤ n d’où l’existence d’un sous-espace F de dimension n contenant ces vecteurs.
On peut même remarquer, puisque la somme des colonnes de M est nulle, que
n+1
X X X n+1
X n+1X n+1
X n+1
X
k wi k2 = < wi |wj >= mi,j = ( mi,j ) = 0 = 0, donc wi = 0 et vu
i=1 1≤i,j≤n+1 1≤i,j≤n+1 i=1 j=1 i=1 i=1
que le rang de B est gale à n, on aura (w1 , ..., wn ) base de F .
2.2.5. • Considérons f une isométrie de F = V ect(w1 , ..., wn ) vers E donc f conserve le produit sca-
laire.(Une telle isométrie existe, il suffit de choisir une base orthonormée de E pour le produit scalaire
(.|.) et une base orthonormée de F pour le produit scalaire < .|. > et considérer l’application linéaire
qui transforme la base de F en la base de E).
1
Alors si on pose vi = f (wi ) pour tous i ∈ {1, ..., n+1}, alors ∀i 6= j ∈ {1, ..., n}, (vi |vj ) =< wi |wj >= −
n
et (vi |vi ) =< wi |wi >= 1, de plus ∀i ∈ {1, ..., n},
n n
X X n−1 1
• (vn+1 |vi ) = (f (wn+1 )|f (wi )) = (f (− wj )|f (wi )) = − < wj |wi >= −1 + =−
j=1 j=1
n n
n
X n
X X
• (vn+1 |vn+1 ) = (f (wn+1 )|f (wn+1 )) = (− f (wi )| − f (wj )) = < wi |wj >=
i=1 j=1 1≤i,j≤n
n X n n
X X 1
= ( < wi |wj >) = = 1.
i=1 j=1 i=1
n
La famille (v1 , ..., vn+1 ) répond à la question.

PROBLÈME

1 ère Partie
Un résultat utile sur les fractions rationnelles

3.1. • L’inégalité est évidement vérifiée sur C \ D.


• Soit a ∈ D qui est fini, donc z0 est isolé, et par suite ∃r > 0 tel que B(a, r) ne rencontre D qu’au point
a et soit une suite (zn )n de B(a, r) \ {z0 } qui converge vers a, alors la passage à la limite dans l’inégalité
|R(zn )| ≤ M |Q(zn )| et grâce à la continuité des applications z 7−→ |R(z)| et z 7−→ M |Q(z)|, entraine que
|R(a)| ≤ M |Q(a)|.
En définitive, l’inégalité est vérifiée pour tout z ∈ C.
3.2. • Si Q(z0 ) = 0, alors l’inégalité précédente, entraine que R(z0 ) = 0, ce qui contredit que R ∧ Q = 1.
R
• Q est sans pôles dans C, donc Q est constant, et par suite la fraction devient un polynôme P de C[X].
Q
Z 2π
3.3. 3.3.1. • ei(k−q)t dt = 2πδk,q où δk,q désigne le symbôle de Kroneker.
0
d
X Z 2π d
X Z 2π d
X
k it −iqt k i(k−q)t
3.3.2. • P = ak X , donc P (re )e dt = ak r e dt = ak rk δk,q = 2πaq rq .
k=1 0 k=1 0 k=1
Z 2π
M
3.3.3. • Soit r > 0, q ∈ {1, ..., d}, alors 2π|aq |rq = | P (reit e−iqt dt| ≤ 2πM , donc |aq | ≤ , ce qui
0 rq
entraine en tendant r vers +∞ que aq = 0 pour tout q ∈ {1, ..., d} et par suite P = a0 .

2 ème Partie
Étude du cas n = 1 et applications

4.1. Étude du cas n = 1

4.1.1. • x 6= 0 et (x, f (x)) liée, donc ∃λx ∈ C tel que f (x) = λx x, c’est à dire λx valeur propre associée à x,
d’où l’unicité.

2
4.1.2 • x vecteur propre associé à λx et (x, y) liée, donc y est aussi vecteur propre associé à λx , d’où
f (y) = λx y = λy y et puisque y 6= 0, on obtient λx = λy .
4.1.3. • D’une part f (x + y) = λx+y (x + y) et d’autre part f (x + y) = f (x) + f (y) = λx x + λy y et par liberté
de (x, y), on aura λx+y = λx = λy .
4.1.4. • On vient de montrer que ∀x, y ∈ E \{0}, λx = λy , c’est à dire ∃λ ∈ C tel que ∀x ∈ E \{0},f (x) = λx,
donc f = λidE .
4.2. Quelques applications
4.2.1. • f laisse stable les droites vectorielles, donc ∀x ∈ E \ {0} f (x) ∈ V ect(x), donc daprès 4.1 f est
une homothétie.
4.2.2. • Soit x, y, z trois vecteurs librent deux à deux de E, alors V ect(x, y)∩V ect(x, z) = V ect(x) est stable
par f , donc f laisse stable toutes les droites vectorielles, et la question précédente entraine que f est
une homothétie.
4.2.3. (i) f n’est pas une homothétie, donc par contraposée de la question 4.1, ∃x0 ∈ E \ {0} tel que
(x0 , f (x0 )) est libre.
(ii) Le théorème de la base incomplète assure l’existence des vecteurs e3 , ..., ep tel que (x0 , f (x0 ), e3 , ..., ep )
soit une base de E.
(iii) h(f (x0 )) = −f (x0 ) et f (h(x0 )) = f (x0 ), or f (x0 ) 6= 0, donc h(f (x0 )) 6= f (h(x0 )) et par suite
f h 6= hf .
4.2.4. • Si f n’est pas une homothétie, la conclusion de la question 4.2.3 conduit à l’existence de h symétrie
vectorielle de E tel que f h 6= hf , donc par contraposée on obtient l’implication demandée.
4.2.5. Traduction matricielle
• =⇒ Si A = λIp est une matrice scalaire, alors elle commute avec toutes les matrices.
• ⇐= Si A commute avec toutes les matrices, considérons f l’endomorphisme canoniquement associé
à A.
Soit g un endomorphisme de Rn de matrice M dans la base canonique de Rn , alors AM = M A, donc
f g = gf c’est à dire f commute avec tous les endomorphismes de Rn , et par la question 4.2.4, f est
une homothétie, donc A est une matrice scalaire.
3 ème Partie
Étude du cas général
5.1. 5.1.1. • L’ensemble L = {q ∈ [[1, n]] / (x, f (x), ..., f q (x)) est liée} est un sous-ensemble de N qui
contient n, donc admet un plus petit élément nx .
/ L, donc (x, f (x), ..., f nx (x)) est liée et (x, f (x), ..., f nx −1 (x)) est libre.
nx ∈ L et nx − 1 ∈
5.1.2. • f (V ect(x, f (x), ..., f nx −1 )) ⊂ V ect(f (x), ..., f nx (x)), or d’après la question précédente
f nx (x) ∈ V ect(x, f (x), ..., f nx −1 (x)), donc V ect(x, f (x), ..., f nx −1 (x)) est stable par f .
5.2. 5.2.1. • La question précédente assure que l’ensemble {nx / x ∈ E \ {0}} est non vide inclu dans
[[1, n]], donc p existe et p ≤ n et p = nx0 où x0 ∈ E \ {0}, donc (x0 , f (x0 ), ..., f p−1 (x0 )) est libre et
(x0 , f (x0 ), ..., f p (x0 )) est liée.
5.2.2. • Par définition de p, f p (x0 ) ∈ V ect(x0 , f (x0 ), ..., f p−1 (x0 )), donc ∃a0 , a1 , ..., ap−1 tel que
p−1
X p−1
X
i
p
f (x) = ai f (x0 ) = P (x0 ) avec P = ai X i .
i=0 i=0
• L’unicité vient de la liberté de la famille (x0 , f (x0 ), ..., f p−1 (x0 )).
• De plus s’il existe Q ∈ Cp−1 [X] non nul tel que Q(f )(x0 ) = 0, alors la famille (x0 , f (x0 ), ..., f p−1 (x0 ))
est liée, ce qui est absurde.
5.3. 5.3.1. • f p (x0 ) ∈ V ect(x0 , f (x0 ), ..., f p−1 (x0 )) et p ≥ ne , donc f p (e) ∈ V ect(e, f (e), ..., f p−1 (e)), ce qui
assure la stabilité de F par f .
5.3.2. • La famille (x0 , f (x0 ), ..., f p−1 (x0 )) est libre de cardinal p, donc dim(F ) ≥ p.
• f p (x0 ) ∈ V ect(x0 , f (x0 ), ..., f p−1 (x0 )) et f p (e) ∈ V ect(e, f (e), ..., f p−1 (e)), donc
F = V ect(x0 , f (x0 ), ..., f p−1 (x0 ), e, f (e), ..., f p−1 (e)), et par suite dim(F ) ≤ 2p.
5.3.3. • La famille (x0 , f (x0 ), ..., f p−1 (x0 )) est libre dans F , on la complète en une base de F .
Une forme linéaire sur F est totalement détérminée par ses images sur une base de F . On considère
pour j ∈ {1, ..., p − 1} la forme linéaire ϕj sur F qui prend 1 sur f j (x0 ) et nulle sur les autres vecteurs
de la base, alors (ϕ0 , ..., ϕp−1 ) répond à la question.
5.4. • Pour i, j ∈ {1, ..., p − 1}, ϕj (f i (vλ )) = δi,j + λϕj (f i (e)).
Si on note M (vλ ) la matrice (ϕj (f i (vλ )))1≤i,j≤p−1 , alors M (vλ ) = Ip + λM (e),donc ∆(λ) = det(Mvλ )
est un polynôme en λ de degré ≤ p.
• ∆(0) = det(Ip ) = 1.

3
5.5. • Par définition de p, p ≥ nvλ , donc f p (vλ ) ∈ V ect(vλ , f (vλ ), ..., f p−1 (vλ )), ce qui assure l’existence de
p−1
X
p
la famille α0 (λ), ..., αp−1 (λ) tel que f (vλ ) = αk (λ)f k (vλ ).
k=0
5.6. 5.6.1. • La linéarité de ϕj donne le système (2).
ϕ0 (f p (vλ ))
   
α0 (λ)
5.6.2. • Le système (2) s’écrit Mvλ  .. ..
=
   
. . 
αp−1 (λ) ϕp−1 (f p (vλ ))
∀λ ∈ C \ Z, ∆(λ) = det(M (vλ )) 6= 0, donc le système admet une solution unique, à savoir
1
αi (λ) = det(A) où A est la matrice M (vλ ) en remplaçant la ième colonne par le second
∆(λ)
membre du système (2). On a donc αi est une fraction rationnelle en λ définie sur C \ Z.
p−1
X
5.7. 5.7.1. • Soit a0 , ..., ap−1 ∈ C tel que ai f i (vλ ) = 0, alors
i=0
Xp−1 p−1
X p−1
X
∀j ∈ {0, ..., p − 1}, 0 = ϕj ( ai f i (vλ )) = ai ϕj (f i (vλ )) = ai δi,j = aj ,
i=0 i=0 i=0
donc la famille (vλ , f (vλ ), ..., f p−1 (vλ )) est libre.
p−1
Y
5.7.2. • Soit pour λ ∈ C \ Z et j ∈ {0, ..., p − 1}, Qj = (X − βk (λ)).
k=i
k6=j

p−1
Qj est de degré p − 1 et la famille (vλ , f (vλ ), ..., f (vλ )) est libre, donc Qj (f )(vλ ) 6= 0.
5.7.3. • L’égalité (1) de la question 5.5, s’écrit 0 = Pλ (f )(vλ ) = (f − βj (λ)idE )(Qj (f )(vλ )),
donc Qj (f )(vλ ) ∈ Ker(f − βj (λ)idE ) et Qj (f )(vλ ) 6= 0, donc βj (λ) ∈ Sp(f ).
x
5.8. 5.8.1. • Soit g ∈ L(E) tel que kgk = 0, alors g = 0 sur la sphère S(0, 1), donc ∀x ∈ F \ {0}, ∈
kxk
x 1
S(0, 1), et par suite g( )= g(x) = 0, donc g = 0 sur F \ {0} et g(0) = 0, on conclut que g = 0
kxk kxk
sur F .
• ∀λ ∈ C, kλgk = sup kλg(x)k = |λ|kgk.
kxk=1
• ∀x ∈ S(0, 1), ∀g, h ∈ L(F ) kg(x) + h(x)k ≤ kg(x)k + kh(x)k ≤ kgk + khk et par passage au sup, on
obtient kg + hk ≤ kgk + khk.
x
5.8.2. • Soit x ∈ F \ {0}, ∀g ∈ L(F ) kg(x)k = kg( )k.kxk ≤ kgk.kxk,
kxk
x
donc ∀x ∈ F \ {0}, k(gh)(x) = g(h(x))k ≤ kgk.kh(x)k ≤ kgk.khk.kxk et par suite kgh( )k ≤
kxk
kgk.khk et le passage au sup entraine que kghk ≤ kgk.khk.
5.8.3. • Soit x un vecteur propre unitaire de fF associé à βj (λ), alors kβj (λ)xk = |βj (λ)| = kfF (x)k ≤
kfF k.
X
5.8.4. • Les formules de Viète s’écrivent ∀k ∈ {1, ..., p}, αp−k = (−1)k−1 βi1 ...βik .
0≤i1 <...<ik ≤p−1
X X
k
• |αp−k | ≤ |βi1 |...|βik | ≤ kfF k = Cpk kfF kk ≤ M = max (Cpk kfF kk ).
1≤k≤p
0≤i1 <...<ik ≤p−1 0≤i1 <...<ik ≤p−1

5.9. • Les αi sont des fractions rationnelles bornées sur C \ Z où Z est fini, donc d’après la première
partie, ces fractions sont constantes, donc ∀λ ∈ C, ∀k ∈ {0, ..., p − 1}, αk (λ) = αk (0) et par suite
p−1
X
Pλ = X p − αk (0)X k , or v0 = x0 et l’égalité (1) de la question 5.5, avec λ = 0 s’écrit Pλ (f )(x0 ) =
k=0
P0 (f )(x0 ) = 0.
• Pλ est unitaire de degré p tel que Pλ (f )(x0 ) = 0, or l’unicité d’un tel polynôme assurée par la question
5.2.2 entraine que Pλ = P .
• Avec λ = 1, Pλ (f )(e) = Pλ (f )(vλ − x0 ) = Pλ (f )(vλ ) − Pλ (f )(x0 ) = 0 − 0 = 0.

4
ÉCOLE POLYTECHNIQUE

CONCOURS D’ADMISSION 2017 FILIÈRE MP

COMPOSITION DE MATHÉMATIQUES – B – (X)

(Durée : 4 heures)

L’utilisation des calculatrices n’est pas autorisée pour cette épreuve.

???

On utilise la notation allégée pour l’intégrale d’une fonction f : R → R continue par morceaux
et intégrable sur R Z Z +∞
f (x)dx = f (x)dx.
−∞

∂f ∂f ∂2f
Si f est une fonction de deux variables réelles t, x, on note ∂t f = , ∂x f = , ∂xx f =
∂t ∂x ∂x2
les dérivées partielles de f (sous réserve de leur existence).
Si n est un entier naturel, on note Cbn l’ensemble des fonctions f : R → R de classe C n et dont
toutes les dérivées f , f 0 , . . . , f (n) , jusqu’à l’ordre n, sont bornées.

On dit qu’une fonction m : R → R est une mesure si elle est continue, positive, intégrable sur R
et telle que Z
m(x)dx = 1.

On considère la fonction h : [0, +∞[ → R définie par h(0) = 0 et pour x > 0,

h(x) = x ln(x).

On dit qu’une fonction f : R → R admet une entropie relativement à une mesure m si f est
continue et h(f 2 )m est intégrable sur R. De même, on dit que f admet une variance relativement
à m si f est continue et f 2 m est intégrable sur R.
On admet que la fonction µ définie sur R par
1 2
µ(x) = √ e−x
π
est une mesure.

Ce problème étudie certaines inégalités fonctionnelles. Dans les parties I et II, on étudie un
opérateur différentiel lié à la mesure µ et on démontre une inégalité pour cette mesure. Dans la
partie III, on voit comment une telle inégalité en entraine une seconde, et on étudie une forme
de réciproque. La partie IV est indépendante des autres, et s’intéresse à une inégalité pour les
fonctions caractéristiques.

1
Préliminaires

Soit m une mesure.

1. Soit f : R → R une fonction qui admet une variance relativement à m. Montrer que f m est
intégrable. En conséquence, le réel
Z Z 2
2
Varm (f ) = f (x) m(x)dx − f (x)m(x)dx

est bien défini. Montrer que Varm (f ) > 0.

2. Soit f : R → R une fonction qui admet une entropie relativement à m.

2a. Montrer que f 2 m est intégrable. En conséquence, le réel


Z Z 
2 2
Entm (f ) = h(f (x) )m(x)dx − h f (x) m(x)dx

est bien défini.

2b. Soit a > 0. Montrer que

∀x > 0, h(x) > (x − a)h0 (a) + h(a),

avec inégalité stricte si x 6= a.

2c. Montrer que Entm (f ) > 0.


Z
On pourra utiliser la question précédente avec a = f (x)2 m(x)dx.

2d. On suppose ici que pour tout x ∈ R, m(x) > 0. Caractériser les fonctions f telles que
Entm (f ) = 0.

Partie I

On note L l’opérateur qui à une fonction f : R → R de classe C 2 , associe la fonction Lf définie


par
1
∀x ∈ R, Lf (x) = f 00 (x) − xf 0 (x).
2
On étend également cette définition aux fonctions f (t, x) de deux variables, en posant
1
Lf (t, x) = ∂xx f (t, x) − x∂x f (t, x),
2
sous réserve que ces quantités soient définies au point (t, x) ∈ R2 .
On rappelle que la mesure µ a été définie dans l’introduction.

2
1 0
3a. Soit f : R → R de classe C 2 . Montrer que Lf = µf 0 .

3b. Soient h1 , h2 deux fonctions de Cb2 . Montrer que


Z Z
1
h1 (x)(Lh2 )(x)µ(x)dx = − h01 (x)h02 (x)µ(x)dx,
2
après avoir justifié l’existence de chacun des termes de la formule.

On considère une fonction f ∈ Cb0 . On définit pour (t, x) ∈ R2


Z
Φf (t, x) = f (x cos t + y sin t)µ(y)dy.

4. Montrer que la fonction Φf : R2 → R est bien définie et continue.

5. On suppose que f ∈ Cb2 .

5a. Montrer que, sur R2 , Φf est de classe C 1 et ∂xx Φf est bien définie, continue et bornée.

5b. Soit (t, x) ∈ R2 . Trouver une relation entre ∂x Φf (t, x) et Φf 0 (t, x).

5c. Montrer que pour tout (t, x) ∈ R2 , on a ∂t Φf (t, x) cos t = LΦf (t, x) sin t.

Z Z
5d. Montrer que pour tout t ∈ R, on a Φf (t, x)µ(x)dx = f (x)µ(x)dx.

On admet pour la suite du problème que cette égalité reste vraie pour tout f ∈ Cb0 .

Partie II

Soit f : R → R+ une fonction de Cb0 positive. On définit pour t ∈ R


Z
J(t) = h(Φf (t, x))µ(x)dx.

π 
6. Montrer que J : R → R est continue, et calculer J(0) et J .
2

7. On suppose dans toute cette question que f ∈ Cb2 et qu’il existe δ > 0 tel que

∀x ∈ R, f (x) > δ.

7a. Montrer que J est alors de classe C 1 sur R et que


(∂x Φf (t, x))2
Z
sin t
∀t ∈ R, J 0 (t) cos t = − µ(x)dx.
2 Φf (t, x)

On note g = (f 0 )2 /f .

3
7b. Soit (t, x) ∈ R2 . Montrer que

Φf 0 (t, x)2 6 Φf (t, x)Φg (t, x).

7c. Conclure que


Z Z  Z
1
h(f (x))µ(x)dx − h f (y)µ(y)dy 6 g(x)µ(x)dx.
4

8. Montrer que pour tout f ∈ Cb2 , f admet une entropie relativement à µ et que
Z
Entµ (f ) 6 |f 0 (x)|2 µ(x)dx.

On pourra considérer la famille de fonctions définies par fδ = δ + f 2 pour δ > 0.

Partie III

Soit m une mesure. On suppose dans cette partie qu’il existe une constante C > 0 telle que, si
f : R → R est de classe C 1 et de dérivée f 0 bornée, alors f admet une entropie relativement à
m et
Z
Entm (f ) 6 C |f 0 (x)|2 m(x)dx. (1)

Z
9. Montrer que (1 + |x| + x2 )m(x)dx < +∞.

10. Soit f ∈ Cb1 . On souhaite montrer que f admet une variance relativement à m et que
Z
C
Varm (f ) 6 |f 0 (x)|2 m(x)dx. (2)
2

10a. Montrer
Z que f m et f 2 m sont
Z intégrables, et qu’il suffit de montrer (2) dans le cas où on
a de plus f (x)m(x)dx = 0 et f (x)2 m(x)dx = 1.

10b. Sous les hypothèses de la question précédente, montrer (2).


On pourra appliquer (1) à la famille de fonctions fε = 1 + εf pour ε > 0.

11. Soit f une fonction de Cb1 , telle que pour tout x ∈ R, on a |f 0 (x)| 6 1. On note, pour
λ ∈ R, Z
H(λ) = eλf (x) m(x)dx.

On admet que H est de classe C 1 et que l’on obtient une expression de H 0 (λ) en dérivant sous
le signe intégral de manière usuelle (on pourrait le démontrer comme précédemment).

4
11a. Montrer que pour tout λ ∈ R,

Cλ2
λH 0 (λ) − H(λ) ln H(λ) 6 H(λ).
4

11b. En déduire que pour λ > 0,

Cλ2
Z  Z 
λf (x)
e m(x)dx 6 exp λ f (x)m(x)dx + . (3)
4

1
On pourra étudier la fonction λ 7→ ln H(λ).
λ

12. Montrer que l’inégalité (3) s’applique à la fonction définie par f (x) = x.
x
On pourra utiliser la suite de fonctions définies par fn (x) = n arctan .
n
Z
13a. Soient M = xm(x)dx et a > M . Montrer que

+∞
(a − M )2
Z  
m(x)dx 6 exp − .
a C

1 2
13b. Conclure que pour tout α < , la fonction x 7→ eαx m(x) est intégrable sur R.
C

Partie IV

14. Soient p, q, r : R → R+
∗ trois fonctions continues, à valeurs strictement positives et
intégrables sur R.

14a. Montrer qu’il existe une fonction u : ]0, 1[ → R de classe C 1 bijective telle que
Z
0
∀t ∈ ]0, 1[ , u (t)p(u(t)) = p(x)dx.

De même, il existe une fonction analogue v : ]0, 1[ → R pour q.

14b. On suppose que


  2
x+y
∀x, y ∈ R, p(x)q(y) 6 r . (4)
2
Montrer que
Z  Z  Z 2
p(x)dx q(x)dx 6 r(x)dx . (5)

On pourra utiliser, après avoir justifié son caractère licite, le changement de variable défini par
u(t) + v(t)
x= dans le membre de droite de l’inégalité (5).
2

5
On admet pour la suite du problème que l’inégalité (5) reste vraie en supposant uniquement que
p, q, r : R → R+ sont des fonctions à valeurs positives, continues par morceaux, intégrables sur
R, et qui vérifient (4).

Si A ⊂ R, on note 1A sa fonction caractéristique définie par 1A (x) = 1 si x ∈ A et 1A (x) = 0 si


x∈/ A. On note d(x, A) = inf{|x − y| : y ∈ A} la distance de x ∈ R à A.
On note Int le sous-ensemble de P(R) dont les éléments sont les réunions finies d’intervalles de
R. Si A ∈ Int, alors 1A est continue par morceaux, et on définit le réel
Z
µ(A) = 1A (x)µ(x)dx ∈ [0, 1].

15. Soit A ⊂ R.

15a. Montrer que pour tous x, y ∈ R, on a

(x + y)2
   
1
d(x, A) − x 1A (y) exp −y 6 exp −
2 2 2

exp .
2 2

15b. On suppose que A ∈ Int et que µ(A) > 0. En déduire que


Z  
1 2 1
exp d(x, A) µ(x)dx 6 .
2 µ(A)

16. Soit A ∈ Int. Pour t > 0, on définit l’ensemble At = {x ∈ R : d(x, A) 6 t}.

16a. Montrer que At ∈ Int pour tout t > 0.

16b. On suppose de plus que µ(A) > 0. Montrer que pour tout t > 0, on a
2
e−t /2
1 − µ(At ) 6 .
µ(A)

∗ ∗

6
Polytechnique MP 2017 - Épreuve B - corrigé
Préliminiaires
√ √
1. Les fonctions m et f m sont L2 (on sous-entendra : "sur R") car m et f 2 m sont L1 , donc leur produit f m est
intégrable.
D’après l’inégalité de Cauchy-Schwarz,
Z 2 Z 2 Z  Z  Z
p p 2
f (x)m(x) dx = f (x) m(x) · m(x) dx ≤ f (x) m(x) dx · m(x) dx = f (x)2 m(x) dx
donc Varm (f ) ≥ 0.
2.
(2a.) On remarque que x ≥ e ⇒ h(x) ≥ x. On en déduit : ∀x ≥ 0, x ≤ min(e, h(x)).
Par conséquent, pour tout x ∈ R, f (x)2 m(x) ≤ min(em(x), h(f (x)2 )m(x)). Comme x 7→ em(x) et x 7→
h(f (x)2 )m(x) sont intégrables, on en déduit que f 2 m est intégrable.
(2b.) On pose ϕ(x) = h(x) − h(a) − h0 (a)(x − a). Pour x > 0, ϕ0 (x) = h0 (x) − h0 (a) et ϕ00 (x) = 1/x > 0. On en
déduit que ϕ0 est strictement croissante sur ]0, +∞[. Comme ϕ0 (a) = 0, ϕ est strictement décroissante sur [0, a]
(continuité en 0) et strictement croissante sur [a, +∞[. Or ϕ(a) = 0, donc ∀x ≥ 0, ϕ(x) > ϕ(a) si x 6= a.
Z
(2c.) On pose a = f (x)2 m(x) dx. Si a = 0, la continuité de f et m montre que f 2 m = 0. Pour tout x, on a donc
Z
f (x)2 = 0 ou m(x) = 0, donc h(f (x))2 m(x) = 0. De là, Entm (f ) = h(f (x)2 )m(x) dx − h(a) = 0.
2 0
Si a > 0, on a d’après (x)2 − a) pour tout x≥ 0. On
Z 2.b. : h(f (x)) ≥ h(a) + h (a)(fZ Z multiplie par m(x)
 et
2 0 2 2
on intègre, il vient : h(f (x)) m(x) dx ≥ h(a) + h (a) f (x) m(x) dx − a = h f (x) m(x) dx , donc
Entm (f ) ≥ 0.
(2d.) Nous montrons que les fonctions d’entropie nulle sont les fonctions constantes.
On reprend les calculs précédents. Dans le cas a = 0, on obtient f = 0 (identiquement) car m ne s’annule pas.
Dans le cas a > 0, l’égalité Entm (f ) = 0 implique que x 7→ [h(f (x)2 ) − h(a) − h0 (a)(f (x)2 − a)]m(x) est nulle
(car elle est positive et continue). Comme m ne s’annule pas, on obtient le cas d’égalité de 2b., donc f (x)2 = a
pour tout x, c’est-à-dire que f est constante (continuité).
Il est immédiat que les fonctions constantes sont d’entropie nulle, ce qui achève la preuve.
Partie I
3.
1 2 1 2
(3a.) Pour tout x ∈ R, (µf 0 )(x) = √ e−x f 0 (x) donc (µf 0 )0 (x) = √ e−x [−2xf 0 (x) + f 00 (x)] = 2µ(x)Lf (x), ce qui
π π
1
montre bien : Lf = (µf 0 )0 .

(3b.) Les fonctions h01 et h02 sont continues et bornées, donc h01 h02 µ est L1 . D’après 3a., (µh02 )0 = 2µLh2 . On intègre
formellement par parties : Z Z
h01 (h02 µ) = [h1 (h02 µ)] − h1 · 2µLh2 .

Le crochet est bien convergent et nul car h1 et h02 sont bornées et µ tend vers 0 en ±∞. On en déduit la
convergence de la seconde intégrale, et l’égalité demandée.
4. Pour tout y ∈ R, (t, x) 7→ f (x cos t + y sin t)µ(y) est continue sur R2 (théorèmes généraux) ;
pour tout (t, x) ∈ R2 , y 7→ f (x cos t + y sin t)µ(y) est continue sur R (donc continue par morceaux) ;
pour tout t, x, y, |f (x cos t + y sin t)µ(y)| ≤ ||f ||∞ µ(y). Or y 7→ ||f ||∞ µ(y) est intégrable, donc Φf est continue sur
R2 .
5.
(5a.) Notons F (t, x, y) = f (x cos t + y sin t)µ(y). Mentionnons une fois pour toute qu’à (t, x) fixé, y 7→ F (t, x, y) est
continue et intégrable.
Fixons t ∈ R. D’après les théorèmes généraux, pour tout y ∈ R, x 7→ F (t, x, y) est C 1 sur R et sa dérivée est
donnée par :
∂F
(t, x, y) = (cos t)f 0 (x cos t + y sin t)µ(y).
∂x
Remarquons que cette formule définit, pour tout x, une fonction continue de y.
Pour tout x ∈ R et y ∈ R, |(cos t)f 0 (x cos t + y sin t)µ(y)| ≤ ||f 0 ||∞ µ(y). Cette dernière fonction est intégrable,
et d’après le Zthéorème de dérivation, la fonction x 7→ Φf (t, x) est C 1 (attention, à t fixé !) et a pour dérivée :
∂Φf
(t, x) = (cos t)f 0 (x cos t + y sin t)µ(y) dy.
∂x
Par un raisonnement identique au 4. (on domine l’intégrande par y 7→ ||f 0 ||∞ µ(y)), on constate que cette dérivée
partielle définit une fonction continue sur R2 .
On fixe maintenant x ∈ R. On a cette fois, pour tout y ∈ R :
∂F
(t, x, y) = (−x sin t + y cos t)f 0 (x cos t + y sin t)µ(y).
∂t
Pour (t, y) ∈ R2 , |(−x sin t + y cos t)f 0 (x cos t + y sin t)µ(y)| ≤ (|x| + |y|)||f 0 ||∞ µ(y). Cette dernière fonction est
continue et intégrable (= o(e−|y| )Z en ±∞), donc le théorème de dérivation montre que t 7→ Φf (t, x) est C 1 (à
∂Φf
x fixé), de dérivée : (t, x) = (−x sin t + y cos t)f 0 (x cos t + y sin t)µ(y) dy.
∂t
L’intégrande est continue par rapport à (t, x). Soit A > 0. Pour tout x ∈ [−A, A], pour tout t ∈ R, on domine :
Φf
|(−x sin t + y cos t)f 0 (x cos t + y sin t)µ(y)| ≤ (A + |y|)µ(y), fonction L1 , et on conclut que est continue sur
∂t
R2 .
Finalement, les deux dérivées partielles de Φf sont continues sur R2 , donc Φf est de classe C 1 sur R2 .
∂2F
Fixons t ∈ R. Pour tout (x, y) ∈ R2 , (t, x, y) = (cos t)2 f 00 (x cos t + y sin t)µ(y). Or, pour tout (x, y) ∈ R2 ,
∂x2
|(cos t)2 f 00 (x cos t + y sin t)µ(y)| ≤ ||f 00 ||∞ µ(y).
∂ 2 Φf
Z
On en déduit que (t, x) = (cos t)2 f 00 (x cos t + y sin t)µ(y) dy.
∂x2
À nouveau, la continuité de ∂xx Φf se démontre comme au 4., en dominant par ||f 00 ||∞ µ(y). Cette domination
montre en outre, pour tout (t, x) ∈ R2 :
Z
|∂xx Φf (t, x)| ≤ ||f 00 ||∞ µ(y) dy = ||f 00 ||∞ , donc ∂xx Φf est bornée sur R2 .

(5b.) La formule du 5a. montre : ∂x Φf (t, x) = (cos t)Φf 0 (t, x).


Z
1
(5c.) LΦf (t, x) = [(cos t)2 f 00 (x cos t + y sin t) − x(cos t)f 0 (x cos t + y sin t)]µ(y) dy
2
(cos t)2
Z Z
donc (sin t)LΦf (t, x) = (sin t)f 00 (x cos t + y sin t)µ(y) dy − x(cos t) (sin t)f 0 (x cos t + y sin t)µ(y) dy
2
On remarque µ0 (y) = −2yµ(y), puis on intègre par parties (le crochet étant "convergent" et nul) :
Z Z Z
(sin t)f (x cos t+y sin t)µ(y) dy = [f (x cos t + y sin t)µ(y) dy]+2 f (x cos t+y sin t)yµ(y) dy = 2 f 0 (x cos t+
00 0 0

y sin t)yµ(y) dy.


Z
2 0
Finalement, (sin t)LΦf (t, x) = (cos t) f (x cos t + y sin t)yµ(y) dy − (cos t) x sin tf 0 (x cos t + y sin t)µ(y) dy =
Z
(cos t) (x sin t − y cos t)f 0 (x cos t + y sin t)µ(y) dy = (cos t)∂t Φf (t, x).

(5d.) Le membre de droite ne dépend pas de t, on va montrer que le membre de gauche est constant.
Z
Notons, pour tout t ∈ R, G(t) = Φf (t, x)µ(x) dx.
Pour tout x ∈ R, t 7→ Φf (t, x)µ(x) est C 1 sur R, de dérivée t 7→ ∂t Φf (t, x)µ(x).
Z Z
Or |∂t Φf (t, x)| ≤ (|x| + |y|)||f 0 ||∞ µ(y) dy = A|x| + B avec A = ||f 0 ||∞ et B = ||f 0 ||∞ |y|µ(y) dy. On en
déduit la dominationZ : |∂t Φf (t, x)µ(x)| ≤ (A|x| + B)µ(x). Cette fonction est intégrable, donc G est de classe
C 1 sur R et G0 (t) = ∂t Φf (t, x)µ(x) dx.
Z
π 0
Pour t 6≡ 2 [π], on a donc, d’après 5c. : G (t) = (tan t)LΦf (t, x)µ(x) dx.
Z
1 1
Avec 3a. puis 5b. : LΦf (t, x)µ(x) dx = [µ(x)∂x Φf (t, x)] = [cos(t)µ(x)Φf 0 (t, x)] = 0 car Φf 0 est bornée
2 2
(par ||f 0 ||∞ ), d’où G0 (t) = 0.
Comme G0 est continue, on en déduit Z que G0 est identiquement nulle,Z donc G est constante.
Z
Or pour tout x ∈ R, Φf (0, x) = f (x)µ(y) dy = f (x), donc G(0) = Φf (0, x)µ(x) dx = f (x)µ(x) dx.

2
Partie II
6. Φf est continue d’après 4., et un calcul immédiat montre qu’elle est positive et bornée sur R2 par ||f ||∞ . Comme
h est continue sur R+ , et en particulier bornée sur [0, ||f ||∞ ], on en déduit que h ◦ Φf est continue et bornée sur
R2 .
Z dominant l’intégrande par x 7→ ||f
En Z ||∞ µ(x), on constate queZ J et continue, et un calcul rapide montre J(0) =
h(f (x))µ(x) dx et, en posant a = f (y)µ(y) dy : J(π/2) = h(a)µ(x) dx = h(a).

On peut remarquer au passage : J(0) − J(π/2) = Entµ ( f ).
7.
(7a.) Par croissance de l’intégrale, δ ≤ Φf (t, x) ≤ ||f ||∞ . Or h est C 1 sur [δ, +∞[, donc pour tout x ∈ R, t 7→
h(Φf (t, x))µ(x) est de classe C 1 sur R, de dérivée :
t 7→ ∂t Φf (t, x)h0 (Φf (t, x))µ(x).
La fonction h0 est continue, donc bornée, mettons par M , sur [δ, ||f ||∞ ] et on a trouvé au 5d. deux constantes
positives A et B telles que |∂t Φf (t, x)h0 (Φf (t, x))µ(x)| ≤ (A|x| + B)M µ(x).
D’après le théorème
Z de dérivation, J est donc de classeZ C 1 sur R, et avec 5c. :
(cos t)J 0 (t) = (cos t)∂t Φf (t, x)h0 (Φf (t, x))µ(x) dx = (sin t)LΦf (t, x)[1 + ln Φf (t, x)]µ(x) dx.
Z
0 (sin t)
Comme t est fixé, on applique 3a. à x 7→ Φ(t, x), donc (cos t)J (t) = ∂x [µ(x)∂x Φ(t, x)]·[1+ln Φf (t, x)] dx.
 2 Z 
0 sin(t) ∂x Φf (t, x)
On intègre par parties : (cos t)J (t) = [µ(x)∂x Φ(t, x)(1 + ln Φf (t, x))] − µ(x)∂x Φ(t, x) dx =
2 Φf (t, x))
∂x Φf (t, x)2
Z
sin t
− µ(x) dx.
2 Φf (t, x))
L’intégration par parties est bien licite car pour tout (t, x) : |µ(x)∂x Φ(t, x)(1 + ln Φf (t, x))| ≤ ||f 0 ||∞ M µ(x), ce
qui entraîne la convergence et la nullité du crochet.
(7b.) Notons que f étantp minorée par δ et f 0 bornée, la fonction
p g est bornée et gµ est intégrable. On fixe (t, x).
Les fonctions y 7→ f (x cos t + y sin t)µ(y) et y 7→ g(x cos t + y sin t)µ(y) sont L2 donc leur produit y 7→
|f 0 (x cos t + y sin t)µ(y)| est intégrable, et d’après l’inégalité de Cauchy-Schwarz :
Z 2 Z 2
Φf 0 (t, x)2 = f 0 (x cos t + y sin t)µ(y) dy ≤ |f 0 (x cos t + y sin t)|µ(y) dy
Z 2
p p
= f (x cos t + y sin t)µ(y) g(x cos t + y sin t)µ(y) dy
Z Z
≤ f (x cos t + y sin t)µ(y) dy g(x cos t + y sin t)µ(y) dy = Φf (t, x) · Φg (t, x).

∂x Φf (t, x)2 Φf 0 (t, x)2


Z Z
0 tan t sin(2t)
(7c.) Pour tout t ∈]0, π/2[, en appliquant 5b. : J (t) = − µ(x) dx = − µ(x) dx.
2 Φf (t, x)) 4 Φf (t, x))
Z Z
sin(2t) sin(2t)
On constate J 0 (t) ≤ 0 et d’après 7b. et 5d. : |J 0 (t)| ≤ Φg (t, x)µ(x) dx = g(x)µ(x) dx.
4 4
Z π/2 Z Z Z
sin(2t) 1
Comme J est de classe C 1 sur R, J(0)−J(π/2) = −J 0 (t) dt ≤ g(x)µ(x) dx dt = g(x)µ(x) dx,
0 4 4
ce qui est l’inégalité attendue d’après les calculs de 6.
8. Comme f est bornée et h continue, h(f 2 ) est bornée donc h(f 2 )µ est intégrable, c’est-à-dire que f admet une
entropie par rapport à µ.
Soit δ > 0. On pose fδ = δ + f 2 , donc fδ0 = 2f f 0 . En particulier, fδ ∈ Cb2 et fδ ≥ δ et on peut appliquer les
résultats de 7. à fδ .
fδ02 4f 2 f 02
Z Z
On pose gδ = = ≤ 4f . On remarque : 4 gδ (x)µ(x) dx ≤ f 0 (x)2 µ(x) dx.
02 1
fδ δ + f2 Z  Z
Z
D’après 7c. : h(δ + f (x)2 )µ(x) dx − h (δ + f (x)2 )µ(x) dx ≤ f 0 (x)2 µ(x) dx.
Montrons que les termes du membre de gauche sont des fonctions continues de δ :
Pour le premier, si on se restreint à δ ≤ 1, on peut majorer grossièrement |h(δ + f (x)2 )| par le maximum M
2 2
de |h|
Z sur [0, 1 + ||f ||∞ ], ce qui permet la domination : ∀x ∈ R, ∀δ ∈ [0, 1], |h(δ + f (x) )µ(x)| ≤ M µ(x), donc
δ 7→ h(δ + f (x)2 )µ(x) dx est continue sur [0, 1].
Z Z
Pour le second, (δ + f (x)2 )µ(x) dx = δ + f (x)2 µ(x) dx, et comme h est continue sur R+ , on peut donc faire
tendre δ vers 0, ce qui donne l’inégalité demandée.

3
Partie III
1
9. Posons f (x) = x. C’est une fonction C à dérivée constante, donc bornée, donc elle admet une entropie d’après
l’hypothèse de cette partie.
D’après 2a. et 1., f 2 m et f m sont intégrables, donc x 7→ (1 + |x| + x2 )m(x) est intégrable.
10.
Z Z
1
(10a.) Supposons (2) prouvée pour des fonctions g ∈ Cb telles que g(x)m(x) dx = 0 et g 2 (x)m(x) dx = 1.
Z Z
On pose E = f (x)m(x) dx et σ ≥ 0 tel que σ 2 = (f (x) − E)2 m(x) dx. En développant, on vérifie que
σ 2 = Varm (f ).
f −E
Si Varm (f ) = 0, l’inégalité (2) est évidente, on suppose donc σ > 0. On pose alors g = . Il est clair
Z Z σ
que g ∈ Cb1 , g(x)m(x) dx = 0 et g(x)2 m(x) dx = 1, donc on peut appliquer (2) : 1 = Varm (g) ≤
|f 0 (x)|2
Z Z
C C
m(x) dx, et donc Varm (f ) = σ 2
≤ |f 0 (x)|2 m(x) dx.
2 σ2 2
Z Z
(10b.) On suppose donc f m = 0 et f 2 m = 1. On fixe ε > 0 et fε = 1 + εf . On va ensuite faire tendre ε vers 0.
Z Z Z
2 2
D’une part, on développe : (1 + εf (x)) m(x) dx = 1 + 2ε f (x)m(x) dx + ε f (x)2 m(x) dx = 1 + ε2 , donc
Z 
2
h (1 + εf (x)) m(x) dx = h(1 + ε2 ) = (1 + ε2 ) ln(1 + ε2 ) ∼ ε2 .
D’autre part, on considère le développement limité h(1 + y)2 = 2(1 + y)2 ln(1 + y) = 2y + 3y 2 + y 3 θ(y), où θ
est une fonction bornée sur un intervalle
Z [−α, +α], mettons par M > 0.Z
Pour ε assez petit, ε||f ||∞ ≤ α. et | ε3 f (x)3 θ(εf (x))m(x) dx| ≤ ε3 M |f (x)3 |m(x) dx.
On en déduit :
Z Z 
2
Entm (fε ) = h(1 + εf (x))µ(x) dx − h (1 + εf (x)) m(x) dx
Z Z
= 2εf (x)µ(x) dx + 3ε2 f (x)2 µ(x) dx + O(ε3 ) − ε2 + o(ε2 ) ∼ 2ε2
Z
Entm (fε )
Or fε0 0
= εf , donc d’après l’inégalité (1) : ≤ C f 0 (x)2 m(x) dx.
Z ε2
C
On fait tendre ε vers 0, d’où 1 ≤ f 0 (x)2 m(x) dx, ce qui est l’inégalité attendue.
2
11.
(11a.) On peut remarquer H(λ) > 0. Par continuité et positivité de l’intégrande, H(λ) = 0 entraîne m identiquement
nulle, ce qui est absurde pour
Z une mesure. Z
Pour tout λ ∈ R, λH 0 (λ) = λf (x)eλf (x) m(x) dx = h(eλf (x) )m(x) dx et
H(λ) ln(H(λ)) = h(H(λ)).
On pose g(x) = eλf (x)/2 et on reconnaît : λH 0 (λ) − H(λ) ln(H(λ)) = Entm (g).
Or g ∈ Cb1 car f ∈ Cb1 , donc par hypothèse de cette partie, g admet bien une entropie et
Cλ2
Z  2 Z
0
Entm (g) ≤ C λ/2f (x)e λf (x)/2
m(x) dx ≤ Cλ /4 f 0 (x)2 eλf (x) m(x) dx ≤
2
H(λ),
4
0
en tenant compte de |f (x)| ≤ 1.
(11b.) L’inégalité est évidente pour λ = 0, on va la prouver pour λ > 0.
ln H(λ) λH 0 (λ) − H(λ) ln H(λ
Pour tout λ > 0, on pose ϕ(λ) = . Cette fonction est C 1 sur R∗+ et ϕ0 (λ) = ≤
λ λ2 H(λ)
C/4. Z
On a H(0) = 1 et H 0 (0) = f (x)m(x) dx. Comme H est C 1 sur R, on a au voisinage de 0 : H(λ) =
1 + H 0 (0)λ + o(λ), donc ϕ(λ) tend vers H 0 (0) quand λ → 0. On prolonge ainsi ϕ par continuité en 0.
C
Soit λ > 0. D’après le théorème des accroissements finis, il existe c ∈]0, λ[ tel que ϕ(λ) − ϕ(0) = ϕ0 (c) ≤ λ,
4
donc ln H(λ) ≤ λH 0 (0) + Cλ2 /4, c’est-à-dire :
 Z 
H(λ) ≤ exp λ f (x)m(x) dx + Cλ2 /4 .

4
12. On ne peut pas appliquer directement 11. car f : x 7→ x n’est pas bornée. On pose donc, pour n ∈ N∗ , fn :
1
x 7→ nArctan(x/n). Pour tout n, fn est bornée (par nπ/2), de classe C 1 , et ∀x ∈ R, fn0 (x) = ∈ [0, 1],
1 + x2 /n2
donc
Z fn vérifie les hypothèses
 Z de 11. On peut donc écrire,
 pour tout λ ≥ 0 :
eλfn (x) m(x) dx ≤ exp λ fn (x)m(x) dx + Cλ2 /4 .
En utilisant l’inégalité |Arctan(x)| ≤ |x| (car Arctan est concave sur R+ et impaire), on remarque pour tout (n, x) :
|fn (x)m(x)| ≤ |x|m(x). Cette fonction est intégrable (domination), et comme pour tout x, fn (x) → x quand n tend
vers +∞ (convergence simple), le théorème  Zde convergence dominée assure (via la continuité de l’exponentielle)
que le second membre tend vers S = exp λ xm(x) dx + Cλ2 /4 .

Z ε > 0. Pour n assez grand, on a donc :


Soit
eλfn (x) m(x) dx ≤ S + ε.
Z b
Si on fixe deux réels a < b, la positivité de x 7→ eλfn (x) m(x) permet d’écrire : eλfn (x) m(x) dx ≤ S + ε.
a
Pour x ∈ [a, b], eλfn (x) m(x) tend vers eλx m(x) (convergence simple) et 0 ≤ eλfn (x) m(x) ≤ eλx m(x) (domination),
cette dernière fonction étant continue donc intégrable sur [a, b]. Par le théorème de convergence dominée, il vient :
Z b
eλx m(x) dx ≤ S + ε.
a
Comme
Z ceci est vrai de tout segment [a, b], la fonction x 7→ eλx m(x), Z
qui est positive, est intégrable sur R, et
eλx m(x) dx ≤ S + ε. Ceci étant vrai pour tout ε > 0, on en déduit eλx m(x) dx ≤ S, ce qui est l’inégalité
attendue.
13.
(13a.) Soit λ ≥ 0. Pour tout x ∈ [a, +∞[, 1 ≤ eλ(x−a) = e−λa eλx et d’après 12., x 7→ eλx est L1 et :
Z +∞ Z +∞ Z
m(x) dx ≤ eλ(x−a) m(x) dx = e−λa eλx m(x) dx ≤ exp(λ(M − a) + Cλ2 /4).
a a
En dérivant l’argument de l’exponentielle, on constate qu’il admet un minimum pour λ = 2(a − M )/C. Pour
cette valeur de λ, on obtient l’inégalité demandée.
2
(13b.) Notons que x 7→ eαx m(x) est continue sur R, il suffit donc de justifier l’intégrabilité au voisinage de ±∞.
Z +∞
On remarque que, comme m est continue sur R, l’application x 7→ − m(t) dt est une primitive de m.
x
On fixe α < 1/C comme dans l’énoncé et a ≥ M comme dans la question précédente. On intègre formellement
par parties :
Z +∞  Z +∞ +∞ Z +∞ Z +∞ 
2 2 2
eαx m(x) dx = −eαx m(t) dt + 2αxeαx m(t) dt dx.
a x a a x
Z +∞
αx2
D’après 12., pour x ≥ a ≤ M : 0 ≤ e m(t) dt ≤ exp[(α − 1/C)x2 + 2M x/C − M 2 /C], qui tend vers 0
x
quand x → +∞ (car α < 1/C), ce qui montre que le crochet est bien convergent.
Fixons β ∈]α − 1/C, 0[. On vérifie (croissances comparées) qu’au voisinage de +∞ : x exp[(α − 1/C)x2 +
2
2M x/C − M 2 /C] = o(eβx ). Ceci assure la convergence de la seconde intégrale.
Z +∞
2 2
On en déduit que eαx m(x) dx est convergente, donc la fonction x 7→ eαx m(x) est intégrable au voisinage
a
de +∞. Z −a
2
Étudions le cas de −∞. Par un changement de variable évident, eαx m(x) dx converge si et seulement si
−∞
Z +∞
2
eαx m(−x) dx converge.
a
Or m1 : x 7→ m(−x) est une mesure (clair). Vérifions qu’elle satisfait l’hypothèse du III.
Soit f : R → R une fonction C 1 de dérivée bornée. On note f1 : x 7→ f (−x), qui est également C 1 , de dérivée
bornée. D’après l’hypothèse de III, f1 admet une entropie par rapport à m, donc x 7→ h(f (−x)2 )m(x) est L1 .
h(f (x)2 )m(−x) est L1 , donc f admet une entropie par rapport à m1 .
On en déduit que x 7→ Z Z
De plus, Entm (f1 ) ≤ C f10 (x)2 m(x) dx. Le même changement de variable assure que Entm1 (f ) ≤ C f 0 (x)2 m1 (x) dx
On
Z en déduit que lesZ résultats
+∞ Z −a à la mesure m1 . En particulier, pour a ≥
prouvés ci-dessus s’appliquent
2 2 2
xm(−x) dx = −M , eαx m(−x) dx converge, donc eαx m(x) dx converge. Finalement, x 7→ eαx m(x)
a −∞
est intégrable au voisinage de −∞, et donc sur R.

5
Partie IV
14. Z
(14a.) Analysons la situation : K = p(x) dx est une constante strictement positive car p est continue et strictement
positive. Si on note P une primitive de p, on peut écrire l’égalité sous la forme P (u)0 = K, soit P (u(t)) = Kt+K0 .
Comme p > 0, ses primitives sont strictement croissantes, donc injectives, on pourra écrire u(t) = P −1 (Kt+K0 ).
Nécessairement, u est strictement croissante (par composition), on veut donc que u tende vers −∞ en 0 et +∞
en 1, c’est-à-dire que P tende vers 0 en −∞ (donc K0 = 0) et K en +∞.
Z x
Ces remarques informelles amènent à poser, pour x ∈ R, P (x) = p(t) dt. L’application P est bien définie
−∞
car p est intégrable, c’est une primitive de p, donc elle est strictement croissante sur R. Ses limites en −∞ et
+∞ sont respectivement 0 et K, donc elle induit une bijection de R sur ]0, K[. Sa réciproque P −1 est de classe
C 1 car P 0 = p ne s’annule pas.
On pose donc, pour t ∈]0, 1[, u(t) = P −1 (Kt). Par composition, u est C 1 , bijective de ]0, 1[ sur R, et un calcul
K K
direct donne u0 (t) = = .
p(P −1 (Kt)) p(u(t))
Z  Z 
(14b.) D’après 14a., pour tout t ∈]0, 1[ : p(x) dx · q(x) dx = p(u(t))q(v(t))u0 (t)v 0 (t).
Notons que les quatre facteurs sont positifs. On considère les racines carrées des deux membres.
D’après l’hypothèse (4) : p(u(t)) q(v(t)) ≤ r u(t)+v(t)
p p
2 .
a2 + b2 p
Rappelons que pour tout (a, b) ∈ R2 : |ab| ≤ , (vient de (|a| − |b|)2 ≥ 0). On en déduit : u0 (t)v 0 (t) ≤
2
u0 (t) + v 0 (t)
.
2 s Z  Z    u0 (t) + v 0 (t)
Finalement, p(x) dx · q(x) dx ≤ r u(t)+v(t)
2 · .
2
On intègre les deux membres sur ]0, 1[, en tant que fonctions de t. Le membre de gauche est constant, donc
inchangé par cette opération.
u(t) + v(t)
Considérons w : t ∈]0, 1[7→ . C’est une fonction C 1 , strictement croissante (somme de fonctions
2
strictement croissante). Comme u et v tendent toutes les deux vers −∞ en 0 et +∞ en 1, il en va de même
pour w, qui définit une bijection croissante de ]0, 1[ sur R.
Z 1 
u(t) + v(t) u0 (t) + v 0 (t)
 Z
Par changement de variable, on a donc : r dt = r(x) dx, et on obtient finale-
0 2 2
ment l’inégalité (5).
15.
(15a.) Si y ∈
/ A, le premier membre est nul et l’inégalité est évidente.
Si y ∈ A, alors d(x, A) ≤ |x − y|, donc 12 d(x, A)2 − x2 − y 2 ≤ (x − y)2 /2 − x2 − y 2 = −(x + y)2 /2 et on obtient
l’inégalité demandée par croissance de exp.
(15b.) L’inégalité de 15a. ressemble fort à (4). Il suffit en effet de poser p(x) = exp(d(x, A)2 /2 − x2 ), q(x) =
1A (x) exp(−x2 ) et r(x) = exp(−x2 ) pour obtenir trois fonctions positives, continues par morceaux, intégrables
sur R et vérifiant (4). On applique donc 14b. :
Z Z Z 2
exp(d(x, A)2 /2 − x2 ) dx × 1A (x) exp(−x2 ) dx ≤ exp(−x2 ) dx .
Z
En divisant les deux membres par π, puis en intégrant le membre de droite, il vient : exp(d(x, A)2 /2)µ(x) dx×
Z Z
1A (x)µ(x) dx = µ(A) exp(d(x, A)2 /2)µ(x) dx ≤ 1, ce qui est l’inégalité attendue.
16.
SN
(16a.) On note A = k=1 I k où I k est un intervalle et N un entier naturel non nul. Par double inclusion, on montre
SN
que At = k=1 Itk :
S’il existe k tel que x ∈ Itk , alors d(x, A) ≤ d(x, Ik ) ≤ t, donc x ∈ At .
Réciproquement, supposons x ∈ At . On se donne une suite (ai ) d’éléments de A telle que d(x, ai ) → d(x, A).
Comme A est une réunion finie d’intervalles, il existe au moins un intervalle I k qui contient une infinité de
termes de cette suite. On en déduit d(x, I k ) ≤ d(x, A) ≤ t, donc x ∈ Itk .
On vérifie maintenant que si I est un intervalle, alors It est encore un intervalle.
Si I est vide, alors It = R. On suppose donc I non vide.

6
Fixons x < w < y avec x, y ∈ It .
Supposons qu’il existe a ∈ I tel que a ≤ w. On a alors deux cas : s’il existe b ∈ I tel que w < b, alors w ∈ I
et d(w, A) = 0 ≤ t ; sinon, I est situé à gauche de w, donc d(w, I) ≤ d(y, I) ≤ t. On procède symétriquement
s’il existe a ∈ I tel que a ≥ w, en considérant cette fois la position de x. Finalement, w ∈ It , donc It est un
intervalle, ce qui montre que At ∈ Int pour tout t ≥ 0.
/ At , exp(d(x, A)2 /2) ≥ exp(t2 /2), donc (1 − 1A (x)) exp(d(x, A)2 /2) ≥ (1 − 1A (x)) exp(t2 /2), puisque les
(16b.) Si x ∈
deux membres sont nuls lorsque x ∈ A. Z
En multipliant par µ(x) et en intégrant, il vient : (1 − 1A (x)) exp(d(x, A)2 /2)µ(x) dx ≥ exp(t2 /2)(1 − µ(At )).
D’autre part, on majore 1 − 1A ≤ 1 et donc
Z Z d’après 15b. :
1
(1 − 1A (x)) exp(d(x, A)2 /2)µ(x) dx ≤ exp(d(x, A)2 /2)µ(x) dx ≤ , d’où l’inégalité attendue.
µ(A)

EDB

7
ÉCOLE POLYTECHNIQUE – ÉCOLES NORMALES SUPÉRIEURES

ÉCOLE SUPÉRIEURE DE PHYSIQUE ET DE CHIMIE INDUSTRIELLES

CONCOURS D’ADMISSION 2016 FILIÈRE PC

COMPOSITION DE MATHÉMATIQUES – (XEULC)

(Durée : 4 heures)

L’utilisation des calculatrices n’est pas autorisée pour cette épreuve.


Toute affirmation doit être clairement et complètement justifiée.

⋆⋆⋆

Les parties I, II et III sont assez largement indépendantes. En particulier la partie II peut
être traitée indépendamment de la partie I en admettant les trois premières questions et la partie
III (exceptée la dernière question) indépendamment de la partie II. Il est cependant vivement
conseillé de suivre la progression naturelle du problème.

Notations

Dans le problème, pour tous entiers positifs non nuls n et k, Mn,k (R) désignera les matrices
àcoefficients
 réels de taille n × k. Un vecteur u ∈ Rn sera considéré comme un vecteur colonne
u1
 .. 
 .  et uT désignera le vecteur ligne obtenu par transposition. De même, pour M ∈ Mn,k (R),
un
MT désignera la transposée de M .

On note ϕ la fonction de [0, +∞[ dans R définie par



0 si t = 0
ϕ(t) = (1)
−t ln(t) sinon.
P
Soit N > 2 un entier. On note ΣN l’ensemble des vecteurs p ∈ RN tels que N i=1 pi = 1 et pi > 0
pour tout 1 6 i 6 N . On remarquera que p peut être interprété comme une loi de probabilité
sur {1, . . . , N }. On note également HN la fonction définie sur ΣN par
N
X
HN (p) = ϕ(pi ) .
i=1

1
Partie I
1. Vérifier que ϕ est de classe C 0 sur [0, +∞[ et C ∞ sur ]0, +∞[. Donner la limite de la
dérivée ϕ′ (t) de ϕ lorsque t tend vers 0 dans ]0, +∞[.
2. Montrer que ΣN est une partie fermée, bornée et convexe de RN .
3. Montrer que HN est positive, continue sur ΣN et calculer la valeur de HN (p) lorsque
pi = 1/N pour tout i ∈ {1, . . . , N } (loi uniforme sur {1, . . . , N }).
4. (a) Soient a et b dans [0, +∞[ tels que a < b. Montrer qu’il existe ǫ ∈]0, b] tel que
ϕ(a + t) + ϕ(b − t) > ϕ(a) + ϕ(b) pour tout t > 0 tel que t 6 ǫ.
(b) En déduire que HN atteint son maximum sur ΣN en un unique point que l’on déter-
minera.
5. On
P+∞ note Σ∞ l’ensemble des suites de réels p = (pi )i>1 telles que pi >P
0 pour tout i > 1 et

i=1 pi = 1. On note H∞ la fonction sur Σ∞ définie par H∞ (p) = i=1 ϕ(pi ) à valeurs
dans R+ ∪ {+∞}.
(a) On considère a ∈]0, 1[ et pi = a(1 − a)i−1 pour i > 1. Calculer H∞ (p) et étudier ses
variations en fonction de a.
(b) Montrer qu’il existe p ∈ Σ∞ telle que H∞ (p) = +∞. (Ind : On pourra utiliser sans
démonstration que la série de terme général n−1 ln(n)−β pour n > 2 converge si et
seulement si β > 1).
6. Soit n un entier strictement positif. On considère une famille (Xk )16k6n de n variables
aléatoires à valeurs dans {1, . . . , N }, deux à deux indépendantes et de même loi, définies sur
un espace probabilisé (Ω, A , P). On suppose de plus que P(X1 =Q i) = pi et que pi > 0 pour
tout i ∈ {1, . . . , N }. Montrer que pour tout ǫ > 0, on a P n1 ln ( nk=1 pXk ) + HN (p) > ǫ
tend vers 0 lorsque n tend vers l’infini.

Partie II
PN
Soient f ∈ RN et Jf : ΣN → R définie par Jf (p) = HN (p) + i=1 pi fi . On note

Jf,∗ = sup{ Jf (p) | p ∈ ΣN }

la borne supérieure de Jf sur ΣN et ΣN (f ) = { p ∈ ΣN | Jf (p) = Jf,∗ } l’ensemble des p de ΣN


pour lesquels la borne supérieure est atteinte.
7. Montrer que ΣN (f ) est non vide.
8. Soit p ∈ ΣN .
(a) On suppose que p1 = 0 et p2 > 0. Montrer alors qu’il existe p′ dans ΣN tel que
Jf (p′ ) > Jf (p) (on pourra chercher p′ proche de p).
(b) En déduire que si p ∈ ΣN (f ), alors pi > 0 pour tout i ∈ {1, . . . , N }.
9. Soit p ∈ ΣN . On
Psuppose maintenant que pi > 0 pour tout i ∈ {1, . . . , N }. On note
N
E0 = {a ∈ RN | a
i=1 i = 0}.
(a) Vérifier que E0 est un sous-espace vectoriel de RN dont on donnera la dimension.
Identifier l’orthogonal E0⊥ de E0 pour le produit scalaire canonique sur RN .
(b) Soient a ∈ E0 et p̃ : R → RN définie par p̃(t) = p + ta. Montrer qu’il existe ǫ > 0 tel
que p̃(t) ∈ ΣN pour tout t ∈] − ǫ, ǫ[. Calculer la dérivée de p̃ en 0.

2
P
(c) On suppose de plus que p ∈ ΣN (f ). Montrer que pour tout a ∈ E0 , on a N i=1 ai (fi −
ln(pi )) = 0. En déduire qu’il existe c ∈ R, tel que ln(pi ) = fi + c pour tout i ∈
{1, . . . , N }.
P
10. Identifier ΣN (f ). Montrer que Jf,∗ = ln( N fi
i=1 e ).
P
On considère maintenant F :]0, +∞[→ R la fonction définie par F (β) = β1 ln( N i=1 e
βfi )

11. Montrer que F est dérivable et calculer sa dérivée F ′ . Montrer de plus que pour tout
β ∈]0, +∞[, il existe p(β) ∈ ΣN (βf ) tel que F ′ (β) = − β12 HN (p(β)).
12. Etudier les limites de F en 0 et en +∞.

Partie III

Soient (Ω, A , P) un espace probabilisé et X : Ω → {1, . . . , N } une variable aléatoire de loi


q ∈ ΣN . On suppose que l’on dispose d’une famille finie g = (gk )1≤k≤d de fonctions sur {1, . . . , N }
à valeurs dans R et de la valeur g k = E(gk (X)) de l’espérance de gk (X) pour tout k ∈ {1, . . . , d}.

On note
N
X
ΣN (g, g) = { p ∈ ΣN | pi gk (i) = g k , 1 6 k 6 d } ,
i=1

et on remarque que q ∈ ΣN (g, g) et que si p ∈ ΣN (g, g) alors pour toute variable aléatoire
Y : Ω → {1, . . . , N } de loi p, on a E(gk (X)) = E(gk (Y )).

On cherche dans cette partie à déterminer les probabilités p de ΣN (g, g) sur lesquelles HN
atteint son maximum.

Soient M ∈ MN,d (R) définie par Mi,j = gj (i) pour (i, j) = {1, . . . , N }× {1, . . . , d}, p ∈ ΣN et
m ∈ Rd . On note A ∈ Md (R) la matrice carrée de taille d×d définie pour tous (k, l) ∈ {1, . . . , d}2
par
XN
Alk = pi (Mil − ml )(Mik − mk ) .
i=1

On note M f = (M |1) ∈ MN,d+1 (R) la matrice augmentée obtenue en ajoutant une colonne
de 1 à droite de M .
13. Vérifier que si Y : Ω → {1, . . . , N } est une variable aléatoire de loi p, alors Alk = E((gl (Y )−
ml )(gk (Y ) − mk )) puis que A est une matrice symétrique telle que θ T Aθ > 0 pour tout
θ ∈ Rd .
14. Soit θ ∈ Rd tel que θ T Aθ = 0. On suppose que pi 6= 0 pour tout 1 6 i 6 N .
(a) P
Montrer qu’il existe c ∈ R, que l’on précisera, tel que pour tout i ∈ {1, . . . , N }, on a
d
l=1 Mil θl = c.
(b) Montrer que si KerM f = {0} alors θ = 0.

3
PN fi (θ)
On note pour tout θ ∈ Rd , f (θ) = M θ ∈ RN , Z(θ) = i=1 e et

ef1 (θ) efN (θ)


p(θ) = ( ,..., ) ∈ ΣN
Z(θ) Z(θ)

où f (θ) = (f1 (θ), . . . , fN (θ)). Enfin, on considère la fonction L : Rd → R définie par

L(θ) = ln(Z(θ)) − q T M θ .

15. Montrer que L est de classe C 1 et calculer son gradient.


16. Montrer que si θ est un point critique de L (c’est-à-dire en lequel le gradient de L s’annule)
alors M T p(θ) = M T q et p(θ) ∈ ΣN (g, g).
17. Montrer que L est de clase C 2 et que pour tous entiers 1 6 l, k 6 d on a

XN
∂2L
(θ) = pi (θ)(Mil − ml (θ))(Mik − mk (θ))
∂θl ∂θk
i=1

où m(θ) = M T p(θ).
f = {0}.
On suppose dorénavant que KerM
18. On s’intéresse dans cette question au nombre de points en lesquels la fonction L atteint
son minimum.
(a) Montrer que si θ et θ ′ sont deux points distincts de RN tels que L admet un point
critique en θ, alors la dérivée de t → L(tθ + (1 − t)θ ′ ) est strictement croissante sur
[0, 1] et s’annulle en t = 1.
(b) En déduire qu’il existe au plus un point critique pour L et conclure sur le nombre de
points en lesquels L atteint son minimum.
19. On suppose que la fonction L a un minimum global atteint en θ∗ .
(a) Montrer que HN (p(θ∗ )) > HN (q) puis que HN (p(θ∗ )) est la valeur maximale de HN
sur ΣN (g, g).
(b) Montrer que p(θ∗ ) est l’unique point de ΣN (g, g) en lequel HN atteint son maximum.

∗ ∗

4
X-ENS PC 2016 : Entropie de Shannon

Gilbert Primet
24 avril 2016

Partie I
1. ϕ, produit de deux fonctions C ∞ est C ∞ sur ]0, +∞[. De plus :

lim t ln(t) = 0
t→0

donc ϕ est continue en 0 (donc C0 sur [0, +∞[.

∀t ∈]0, +∞[ϕ 0 (t) = −1 − ln(t)

et
lim ϕ 0 (t) = +∞.
t→0

(n) (n)
2. (a) Soit une suite convergente (pn ) = (p1 , · · · , pN ) d’éléments de ΣN de limite p = (p1 , · · · , pN ). On a
donc :
N
(n) (n)
X
∀i ∈ [|1, N |]∀n ∈ Npi > 0, ∀n ∈ N pi = 1.
i=1
Par passage à la limite et conservation des inégalités larges, on obtient :
N
X
∀i ∈ [|1, N |]pi > 0. pi = 1.
i=1

Donc p ∈ ΣN . Ceci prouve par caractérisation séquentielle que ΣN est fermé.


N
X
(b) Si p ∈ ΣN , alors kpk1 = pi = 1 donc ΣN est borné (On peut aussi remarquer que p ∈ ΣN ⇒ kpk∞ 6
i=1
1.)
(c) Enfin, si (p, q) ∈ Σ2N , p = (p1 , · · · , pN ), q = (q1 , · · · , qN ), et λ ∈ [0, 1], alors, λp + (1 − λ)q = (λpi + (1 −
λqi )i∈[|1,N |] , et :
N
X N
X N
X
λpi + (1 − λqi = λ pi + (1 − λ) qi = λ + (1 − λ) = 1
i=1 i=1 i=1
et
∀i ∈ [|1, N |]λpi + (1 − λ)qi > 0.
donc
∀(p, q) ∈ Σ2N ∀λ ∈ [0, 1]λp + (1 − λ)q ∈ ΣN .
ΣN est donc convexe.
3. ϕ est positive sur [0, 1], donc par somme HN est positive. Les projections p 7→ pi sont continues, ϕ est
continue, donc par composée et somme, HN est continue.
Lorsque pi = N1 , alors
1
 
HN (p) = N ϕ = ln(N )
N

1
4. (a) Posons g(t) = ϕ(a+t)+ϕ(b −t)−ϕ(a)−ϕ(b) pout t ∈ [0, b]. g est définie et continue sur [0, b], dérivable
sur ]0, b[, g(0) = 0 et ∀t ∈]0, b[ g 0 (t) = ϕ 0 (a + t) − ϕ 0 (b − t). Or, par stricte croissance de ln, ϕ 0 décroît
strictement sur ]0, +∞[. Pour a + t < b − t, soit t < a+b 0 a+b
2 , g (t) > 0. g croît donc strictement sur [0, 2 [,
et comme g(0) = 0, on a donc : # "
a+b
∀t ∈ 0, g(t) > 0
2
ce qui montre le résultat demandé pour ε = a+b 2
(b) La fonction HN est continue sur ΣN . De plus ΣN est fermé et borné. HN est donc bornée sur ΣN et
atteint ses bornes. D’après la question précédente, si HN atteint son maximum en (p1 , · · · , pN ), tous
les pi sont égaux. Si ce n’était pas le cas, avec pi < pj , il existerait t ∈]0, pj [ tel que
ϕ(pi + t) + ϕ(pj − t) > ϕ(pi ) + ϕ(pj ),
et l’on aurait alors
HN (q1 , · · · , qN ) > HN (p1 , · · · , pN ),
où (q1 , · · · , qN ) s’obtient à partir de (p1 , · · · , pN ), en posant
qi = pi + t, qj = pj − t, qk = pk si k ∈ [|1, N |] \ {i, j}.
On vérifie aisément que q ∈ ΣN .
Le maximum est donc atteint au seul élément p = ( N1 , · · · , N1 ), qui correspond à la probabilité uni-
forme sur {1, · · · , N }. et ce maximum est ln(N ).
5. (a) ∀i ∈ N∗ ϕ(pi ) = −a(1 − a)i−1 (ln(a) + (i − 1) ln(1 − a)). On sait que

X 1
∀x ∈] − 1, 1[ xn = ,
1−x
n=0

et donc par dérivation d’une série entière à l’intérieur de son intervalle de convergence :

X 1
∀x ∈] − 1, 1[ nxn−1 = ,
(1 − x)2
n=1

d’où :

X x
∀x ∈] − 1, 1[ nxn = .
(1 − x)2
n=0
Par le changement d’indice n = i − 1, on obtient donc :
(1 − a) ln(1 − a) ϕ(a) + ϕ(1 − a)
H∞ (p) = − ln(a) − = .
a a
Notons h(a) cette quantité. h est dérivable sur ]0, 1[ et :
a(ϕ 0 (a) − ϕ 0 (1 − a)) − (ϕ(a) + ϕ(1 − a))
∀a ∈]0, 1[h0 (a) = .
a2
h0 (a) est donc du signe du numérateur qui vaut :
N (a) = (ϕ 0 (a) − ϕ 0 (1 − a)) − (ϕ(a) + ϕ(1 − a)) = a(− ln(a) + ln(1 − a) + ln(a)) + (a − 1) ln(1 − a),
et :
N (a) = a ln(1 − a) < 0.
h décroît donc sur ]0, 1[ de +∞ à 0.
C
(b) On peut choisir la série de Bertrand convergente de terme général pi = .où C est choisi de façon
i ln2 (i)
+∞
X
que pi = 1 On a alors, lorsque i tend vers +∞ :
i=1

ln(pi ) ∼ − ln(i)
donc
C
ϕ(pi ) ∼
i ln(i)
et donc
Hn (p) = +∞.

2
6. Les variables aléatoiresYk = ln(pXk ) = (ln ◦p)(Xk ) sont deux à deux indépendantes et de même loi :

Yk (Ω) = {ln(p1 ), · · · , ln(pN )}, ∀i ∈ [|1, N |]p([Yk = ln(pi )]) = pi .

. Donc :
n
X
E(ln(pXk )) = pi ln(pi ) = −HN (p).
i=1

(On aurait pu aussi simplement utiliser le théorème de transfert).


Enfin, pour tout entier naturel non nul n :
 n 
Y1 + · · · + Yn 1 Y 
= ln  pXk  .
n n
k=1

Ceci acquis, la propriété demandée traduit donc la loi faible des grands nombres sur la suite de va-
riables aléatoires indépendantes Yi . (Ces variables, qui sont finies , possèdent toutes une variance, et le
théorème peut donc s’appliquer).

Partie II
7. Jf , somme de HN et d’une fonction polynôme est continue sur ΣN qui est fermé et borné. JN est donc
bornée et atteint ses bornes. ΣN (f ) est donc non vide.
8. (a) D’après la question (4), il existe  ∈]0, p2 [ tel que pour tout t ∈]0, [, ϕ(t) + ϕ(p2 − t) > ϕ(0) + ϕ(p2 ).
Pour t ∈]0, [, on a :
tf1 + (p2 − t)f2 = t(f1 − f2 ) + p2 f2 .
Donc , en posant p0 = p + (t, p2 − t, 0, · · · , 0), alors p0 ∈ ΣN et Jf (p0 ) − Jf (p) = t(f1 − f2 ) + A(t), où :

A(t) = ϕ(t) + ϕ(p2 − t) − ϕ(0) − ϕ(p2 ) = −t ln(t) + o0 (t ln(t)).

Donc
Jf (p0 ) − Jf (p) ∼0 −t ln(t).
Donc, au voisinage de 0 on a : Jf (p0 ) − Jf (p) > 0, cequi contredit le fait que p ∈ ΣN (f ).
(b) On peut bien sûr répéter le même raisonnement si pi = 0 et pj > 0. Comme les composantes d’un
élément de ΣN ne peuvent pas être toutes nulles (leur somme vaut 1), on obtient le résultat demandé.
9. On munit RN du produit scalaire canonique.
(a) E0 = Vect(1, 1, · · · , 1)⊥ , donc E0 est un sous espace vectoriel de RN de dimension N − 1 et E0⊥ =
(Vect(1, · · · , 1)⊥ )⊥ = Vect(1, · · · , 1), puisque RN est de dimension finie.
(b) Si a = (0, · · · , 0), la question est triviale et tout  > 0 convient.
Supposons a , (0, · · · , 0). Notons M = maxi∈[|1,N |] |ai |. M > 0 puisque les ai sont non tous nuls par
hypothèse.
p + ta = (pi + tai )i∈[|1,N |] . On a :

N
X N
X N
X
pi + tai = pi + t ai = 1.
i=1 i=1 i=1

p
D’autre part : Si |tai | < pi , alors pi + tai > 0. En particulier, si |t| < Mi , pour tout i ∈ [|1, N |, alors
m
cette condition est réalisée. En posant m = mini∈[|1,N |] pi > 0, et  = M , on a bien p̃ ∈ ΣN pour tout
t ∈] − , +[.
Enfin ∀t ∈ Rp̃0 (t) = a. Ceci est vrai en particulier pour t = 0.
N
X
(c) ∀t ∈] − , [ u(t) = Jf (p̃(t)) = HN (p̃(t)) + (pi + tai )fi . u est dérivable sur ] − , [, et :
i=1

N
X N
X
∀t ∈] − , [u 0 (t) = −ai (1 + ln(pi + tai ) + ai f i .
i=1 i=1

3
Comme u admet un maximum en 0 (point intérieur à l’intervalle) , u 0 (0) = 0, et compte-tenu du fait
que a ∈ E0 , on obtient :
Xn
ai (fi − ln(pi )) = 0.
i=1

Donc le vecteur (ln(pi )−fi )i∈[|1,N |] est élément de E0⊥ = Vect(1, · · · , 1), et il existe donc c ∈ R tel que :∀i ∈
[|1, n|]ln(pi ) = fi + c.
10. Comme p ∈ ΣN (f ), on a de plus
N
X N
X
pi = efi +c = 1.
i=1 i=1
N 
X 
Donc c = − ln  efi  . et ΣN (f ) possède un unique élément
i=1

p = (ef1 +c , · · · , efN +c ).

On obtient N 
N
X N
X X 
Jf ,∗ = Jf (p) = pi (fi − ln(pi )) = −c pi = −c = ln  efi  .
i=1 i=1 i=1

N (β)
11. F est dérivable comme quotient de fonctions dérivables et : ∀β ∈]0, +∞[ F 0 (β) = − β 2 où N (β) =
P  PN βf eβfi P 
− ln N i=1 e βfi − P i=1 i
. Or , en posant d = − ln N i=1 e
βfi , l’unique élément de Σ (f ) est : p(β) =
N
( Ni=1 eβfi )
(ef1 +d , · · · , efN +d ), et
N
X XN
HN (p(β)) = ln( eβfi ) − pi βfi
i=1 i=1
, où
eβfi
pi = eβfi +d = PN .
βfi
i=1 e
C’est bien le résultat demandé !
12. Comme N > 2, le numérateur de F(β) tend vers ln(N ) > 0 lorsque β tend vers 0, et donc

lim F(β) = +∞.


β→0
P  P 
N βfi N β(fi −f )
Soit f = maxi∈[|1,N |] fi , et k = card{i ∈ [|1, N |]|fi = f }. Alors ln i=1 e = βf + ln i=1 e . Le
deuxième terme tend vers ek lorsque β tend vers +∞. On a donc

lim F(β) = f = max fi


β→+∞ i∈[|1,N |]

Partie III
13. On a, d’après le théorème de transfert :
N
X
Al,k = P(Y = i)(gl (i) − ml )(gk (i) − mk ) = E ((gl (Y ) − ml )(gk (Y ) − mk )) .
i=1

A est évidemment symétrique. Si θ = (θ1 , · · · , θd ) ∈ Rd , alors


 2 
X Xd  
θ T Aθ = Al,k θl θk = E  θj (gj (Y ) − mj )  > 0.
  
   
2 (l,k)∈[|1,d|] j=1

(On développe le carré et on utilise la linéarité de l’espérance. On sait de plus que l’espérance d’une
variable positive est positive.)

4
Pd
14. (a) D’après la solution de la question précédente, cela signifie que la variable aléatoire j=1 θj (gj (Y ) −
mj ) est nulle. Donc, en reprenant l’indexation de l’énoncé :

d
X d
X
θl gl (Y ) = ml θl = c.
l=1 l=1

Comme pout tout i ∈ [|1, N |]pi , 0, Y peut prendre toute valeur dans [|1, N |] avec une probabilité
non nulle. On a donc pour tout i ∈ [|1, N |] :
d
X
θl gl (i) = c
l=1

(Pour ceux que cette solution probabiliste rebute, on peut écrire l’espérance précédente comme
somme de N termes positifs, pondérés par les pi > 0. Le coefficient de chaque pi est donc nul.)
(b) on a : (θ1 , · · · , θd , −c) ∈ ker M̃, donc θ = 0 (et c = 0).
15. f1 , · · · , fN sont des polynômes, de même que θ 7→ qT Mθ (ce sont même des formes linéaires.) Plus pré-
cisément,
d
X
∀i ∈ [|1, N |] fi (θ) = Mi,l θl
l=1
et :
N
X X
qT Mθ = qi fi (θ) = qi Mi,l θl
i=1 (i,l)∈[|1,N |]×[|1,d|]

L admet donc d’après les théorèmes usuels des dérivées partielles, et :


PN fi (θ) N
∂L i=1 Mi,j e
X
∀j ∈ [|1, d|] (θ) = PN f (θ) − qi Mi,j
∂θj i=1 e
i
i=1

Ces dérivées partielles sont continues, d’après les théorèmes usuels, et en reprenant les notations de
l’énoncé :
grad(f ) = Mp(θ)T − qT M
16. Si θ est point critique de L, alors, on obtient l’égalité demandée par transposition de Mp(θ)T − qT M = 0.
Or, d’après la définition, p ∈ ΣN (ḡ, g) ⇐⇒ p ∈ ΣN , M T p = M T q. La condition matricielle est réalisée et
p(θ) ∈ ΣN . On a donc bien p(θ) ∈ ΣN (ḡ, g).
17.
PN fi (θ) N
∂L i=1 Mi,k e
X
∀k ∈ [|1, d|] (θ) = PN f (θ) − qi Mi,k
∂θk i=1 e
i
i=1
On voit que ces fonctions admettent des dérivées partielles, et que :
P  
PN N fi (θ) PN M efi (θ)
∂ 2L M M
i,l i,k e fi (θ)
i=1 Mi,k e i=1 i,l
∀(l, k) ∈ [|1, d|]2 (θ) = i=1PN − 2 .
∂θl ∂θk i=1 e
fi (θ) P
N
e fi (θ)
i=1

Donc : N  N 
N
2 ∂2 L X X  X 
∀(l, k) ∈ [|1, d|] (θ) = Mi,l Mi,k pi (θ) −  Mi,l pi (θ) 
  Mi,k pi (θ)
∂θl ∂θk
i=1 i=1 i=1

On reconnaît là la covariance des variables aléatoires gk (Y ) et gl (Y ) pour la probabilité de distribution


pi (θ). Or mk (θ) est précisément l’espérance de gk (Y ) pour cette probabilité. On obtient donc , par une
autre expression de la covariance :

Cov(X, Y ) = E(XY ) − E(X)E(Y ) = E(X − E(X))(Y − E(Y ))

la formule demandée.

5
18. (a) Posons , pour t ∈ [0, 1] :

u(t) = L(tθ + (1 − t)θ 0 ) = L((t(θi − θi0 ) + θ 0 )i∈[|1,d|]) ).

Alors u est deux fois dérivable sur [0, 1], et, avec des notations pas très heureuses :
d
X ∂L
∀t ∈ [0, 1] u 0 (t) = (θi − θi0 ) (tθ + (1 − t)θ 0 ).
∂θi
i=1

En particulier u 0 (1) = 0, puisque toutes les dérivées partielles s’annulent en θ. Puis :


 
d d 2L
X
0 
X
0 ∂ 0

∀t ∈ [0, 1] u”(t) = (θi − θi )  (θj − θj ) (tθ + (1 − t)θ ) .

 ∂θj ∂θi 
i=1 j=1

On reconnaît ( ?) que cette quantité est : V ( dk=1 (θk − θk0 )gk (X)), pour la distribution de probabilités
P
0
p(tθ + (1 − t)θ ) , et cette variance est strictement positive. En effet, si elle s’annulait, d’après la
question 14,(qui s’applique ici puisque pi (tθ + (1 − t)θ 0 ) > 0 pour tout i), on aurait θ − θ 0 = 0, ce qui
n’est pas. u 0 est donc strictement croissante.
(b) Or u 0 (0) = 0, puisque L admet un point critique en θ 0 . Ceci contredit donc la stricte croissance de u 0 .
Il existe donc au plus un point critique, donc au plus un point où L atteint sont minimum (Si un tel
minimum existe).
19. (a) Supposons d’abord toutes les composantes qi de q non nulles.
On suppose que L a un minimum global atteint en θ∗ . Le gradient de L en θ∗ est alors nul. Donc
M T p(θ∗ )T = qT M, et p(θ∗ ) ∈ ΣN (ḡ, g).

D’autre part
N N
1 X X
HN (p(θ∗ )) = − (fi (θ∗ ) − ln(Z(θ∗ ))) efi (θ∗ ) = ln(Z(θ∗ )) − E(f (θ∗ )) = ln(Z(θ∗ )) − qi fi (θ∗ ).
Z(θ∗ )
i=1 i=1

Donc
N
efi (θ∗ )
X !
HN (p(θ∗ )) − HN (q) = ln(Z(θ∗ ) − qi ln .
qi
i=1

Or ln est concave (hors programme PC), donc :


N
N 
efi (θ∗ )
X !
X 
f (θ )  = ln (Z(θ )) .
qi ln > ln  e i ∗

qi

i=1 i=1

Donc :
HN (p(θ ∗ )) > HN (q).
Si certaines composantes qi sont nulles, effectue le même calcul en gardant les composantes non
nulles. On obtient alors :

efi (θ∗ )
X !
HN (p(θ∗ )) − HN (q) = ln(Z(θ∗ ) − qi ln .
qi
i∈[|1,N |],qi ,0

Et :  
efi (θ∗ )
X !  X 
fi (θ∗ 
qi ln 6 ln  e  < ln(Z(θ∗ ).
 
qi  
i∈[|1,N |],qi ,0 i∈[|1,N |],qi ,0

Donc cette fois-ci :


HN (p(θ∗ )) > HN (q).
On obtient cette fois-ci une inégalité stricte. On peut remplacer dans la définition de ΣN (g, ḡ), l’élé-
ment q par n’importe quel élément q0 ∈ ΣN (g, ḡ). L’inégalité vaut donc pour tout q0 ∈ ΣN (g, ḡ) et
HN (p(θ∗ )) est donc bien la valeur maximale de HN sur ΣN (g, ḡ). Remarquons que cette valeur maxi-
male ne peut être atteinte qu’en un n − uplet dont les composantes sont strictement positives.

6
(b) Réciproquement, supposons HN atteint son maximum en un point p de ΣN (g, ḡ) Remarquons d’abord
que toutes les composantes de p (où HN atteint son maximum) sont strictement positives puisque
l’on a vu à la question précédente que c’est une condition nécessaire pour que HN ait un maximum
en p.
Remarquons ensuite que pour tout p de ΣN , on a :

p ∈ ΣN (g, ḡ) ⇐⇒ M T p = qM T ,

p ∈ ΣN (g, ḡ) ⇐⇒ M T (p − q) = 0 ⇐⇒ (p − q) ∈ ker(M T ).


En considérant la somme des composantes de p − q, qui est nulle, on voit que les éléments ΣN (g, ḡ)
sont les éléments sont les éléments q + x, où x ∈ ker M̃ T , qui ont leurs composantes positives ou
nulles.
On en déduit aisément que ΣN (g, ḡ) est convexe.

Si HN admet un maximum en deux points p et p0 distincts de ΣN (g, ḡ), alors, pour tout t ∈ [0, 1]tp +
(1 − t)p0 ∈ ΣN (g, ḡ). Comme p et p0 sont à composantes non nulles, cette cette propriété est même
encore vraie sur un certain intervalle [−, 1 + ], avec  > 0 (On choisit  pour que toutes les compo-
santes des N -uplets restent strictement positives.) Donc w : t 7→ HN (tp + (1 − t)p0 ) est définie et de
classe C 1 sur [−, 1 + ] et
N
X
∀t ∈ [0, 1] w0 (t) = − (pi − pi0 ) ln(tpi + (1 − t)pi0 ).
i=1

w admet un extrêmum en 0 et 1, donc w0 (0) = w0 (1) = 0. Mais alors :


N
X
w0 (0) − w0 (1) = − (pi − pi0 )(ln(pi ) − ln(pi0 )) = 0
i=1

Or tous les termes de cette somme sont négatifs ou nuls : ils sont donc tous nuls et donc

∀i ∈ [|1, N |]pi = pi0 .

On a donc bien montré l’unicité demandée.

7
ÉCOLE POLYTECHNIQUE - ÉCOLES NORMALES SUPERIEURES

CONCOURS D’ADMISSION 2016 FILIÈRE MP

COMPOSITION DE MATHÉMATIQUES - C - (ULRC)

(Durée : 4 heures)

L’utilisation des calculatrices n’est pas autorisée pour cette épreuve

???

On note

• RN [X] l’ensemble des polynômes à coefficients réels, de degré 6 N,


n
X
• h., .i le produit scalaire euclidien sur Rn : hx, yi = xj yj pour tout x = (x1 , ..., xn ) , y = (y1 , ..., yn ) ∈ Rn ,
j=1

• Mn (R) l’algèbre des matrices carrées n × n à coefficients réels,


• In son élément unité,
+∞
X Ak
• exp l’exponentielle sur Mn (R) : exp (A) = , ∀A ∈ Mn (R) ,
k!
k=0

• f (k) la dérivée k-ième de la fonction f, pour k > 1, lorsque I est un intervalle ouvert de R et f : I → R
est k-fois dérivable sur I; par convention f (0) = f,
• C m ([0, R] , R), pour m > 1, l’ensemble des fonctions continues sur l’intervalle fermé [0, T ] , de classe C m
sur l’intervalle ouvert ]0, T [ , admettant des dérivées à droite en 0, et à gauche en T, jusqu’à l’ordre m et
telles que f (k) soit continue sur [0, T ] pour k = 1, ..., m,
\
• C ∞ ([0, T ] , R) = C m ([0, T ] , R) ,
m∈N
   
n n n!
• les coefficients binômiaux : = , ∀n ∈ N∗ , k ∈ {0, ..., n} .
k k k! (n − k)!

Les relations entre les 5 parties sont :


1⇒2 3 ⇒ 4 ⇒ 5.
Ainsi, la partie 1 est utile pour résoudre la partie 2 mais les parties (3, 4, 5) sont indépendantes des parties
(1, 2) , etc.

1 Équation différentielle scalaire


Dans cette partie, on fixe n ∈ N∗ , T > 0, a0 , ..., an−1 , c0 , ..., cn−1 ∈ R. Le but de cette partie est de montrer le
résultat suivant.

Proposition 1 Il existe u ∈ C 0 ([0, T ] , R) tel que la solution f du système


( (n)
f (t) + an−1 f (n−1) (t) + · · · + a0 f (t) = u (t) , ∀t ∈ [0, T ] ,
(Σ) :
f (k) (0) = ck pour k = 0, ..., n − 1,

vérifie f (k) (T ) = 0 pour k = 0, ..., n − 1.

1. Justifier, pour tout u ∈ C 0 ([0, T ] , R) , l’existence et l’unicité de f ∈ C n ([0, T ] , R) vérifiant (Σ) .

1
2. Montrer que l’application suivante est un isomorphisme

L : R2n−1 [X] → R2n


 .
P 7→ P (0) , ..., P (n−1) (0) , P (T ) , ..., P (n−1) (T )

3. Montrer qu’il existe f ∈ C ∞ (R, R) telle que f (k) (0) = ck et f (k) (T ) = 0 pour k = 0, ..., n − 1.
4. Montrer la Proposition 1.
5. La fonction u évoquée dans la Proposition 1 est-elle unique ?

2 Système différentiel
Dans cette partie, on fixe n ∈ N∗ , T > 0, A ∈ Mn (R) et b ∈ Rn . Le but de cette partie est de montrer
l’équivalence entre les énoncés suivants.


• (E1 ) : b, Ab, ..., An−1 b est une base de Rn .

• (E2 ) : Pour tout y 0 ∈ Rn , il existe u ∈ C 0 ([0, T ] , R) tel que la solution de

 dy (t) = Ay (t) + u (t) b, ∀t ∈ [0, T ]



dt (2, 1)
y (0) = y 0

vérifie y (T ) = 0.

1. Justifier, pour tout u ∈ C 0 ([0, T ] , R) et tout y 0 ∈ Rn , l’existence et l’unicité de y ∈ C 1 ([0, T ] , R) vérifiant


(2.1) .
2. Exprimer y (T ) en fonction de A, b, u et y 0 . En déduire une reformulation de l’égalité y (T ) = 0 de la
forme y 0 = Φ (A, b, u) .
3. Montrer que, pour tout k > n, il existe un polynôme Pk ∈ Rn−1 [X] tel que Ak = Pk (A) .

4. Le but de cette question est de démontrer (E2 ) ⇒ (E1 ) . On suppose que b, Ab, ..., An−1 b n’est pas une
base de Rn .

(a) Justifier l’existence de z ∈ Rn \ {0} tel que z, Ak b = 0 pour tout k ∈ N.


(b) Que dire de hz, exp (At) bi pour t ∈ R ?
(c) Soit y 0 ∈ Rn tel que z, y 0 6= 0. Montrer qu’il n’existe pas de fonction u ∈ C 0 ([0, T ] , R) pour
laquelle la solution de (2.1) vérifie y (T ) = 0.
(d) Conclure.

Juqu’à la fin de la partie 2, notre but est de démontrer (E1 ) ⇒ (E2 ) . On suppose donc que (E1 ) est
vérifié. On note a0 , ..., an−1 les coefficients du polynôme caractéristique de A :

det (XIn − A) = X n + an−1 X n−1 + · · · + a1 X + a0

et on définit une famille (v1 , ..., vn ) de Rn par récurrence descendante, de la façon suivante
(
vn := b
vk := Avk+1 + ak vn pour k = n − 1, n − 2, ..., 1.

5. Exprimer vk en fonction de A et b pour k = 1, ..., n.


6. Montrer que Aj b ∈ Vect {v1 , ..., vn } pour j = 0, ..., n − 1. En déduire que (v1 , ..., vn ) est une base de Rn .
7. Montrer que Av1 = −a0 vn .

2
8. En déduire l’existence de U ∈ GLn (R) telle que
 
0 1 0 ··· 0  
 .. .. .. ..  0
 . 0 . . .   .. 
e := U −1 AU =  . . .
 et U −1 b = 
 
A .. ..

 .. . . 0 0
 
 
 0 ··· ··· 0 1 
1
−a0 −a1 ··· ··· −an−1

9. Soit y 0 ∈ Rn , u ∈ C 0 ([0, T ] , R) et y la solution de (2.1) .

(a) Quel problème de Cauchy la fonction

F : [0, T ] → Rn
t 7→ U −1 y (t)

résout-elle ?
(b) Notons f (t) la première composante de F (t) . Quel problème de Cauchy la fonction f résout-elle ?

10. Conclure.

3 Classe de Gevrey : résultats généraux


Dans cette partie, on fixe s ∈ [1, +∞[ et T > 0.
Définition : Une fonction f : [0, T ] → R est dans la classe de Gevrey d’ordre s sur [0, T ] si f ∈
C ∞ ([0, T ] , R) et s’il existe M, R > 0 tels que
s
M (n!)
f (n) (t) 6 , ∀n ∈ N, t ∈ [0, T ] .
Rn
On note alors f ∈ G s (0, T ) .

1. Montrer que, si f ∈ G s (0, T ) alors la fonction

g: [0, T ] → Rn
t 7→ f (T − t)

est dans la classe de Gevrey d’ordre s sur [0, T ] .


2. Montrer que G s (0, T ) contient les fonctions polynomiales.
3. Montrer que G s (0, T ) est un espace vectoriel.
4. Montrer que si f1 , f2 ∈ G s (0, T ) alors leur produit f1 f2 est dans la classe de Gevrey d’ordre s sur [0, T ] .

5. Soit f ∈ G s (0, T ) et M, R des constantes associées comme dans la définition ci-dessus. On suppose qu’il
existe δ > 0 tel que f (t) > δ pour tout t ∈ [0, T ] .
 (n) n    (n−k)
1 1 X n (k) 1
(a) Montrer que, pour tout n ∈ N∗ , =− f .
f f k f
k=1
(b) Montrer qu’il existe ε > 0 tel que, pour tout n ∈ N∗
n  1−s
X n δ
εk 6 .
k M
k=1

1 1
(c) Montrer que ∈ G s (0, T ) avec, par exemple, les constantes M 0 = et R0 = εR.
f δ

3
4 Classe de Gevrey : exemples
On fixe T > 0. Le but de cette partie est de montrer que les fonctions h et φ : [0, T ] → R définies par


 1 si t = 0,

1


 
 0 si t = 0
 2
e (T −t)


h (t) = φ (t) = 1 si t ∈ ]0, T [ ,
 e− t12 si t ∈ ]0, T ] − 1
(T −t)2 + e− t2
e







0 si t = T.

3
sont de classe de Gevrey d’ordre sur [0, T ] .
2
1. Montrer que h(k) (t) → 0 pour tout k ∈ N.
t→0+

2. Montrer que h ∈ C ∞ ([0, T ] , R) .


X
3. Soit (an )n∈N une suite de nombres réels tels que la série entière an z n ait un rayon de convergence
+∞
X
ρ > 0. Pour z ∈ C tel que |z| < ρ, on note F (z) = an z n . Montrer que, pour tout r ∈ ]0, ρ[ , on a
n=0

Z2π 
1 F reiθ
an = n dθ.
2π (reiθ )
0

t0
4. Dans cette question, on fixe t0 ∈ ]0, T ] et r := .
3
(a) Montrer que, pour tout z ∈ C\ {t0 } , on a
1 +∞ n

(t0 −z)2
X (−1) 1
e = 2n .
n!t2n

n=0 0 z
1−
t0

1 +∞
− X
(b) En déduire qu’il existe une suite (an )n∈N de nombres réels tels que e (t0 −z)2 = an z n pour tout
n=0
z ∈ C vérifiant |z| < t0 .
(c) Montrer que
1
Z2π −
(t0 −reiθ )2
h(n) (t0 ) 1 e
= n dθ .
n! 2π (reiθ )
0

(d) Montrer que, pour un certain λ > 0 indépendant de t0 , on a


λ
e− r 2
(n)
h (t0 ) 6 n! n .
r
 α
α
(e) Montrer que, pour tout α, β, x > 0 alors xα e−βx 6 .

(f) Montrer que nn 6 en−1 n! pour tout n ∈ N∗ .
3
(g) En déduire que h ∈ G 2 (0, T ) .
3
5. Montrer que φ ∈ G 2 (0, T ) .
3
6. Soit P ∈ R [X] . Montrer que la fonction t 7→ P (t) φ (t) est dans la classe de Gevrey d’ordre sur [0, T ] .
2
(n) (n)
7. Calculer φ(n) (0) , φ(n) (T ) , (P φ) (0) et (P φ) (T ) pour tout n ∈ N.

4
5 Équation de la chaleur
Dans cette partie, on fixe s ∈ [1, 2[ , T > 0 et une fonction f ∈ G s (0, T ) .

1. Montrer que
+∞
X x2n
H (t, x) = f (n) (t)
n=0
(2n)!

est bien défini, pour tout (t, x) ∈ [0, T ] × [0, 1] .


2. Montrer que toutes les dérivées partielles de H sont définies et continues sur [0, T ] × [0, 1] .
∂H ∂2H ∂H
3. Montrer que − = 0 sur [0, T ] × [0, 1] et que (t, 0) = 0 pour tout t ∈ [0, T ] .
∂t ∂x2 ∂x
4. En déduire que, pour tout polynôme pair H 0 , il existe une fonction u ∈ C 0 ([0, T ] , R) et une solution H
du système
∂2H

∂H

 (t, x) − (t, x) = 0, ∀ (t, x) ∈ [0, T ] × [0, 1] ,
∂t ∂x2




∂H


∀t ∈ [0, T ] ,


 (t, 0) = 0,
∂x
 ∂H
∀t ∈ [0, T ] ,


 (t, 1) = u (t) ,



 ∂x
H (0, x) = H 0 (x) , ∀x ∈ [0, 1]


qui satisfasse H (T, x) = 0 pour tout x ∈ [0, 1] .

5
CORRIGÉ EPREUVE C ENS MP 2016 (ULRC)

1 Équation différentielle scalaire


1. Cette équation différentielle est linéaire scalaire d’ordre n avec les fonctions u et (t 7→ ak )06k6n−1 qui
sont continues sur l’intervalle [0, T ] et (0, c0 , ..., cn−1 ) ∈ [0, T ] × Rn . L’application du théorème de Cauchy-
Lipschitz prouve l’existence et l’unicité de la fonction f qui est, par définition, n fois dérivable sur [0, T ]
n−1
X
(n)
et comme f =u− ak f (k) ∈ C 0 ([0, T ] , R) , on peut affirmer que f ∈ C n ([0, T ] , R) .
k=0

2. L est clairement linéaire et comme dim (R2n−1 [X]) = 2n = dim R2n , il suffit de montrer que L est
injective, i.e. ker (L) = {0}, pour démontrer qu’il s’agit d’un isomorphisme. Soit P ∈ ker (L) alors
∀k ∈ {0, ..., n − 1} , P (k) (0) = P (k) (T ) = 0 donc 0 et T sont racines d’ordre au moins n de P. Puisque
0 6= T, on en déduit que P admet au moins n + n = 2n racines (en comptant la multiplicité) et comme
deg (P ) < 2n, on peut affirmer que P = 0 d’où ker (L) = {0} .
3. Soit P0 = L−1 (c0 , ..., cn−1 , 0, ..., 0) ∈ R2n−1 [X] et vérifie ∀k ∈ {0, ..., n − 1} , P (k) (0) = ck et P (k) (T ) = 0.
Sa fonction associée f est clairement C ∞ sur R et vérifie les conditions demandées.
n−1
X
4. Soit f la fonction obtenue à la question précédente et u = f (n) + ak f (k) ∈ C 0 ([0, T ] , R) alors f est
k=0
solution de (Σ) et vérifie f (k) (T ) = 0 pour k = 0, ..., n − 1.
L1 : C ∞ ([0, T ] , R) → R2n
5. On pose  qui est linéaire et sur-
f 7→ f (0) , ..., f (n−1) (0) , f (T ) , ..., f (n−1) (T )
jective (d’après la question 3 en identifiant les polynômes de R [X] à leur fonction polynomiale as-
sociée sur R). Si f ∈ C ∞ ([0, T ] , R) alors P = L−1 (L1 (f )) ∈ R2n−1 [X] et f − P ∈ ker (L1 ) , on en
déduit aisément que C ∞ ([0, T ] , R) = R2n−1 [X] ⊕ ker (L1 ) donc ker (L1 ) est de dimension infinie (car
H : ker (L1 ) → C 0 ([0, T ] , R)
C ∞ ([0, T ] , R) l’est mais pas R2n−1 [X]). L’application linéaire n−1
X
f 7→ f (n) + ak f (k)
k=0
est non identiquement nulle (son noyau est un sous-espace vectoriel des solutions de l’équation différentielle
n−1
X
y (n) + ak y (k) = 0 qui est de dimension n (via Cauchy-Lipschitz). Ainsi, son noyau est aussi un espace
k=0
vectoriel de dimension finie qui ne peut être égal à ker (L1 ) car celui-ci est de dimension infinie). L’espace
vectoriel Im (H) est non nul donc infini (et même de dimension infinie). Soit f0 la fonction obtenue à la
question 1.4 alors, pour toute fonction g ∈ ker (L1 ) , la fonction fg = f0 + g vérifie :
(k) (k)
∀k ∈ {0, ..., n − 1} , (fg ) = ck , (fg ) (T ) = 0

et la fonction u = H (f0 + g) = H (f0 ) + H (g) convient pour la Proposition 1. Comme l’ensemble


{H (f0 ) + H (g) , g ∈ ker (L1 )} = {H (f0 ) + h, h ∈ Im (H)} est infini, on en déduit que la fonction u
n’est pas unique.

2 Système différentiel
1. Soit u ∈ C 0 ([0, T ] , R) et y 0 ∈ Rn . Le problème (2.1) est un problème  de Cauchy linéaire associé aux
fonctions t 7→ A et t 7→ u (t) b qui sont continues sur [0, T ] et 0, y 0 ∈ [0, T ] × Rn donc le théorème de
Cauchy-Lipschitz démontre l’existence et l’unicité de la solution à (2.1) . Par définition, cette solution y
est dérivable sur [0, T ] et y 0 = Ay + ub ∈ C 0 ([0, T ] , Rn ) donc y ∈ C 1 ([0, T ] , Rn ) .

2. Soit y une telle solution, on a :


d −At
= e−At (−A) y (t) + e−At y 0 (t) = e−At (−Ay (t) + y 0 (t))

e y (t)
dt
= e−At u (t) b = u (t) e−At b.
u(t)∈R

1
(on mime la variation de la constante qui reste valable en dimension finie dans le cas des matrices à
coefficients constants). En intégrant cette relation sur [0, T ] , on obtient :
  T 
ZT Z
e−AT y (T ) − e−A0 y (0) = u (t) e−At bdt ⇔ y (T ) = eAT y 0 +  u (t) e−At bdt = Φ (A, b, u) .
0 0

3. Le polynôme caractéristique χA annule A (théorème de Cayley-Hamilton) et est de degré n. Soit k ∈ N,


la division euclidienne de X k par χA fournit l’égalité X k = χA (X) Qk (X) + Pk (X) avec deg (Pk ) <
deg (χA ) = n (i.e. Pk ∈ Rn−1 [X]) donc Ak = χA (A) Qk (A) + Pk (A) = Pk (A) .

4. (a) La famille b, Ab, ..., An−1 b est de cardinal n et n’est pas une base de Rn donc ce n’est pas une
famille génératrice de Rn i.e. F = Vect b, Ab, ..., An−1 b Rn et on a F ⊥ 6= {0} . Soit z ∈ F \ {0}
alors, en utilisant la question 2.3 :

∀k ∈ N, Ak b = Pk (A) b ∈ Vect b, Ab, ..., An−1 b = F ⇒ z, Ak b = 0.




(b) Soit t ∈ R, on a :
* +∞
+ * N
+ * N
+ +∞ k
X Ak t k X Ak t k X Ak t k X t
hz, exp (At) bi = z, b = z, lim b = lim z, b = z, Ak b = 0
k! N →+∞ k! (∗) N →+∞ k! k!
k=0 k=0 k=0 k=0

(∗) : y ∈ Rn 7→ hz, yi est continue (soit utiliser Cauchy-Schwarz, soit linéaire en dimension finie).
(c) On procède par l’absurde en supposant qu’il existe une fonction u ∈ C 0 ([0, T ] , R) pour laquelle la
solution de (2.1) vérifie y (T ) = 0. D’après la question 2.2, on a :

ZT ZT ZT
−At −At
y (T ) = 0 ⇔ y + 0
u (t) e bdt = 0 ⇔ y = − 0
u (t) e bdt ⇒ z, y 0
=− u (t) z, e−At b dt = 0
(∗) | {z }
0 0 0 =0

ce qui est absurde.


(∗) : En effet, soit f : [a, b] → E où (E, h., .i) est un espace euclidien (donc un espace vectoriel normé
Zb
de dimension finie), par définition de l’intégration de fonctions à valeurs vectoriels, on a f (t) dt =
a
n
X Zb
εi fi (t) dt où (εi )16i6n est une base de de E et, pour tout t ∈ [a, b] , (f1 (t) , ..., fn (t)) les
i=1 a
coordonnées de f (t) dans la base (εi )16i6n donc :
* Zb + n Zb Zb X
n Zb * X n
+ Zb
X
v, f (t) dt = hv, εi i fi (t) dt = hv, εi i fi (t) dt = v, εi fi (t) dt = hv, f (t)i dt.
a i=1 a a i=1 a i=1 a


(d) Supposons que (E2 ) est vérifiée. Si b, Ab, ..., An−1 b n’est pas une base de Rn , d’après la question
2.4.a, il existe y 0 ∈ Rn tel que ∀k ∈ N, z, Ak b = 0. D’après la question 2.4.c, il n’existe pas de
fonction u ∈ C 0 ([0, T] , R) pour laquelle la solution de (2.1) vérifie y (T ) = 0 ce qui contredit (E1 )
donc b, Ab, ..., An−1 b est une base de Rn d’où (E1 ) est vérifiée. Par conséquent, on vient d’établir
que (E2 ) ⇒ (E1 ) .
5. En itérant la relation proposée, on obtient

vn−1 = Avn + an−1 vn = Ab + an−1 b,


vn−2 = Avn−1 + an−2 vn = A2 b + an−1 Ab + an−2 b,
vn−3 = Avn−2 + an−3 vn = A3 b + an−1 A2 b + an−2 Ab + an−3 b.

On conjecture rapidement que

(Hk ) : vn−k = Ak b + an−1 Ak−1 b + an−2 Ak−2 b + · · · + an−k b.

2
L’initialisation k = 1 est immédiate et pour l’hérédité, supposons (Hk ) vraie pour un certain k ∈
{1, ..., n − 2} alors on a k + 1 6 n − 1 donc :

vn−(k+1) = vn−k−1 = Avn−k + an−k−1 vn = A Ak b + an−1 Ak−1 b + an−2 Ak−2 b + · · · + an−k b + an−k−1 b


= Ak+1 b + an−1 Ak b + an−2 Ak−1 b + · · · + an−k−1 b

ce qui démontre (Hk+1 ) et achève la récurrence. Par conséquent, (Hk ) est vraie pour k ∈ {1, ..., n − 1}
donc, on posant j = n − k, on a :

∀j ∈ {1, ..., n − 1} , vj = An−j b + an−1 An−j−1 b + an−2 An−j−2 b + · · · + aj b et vn = b.

6. On pose F = Vect {v1 , ..., vn } . On procède par récurrence forte en posant, pour tout j ∈ {0, ..., n − 1} ,
(Hj ) : Aj b ∈ F. Il est immédiat que (H0 ) est vraie. Soit j ∈ {1, ..., n − 2} . Supposons que (Hk ) est vraie
pour tout k ∈ {0, ..., j} alors, d’après la question précédente, on a :
j
X
Aj+1 b = vn−(j+1) − an−j−1+i Ai b
|{z} ∈F
i=0
| {z }
∈F car 06i6j6n−1
∈F car 16n−(j+1)6n

ce qui démontre (Hj+1 ) et achève la récurrence. On en déduit que :

Rn = Vect b, Ab, ..., An−1 b ⊂ Vect {v1 , ..., vn } ⊂ Rn ⇒ Vect {v1 , ..., vn } = Rn .

(E1 )

Puisque la famille (v1 , ..., vn ) est de cardinal n = dim (Rn ) et qu’elle est génératrice, on en déduit que
c’est une base de Rn .
7. D’après la question 2.5, on a :

v1 = An−1 b + an−1 An−2 b + an−2 An−3 b + · · · + a1 b


Cayley-
Av1 = An b + an−1 An−1 b + an−2 An−2 b + · · · + a1 Ab = (χA (A) − a0 Id) b = −a0 b = −a0 vn .
Hamilton

8. Soit l’application linéaire ϕ : x ∈ Rn 7→ Ax ∈ Rn . Puisque l’on a :

ϕ (v1 ) = −a0 vn , ∀k ∈ {2, ..., n} , ϕ (vk ) = Avk = vk−1 − ak−1 vn ,

sa matrice A
e dans la base (v1 , v2 , ..., vn ) est :

ϕ (v1 ) ϕ (v2 ) ϕ (v3 ) ϕ (vn )



0 1 0 ··· 0
 v1
 .. .. .. ..  v2
 .
 0 . . .  
e=  .
A .. .. .. .
 ..

 . . 0   .
 0 ··· ··· 0 1  vn−1
−a0 −a1 ··· ··· −an−1 vn

La matrice de ϕ dans la base canonique est .... A (ϕ (ei ) = AEi,i = Ci la i-ième colonne  de  A) et la
0
 .. 
matrice des coordonnées de b = vn = 0v1 + · · · + 0vn−1 + 1vn dans la base (v1 , ..., vn ) est eb =  .  . Si on
 
0
1
note U la matrice de passage de la base canonique de Rn à (v1 , ..., vn ) qui est une matrice inversible alors,
d’après les formules de changement de base, on a A e = U −1 AU et U −1 b = eb.

9. (a) L’application c ∈ Rn 7→ U −1 c étant linéaire, on a pour tout t ∈ [0, T ] :


u(t)∈R
F 0 (t) = U −1 y 0 (t) = U −1 (Ay (t) + u (t) b) = U −1 Ay (t) + u (t) U −1 b
   
0 0
 ..   .. 
e −1 y (t) + u (t) 
= AU  .  = AF
e (t) +  .  et F (0) = U −1 y 0 .
 
0  0 
1 u (t)

3
 
f1 (t)
 f2 (t) 
(b) On pose F (t) =  .  (avec f1 = f ) alors la question précédente montre que :
 
 .. 
fn (t)

f10 (t) = f2 (t) f2 (t) = f10 (t)


 
 
f20 (t) = f3 (t) f3 (t) = f100 (t)

 


 

..
 

 .. 


.
 .
∀t ∈ [0, T ] , 0 ⇔ (n−1)

 fn−1 (t) = fn (t) 
 fn (t) = f1 (t)

 n
X 
 n
0 (n)
  X
 fn (t) = − ak−1 fk (t) + u (t)  f1 (t) = − ak−1 f (k−1) (t) + u (t)

 

 
k=1 k=1

n−1
X
donc f vérifie l’équation différentielle y (n) (t) + ak y (k) (t) = u (t) . En outre, on a F (0) = U −1 y 0
  k=0
c0
 c1 
donc, si on pose U −1 y 0 =  .  , on a :
 
 .. 
cn−1
 
 f1 (0) = c0
  f (0) = c0
f 0 (0) = c1

 f2 (0) = c1
 

.. ⇔ ..


 . 

 .
 (n−1)
fn (0) = cn−1 f (0) = cn−1

ce qui prouve que f vérifie le problème de Cauchy :


 n−1

 (n) X
y (t) + ak y (k) (t) = u (t) , ∀t ∈ [0, T ]
.
 k=0
 (k)
y (0) = ck , ∀k ∈ {0, ..., n − 1}
 
c0
  c1
10. Il existe U ∈ GLn (R) telle que A e = U −1 AU. Soit y 0 ∈ Rn , il existe (ck ) −1 0
06k6n−1 tel que U y = .
 
..
  .
cn−1
D’après la Proposition 1, il existe u ∈ C 0 ([0, T ] , R) tel que la solution f du système
( (n)
f (t) + an−1 f (n−1) (t) + · · · + a0 f (t) = u (t) , ∀t ∈ [0, T ] ,
(Σ) :
f (k) (0) = ck pour k = 0, ..., n − 1,
 
f (t)
 f 0 (t)

vérifie f (k) (T ) = 0 pour k = 0, ..., n − 1. On pose alors z : t ∈ [0, T ] 7→   ∈ Rn qui est
..
 
 .
(n−1)
f (t)
clairement dérivable sur [0, T ] et vérifie, pour tout t ∈ [0, T ] :
   
0 0
 ..   .. 
0
z 0 (t) = Az  .  ⇔ (U z (t)) = A (U z (t)) + u (t) U  .  ⇔ y 0 (t) = Ay (t) + u (t) b
e (t) +    
 0  0
u (t) 1
   
c0 f (T )
 c1
  f 0 (T )

si l’on pose y = U z. En outre, on a z (0) =   = U −1 y 0 donc y (0) = y 0 et y (T ) =  =
..
   
..
 .  .
cn−1 f (n−1) (T )
0Rn ⇒ z (T ) = 0Rn donc (E2 ) est vérifiée, ce qui prouve l’implication (E1 ) ⇒ (E2 ) .

4
3 Classe de Gevrey : résultats généraux
1. Il est immédiat que g ∈ C ∞ ([0, T ] , R) et :
s
n M (n!)
∀n ∈ N, ∀t ∈ [0, T ] , g (n) (t) = (−1) f (n) (T − t) = f (n) (T − t) 6
Rn
donc g ∈ G s (0, T ).
2. Soit f ∈ R [X] alors il existe N ∈ N tel que ∀n > N, f (n) = 0. Pour chaque k ∈ {0, ..., N } , f (k) est con- !
1 (k)
tinue sur le segment [0, T ] donc elle y est bornée, ce qui assure l’existence de M = max s sup f (t) .
06k6N (k!) t∈[0,T ]
Il est alors immédiat que :
s
f (n) (t) s M (n!)
∀t ∈ [0, T ] , ∀n ∈ N, 6 M ⇔ f (n) (t) 6 M (n!) =
n! 1n
ce qui assure que f ∈ G s (0, T ) .
3. G s (0, T ) ⊂ C ∞ ([0, T ] , R) qui est un R-espace vectoriel. 0C ∞ ([0,T ],R) ∈ G s (0, T ) (car c’est une fonction
polynomiale par exemple). Soient f, g ∈ G s (0, T ) et λ, µ ∈ R, il existe M, R, M 0 , R0 ∈ R∗+ tels que :
s s
M (n!) M 0 (n!)
∀t ∈ [0, T ] , ∀n ∈ N, f (n) (t) 6 , g (n) (t) 6 n ⇒
Rn (R0 )
(n)
(λf + µg) (t) = λf (n) (t) + µg (n) (t) 6 |λ| f (n) (t) + |µ| g (n) (t)
|µ| M 0 |λ| M + |µ| M 0
   
s |λ| M s
6 (n!) + s 6 (n!) n
Rn (R0 ) min (R, R0 )

donc λf + µg ∈ G s (0, T ) ce qui prouve que G s (0, T ) est un R-espace vectoriel.


4. Soit f1 , f2 ∈ G s (0, T ) alors f1 , f2 appartiennent à C ∞ ([0, T ] , R) donc f1 f2 aussi. il existe M1 , R1 , M2 , R2 ∈
R∗+ tels que :
s s
M1 (n!)
(n) M2 (n!)
∀t ∈ [0, T ] , ∀n ∈ N, f1 n , (t) 6g (n) (t) 6 n ⇒
(R1 ) (R2 )
n   n   n   s s
(n)
X n (k) (n−k)
X n (k) (n−k)
X n M1 (k!) M2 ((n − k)!)
(f1 f2 ) (t) = f1 (t) f2 (t) 6 f1 (t) f2 (t) 6 ×
k k k (R1 )k (R2 )
n−k
k=0 k=0 k=0
n     s n  
s
X n k! (n − k)! 1 s
X n 1 1
= M1 M2 (n!) k n−k
= M 1 M 2 (n!)  s
k=0
k n! (R1 ) (R2 ) k=0
k n (R1 ) (R2 )n−k
k

k
| {z }
>1
n    n
s
X n 1 s 1 1
6 M1 M2 (n!) = M1 M2 (n!) +
k (R1 ) (R2 )n−k
k R1 R2
k=0

ce qui permet de conclure à f1 f2 ∈ G s (0, T ) .


1
5. (a) Puisque f × = 1 alors, pour n > 1, la formule de Leibniz montre que :
f
 n X n    (n−k)    n X n    (n−k)
1 n (k) 1 n 0 1 n (k) 1
0 = 1(n) = f× = f = f + f .
f k f 0 f k f
k=0 | {z } k=1
=f

On conclut en isolant f (n) et en divisant par f.


   1−s
n n
(b) Puisque 1 − s 6 0 et que > 1, on a 6 1 donc, pour x ∈ ]0, 1[ , on a :
k k
n  1−s n +∞
X n X X x
xk 6 xk 6 xk = → 0.
k 1 − x x→0
k=1 k=1 k=1

5
n  1−s
δ X n δ
Puisque > 0, il existe α > 0 tel que ∀x ∈ [0, α] , xk 6 ce qui permet de conclure
M k M
k=1
(ε = α convient).
s
(n!)
(c) Prouvons par récurrence sur n ∈ N la propriété (Hn ) : ∀t ∈ [0, T ] , ∀n ∈ N, f (n) (t) 6 n .
δ (εR)
Pour n = 0, on a sur [0, T ] ,
s
1 1 1 (0!)
f >δ>0⇒ = 6 = 0
f f δ δ (εR)

ce qui prouve (H0 ) . Soit n > 1 et supposons (Hk ) vraie pour tout entier k < n alors, pour tout
t ∈ [0, T ] , on a :
 (n) n    (n−k) n    (n−k)
1 1 X n (k) 1 1 X n (k) 1
(t) = − f = f
f 3.5.a) f k f f k f
k=1 k=1
n     (n−k) d’après (H n   s s
1X n 1 n−k ) 1 X n M (k!) ((n − k)!)
6 f (k) 6 × n−k
δ k f car n−k<n δ k Rk δ (εR)
k=1 k=1
s n   s s n  1−s s s
M (n!) X n k! (n − k)! k M (n!) X n k M (n!) δ (n!)
= 2 n ε = n ε 6 n × = n
δ (εR) k n! δ 2 (εR) k 2
3.5.b) δ (εR) M δ (εR)
k=1 k=1

1
ce qui démontre (Hn ) et permet de conclure d’où ∈ G s (0, T ) .
f

4 Classe de Gevrey : exemples


1
1. La fonction h est de classe C ∞ sur ]0, T ] (car t 7→ − 2 l’est et exp est C ∞ sur R). Prouvons par récurrence
t
sur k ∈ N la propriété :
 
1
(Hk ) : ∃Pk ∈ R [X] , ∀t ∈ ]0, T ] , h(k) (t) = Pk h (t) .
t

Pour k = 0, le polynôme P0 = 1 convient. Supposons la propriété vraie pour un entier k alors, pour tout
t ∈ ]0, T ] , on a :
0   1  0       
(k+1)

(k) 1 0 1 1 2 0 1
h (t) = h (t) = Pk h (t) = − 2 Pk h (t) + Pk h (t) = Pk+1 h (t)
t t t t t3 t

si l’on choisit Pk+1 (X) = −X 2 Pk0 (X) + 2X 3 Pk (X) ∈ R [X] ce qui démontre (Hk+1 ) et achève la
récurrence. On en déduit que :
√  −x √ nk −x
 
(k) 1 − 12 x=1/t2
h (t) = Pk e t = Pk x e ∼ ank x e → 0
t t→0+ ⇔x→+∞ x→+∞ x→+∞

par les croissances comparées. (∗) : nk désigne le degré de Pk et ak son coefficient dominant
2. On a établi à la question précédente que h ∈ C ∞ (]0, T ] , R) et ∀k ∈ N, lim h(k) = 0 donc, d’après
0+
le théorème de prolongement continu de la dérivée (ou limite de la dérivée), on peut affirmer que h ∈
C ∞ ([0, T ] , R) .
3. Soit r ∈ ]0, ρ[ , on a :
 +∞ +∞
F reiθ 1 X
iθ k
X k−n
ak reiθ

∀θ ∈ [0, 2π] , iθ n = iθ n ak re = .
(re ) (re ) k=0 k=0

n−k
Pour tout k ∈ N, posons Fk : θ 7→ ak reiθ qui est continue sur [0, 2π]. En outre, on a :
X X 1 X
sup |Fk (θ)| = |ak | rk−n = |ak | rk .
rn
k>0 θ∈[0,2π] k>0 k>0

6
X X
Puisque la série ak z k a pour rayon de convergence ρ et que r < ρ, on peut affirmer que la série |ak | rk
k k>0
converge (convergence absolue à l’intérieur du disque ouvert de convergence). Par conséquent, la série de
X
fonctions Fk converge normalement, donc uniformément, sur [0, 2π], ce qui permet de permuter les
k>0
symboles série et intégrale :
Z2πX
+∞ +∞ Z 2π Z2π  +∞ Z2π
X F reiθ X
k−n
Fk = Fk ⇔ n dθ = ak r ei(k−n)θ dθ.
(reiθ )
0 k=0 k=0 0 0 k=0 0

Z2π Z2π Z2π θ=2π


eimθ

imθ
Pour m = 0, on a e dθ = 1dθ = 2π et si m 6= 0, on a : eimθ dθ = = 0 (car θ 7→ eimθ
im θ=0
0 0 0
est 2π-périodique), ce qui nous fournit l’égalité :
Z2π 
F reiθ n−n
n dθ = an r 2π = 2πan
(reiθ )
0

+∞
X wn
4. (a) Pour tout w ∈ C, on a ew = donc, pour tout z ∈ C\ {t0 } , on a :
n=0
n!

+∞
!n +∞ +∞ +∞
1 n n n

(t0 −z)2
X 1 1 X (−1) X (−1) X (−1) 1
e = − 2 = 2n = 2n = 2n 2n .
n! (t0 − z) n! (t0 − z) n!t0
  
n=0 n=0 n=0 z n=0 z
n! t0 1 − 1−
t0 t0
+∞
1 X 1
(b) Pour tout w ∈ C avec |w| < 1, on a : = wk . Fixons w ∈ C∗ alors la fonction t 7→
1−w 1 − tw
k=0
1
est développable en série entière avec un rayon de convergence R = car :
|w|
+∞
1 X
∀t ∈ ]−R, R[ , = tk w k
1 − tw
k=0

En dérivant terme à terme n fois cette série entière (ce qui est licite à l’intérieur du disque ouvert de
convergence), on obtient :
+∞
n!wn X
(D) : ∀t ∈ ]−R, R[ , n+1 = k (k − 1) · · · (k − n + 1) tk−n wk
(1 − tw) k=n

Si |w| < 1 alors, en choisissant t = 1 dans la formule ci-dessus, en divisant pas wn et en effectuant le
changement d’indice i = k − n, on en déduit la formule :
+∞
1 1 X
∀w ∈ C avec |w| < 1, n+1 = (k + n) (k + n − 1) · · · (k + 1) wk .
(1 − w) n!
k=0

En utilisant la question précédente, on a pour tout z ∈ C vérifiant |z| < t0 :


1 +∞ n +∞ n +∞  k

(t0 −z)2
X (−1) 1 X (−1) 1 X z
e = 2n  2n = 2n (2n − 1)! (k + 2n − 1) (k + 2n − 2) · · · (k + 1)
n=0
n!t 0 z n=0
n!t0 t0
1− k=0
t0
+∞ +∞
X X (−1) (k + 2n − 1) (k + 2n − 2) · · · (k + 1)  z k
n
= 2n
n=0 k=0 n! (2n − 1)! (t0 ) t0

Prouvons que la famille de complexes


n  k !
(−1) (k + 2n − 1) (k + 2n − 2) · · · (k + 1) z
(an,k )(n,k)∈N2 = 2n
n! (2n − 1)! (t0 ) t0
(n,k)∈N2

7
est sommable en utilisant le théorème de Fubini. Soit n ∈ N, la série
k
X 1 X z
|an,k | = 2n (k + 2n − 1) (k + 2n − 2) · · · (k + 1)
n! (2n − 1)! |t0 | t0
k>0 k>0

z
converge (car < 1 d’après (D)). Posons :
t0
+∞ +∞ k
X 1 X z
Sn = |an,k | = 2n (k + 2n − 1) (k + 2n − 2) · · · (k + 1)
n! (2n − 1)! |t0 | t0
k=0 k=0
1 1 1
= 2n × 2n (d’après (D) ) =  n .
n! |t0 | z n! (|t0 | − |z|)
2
1−
t0
X 1
Puisque la série n converge (série exponentielle de rayon de convergence infini),
2
n! (|t0 | − |z|)
n>0
X
on en déduit la série Sn converge donc la famille (an,k )(n,k)∈N2 est sommable lorsque |z| < t0 .
n>0
D’après le théorème de Fubini, on a pour tout z ∈ C vérifiant |z| < t0 :

1 +∞ X
+∞ +∞ X
+∞ +∞ +∞ n

(t0 −z)2
X X X X (−1) (k + 2n − 1) (k + 2n − 2) · · · (k + 1)
e = an,k = an,k = zk 2n+k
.
n=0 k=0 k=0 n=0 k=0 n=0 n! (2n − 1)! (t0 )
| {z }
=ak

+∞
X
(c) D’après la question précédente, la fonction g : z 7→ h (t0 − z) = ak z k étant développable en série
k=0
entière avec un rayon de convergence R > t0 , on a :
n
g (n) (0) (−1) h(n) (t0 )
∀n ∈ N, = an ⇔ = an ⇒
n! n!
1
Z2π  Z2π − (t −re 2
h(n) (t0 ) 1 g reiθ 1 e 0 iθ )
= |an | = n dθ = n dθ
n! 4.3 2π (reiθ ) 2π (reiθ )
0 0

t0
(d) D’après la question précédente et en tenant compte que r = ⇔ t0 = 3r, on a pour tout entier n :
3
Z2π 1 Z2π !
h(n) (t0 ) 1 1 −
(3r−reiθ )2
1 1
6 n e dθ = exp − 2 dθ
n! 2π (reiθ ) 2πrn r2 (3 − eiθ )
0 0
Z2π !! Z2π !!
1 1 1 1 1
= exp Re − 2 dθ = exp − 2 Re 2 dθ
2πrn r2 (3 − eiθ ) 2πrn r (3 − eiθ )
0 0

La fonction
   
!
2
1  (3 − eiθ ) 9 − + 6eiθ e2iθ  9 − 6 cos (θ) + cos (2θ)
ψ : θ 7→ Re = Re  2  = Re  =

2 2 2
(3 − eiθ ) (3 − eiθ )
2
(3 − eiθ )
2
(3 − eiθ )
2

étant continue sur le segment [0, 2π], elle y atteint ses bornes et on pose λ = min ψ. En outre, puisque
[0,2π]
|−6 cos (θ) + cos (2θ)| 6 7 < 9, on est assuré que ψ ne s’annule pas donc λ > 0 et on en déduit que :

Z2π
h(n) (t0 )
   
1 λ 1 λ
6 exp − 2 dθ = n exp − 2 .
n! 2πrn r r r
0

8
(e) Une étude rapide de la fonction x 7→ xe−x montre que ∀x ∈ R+ , 0 6 xe−x 6 e−1 donc :
α  α  α  α
 α −t α −1 α
xα e−βx = xe−βx/α 6 te 6 e = .
t=βx/α>0 β β eβ

nn
(f) Posons un = alors, pour tout entier n > 1, on a :
n!
n+1 n+1 n  n
un+1 (n + 1) n! (n + 1) 1 (n + 1) 1
= × n = × n = = 1+
un (n + 1)! n (n + 1) n nn n
    
1 1
= exp n ln 1 + 6 exp n × = e.
n ln concave n
On en déduit que pour tout entier N > 2, on a :
N −1 N −1
uN Y un+1 Y
= 6 e = eN −1 ⇒ uN 6 eN −1 u1 = eN −1 ,
u1 n=1
un n=1

cette formule étant évidemment vraie pour n = 1.


(g) Soit t0 ∈ ]0, T ] , d’après la question 4.4.d), il existe λ (indépendant de t0 ) tel que :
   n/2     n/2
n! λ 1 1 n/2
∀n ∈ N, h(n) (t0 ) 6 exp − = n! exp −λ 6 n!
rn r2 r2 r2 4.4.e eλ
n! √ n! √ e −1/2
(n!)
3/2
= √ n n n 6 √ n en−1 n! = √ n .
2eλ 4.4.f 2eλ 2λ

Cette inégalité restant vraie pour t0 = 0 car , d’après la question 4.1, on a ∀n ∈ N, h(n) (0) = 0
3
d’où h ∈ G2 (0, T ) .
3 3
5. Puisque h ∈ G 2 (0, T ) , la fonction g : t 7→ h (T − t) appartient aussi à G 2 (0, T ) donc g + h appartient
3
à G 2 (0, T ) (car c’est un espace vectoriel). Puisque g + h ne s’annule pas sur [0, T ] (chaque fonction
est positive sur cet intervalle et elles ne s’annulent pas simultanément) et qu’elle est continue sur ce
segment, on est assuré de l’existence de δ = min (g + h) > 0. On a alors g + h > δ sur [0, T ] donc,
[0,T ]
1 3
d’après la question 4.5.c, on a ∈ G 2 (0, T ) . En utilisant la question 4.4, on peut affirmer que
g+h
1 g 3
g× = = φ ∈ G 2 (0, T ) .
g+h g+h
3 3
6. Puisque P ∈ G 2 (0, T ) (question 4.2) et φ aussi, on en déduit que P φ ∈ G 2 (0, T ) (question 4.4).
7. En utilisant les notations de la question 5.5 ainsi que la question 5.1, on a :

(R) : φ (g + h) = g
n
∀n ∈ N, h(n) (0) = 0, g (n) (T ) = (−1) h(n) (0) = 0,
(n) n (n)
(g + h) (0) = (−1) h(n) (T ) , (g + h) (T ) = h(n) (T )

En dérivant n fois la relation (R) et en l’évaluant en T, on obtient pour tout n > 1 :


n   n−1
X n
X n (k)
φ (T ) h(n−k) (T ) = 0 ⇔ φ(n) (T ) h (T ) = − φ(k) (T ) h(n−k) (T )
k k
k=0 k=0

Puisque φ (T ) = 0 et que h (T ) 6= 0, une récurrence immédiate prouve que φ(n) (T ) = 0 pour tout entier
n donc, toujours pour tout entier n,
n  
(n)
X n (k)
(P φ) (T ) = φ (T ) P (n−k) (T ) = 0.
k
k=0

On remarque ensuite que :


(R0 ) : (1 − φ) (g + h) = h

9
En dérivant n fois la relation (R0 ) et en l’évaluant en 0, on obtient pour tout n > 1 :
n   n−1
X n
X n (k) n−k (n−k) (n) (k)
(1 − φ) (0) (−1) h (T ) = 0 ⇔ (1 − φ) (0) h (T ) = − (1 − φ) (0) h(n−k) (T )
k k
k=0 k=0

(n)
Puisque (1 − φ) (0) = 0 et que h (T ) 6= 0, une récurrence immédiate prouve que (1 − φ) (0) = 0 pour
tout entier n. Ainsi, on vient d’établir que :
 n  
(n) 1 si n = 0 (n)
X n (k)
∀n ∈ N, φ (0) = ⇒ (P φ) (0) = φ (0) P (n−k) (0) = φ (0) P (n) (0) = P (n) (0) .
0 si n > 1 k
k=0

5 Équation de la chaleur
1. Il existe M, R > 0 tel que :
s s s
M (n!) x2n M (n!) 12n M (n!)
∀t ∈ [0, T ] , f (n) (t) 6 n
⇒ f (n)
(t) 6 n
× = n .
R (2n)! R (2n)! R (2n)!
s
M (n!)
On pose pour tout entier n, un = alors :
Rn (2n)!
s
un+1 (n + 1) ns 1 2−s>0
= ∼ 2
= → 0.
un R (2n + 2) (2n + 1) n→+∞ 4Rn 4Rn2−s n→+∞
X
D’après le critère de d’Alembert, la série un converge ce qui prouve la convergence absolue, donc la
n>0
X
(n) x2n
convergence, de la série f (t) d’où la bonne définition de H (t, x) pour tout (t, x) ∈ [0, T ]×[0, 1] .
(2n)!
n>0

x2n
2. Pour tout entier n, posons Hn : (t, x) 7→ f (n) (t) . Cette fonction est de classe C ∞ sur [0, T ] × [0, 1] .
(2n)!
Pour tout couple (p, q) ∈ N2 et tout couple (t, x) ∈ [0, T ] × [0, 1] , on a :
∂ p+q Hn (2n) (2n − 1) · · · (2n − q + 1) 2n−p
(t, x) = f (n+q) (t) x
∂xp ∂tq (2n)!

 0 si p > 2n
s
6 M ((n + q)!) (2n) (2n − 1) · · · (2n − q + 1) = u(p,q)
n
× si p 6 2n
Rn+q

(2n)!
∂ p+q H (p,q) p
donc sup (t, x) 6 un . Pour tout n > , on a :
(t,x)∈[0,T ] ∂xp ∂tq 2
(p,q) s
un+1 (n + q + 1) (2n + 2) (2n + 1) ns 1 2−s>0
= × ∼ 2
= → 0.
(p,q)
un R (2n + 2) (2n + 1) (2n − q + 3) (2n − q + 2) n→+∞ 4Rn 4Rn2−s n→+∞

(p,q) (p,q)
X X
D’après le critère de d’Alembert, la série un converge donc la série un aussi. Ainsi, pour
n>p/2 n>0
X ∂ p+q Hn
tout couple (p, q) ∈ N2 , la série converge normalement, donc uniformément, sur [0, T ] × [0, 1]
∂xp ∂tq
n>0
+∞
X
ce qui démontre que la fonction Hn = H est de classe C ∞ sur [0, T ] × [0, 1] d’où l’existence et la
n=0
continuité de toutes ses dérivées partielles sur [0, T ] × [0, 1] .
3. D’après la question précédente, on a pour tout (t, x) ∈ [0, T ] × [0, 1] :
+∞ +∞
∂H X ∂Hn X x2n
(t, x) = (t, x) = f (n+1) (t)
∂t n=0
∂t n=0
(2n)!
+∞ 2 +∞ +∞
∂2H X ∂ Hn X
(n) x2n−2 X x2k
(t, x) = (t, x) = f (t) = f (k+1) (t)
∂x2 n=0 |
∂x2{z } n=1 (2n − 2)! k=n−1 (2k)!
k=0
=0 si n=0

10
∂H ∂2H
d’où l’égalité − = 0 sur [0, T ] × [0, 1] . En outre, pour tout t ∈ [0, T ] , on a :
∂t ∂x2
+∞ +∞
∂H X ∂Hn X 02n−1
(t, 0) = (t, 0) = f (n) (t) = 0.
∂x ∂x
n=0 | {z } n=1
(2n − 1)!
| {z }
=0 si n=0 =0 car 2n−1>0

N
X
4. Le polynôme H 0 étant pair, il existe un entier N et des réels (cn )06n6N tels que H 0 = cn X 2n . On
n=0
N
X (2n)!
pose P = cn X n ∈ R [X] alors, d’après la formule de Taylor, on a :
n=0
n!

P (n) (0) (2n)! P (n) (0) P (n) (0) P (n) (0)


∀n ∈ {0, ..., N } , = cn ⇔ = cn , ∀n > N, =0⇔ =0
n! n! (2n)! n! (2n)!
3 3
Soit φ la fonction définie par la partie 4 et f = P φ. Puisque φ ∈ G 2 (0, T ) (question 4.5), alors f ∈ G 2 (0, T )
+∞
X x2n
(question 3.2 et 3.4). D’après la question précédente, la fonction H : (t, x) 7→ f (n) (t) vérifie les
n=0
(2n)!
deux premières équations proposées. En outre, on a :
+∞ +∞ N
X x2n question X x2n X
∀x ∈ [0, 1] , H (0, x) = f (n) (0) = P (n) (0) = cn x2n = H 0 (x)
n=0
(2n)! 4.7
n=0
(2n)! n=0
+∞ 2n +∞
X
(n) x question X x2n
∀x ∈ [0, 1] , H (T, x) = 0 = f (T ) = 0 = 0.
n=0
(2n)! 4.7
n=0
(2n)!

∂H ∂H
Pour finir, on pose u : t 7→ (t, 1) qui est continue sur [0, T ] donc ∀t ∈ [0, T ] , (t, 1) = u (t), ce
∂x ∂x
qui permet de conclure.

11
M THÉMATIQUES II 2011/2012

Concours National Commun


Mathématiques II
AQALMOUN Mohamed agrégé de mathématiques CPGE Khouribga

L’objet de ce problème est d’établir le résultat suivant dû à FARAHAT et LEDERMAN en 1958 :

P€oŠuˆr‡ ˆtoŠuˆt Œp€o†l“y“n€ô”m€e ˆu’nˆiˆtaˆiˆr€e P €d€e €d€eg‰r€é n ≥ 2 €à‡ €coe f f‰i€c‰i€e“nˆt €d€a’nŒš K (K = R €oŠu‡ C)
€e‰t ˆtoŠuˆte •m€aˆt‰rˆi€ce N ∈ Mn−1 (K) €d€o”nˆt „le Œp€o†l“y“n€ô”m€e •mˆi’nˆi’m€a„l €esˆt €d€e €d€eg‰r€é n − 1, ˆi„l €eœx‰iŒsˆte

ˆu’n€e •m€aˆt‰rˆi€ce M ∈ Mn (K) ˆte l le €qˆu€e N Œs€oŠiˆt ˆu’n€e Œs€oŠuŒš-•m€aˆt‰rˆi€ce €d€e M €e‰t €qˆu€e „le Œp€o†l“y“n€ô”m€e

n
€caˆr€a€c‰té‰rˆiŒsˆt‰i€qˆu€e €d€e M Œs€oŠiˆt €éga„l €à‡ (−1) P

La troisième partie du problème utilise le résultat de la seconde ; la dernière partie utilise les résultats
de première et de la troisième partie.
Notations et rappels
Dans tout ce problème ,K désigne R ou C. On note K[X] la K-algèbre des polynômes à coefficient dans
K et, pour tout m ∈ N, Km [X] désigne le K-sous espace vectoriel de K[X] formé des polynômes de degré
≤ m.
Pour tout couple (n, p) d’entiers naturels non nuls, on note Mn,p (K) le K-espace vectoriel des matrices
à coefficients dans K, à n lignes et p colonne ; Mn,n (K) est noté simplement Mn (K), c’est la K-algèbre
des matrices carrées d’ordre n à coefficient dans K ; on note aussi In (resp. 0n ) la matrice identité (resp.
la matrice nulle ) de Mn (K).
Si (n, p) ∈ N∗ × N∗ et A ∈ Mn,p (K), on note t A la matrice transposée de A et rg(A) son rang ; si de plus
n = p, la trace de A est noté T r(A) et son déterminant est noté det(A) ou |A|.
Le polynôme minimal d’une matrice A ∈ Mn (K) est noté πA est sont polynôme caractéristique est noté
χA ; on rappelle que, pour tout λ ∈ K, χA = det(A − λIn ) = |A − λIn |.
Si A ∈ Mn (K), la comatrice de A est noté A e = t AA
e ; on rappelle que At A e = |A|In .

Première partie : Expression d’un déterminant

1. (a) On suppose ici que la matrice B est inversible


i. En effectuant un produit matriciel par blocs,déterminer w ∈ Mn,1 (K) et λ ∈ K tels que
    
B v B 0 In w
t = t t
u b u 1 u λ

B 0
ii. Exprimer le déterminant t en fonction du déterminant de la matrice B
u 1
iii. Exprimer l’inverse B −1 de B en fonction de sa comatrice B
e et de son déterminant |B|.
iv. Montrer que
B v
t = b|B| −t ut Bv
e (1)
u b
(b) On revient au cas général et on ne suppose plus que la matrice B est inversible.
i. Montrer qu’il existe ε > 0 tel que pour tout x ∈]0, ε[, la matrice Bx = B − xIn est inver-
sible.
ii. • Si m est un entier ≥ 1, montrer que les applications A → t A et A → |A| ,définie sur
Mn (K) ,sont continues.
On admet que l’application A → A,e définie sur Mn (K), est aussi continue.

Filière MP 1/4 Aqalmoun Mohamed


M THÉMATIQUES II 2011/2012

iii. Montrer alors que la formule (1) ci-dessus est encore valable dans ce cas.
Deuxième partie : Réunion de sous-espaces vectoriels
Soit E un K espace vectoriel non nécessairement de dimension finie ; os suppose un entier naturel
r ≥ 2 et des sous espaces vectoriels F1 , F2 , ..., Fr de E tels que

E = F1 ∪ F2 ∪ ... ∪ Fr

2. (a) Si r = 2 , montrer que E = F1 ou E = F2 . On pourra raisonner par l’absurd et considé-


rer,après en avoir justifier l’existence, un vecteur x1 + x2 où x1 et x2 sont tels que x2 ∈ E \ E1
et x1 ∈ E \ E2
Dans la suite de cette partie , on suppose r ≥ 2 et on pose F = F1 ∪ F2 ∪ ... ∪ Fr−1
(b) On suppose ici que E 6= F et E 6= Fr , et on considère deux vecteurs x ∈ E \ F et y ∈ E \ Fr .
i. Justifier que x ∈ Fr et montrer que , pour tout λ ∈ K, y + λx ∈
/ Fr
ii. En déduire que pour tout λ ∈ K , y + λx ∈ F puis montrer alors qu’il existe deux scalaires
α et β distincts, et un entier k compris entre 1 et r − 1 , tels que y + αx ∈ Fk et y + βx ∈ Fk .
iii. Trouver une contradiction et conclure.
(c) Montrer qu’il existe un entier i ∈ {1, ..., r} tel que E = Fi

Troisième partie :À propos du polynôme minimal d’une matrice


m
X m
X
Soit n ∈ N∗ .Si A ∈ Mn (K) et P = ak X k ∈ K[X],avec m ∈ N, P (A) désigne la matrice ak Ak
k=0 k=0
avec la convention A0 = In ; P est dit un polynôme annulateur de A si P (A) = 0. On rappelle que
le polynôme minimal πA de A est le polynôme unitaire de degré minimal annulateur de A,c’est le
générateur unitaire de l’idéal des polynômes annulateurs de A.
3. (a) Montrer que le polynôme minimal d’une matrice A ∈ Mn (K) est de degré ≤ n
(b) Montrer que le degré du polynôme minimal d’une matrice A ∈ Mn (K) est égal à n si, et
seulement si, (In , ..., An−1 ) est une famille libre de Mn (K).


(c) Soit A ∈ Mn (K) ; pour tout ∈ Mn,1 (K), on pose

IA,v = {P ∈ K[X]; P (A)v = 0}

i. Si v ∈ Mn,1 (K), montrer que IA,v est un idéal de K[X], puis en déduire qu’il existe un
unique polynôme unitaire de K[X] engendrant cet idéal.
Dans la suite du problème, ce polynôme sera noté πA,v .
ii. Montrer que, pour tout v ∈ Mn,1 (K), πA,v divise πA puis en déduire que l’ensemble
{πA,w ; w ∈ Mn,1 (K)} est fini.
On considère donc un entier r ∈ N∗ et des vecteurs v1 , ..., vr de Mn,1 (K) tels que

{πA,w ; w ∈ Mn,1 (K)} = {πA,v1 , ..., πA,vr }.

On pose enfin Fk = {v ∈ Mn,1 (K) ; πA,vk (A)v = 0}, k ∈ {1, ..., r}.
iii. Vérifier que, pour tout k ∈ {1, ..., r} , Fk est un sous espace vectoriel de Mn,1 (K) et
justifier que
Mn,1 (K) = F1 ∪ ... ∪ Fr
iv. Montrer alors qu’il existe w ∈ Mn,1 tel que πA,w = πA
(d) Déterminer un vecteur e ∈ M3,1 (R) tel que πA,w = πA où A est la matrice définie par
 
0 0 c
A =  1 0 b  , (a, b, c) ∈ R3
0 1 a

Filière MP 2/4 Aqalmoun Mohamed


M THÉMATIQUES II 2011/2012

(e) Soit A ∈ Mn (K).On considère les assertions (i), (ii) et (iii) suivantes dont on veut montrer
qu’elle sont équivalentes :
(i) Le polynôme minimal πA de la matrice A est de degré n.
(ii) Il existe v ∈ Mn,1 (K) tel que ,pour tout x ∈ Kn , il existe u ∈ Mn,1 (K) vérifiant

x = (t uv, t uAv, ..., t uAn−1 v)

(iii) Pour tout x ∈ Kn , il existe (u, v) ∈ (Mn,1 (K))2 tel que x = (t uv, t uAv, ..., t uAn−1 v)
Comme il est évident que l’assertion (ii) entraîne l’assertion (iii) , il suffit de montrer que
l’assertion (i) entraîne (ii) et que l’assertion (iii) entraîne (i)
i. Montrer que l’assertion (i) entraîne l’assertion (ii). On pourra considérer la matrice de
Mn (K) dont les colonnes sont les vecteurs v, Av, ..., An−1 v,pris dans cet ordre, ou v est
un vecteurs bien choisi dans Mn,1 (K)
ii. Montrer que l’assertion (iii) entraîne l’assertion (i) . On pourra utiliser la caractérisation
de la question 3.2
Quatrième partie : Démonstration du résultat proposée
n
X
Dans cette parte , on se donne un entier n ≥ 2, un polynôme P = X n + ck X n−k ∈ K[X],
k=1
unitaire de degré n, et une matrice B ∈ Mn−1 (K) dont le polynôme minimal est de degré n − 1.
n
On se propose de montrer l’existence d’une matrice A ∈ Mn (K)  telle que
 χA = (−1) P et dont B
B v
soit une sous matrice ; pour cela , on cherche A sous la forme t , où les inconnues b ∈ K
u b
et u, v ∈ Mn−1,1 (K) sont à déterminer en fonction des coefficients des données P et B
4. (a) Justifier que si A répond à la question alors le coefficient b de la matrice A est entièrement
déterminer et en donner l’expression en fonction des coefficients des données P et B
n−1
X
Dans la suite on écrit χB = (−1)n−1 αk X n−1−k avec α0 = 1, et on pose b = α1 − c1 .
k=0
On cherche à justifier l’existence u, v ∈ Mn−1,1 (K) tels que la matrice A répond à la question.
n−2−p
X
(b) Une famille de polynômes : On pose Up = αk X n−2−k−p , p ∈ {0, ..., n − 2}.
k=0
i. Montrer que (U0 , ..., Un−2 ) est une famille libre de Kn−2 [X]
n−2
X
t
ii. Montrer que tout Q ∈ Kn−2 [X] peut s’écrire Q = yB k zUk avec y, z ∈ Mn−1,1
k=0
(c) Expression d’une matrice :
n−2
X
i. Montrer que, pour tout (x, λ) ∈ K2 , χB (x) − χB (λ) = (−1)n−1 (x − λ) Up (x)λp
p=0
n−2
X
ii. En déduire que , pour tout x ∈ K, χB (x)In−1 = (−1)n (B − xIn−1 ) Up (x)B p
p=0
n−2
X
iii. Montrer que la transposée de la comatrice de (B − xIn−1 ) vaut (−1)n Up (x)B p , pour
p=0
tout x ∈ K
(d) Résolution du problème : A désigne toujours la matrice ci-dessus avec b = α1 − c1
i. Montrer que pour tout x ∈ K ,
n−2
X
χA (x) = (−1)n (xn + (α1 − b)xn−1 + H(x)) − (−1)n Up (x)t uB p v
p=0

où les coefficients de H ne dépendent que du scalaire b et des coefficients de χB .

Filière MP 3/4 Aqalmoun Mohamed


M THÉMATIQUES II 2011/2012

n
X n−2
X
n−k t
n
ii. Montrer que χA = (−1) P si, et seulement si, H − ck X = uB p vUp
k=2 p=0

iii. Justifier alors l’existence d’au moins deux vecteurs u et v de Mn−1,1 (K) tels que la matrice
A répond au problème posé.

Fin de l’épreuve

Filière MP 4/4 Aqalmoun Mohamed


C PGEA.H.RÉDA
.AGADIR.-
FA
. SLAOUI .- C ORRECTION M ATHS :2
MP2012
Première partie :Expression d’un déterminant
Dans cette partie une matrice carrée B d’ordre n à coefficients dans K , u , v deux vecteurs colonnes de Mn,1 (K) et un scalaire b , et n un entier
naturel non nul
I.1. On suppose dans cette question que B est une matrice inversible
( (
w = B −1 v
         
B v B 0 I w B v B Bw Bw = v
I.1.1. On a t = t . n ⇐⇒ t = t t uw + λ ⇐⇒ ce entraine que
u b u 1 0 λ u b u b =t uw + λ λ = b −t uB−1 v
B 0
I.1.2. En développant par rapport à la dernière colonne on a tu = | B|
1
1 te
I.2.3. La formule demandée est B−1 = | B|
B
B v B 0 In w
= b −t uB−1 v | B| =t u| B| − t uB−1 v | B| = b| B| −t ut Bv
 
I.1.4. On a = t .
tu
b −t uB−1 v
e
b u 1 0
I.2. On revient a cas général et on ne suppose plus que la matrice B est inversible
I.2.1. La matrice B a un nombre fini de valeur propres dans C si on pose ε = min {|λ| , λ ∈ S pC ( B) , λ 6= 0} , alors il est clair que
∀ x ∈]0, ε[ ; Bx = B − x.In ∈ G Ln (K)
1.2.2. Soit m un entier naturel non nul .
d.Reamqrquons d’abord que Mm (K) est un espace de dimension finie donc toutes ses normes sont équivalentes et par suite chaque
condidat est libre d’utiliser
( sa norme bienaimée
Mm (K) −→ Mm (K)
..L’application T : est un endomorphisme en dimension finie donc il est continue sur Mm (K)
A 7−→t A
( 2
Mm (K) −→ Km
..L’application f :  est une application linéaire en dimension finie donc elle est continue sur
aij 1≤i,j≤m 7−→ ( a11 , a12 , . . . , amm )
 2
K −→ K
 m
m 2
Mm (K) et l’application g : est une fonction polynomiale de Kn donc elle est continue sur
 aij 1≤i,j≤m 7−→ ∑ ε(σ ) ∏ akσ(k)


σ∈Sm k =1
2
Km donc l’application det : A 7−→ | A| est continue sur Mm (K) car det = go f

I.2.3. Soit x ∈]0, ε[ ona Bx ∈ G Ln (K) , en appliquant le résultat de la question ( I ) à la matrice Bx on a


Bx v
tu = b | Bx | − t u t B
fx v
b
Donc par continuité des applications A 7−→t A , A 7−→ | A| et A 7−→ A e et par passage à la limite quand x tend vers 0 on a
B v t t
t u b = b | B | − u Bv, d’ou le résultat dans le cas général
e

Deuxième partie :Réunion de sous espaces vectoriels


Soit E un K-espace vectoriel non nécessairement de dimension finie ,on suppose qu’il existe un entier naturel r ≥ 2 et des sous espaces F1 , . . . , Fr de
E tels que E = F1 ∪ . . . ∪ Fr
2.1. Dans cette question on suppose que r = 2 , donc E = F1 ∪ F2 .Si E 6= F1 et E 6= F2 , c’est à dire F1 et F2 sont inclus strictement dans E ce qui
entraine alors qu’il existe deux vecteurs x1 et x2 de E tels que x1 ∈ / F1 et x2 ∈
/ F2 et comme E = F1 ∪ F2 , alors x1 ∈ F2 et x2 ∈ F1 . Le vecteur
x1 + x2 est un vecteur de F1 ∪ F2 ce qui entraine que x1 + x2 ∈ F1 ou x1 + x2 ∈ F2 .
..Si x1 + x2 ∈ F1 ,alors ( x1 + x1 ) − x1 = x2 serait un élément de F2 ce qui contredit la définition de x2
..Si x1 + x2 ∈ F2 , alors x1 = ( x1 + x2 ) − x2 serait un élément de F1 ce qui contredit la définition de x1 .On conclut alors que E = F1 ou E = F2
Dans la suite on suppose que r ≥ 2 et on pose F = F1 ∪ . . . ∪ Fr
2.2. On suppose que E 6= F et E 6= Fr et considère deux vecteurs x et y de E tels que x ∈
/ F et y ∈
/ Fr
2.2.1. Comme x ∈ E = F ∪ Fr et x ∈ / F ,alors nécessairement x ∈ Fr .Supposons qu’il existe un scalaire λ tel que y + λ.x ∈ Fr , alors comme
y ∈ Fr et x ∈ Fr ,alors comme Fr est un sous espace vectoriel de E ,on a y = (y + λ.x ) − λ.x ∈ Fr ce qui contredit la définition de y , on
conclut alors que ∀λ ∈ K , y + λ.x ∈ / Fr
2.2.2. Comme E = F ∪ Fr et ∀λ ∈ K , y + λ.x ∈
/ Fr ,alors ∀λ ∈ K , y + λ.x ∈ F. L’ensemble {y + λ.x , λ ∈ K} est un ensemble infini
−1
r[
inclus dans la réunion fini F = Fk ,alors nécessairement il existe deux scalaires distincts α , β et un entier k ∈ [[ 1, r − 1 ]] tel que
k =1
(y + α.x , y + β.x ) ∈ Fk2
2.2.3. Comme Fk est un sous espace vectoriel de E ,alors (y + α.x ) − (y + β.x ) ∈ Fk c’est à dire (α − β).x ∈ Fk et comme α 6= β alors x ∈ Fk et
par suite x ∈ F ce qui contredit la définition de x on conclut alors que E = F ou E = Fr
2.3. On raisonne par récurrence sur r .
..L’initialisation r = 2 est vérifiée à la question (2.1).
..Soit r ≥ 2 ,on suppose le résultat vrai à l’ordre r − 1 et considérons une famille de sous espace vectoriels F1 , . . . Fr tels que
E = F1 ∪ . . . ∪ Fr et montrons qu’il existe k ∈ [[ 1, r ]] , E = Fk .D’après la question précédente on a E = Fr ou E = F = F1 ∪ F1 . . . ∪ Fr−1 .Si
E = Fr c’est terminé si non E = F1 ∪ . . . ∪ Fr−1 ,alors l’hypothèse de récurrence nous permet d’affirmer qu’il existe k ∈ [[ 1, r − 1 ]] , E = Fi
d’ou le résultat à l’ordre r

Troisième partie :A propos du polynôme minimal d’une matrice


3.1. Soit A une matrice d’ordre n à coefficients dans K.D’après le théorème du C aley − Hamilton le polynôme minimal de A divise son polynôme
caractéristique et comme le polynôme caractéristique est de degré n , alors deg π A ≤ n

Page :1
C ORRECTION M ATHS :2 C PGEA.H.RÉDA
.AGADIR.-
FA
. SLAOUI .-
MP2012

n −1
3.2. (⇒).Supposons que le degré du polynôme minmale π A est égale à n et soit (α0 , . . . , αn−1 ) ∈ Kn tel que ∑ αk Ak = 0n , si on pose
k =0
n −1
P= ∑ αk X k si on suppose que ce polynôme est non nul ,alors par définition de π A , on a π A divise P ce qui entraine que deg π A ≤ n − 1 ce
k =0
qui est absurde , on conclut alors que le polynôme P est nul et par suite tous ses coefficients sont nuls c’est à dire ∀k ∈ [[ 0, n − 1 ]] , αk = 0 ce
qui prouve que la famille ( In , A . . . , An−1 ) est une famille libre
r
(⇐).Supposons que la famile ( In , A . . . , An−1 ) est libre est supposons que deg π A = r ≤ n − 1 .Posons π A = ∑ ak X k , comme π A ( A) = 0n
k =0
r
, alors ∑ ak Ak = 0n , or la famille ( In , A, . . . , Ar ) est libre comme sous famille d’une famille libre donc ∀k ∈ [[ 0, r ]] , ak = 0 et par suite le
k =0
polynôme π A est nulle ce qui contredit la définition de π A , on conclut alors que deg π A = n et par suite l’équivalence
3.3. Soit A ∈ Mn (K), pour tout v ∈ Mn,1 (K) on pose I A,v = { P ∈ K[ X ] , P( A)v = 0n,1 }
3.3.1. Il est clair que I A,v contient le polynôme nul .Soit ( P, Q) ∈ (I A,v )2 ,alors ( P − Q)( A)v = P( A)v − Q( A)v = 0n,1 et par suite P − Q ∈ I A,v
donc I A,v est un sous groupe du groupe additif (K[ X ], +).Soit maintenant S ∈ K [ X ] , alors
(SP)( A)v = (S( A) P( A)) v = S( A) ( P( A)v) = S( A)0n,1 = 0n,1
Et par suite I A,v est un idéal de K[ X ] et comme l’anneau K[ X ] est principal , c’est à dire chaque idéal de K[ X ] est engendré par un seul
élément , et si de plus cet idéal est non nul , alors il existe un unique polynôme unitaire qui engendre cet idéal et c’est notre cas puisque
π A est non nul et c’est un élément de I A,v puisque π A ( A) = 0n et par suite π A ( A)v = 0n,1 .Dans la suite le générateur unitaire de I A,v
sera noté π A,v
3.3.2 On a montrer avant que π A ∈ I A,v = π A,v K[ X ] ce qui entraine que π A,v divise le polynôme minimal π A .Comme l’ensemble Du des
diviseurs unitaires du polynôme π A est fini ,et comme {π A,w , w ∈ Mn,1 } ⊂ Du alors il est aussi fini
(
Mn,1 (K) −→ Mn,1 (K)
3.3.3. Soit k ∈ [[ 1, r ]] et Fk = v ∈ Mn,1 (K) ; π A,vk ( A)v = 0n,1 .Il est clair que l’application ϕk :

est un endomor-
v 7−→ π A,vk ( A)v
phisme de Mn,1 (K) de noyau Fk ce qui prouve alors que Fk est un sous espace vectoriel de Mn,1 (K).
Montrons maintenant que Mn,1 (K) = F1 ∪ . . . ∪ Fr .Soit v ∈ Mn,1 (K) , alors ∃k ∈ [[ 1, r ]] , π A,v = π A,vk et comme π A,v ( A)v = 0 , alors
π A,vk ( A)v = 0 c’est à dire que v ∈ Fk d’ou Mn,1 (K) ⊂ F1 ∪ . . . ∪ Fr et par suite on l’égalité
3.3.4. D’après la deuxième partie ∃k ∈ [[ 1, r ]] , Mn,1 (K) = Fk , c’est à dire que ∀v ∈ Mn,1 (K) , π A,vk ( A)v = 0n,1 ce qui permet de dire que
π A,vk ( A) = 0n et par suite π A divise π A,vk et comme π A,vk divise π A et les deux polynômes sont unitaires alors π A = π A,vk
 
0 0 c
3.4. Soit A = 1 0 b avec ( a, b, c) ∈ R3 .Si on désigne par (e1 , e2 , e3 ) la base canonique de M3,1 (R) alors on a Ae1 = e2 , A2 e1 = e3 , ce
0 1 a
qui entraine que (e1 , Ae1 , A2 e1 ) est une base de M3,1 (R) et par suite la famille ( I3 , A, A2 ) est libre et d’après la question (3.2) le degré du
polynôme minimal de A est égale à 3 .Ceci d’une part d’autre part si Q = a0 + a1 X + a1 X 2 est un polynôme à coefficients dans K tel que
Q( A)e1 = 0 , alors comme la famille (e1 , Ae1 .A2 e1 ) est libre alors a0 = a1 = a2 = 0 et par suite Q est le polynôme nul ce qui entraine que
deg π A,e1 ≥ 3 et comme π A,e1 divise π A alors deg π A,e1 = 3 et puisque π A,e1 et π A sont unitaires , alors π A,e1 = π A
3.5. Soit A une matrice d’ordre n à coefficients dans K
3.5.1 Supposons que deg π A = n .Considérons un vecteur v de Mn,1 (K) tel que π A = π A,v , commençons par montrer que la famille
n −1 n −1
(v, Av, . . . , An−1 v) est une base de Mn (K) . Soit α0 , . . . , αn−1 des scalaires tels que ∑ αk Ak v = 0n,1 , si on pose P = ∑ αk X k , alors il est
k =0 k =0
clair que P ∈ I A,v et par suite π A divise P et comme deg P ≤ n − 1 et deg π A = n ,alors nécessairement P = 0 ce qui entraine alors que
∀i ∈ [[ 0, n − 1 ]] , αi = 0 et par suite la famille (v, Av, . . . , An−1 v) est libre et comme son cardinal est égale à la dimension de Mn,1 (K) ,
alors c’est une base de Mn,1 (K) . (
Mn,1 (K) −→ Kn
Considérons maintenant l’application ϕ A,v :  . Montrons que ϕ A,v est un isomorphisme d’espace
u 7−→ t uv,t uAv, . . . ,t uAn−1 v
vectoriel.Il est clair que ϕ A,v est une application linéaire et puisque les espaces Mn,1 (K) et Kn ont la même dimension , alors il suffit de
montrer que ϕ A,v est injective .
.Soit u ∈ Mn,1 tel que ϕ A,v (u) = 0Kn , donc ∀k ∈ [[ 0, n − 1 ]] , t uAk v = 0 ce qui entraine u ∈ R⊥ = {0} ,avec R⊥ est l’orthogonal de R
relativement à son produit scalaire canonique , ce qui entraine alors que u = 0 et par suite ϕ A,v est injective donc c’est un isomorphisme
d’espace vectoriel et par suite ∀ x ∈ Kn , ∃u ∈ Mn,1 (K) , x = t uv,t uAv, . . . ,t An−1 v


3.5.2. Supposons (iii ) , et montrons que deg π A = n , pour cela montrons que la famille ( In , A, . . . , An−1 ) est libre , soit alors (α0 . . . , αn ) ∈ Kn+1
n −1
tel que ∑ αk Ak = 0n : (∗) .Soit i ∈ [[ 0, n − 1 ]] , posons ei = (δi,k )0≤k≤n−1 , d’après l’hypothèse il existe (ui , vi ) ∈ (Mn,1 (K))2 tel que
k =0
tu t ui An−1 vi , remarquons que cette égalité veut dire que t ui Ak vi = δi,k .L’égalité (∗) entraine que
t

ei = i vi ,!ui Avi , . . . ,
n −1 n −1  
tu
i ∑ αk Ak vi = 0 c’est à dire que ∑ αk t ui Ak vi = 0 ce qui entraine que αi = 0 et par suite que la famille ( In , A, . . . , An−1 ) est
k =1 k =0
libre d’ou le résultat

Quatrième partie :Démonstration du résultat proposé


!
n
4.1. On suppose que A répond à la question , alors on a tr ( A) = b + tr ( B) et comme χ A = (−1)n Xn + ∑ ck X n−k ,alors tr ( A) = −c1 et par
k =1
suite b = −c1 − tr ( B)
4.2.1. Il est clair que ∀ p ∈ [[ 0, n − 2 ]] , deg U p = n − 2 − p et par suite la famille (U0 , . . . , U p ) est une famille de degré echellonné de Kn−2 [ X ] donc
c’est une famille libre

Page :2
C PGEA.H.RÉDA
.AGADIR.-
. SLAOUI .-
FA
C ORRECTION M ATHS :2
MP2012

4.2.2. Le cardinal de la famille (U1 , . . . , U p ) est égale à la dimension de Kn−2 [ X ] donc c’est une base de Kn−2 [ X ].Soit Q un élément de Kn−2 [ X ] et
n −2
( a0 , . . . , an−2 ) ∈ Kn−1 telle que Q = ∑ ak Uk .D’après la question (3.5) il existe un couple (y, z) éléments de Mn−1,1 (K) tel que
k =0
n −2
t yz,t yBz, . . . ,t yBn−2 z ce qui entraine alors que Q = ∑ t yBk zUk

( a 0 , . . . , a n −2 ) =
k =0

4.3. Expression d’une comatrice


4.3.1. Soit ( x, λ) ∈ K2 , on a :
n −2− p
! !
n −1   n −2 n −2− k n −2
χ B ( x ) − χ B (λ) = (−1) n −1
∑ αk x n −1− k
−λ n −1− k
= (−1) n −1
( x − λ) ∑ ∑ X n −2− k p
λ = (−1) n −1
( x − λ) ∑ ∑ αk X n −2− k
λp
k =0 k =0 p =0 p =0 k =0
n −2
Ce qui veut dire : χ B ( x ) − χ B (λ) = (−1)n−1 ∑ λ p U p ( x ).
p =0

4.3.2. Si on pose T (λ) = χ B ( x ) − χ B (λ) , alors on a


n −2
T ( B) = χ B ( x ) In−1 − χ B ( B) = χ B ( x ) In−1 = (−1)n ( B − xIn−1 ) ∑ Up (x)B p
p =0

4.3.3. Si A est une matrice d’ordre n à coefficients dans K et (i, j) ∈ [[ 1, n ]]2 , alors la notation [ A]i,j désigne l’élément générique de la matrice A.
!
n −2
t ^ n
On a (∗) : ( B − xIn−1 )( B − xIn−1 ) = χ B ( x ).In−1 = (−1) ( B − x.In−1 ) ∑
U p ( x ) B .Soit (i, j) ∈ [[ 1, n − 1 ]]2 , on note
p
p =0
" #
h  i n −2
Pi,j ( x ) = t
B−
^ xIn−1
i,j
et Qi,j ( x ) = (−1)n ∑ Up (x)B p
p =0 i,j
Il est clair que Pi,j et Qi,j sont deux fonctions polynômiales en x. Si x est un scalaire qui n’est pas un e valeur propre de B , alors la matrice
B − xIn−1 est inversible et par suite (∗) entraine que
n −2
t ( B − xIn−1 ) = (−1)n ∑ Up (x) B p et par suite Pi,j (x) = Qi,j ( x ) et comme le spectre de la matrice B est fini , alors les deux fonctions
p =0
polynômiales Pi,j et Qi,j sont égales sur un ensemble infini donc elles sont égales par tout et par suite ∀ x ∈ K , Pi,j ( x ) = Qi,j ( x ) et ceci
entraine alors que
  n −2
∀x ∈ K , t B −^ x.In−1 = (−1)n ∑ U p ( x ) B p
p =0
4.4. Résolution du problème. A désigne toujours la matrice ci-dessus avec b − α1 − c1
4.4.1. Soit x un élément de K.D’après la question (1.1.4) , on a !
  n −1 n −2
χ A ( x ) = (b − x )χ B ( x ) −t ut B −
^ x.In−1 v = (b − x ) (−1)n−1 ∑ α k X n −1− k − (−1)n ∑ Up (x)t uB p v
k =0 p =0
Ce qu’est équivalent à !
n −1 n −2
χ A ( x ) = (−1) ( x − b)n
x n −1
+ ∑ αk X n −1− k
− (−1)n ∑ Up (x)t uB p v , donc
k =1 p =0
 
 n −1 n −1  n −2
χ A ( x ) = (−1)n  n −1
∑ k n−k
∑ k  − (−1) ∑ U p ( x ) uB v
n −1− k 
 n  n t p
 x + ( α 1 − b ) x + α x − b α x
 k =2 k =1  p =0
| {z }
H (x)
! !
n n −2
4.4.2. ..(⇒).Supposons que χ A ( x ) = (−1)n P, alors on a (−1)n xn + ∑ ck x n−k
= (−1)n xn + ( α1 − b ) x n −1 + H (x) − ∑ Up (x) uB t p
v ,
k =1 p =0
n n −2
comme α1 − b = c1 , alors l’égalité précédente devient après simplification ∑ ck xn−k = H (x) − ∑ Up (x)t uB p v , ce qui entraine alors
k =2 p =0
!
n n −2 n n −2
que H ( x ) − ∑ ck x n−k = ∑ U p ( x )t uB p v , ce étant vrai pour tout x dans K et par suite le polynôme H− ∑ X n−k − ∑ t
uB p vU p
k =2 p =0 k =2 p =0
admet une infinité de racines et par suite il est nul d’ou le résultat
(⇐).Est clair
n
4.4.3. Le polynôme H − ∑ ck X n−k est un élément de Kn−2 [X ] donc d’après la question (4.2.2) , il existe un couple (u, v) d’éléments de
k =2
n n −2
Mn−1,1 (K) tel que H − ∑ ck X n−k = ∑ t uB p v.U p , d’après la question précédente on a χ A = (−1)n P et par suite la matrice
k =2 p =0
 
B v
A= t répond à la question posée
u b

d.j...F IN DE L’ ÉPREUVE ...j.d

Page :3
République Tunisienne
Ministère de l’Enseignement Supérieur
et de la Recherche Scientifique
Concours Nationaux d’Entrée aux Cycles
de Formation d’Ingénieurs
Session 2020

Concours Mathématiques et Physique


Epreuve de Mathématiques I

Session 2020 Date : 16/07/2020 Heure : 8H Durée : 4 heures

La qualité de la rédaction et de la présentation, la clarté et la précision des raisonnements entreront pour une part importante dans l’appré-
ciation des copies. L’usage des calculatrices n’est pas autorisé. Tout résultat fourni dans l’énoncé peut être admis et utilisé par la suite, même
s’il n’a pas été démontré.

Exercice (4,5 points)


On note Mn (R) l’espace vectoriel des matrices carrées réelles d’ordre n ∈ N∗ , GLn (R) le groupe
des matrices inversibles de Mn (R) et In la matrice identité de Mn (R).
On considère l’ensemble D des matrices A = ( aij )1≤i,j≤n ∈ Mn (R) vérifiants
n
aii > ∑ |aij |, pour tout i ∈ {1, 2, . . . , n} .
j =1
j,i

1. Pour p ∈ {1, 2, . . . , n}, on considère l’application f p définie par

fp : M n (R) −→ R
n
A = ( aij )1≤i,j≤n 7−→ a pp − ∑ | a pj |
j =1
j,p

(a) Montrer que, pour tout p ∈ {1, 2, . . . , n} , l’application f p est continue sur Mn (R).
(b) En déduire que D est un ouvert de Mn (R).
2. Soit A ∈ D et X ∈ Mn,1 (R) tels que AX = 0. Montrer que X = 0. (On pourra considérer la
composante de valeur absolue maximale du vecteur X). En déduire que D ⊂ GLn (R).
3. Montrer que D est une partie convexe de Mn (R).
4. Montrer que l’application
det : Mn (R) −→ R

A 7−→ det( A)
est continue.
5. En déduire que det( A) > 0, pour tout A ∈ D .
Concours Mathématiques et Physique Epreuve de Mathématiques I Page 2 sur 5

n
6. Soit A = ( aij )1≤i,j≤n ∈ Mn (R) telle que : aii ≥ ∑ |aij |, pour tout i ∈ {1, 2, . . . , n} .
j =1
j,i
Montrer que det( A + εIn ) > 0, pour tout ε > 0. En déduire que det( A) ≥ 0.

Problème
Dans ce problème, on désigne par Φ la fonction définie sur ]−∞, 1[ par

1 1
 + si t , 0.


t ln (1 − t)
Φ (t) =
 1

si t = 0.

2

Partie 1 (1,5 points) : Préliminaires


Les deux questions suivantes sont indépendantes.
n
1
1. Montrer que la suite (γn )n∈N∗ définie par γn = − ln(n) + ∑ k , pour tout n ∈ N∗ , est conver-
k =1
gente. On note γ sa limite.
Z +∞
2. On pose pour tout x > 0, Γ ( x ) = t x−1 e−t dt.
0

(a) Montrer que Γ est bien définie sur R∗+ et que, pour tout x > 0, Γ( x + 1) = xΓ( x ).
(b) En déduire que, pour tout k ∈ N, Γ(k + 1) = k!.

Partie 2 (5 points) : Etude d’une intégrale


1. (a) Montrer que Φ est une fonction continue sur ]−∞, 1[.
Z 1 Z 1
(b) Montrer que Φ (t) dt converge. On note J = Φ (t) dt.
0 0
2. Soit ε > 0.
Z +∞ −αt
e
(a) Montrer que, pour tout α > 0, l’intégrale dt converge et
ε t
Z +∞ −αt Z +∞ −u
e e
dt = du.
ε t εα u
Z +∞ −αt
e − e− βt
(b) En déduire que, pour tout α > 0 et β > 0, l’intégrale dt converge et
ε t
Z +∞ −αt
− e− βt
Z εβ −u
e e
dt = du.
ε t εα u

3. Soient α et β deux réels strictement positifs.


Z εβ −u
e β
(a) Montrer, à l’aide d’un encadrement, que lim du = ln( α ).
ε →0+ εα u
Concours Mathématiques et Physique Epreuve de Mathématiques I Page 3 sur 5

Z +∞ −αt
e − e− βt
(b) En déduire que dt converge et déterminer sa valeur.
0 t
Z +∞
e−u 1 − e−u
4. Montrer que J = h ( u ) du, où h ( u ) = 1 − , pour tout u > 0.
0 1 − e−u u
Indication : On pourra effectuer le changement de variable u = − ln(1 − t).
5. (a) Pour tout n ∈ N∗ , on pose ψn : ]0, +∞[ → R, u 7→ h (u) e−nu .
Montrer que ψn est intégrable sur ]0, +∞[. Et que
Z +∞
1
|ψn (u)| du = + ln (n) − ln (n + 1) .
0 n

(b) En déduire que J = γ.

Partie 3 (3 points) : Développement en série entière de Φ


On considère la suite de fonctions définies sur [0, 1] par

x (1 − x ) · · · ( n − x )
u0 ( x ) = x et ∀n ≥ 1, un ( x ) = , pour tout x ∈ [0, 1] .
( n + 1) !
1
1. Montrer que, pour tout x ∈ [0, 1] et tout n ∈ N, 0 ≤ un ( x ) ≤ .
n+1
En déduire que la suite de fonctions (un )n∈N converge uniformément vers la fonction nulle
sur [0, 1].
2. Montrer que, pour tout t ∈ ]−1, 1[,
Z 1
1− (1 − t) x dx = tΦ (t) .
0

3. (a) Soit x ∈ [0, 1]. Justifier que, pour tout t ∈ ]−1, 1[,
+∞
(1 − t ) x = 1 − ∑ u n −1 ( x ) t n .
n =1

(b) En déduire que, pour tout t ∈ ]−1, 1[,


+∞ Z 1
Φ (t) = ∑ an tn , où an =
0
un ( x ) dx.
n =0

+∞
an
4. Montrer que γ = ∑ n + 1 . (On pourra considérer une primitive de Φ sur ]−1, 1[).
n =0

Partie 4 (6 points) : Formule de type Taylor et applications


Dans toute la suite, on considère une fonction f : R+ −→ R vérifiant la propriété

∃ A > 0 et β ∈ R tels que ∀t ∈ R+ : | f (t)| ≤ Ae βt .

1. Soit n ∈ N∗ .
Concours Mathématiques et Physique Epreuve de Mathématiques I Page 4 sur 5

k (nt)k
(a) Montrer que, pour tout t ∈ R+ , la série ∑ f ( ) converge absolument.
k ≥0
n k!
On pose alors, pour tout t ∈ R+ ,
+∞
k (nt)k
f n (t) = e−nt ∑ n ) k! .
f (
k =0

(b) Montrer que la fonction f n est continue sur R+ .


2. On fixe x un réel strictement positif, on considère une suite ( Xk )k∈N∗ de variables aléa-
toires indépendantes sur l’espace probabilisé (Ω, T , P) et on suppose qu’elles suivent toutes
la loi de Poisson de paramètre x.
Pour tout n ∈ N∗ , on pose Sn = X1 + · · · + Xn .
(a) Montrer que Sn suit la loi de Poisson de paramètre nx.
   
Sn Sn Sn 2 x
(b) Déterminer l’espérance E de et en déduire que E ( − x ) = .
n n n n
Sn  
(c) Montrer que f ( ) admet une espérance finie donnée par E f ( Snn ) = f n ( x ).
n
3. Dans cette question, on fixe n ∈ N∗ et x un réel strictement positif. On suppose que β ≤ 0
et que f est continue en x.
(a) Soit ε > 0.
i) Montrer qu’il existe α > 0 tel que, pour tout t ∈ R+ ,
A ε
| f (t) − f ( x )| ≤ 2 2
( t − x )2 + .
α 2

ii) En déduire que


Ax ε
| f n ( x ) − f ( x )| ≤ 2 + .
α2 n 2
(b) Montrer alors que
+∞
k (nx )k
f ( x ) = lim e−nx
n→+∞
∑ f(n) k!
.
k =0

k
4. Dans cette question on fixe n ∈ N∗ et on suppose que la série ∑ f ( n ) converge absolument.
k ≥0
Montrer que
1 +∞ k
Z +∞

n k∑
f n ( x )dx = f ( ).
0 =0
n

Dans la suite, on suppose que f est continue sur R+ et que β < 0.

k
5. Montrer que, pour tout n ∈ N∗ , la série ∑ f ( n ) converge absolument.
k ≥0
β
6. Montrer qu’il existe n0 ∈ N∗ tel que n(e − 1) ≤ 2 , ∀n ≥ n0 . En déduire que
β
n

β
| f n ( x )| ≤ Ae 2 x , ∀ x ∈ R+ , ∀ n ≥ n 0 .
Concours Mathématiques et Physique Epreuve de Mathématiques I Page 5 sur 5

7. Montrer alors que f est intégrable sur R+ et que

1 +∞ k
Z +∞

0
f ( x ) dx = lim
n→+∞ n
∑ f ( n ).
k =0

8. En appliquant le résultat de la question précédente à une fonction f bien choisie, montrer


que
+∞
!
1 1
γ = lim ln(1 − e− n ) + ∑ k
.
n→+∞
k =1 n ( e n − 1 )

Fin de l’énoncé
République Tunisienne
Ministère de l’Enseignement Supérieur
et de la Recherche Scientifique
Concours Nationaux d’Entrée aux Cycles
de Formation d’Ingénieurs
Session 2020

Concours Mathématiques et Physique


Corrigé de l’épreuve de Mathématiques I

Exercice
1. (a) Soit p ∈ {1, 2, . . . , n}. Pour tout 1 ≤ j ≤ n, l’application A 7−→ a p,j est continue comme
composée de deux applications continues. Par suite, f p est continue comme combinai-
son linéaires de fonctions continues.

(b) Comme ]0, ∞[ est un ouvert de R et, pour tout p ∈ {1, 2, . . . , n} , f p est continue alors
n
f p−1 (]0, ∞[) est un ouvert pour chaque p ∈ {1, 2, . . . , n} . Donc D = f p−1 (]0, ∞[) est un
T
p =1
ouvert de Mn (R) comme intersection finie d’ouverts de Mn (R).

n
2. Soit X = t ( x1 x2 · · · xn ) ∈ Mn,1 (R) tel que AX = 0. Donc, pour tout i ∈ {1, 2, . . . , n}, ∑ aij x j = 0.
j =1
Soit i0 ∈ {1, 2, .., n} tel que xi0 = max | xi |. On a xi0 = 0 car, sinon :
1≤ i ≤ n
!
n n n
a i0 i0 x i0 = − ∑ a i0 j x j ≤ ∑ a i0 j x j ≤ ∑ a i0 j x i0 < a i0 i0 x i0 ,
j=1,j,i0 j=1,j,i0 j=1,j,i0

qui est absurde. Du fait que max | xi | = xi0 = 0, on déduit que xi = 0, pour tout i ∈
1≤ i ≤ n
{1, 2, . . . , n}. Par suite X = 0.
n o
Soit A ∈ D . Alors, d’après le résultat précédent, ker A = 0 Mn,1 (R) . Donc 0 n’est pas une
valeur propre de A et, par conséquent, A ∈ GLn (R). Il s’ensuit que D ⊂ GLn (R).
 
3. Soient A = aij 1≤i,j≤n
, B = bij 1≤i,j≤n
∈ D et t ∈ [0, 1]. Pour chaque 1 ≤ i ≤ n :

taii + (1 − t)bii > t ∑ aij + (1 − t) ∑ bij = ∑ taij + (1 − t)bij ≥ ∑ taij + (1 − t)bij



j,i j,i j,i j,i

Ce qui montre que tA + (1 − t) B ∈ D .

4. La fonction det est une fonction polynômiale en les coefficients de la matrice A. Ce qui
justifie la continuité de l’application déterminant.
Concours Mathématiques et Physique Corrigé de l’épreuve de Mathématiques I Page 2 sur 13

5. Comme D est convexe alors D est connexe par arcs. De plus, l’application det est continue
donc l’image det(D) de D par l’application det est un intervalle de R. Puisque, d’après
la question 2-, D ⊂ GLn (R) cette image ne contient pas 0 . Par suite det(D) ⊂ R∗+ ou
det(D) ⊂ R∗− . Or In ∈ D et det( In ) = 1 > 0 donc det(D) ⊂ R∗+ . On en déduit que det( A) > 0
pour tout A ∈ D .

n n
6. Soit ε > 0. On a aii ≥ ∑ | aij | donc aii + ε > ∑ | aij |, pour tout i ∈ {1, 2, . . . , n}. Donc A + εIn ∈ D
j,i j,i
j =1 j =1
et, par conséquent, det( A + εIn ) > 0. Ceci étant vrai pour tout ε > 0, lorsque ε → 0, par
continuité de l’application det on obtient det( A) ≥ 0.

Problème
Partie 1 : Préliminaires
1. On a
 
1 1 1 1 1 1
γn − 1 − γn = − − ln(1 − ) = − − − − 2 + o ( 2 )
n n n→+∞ n n 2n n
1 1 1
= + o( 2 ) ∼ ≥ 0.
2n2 n n→+∞ 2n2

Comme la série ∑ 2n1 2 converge alors la série ∑ (γn−1 − γn ) converge. Il s’ensuit que la suite
(γn )n converge.

2. (a) Soit x > 0.


• i) La fonction t 7−→ t x−1 e−t est cpm sur R∗+ .
Z 1
dt
ii) Au voisinage de 0, t x −1 e − t ∼ t x −1 = 1
t1− x
≥ 0 et converge (car 1 − x < 1) donc
Z 1 0 t 1− x
t x−1 e−t dt converge.
0
Z +∞
  dt
iii) Au voisinage de +∞, t 2 t x −1 e − t −→ 0 donc t x −1 e − t = o 1
t2
. Comme
Z +∞ t→+∞ t→+∞ 1 t2
converge et 1
t2
≥ 0 alors t x−1 e−t dt converge.
1
Z +∞
i), ii) et iii) prouvent que t x−1 e−t dt converge.
0
• On pose, pour t > 0, u (t) = t x et v (t) = e−t .
On a lim u (t) v (t) = lim u (t) v (t) = 0 donc le crochet [u (t) v (t)]0+∞ existe et vaut 0.
t →0 t→+∞
Z +∞ Z +∞
0
De plus u (t) v (t) dt = x t x−1 e−t dt converge et vaut xΓ ( x ) . D’après la formule
Z +0∞ 0

d’IPP u (t) v0 (t) dt converge et


0
Z +∞ Z +∞
0
u (t) v (t) dt = [u (t) v (t)]0+∞ − u0 (t) v (t) dt = − xΓ ( x ) .
0 0
Concours Mathématiques et Physique Corrigé de l’épreuve de Mathématiques I Page 3 sur 13

Z +∞
Comme u (t) v0 (t) dt = −Γ ( x + 1) on obtient la formule Γ ( x + 1) = xΓ ( x ).
0
Z +∞
(b) Pour k = 0, Γ (0 + 1) = e−t dt = 1 = 0!.
0
Soit k ∈ N et supposons que Γ (k + 1) = k!. Alors

Γ (k + 2) = (k + 1) Γ (k + 1) = (k + 1) k! = (k + 1)!.

D’après le principe de récurrence, pour tout k ∈ N, Γ (k + 1) = k!.

Partie 2 : Etude d’une intégrale


1. (a) La fonction Φ est continue sur ]−∞, 1[ \ {0} . De plus

ln (1 − t) + t −t − 21 t2 + o t2 + t − 21 + o (1)

1 1 1
Φ (t) = + = = = −→ = Φ (0) .
t ln (1 − t) t ln (1 − t) t (−t + o (t)) −1 + o (1 ) t →0 2

Donc Φ est continue en 0. On en déduit que Φ est continue sur ]−∞, 1[ .

(b) La fonction Φ est continue sur [0, 1[ et prolongeable par continuité en 1 (lim Φ (t) = 1).
t →1
R1
Donc 0 Φ (t) dt est convergente.

2. Soit ε > 0.
(a) Soit α > 0.
e−αt e−αt e−αt
* La fonction t 7→ est cpm sur [ε, +∞[ . De plus t2 = te−αt −→ 0 donc =
t t t→+∞ t t→+∞
Z +∞ Z +∞ −αt
 
1 dt 1 e
o t2 . Comme 2
converge et t2 ≥ 0 alors dt converge.
ε t ε t
Z +∞ −u Z +∞ −u
e e
* Le changement de variable u = αt montre que I (α) = αdu = du.
εα αu εα u
Z +∞ −αt Z +∞ − βt
e e
(b) Soient α, β > 0. D’après 2-(a) les intégrales dt et dt convergent donc
ε t ε t
Z +∞ −αt
e − e− βt
dt converge et
ε t
Z +∞ −αt Z +∞ −αt Z +∞ − βt
e − e− βt e e
dt = dt − dt
ε t ε t ε t
Z +∞ −u Z +∞ −u
e e
= du − du (d’après 2-(a))
εα u εβ u
Z εβ −u Z +∞ −u Z +∞ −u
e e e
= du + du − du
εα u εβ u εβ u
Z εβ −u
e
= du.
εα u
Concours Mathématiques et Physique Corrigé de l’épreuve de Mathématiques I Page 4 sur 13

e−εβ e−u e−εα


3. (a) Supposons que α ≤ β. Pour tout u ∈ [εα, εβ], ≤ ≤ . Donc
u u u
Z εβ −εβ Z εβ −u Z εβ −εα
e e e
du ≤ du ≤ du.
εα u εα u εα u
Par suite   Z εβ −u  
−εβ β e −εα β
e ln ≤ du ≤ e ln .
α εα u α
Z εβ −u  

 
β −
 
β
 
β e β
Comme lim e εβ ln α = lim e εα ln α = ln α alors lim du = ln .
ε →0 ε →0 ε→0 εα u α
Z εα −u  
e α
Si β < α alors lim du = ln et par conséquent
ε→0 εβ u β
Z εβ −u Z εα −u    
e e α β
du = − du −→ − ln = ln .
εα u εβ u ε →0 β α
Z εβ −u  
e β
Donc, dans tous les cas, lim du = ln .
ε→0+ εα u α
Z +∞ −αt Z +∞ −αt
− e− βt − e− βt
Z εβ −u
e e e
(b) D’après 2-(b), l’intégrale dt converge et dt = du.
ε t ε t εα u
De plus, d’après 3-,
Z +∞ −αt
− e− βt
Z εβ −u  
e e β
lim dt = lim du = ln .
ε →0+ ε t ε→0+ εα u α
Z +∞ −αt Z +∞ −αt
− e− βt − e− βt
 
e e β
Donc dt converge et dt = ln .
0 t 0 t α

4. On effectue le changement de variable u = − ln (1 − t), on obtient


Z +∞  Z +∞ 
1 − e−u e−u
Z 1   
1 1 1 1 −u
J= + dt = − e du = 1 − du.
0 t ln (1 − t) 0 1 − e−u u 0 u 1 − e−u

5. (a) Les ψn sont intégrables sur ]0, +∞[ . En effet


Sur ]0, 1] : ψn (u) ∼ h (u) → 0 donc ψn est prolongeable par continuité en 0. Donc ψn
u →0 u →0
est intégrable sur ]0, 1]
 
Sur [1, +∞[ : u2 ψn (u) = u2 e−nu − ue−nu − ue−(n+1)u → 0 et donc ψn (u) = o 1
u2
.
u→+∞ u→+∞
Comme u 7−→ u12 est intégrable sur [1, +∞[ alors il en est de même pour ψn .
Donc ψn est intégrable sur ]0, +∞[. (Autres justifications est possibles).
Comme h(u) ≥ 0 sur ]0, +∞[, on obtient alors :
Concours Mathématiques et Physique Corrigé de l’épreuve de Mathématiques I Page 5 sur 13

Z +∞ Z +∞
|ψn (u)| du = ψn (u) du
0 0
Z +∞
u − 1 + e−u −nu
= e du
0 u !
Z +∞
e−(n+1)u − e−nu
= e−nu + du
0 u
Z +∞ +∞ e−(n+1)u − e−nu
Z
= e−nu du + du
0
  0 u
1 n
= + ln
n n+1
1
= + ln (n) − ln (n + 1) . (∗)
n

e−u +∞
(b) Pour tout u > 0, on a = ∑ e−nu (série géométrique de raison e−u et 0 ≤ e−u <
1 − e − u n =1
1), par suite
Z +∞ Z +∞ +∞
e−u
J= h (u) du = ( ∑ ψn (u))du.
0 1 − e−u 0 n =1
D’autre part,
i) Pour tout n ∈ N∗ , ψn est cpm et intégrables sur ]0, +∞[ .
ii) La série de fonctions ∑ ψn converge simplement sur ]0, +∞[ et sa somme
n ≥1
e−u
S : ]0, +∞[ → R, u 7→ h (u) est cpm.
1 − e−u
Z +∞ Z +∞
1 n+1 1
iii) Comme |ψn (u)| du = − ln( ) ∼ 2 , donc la série ∑ |ψn (u)| du
0 n n 2n 0
converge.
D’après le théorème d’intégration terme à terme on en déduit que
Z +∞ +∞ +∞ Z +∞
J = ( ∑ ψn (u))du = ∑ ψn (u) du
0 n =1 n =1 0
+∞ N
1 1
= ∑ + ln (n) − ln (n + 1) = lim ∑ ( + ln (n) − ln (n + 1))
n =1
n N −→+∞
n =1
n
N
1 N+1
= lim
N −→+∞
∑ n − ln( N + 1) = N −→+
lim γ N − ln(
∞ N
)=γ
n =1

Partie 3 : Développement en série entière de Φ


1. Soit x ∈ [0, 1] .
1
Pour n = 0, on a 0 ≤ u0 ( x ) = x ≤ 1 = 1+0 .
Concours Mathématiques et Physique Corrigé de l’épreuve de Mathématiques I Page 6 sur 13

Soit n ∈ N et supposons que 0 ≤ un ( x ) ≤ 1


n +1 . Alors

x (1 − x ) ... (n + 1 − x ) (n + 1 − x ) 1 (n + 1 − x ) 1
u n +1 ( x ) = = un ( x ) ≤ ≤ .
( n + 2) ! ( n + 2) n + 1 ( n + 2) n+2

D’après le principe de récurrence, pour tout n ∈ N, 0 ≤ un ( x ) ≤ 1


n +1 .
cu
On a ∀ x ∈ [0, 1] et ∀n ∈ N, 0 ≤ un ( x ) ≤ 1
n +1 . Comme 1
n+1 n−→ 0 alors un −→ 0 sur [0, 1] .
→+∞
Autrement : pour tout k ∈ {1, 2, ..n} 0 ≤ k − x ≤ k donc x (1 − x ) ... (n − x ) ≤ n! par suite
1
un ( x ) ≤ n+ 1.

2. Soit t ∈ ]−1, 1[ . Comme 1 − t > 0 alors la fonction x 7−→ (1 − t) x est continue sur le segment
Z 1
[0, 1] . Donc (1 − t) x dx existe et
0
" #1
e x ln(1−t)
Z 1
t
1− (1 − t) x dx = 1 − =1+ = tΦ (t) , t , 0.
0 ln (1 − t) ln (1 − t)
0

Clairement vrai pour t = 0.

3. (a) Pour x ∈ [0, 1], un développement en série entière donne : ∀t ∈ ]−1, 1[,
+∞
x ( x − 1) ... ( x − n + 1) n
(1 − t ) x = 1 + ∑ (−1)n n!
t
n =1
+∞ +∞
x (1 − x ) ... (n − 1 − x ) n
= 1− ∑ t = 1 − ∑ u n −1 ( x ) t n .
n =1
n! n =1

(b) Pour t ∈ ]−1, 1[ ,

+∞ +∞
Z 1 Z 1
! Z 1
!

0
(1 − t) x dx =
0
1− ∑ u n −1 ( x ) t n dx = 1 −
0
∑ u n −1 ( x ) t n dx.
n =1 n =1

On pose vn ( x ) = tn un−1 ( x ), x ∈ [0, 1] .


i) Les vn sont continues sur [0, 1] .
ii) Pour tout x ∈ [0, 1], |vn ( x )| = un−1 ( x ) |t|n ≤ |t|n . Comme ∑ |t|n converge alors la série
de fonctions ∑ vn converge normalement et donc uniformément sur le segment [0, 1] .
Z 1
On en déduit que la série ∑ vn ( x ) dx converge et
n ≥1 0

+∞ Z 1 Z 1 +∞ Z 1
∑ vn ( x ) dx = ∑ vn ( x ) dx = 1 − (1 − t) x dx = tΦ (t) .
n =1 0 0 n =1 0

Z 1 Z 1

D’autre part, pour tout n ∈ N∗ , vn ( x ) dx = un−1 ( x ) dx tn ; donc
0 0

+∞  Z 1  +∞  Z 1  +∞  Z 1 
tΦ (t) = ∑ 0
un−1 ( x ) dx t = tn
∑ 0
un−1 ( x ) dx t n −1
=t∑
0
un ( x ) dx tn .
n =1 n =1 n =0
Concours Mathématiques et Physique Corrigé de l’épreuve de Mathématiques I Page 7 sur 13

+∞  Z 1 
On en déduit que, pour t , 0, Φ (t) = ∑ 0
un ( x ) dx tn et par continuité le résultat
n =0
reste vrai pour t = 0. Donc
+∞ Z 1
∀t ∈ ]−1, 1[ , Φ (t) = ∑ an tn , où an =
0
un ( x ) dx.
n =0

+∞
4. La fonction Φ est DSE sur ]−1, 1[ et ∀t ∈ ]−1, 1[ , Φ (t) = ∑ an tn . Comme F : ]−1, 1[ −→ R,
n =0
Z t
t 7−→ φ (s) ds est une primitive de Φ sur ]−1, 1[ alors F est aussi DSE sur ]−1, 1[ et
0

+∞ +∞
t n +1 t n +1
∀t ∈ ]−1, 1[ , F (t) = F (0) + ∑ an = ∑ an
n + 1 n =0 n + 1
.
n =0

t n +1
Posons, pour tout n ∈ N, gn : ]−1, 1[ −→ R, t 7−→ an .
n+1
an
i) Pour tout n, gn (t) −→ .
t →1 n+1
ii) Pour tout t ∈ ]−1, 1[ ,
Z 1
an 1 1
| gn (t)| ≤ = un ( x )dx ≤ .
n+1 n+1 0 | {z } ( n + 1)2
1
≤ n +1

1
Comme la série ∑ converge alors la série de fonctions ∑ gn converge normalement et
( n +1)2
donc uniformément sur ]−1, 1[.
+∞
an
i) et ii) permettent de conclure que F admet une limite en 1 donnée par lim F (t) =
t →1
∑ n + 1.
n =0
Z 1
Or lim F (t) = Φ (t) dt = J = γ, donc
t →1 0

+∞
an
γ= ∑ n + 1.
n =0

Partie 4 : Formule de Taylor et applications


1. (a) Soit t ∈ R+ . Pour tout k ∈ N,
 β k
 
k (nt) k
k ( nt )
k nte n
f ≤ Ae β n =A .
n k! k! k!
Concours Mathématiques et Physique Corrigé de l’épreuve de Mathématiques I Page 8 sur 13

 β
k
nte n k (nt)k
 
Comme la série ∑ k!
converge (série exponentielle) alors la série ∑ f
n k!
k ≥0 k ≥0
converge absolument.

  (nt)k
(b) Pour tout k ∈ N, on pose uk : R+
−→ R, t 7−→ f nk e−nt .
k!
i) Pour tout k, uk est continue sur R+ .
ii) Soit [ a, b] ⊂ R+ . Pour tout t ∈ [ a, b] ,

k (nt)k −nt k (nb)k


   
|uk (t)| = f e ≤ f .
n k! n k!

  (nb)k
Comme la série ∑ f nk converge alors ∑ uk converge normalement et donc
k ≥0 k! k ≥0
uniformément sur [ a, b] . Donc ∑ uk converge uniformément sur tout segment de R+ .
+∞
D’après i) et ii) la somme f n = ∑ uk de cette série de fonctions est continue sur R+ .
k =0

2. (a) Soit n ∈ N∗ . Comme X1 , ..., Xn sont indépendantes alors pour tout t ∈ [−1, 1],

GSn (t) = GX1 (t) × ... × GXn (t) = ( GX1 (t))n .

Or, pour tout t ∈ [−1, 1],


∞ ∞
( x )n n ( xt)n
G X1 ( t ) = ∑ e− x
n!
t = e− x ∑
n!
= e ( t −1) x
n =0 n =0

Donc, pour tout t ∈ [−1, 1],


 n
x ( t −1)
GSn (t) = e = enx(t−1)

Donc Sn suit la loi de Poisson de paramètre nx.

(b) Soit n ∈ N∗ . Comme Sn suit la loi de Poisson de paramètre nx alors


 
Sn 1 1
E = E (Sn ) = nx = x.
n n n

Donc
 2 !   2 !  
Sn Sn Sn Sn 1 1 x
E −x =E −E =V = 2 V (Sn ) = 2 nx = .
n n n n n n n

(nx )k −nx
N∗ . R+ −→ R, t 7−→ f t
. Comme la série ∑ g (k )

(c) Soit n ∈ On pose g : n e =
k ≥0
k!
Concours Mathématiques et Physique Corrigé de l’épreuve de Mathématiques I Page 9 sur 13

k (nx )k −nx
 
∑ f n k! e converge absolument alors la famille
k ≥0 !
(nx )k −nx
( g (k) P (Sn = k))k∈N = g (k) e est sommable. D’après le théorème de
k!
k ∈N
transfert, g (Sn ) admet une espérance finie donnée par
∞ ∞
(nx )k −nx k (nx )k
 
E ( g (Sn )) = ∑ g (k ) e = e−nx ∑ f = f n (x) .
k =0
k! k =0
n k!
 
Sn
Donc f n = g (Sn ) admet une espérance finie donnée par
  
Sn
E f = f n (x) .
n

3. (a) i) Comme f est continue au point x alors il existe α > 0 tel que, pour tout t ∈ R+ ,
ε
|t − x | ≤ α =⇒ | f (t) − f ( x )| ≤ .
2
Donc, pour tout t ∈ R+ ,

si |t − x | ≤ α alors | f (t) − f ( x )| ≤ ε
2 ≤ 2ε + 2 αA2 (t − x )2 ,

si |t − x | > α alors | f (t) − f ( x )| ≤ | f (t)| + | f ( x )|

≤ 2A

≤ 2 αA2 (t − x )2

≤ 2ε + 2 αA2 (t − x )2 .

Donc il existe α > 0 tel que, pour tout t ∈ R+ ,


ε A
| f (t) − f ( x )| ≤ + 2 2 ( t − x )2
2 α

   2
Sn Sn Sn
ii) Comme n (Ω) ⊂ R+ , donc f n − f ( x ) ≤ 2ε + 2 αA2 n −x . Par suite
Concours Mathématiques et Physique Corrigé de l’épreuve de Mathématiques I Page 10 sur 13

  
Sn Sn
| f n ( x ) − f ( x )| = E( f ) − f (x) ≤ E f − f (x)
n n
 2
ε A Sn
≤ + 2 2E −x
2 α n
ε Ax
≤ +2 2 .
2 α n

Autrement,

∞ ∞
k (nx )k −nx (nx )k −nx
 
| f n ( x ) − f ( x )| = ∑ f n k! e − f (x) ∑ k! e
k =0 k =0
| {z }
=1
∞ ∞
k (nx )k −nx (nx )k −nx
 
= ∑ f n k! e − ∑ f (x) k! e
k =0 k =0

(nx )k −nx
   
k
= ∑ f n − f (x) k! e
k =0

(nx )k −nx
 
k
≤ ∑ f n − f (x) k! e
k =0
∞ 2 !
(nx )k −nx

ε A k
≤ ∑ 2 + 2 α2 n − x k!
e
k =0

ε ∞ (nx )k −nx A ∞ k (nx )k −nx


 2

2 k∑ 2 ∑
= e + 2 − x e .
k! α n k!
|=0 {z } k =0
=1

D’après le théorème de transfert on montre que


 2 ! + ∞  2
x Sn k
=E −x =∑ − x P ( Sn = k ) ,
n n k =0
n

donc
ε Ax
| f n ( x ) − f ( x )| ≤ +2 2 .
2 α n

(b) Soit ε > 0. D’après la question précédente, il existe α > 0 tel que,
ε Ax
∀n ∈ N, | f n ( x ) − f ( x )| ≤ +2 2 .
2 α n

Comme 2 Aα nx −→ 0 alors il existe un rang n0 tel que ∀n ≥ n0 , 2 αA2 nx ≤ 2ε . Donc


n→+∞

ε ε
∀n ≥ n0 , | f n ( x ) − f ( x )| ≤ + = ε.
2 2
Concours Mathématiques et Physique Corrigé de l’épreuve de Mathématiques I Page 11 sur 13

On en déduit que f n ( x ) −→ f ( x ) .
n→+∞

  (nx )k
4. On pose, pour tout k ∈ N, vk : [0, +∞[ −→ R, x 7−→ f nk e−nx .
k!
i) Pour tout k ∈ N, vk est cpm sur [0, +∞[.
+∞
ii) ∑ vk converge simplement sur [0, +∞[ et sa somme f n = ∑ vk est cpm sur [0, +∞[ .
k =0
  nk
iii) Pour tout k ∈ N, lim x2 v k
k ( x ) = lim f n x k+2 e−nx = 0 car n > 0. Donc vk ( x ) =
  x →+∞ x →+∞ k! x →+∞
o x2 . Comme x 7−→ x2 est intégrable sur [1, +∞[ alors vk est intégrable sur [1, +∞[ et
1 1

donc vk est aussi intégrable sur [0, +∞[ .


iv) Pour tout k ∈ N,

(nx )k −nx
Z +∞   Z +∞   Z +∞  
k 1 k 1 k −y 1 k
|vk ( x )| dx = f e dx = f y e dy = f
0 n 0 k! n n k! | 0 {z } n n
=k!
 
k
≤ f .
n
+∞
  R 
Comme la série ∑ f nk converge absolument alors ∑ 0 | v k ( x )| dx converge.
k ≥0 k ≥0
i), ii), iii) et iv) permettent alors d’intervertir les symboles ∑ et :
R

+∞ +∞
!
Z +∞ Z +∞ Z +∞

0
f n ( x ) dx =
0
∑ vk (x) dx = ∑ vk ( x ) dx
k =0 k =0 0
+∞
(nx )k −nx
  Z +∞
k
= ∑f e dx
k =0
n 0 k!
+∞  
k 1 1 +∞ k −y
Z
= ∑f y e dx
k =0
n k! n | 0
{z }
=Γ(k+1)=k!
+∞
 
1 k
= ∑
n k =0
f
n
.

   β
k  β
k
k k
5. Pour tout k, f n ≤ Ae β n =A en . Comme la série ∑ en converge (série géomé-
k ≥0
β  
k
trique de raison e n ∈ [0, 1[) alors la série ∑ f n converge absolument.
k ≥0
β β
6. * On a lim n(e n − 1) = β donc pour ε = − 2 il existe n0 tel que pour n ≥ n0 on a :

β −β
n ( e n − 1) − β ≤
2
β β
donc n(e n − 1) ≤ 2 .
Concours Mathématiques et Physique Corrigé de l’épreuve de Mathématiques I Page 12 sur 13

* Pour tout x ∈ R+
+∞ +∞ β
k (nx )k (nx )k β
| f n ( x )| ≤ e−nx ∑ f ( ) ≤ Ae−nx ∑ (e n )k = Aenx(e n −1) .
k =0
n k! k =0
k!

β 1
D’après la question précédente, pour tout n ≥ n0 , nx (e n − 1) ≤ βx. On en déduit que, pour
2
β βx
nx ( e n −1)
tout n ≥ n0 , e ≤ e 2 et par la suite
β
∀n ≥ n0 , ∀ x ∈ R+ : | f n ( x )| ≤ Ae 2 x .

R +∞ R +∞
7. Vue la question 4-, il suffit de montrer que 0 f n ( x ) dx −→ f ( x ) dx.
n→+∞ 0
cs
i) Comme f est continue sur R+ alors, d’après la question 3-b), f n −→ f sur R+ .
ii) Les f n et f sont cpm sur R+ .
 
β
iii) ∀n ≥ n0 , ∀ x ∈ R : | f n ( x )| ≤ A exp
+ x = ϕ ( x ).
2
La fonction ϕ est intégrable sur R+ car β < 0.
D’après le théorème de la convergence dominée, on peut déduire que les f n et f sont inté-
R +∞ R +∞
grables sur R+ et que 0 f n ( x ) dx −→ 0 f ( x ) dx.
n→+∞
+∞  
R +∞ k
Donc f est intégrable sur R et 0 f ( x ) dx = lim n ∑ f
+ 1
.
n→+∞ n
k =0
Z +∞
e− x
8. D’après les questions 4- et 5- de la partie 2, γ = J = h ( x ) dx, où h ( x ) = 1 −
0 1 − e− x
1 − e− x
. Soit f : R+ −→ R définie par
x
 −x
 e
 
−x 1
− 1
h ( x ) = e x , si x , 0.

1 − e− x 1− e − x
f (x) =
 1

si x = 0.
2

On a f est continue sur R+ , puisque lim f ( x ) = 1


2 = f (0). Posons g ( x ) = e x f ( x ); on a g est
x −→0
continue sur R+ et lim g = 1, donc g est bornée sur R+ . Il vient que | g( x )| ≤ M et par la
+∞
Concours Mathématiques et Physique Corrigé de l’épreuve de Mathématiques I Page 13 sur 13

suite | f ( x )| = e− x | g( x )| ≤ Me− x . En utilisant la question 7-,

1 +∞
Z +∞  
k
0
f ( x )dx = lim
n→+∞ n
∑f n
k =0
+∞  !
1 1 − nk 1 n
2 k∑
= lim + e −k

n→+∞ n 1−e n k
=1
 1 k
+∞ e− n
1 +∞ 1
= lim ( ∑ k − ∑ )
n→+∞ n k
k =1 e n − 1 k =1
| {z }
1
 
=− ln 1− e − n
 
+∞
1  1

= lim  ∑   + ln 1 − e− n  .
n→+∞ k
k =1 n e −1
n
Centrale Mathématiques 1 Calculatrice autorisée 2020

Fonctions arithmétiques multiplicatives et applications


La première partie établit des résultats utiles dans les parties suivantes, qui sont indépendantes entre elles.

Notations
On note bxc la partie entière du nombre réel x, c'est à dire le plus grand nombre entier inférieur ou égal à x.
On note P l'ensemble des nombres premiers.
On note m ∧ n le plus grand commun diviseur (pgcd) des entiers naturels n et m.
Si a et b sont des entiers relatifs, on note [[a, b]] = {k ∈ Z, a 6 k 6 b}.
L'ensemble des matrices carrées de taille n à coecients dans C est noté Mn (C).
La matrice identité de Mn (C) est notée In .
Le terme d'indice (i, j) d'une matrice M ∈ Mn (C) est noté mi,j et on note M = (mi,j )(i,j)∈[[1,n]]2 ou plus simplement
M = (mi,j ) lorsque la taille de M est implicite.
Pour n ∈ N∗ , on note Dn l'ensemble des nombres entiers naturels divisant n et on écrit la somme sur
X X
=
d|n d∈Dn
tous les nombres entiers naturels d divisant n.
Une fonction arithmétique est une fonction f : N∗ → C. L'ensemble des fonctions arithmétiques est noté A.
On dit qu'une fonction arithmétique f ∈ A est multiplicative si
(
f (1) 6= 0
∀(m, n) ∈ (N∗ )2 , m ∧ n = 1 =⇒ f (mn) = f (m)f (n)

On note M l'ensemble des fonctions arithmétiques multiplicatives.


On note 1, δ et I les fonctions arithmétiques


N →(C
( (
N∗ → C N∗ → C

1: , δ: 1 si n = 1 , I:
n 7→ 1 n 7→ n 7→ n
0 si n > 2

On remarque que ces trois fonctions arithmétiques sont multiplicatives.


Si f et g sont deux fonctions arithmétiques, le produit de convolution de f et g est la fonctions arithmétique notée
f ∗ g dénie par X   n
∀n ∈ N∗ , (f ∗ g)(n) = f (d)g
d
d|n

I. Quelques résultats utiles


I.A. Propriétés générales de la loi ∗
1. Vérier que δ est un élément neutre pour la loi ∗.
Pour tout n ∈ N∗ , on note Cn = {(d1 , d2 ) ∈ (N∗ )2 , d1 d2 = n}.
2. Justier que, pour tout n ∈ N∗ , X
(f ∗ g)(n) = f (d1 )g(d2 )
(d1 ,d2 )∈Cn

3. En déduire que ∗ est commutative.


4. De même, en exploitant l'ensemble Cn0 = {(d1 , d2 , d3 ) ∈ (N∗ )3 , d1 d2 d3 = n}, montrer que ∗ est associative.
5. Que peut-on dire de (A, +, ∗) ?

1/6
Centrale Mathématiques 1 Calculatrice autorisée 2020

I.B. Groupe des fonctions multiplicatives

6. Soient f et g deux fonctions multiplicatives. Montrer que si


∀p ∈ P, ∀k ∈ N∗ , f (pk ) = g(pk )
alors f = g .
7. Soient m et n deux entiers naturels premiers entre eux. Montrer que l'application
(
Dn × Dm → Dmn
π:
(d1 , d2 ) 7→ d1 d2
est bien dénie et réalise une bijection entre Dn × Dm et Dmn .
8. En déduire que si f et g sont deux fonctions multiplicatives, alors f ∗ g est encore multiplicative.
9. Soit f une fonction multiplicative. Montrer qu'il existe une fonction multiplicative g telle que
k
X
∀p ∈ P, ∀k ∈ N∗ , g(pk ) = − f (pi )g(pk−i )
i=1

et qu'elle vérie f ∗ g = δ .
10. Que dire de l'ensemble M muni de la loi ∗ ?

I.C. La fonction de Möbius

Soit µ la fonction arithmétique dénie par


si n = 1

1

µ(n) = (−1)r si n est le produit de r nombres premiers distincts
sinon

0

11. Montrer que µ est multiplicative.


12. Montrer que µ ∗ 1 = δ .
13. Soit f ∈ A et soit F ∈ A telle que, pour tout n ∈ N∗ , F(n) = f (d). Montrer que pour tout n ∈ N∗ ,
X

d|n
X n
f (n) = µ(d)F
d
d|n

On note ϕ la fonction indicatrice d'Euler, dénie par :


∀n ∈ N∗ , ϕ(n) = card{k ∈ [[1, n]], k ∧ n = 1}
14. Démontrer que ϕ = µ ∗ I.

I.D. Déterminant de Smith

Soient f une fonction arithmétique, n ∈ N∗ et g = f ∗ µ. On note M = (mi,j ) la matrice de Mn (C) de terme général
mi,j = f (i ∧ j). On dénit aussi la matrice des diviseurs D = (di,j ) par

1 si j divise i,
(
di,j =
0 sinon

g(j) si j divise i,
(
Soit M0 la matrice de terme général m0i,j = .
0 sinon

2/6
Centrale Mathématiques 1 Calculatrice autorisée 2020

15. Montrer que M = M0 D> où D> est la transposée de D.


16. En déduire que le déterminant de M vaut
n
Y
det(M) = g(k)
k=1

I.E. Séries de Dirichlet

Soit f une fonction arithmétique. On dénit, pour tout réel s tel que la série converge,

X f (k)
Lf (s) =
ks
k=1

On appelle abscisse de convergence de Lf


X f (k)
Ac (f ) = inf{s ∈ R, la série converge absolument}
ks
On convient que Ac (f ) = +∞ si l'ensemble ci-dessus est vide.
X f (k)
17. Montrer que si s > Ac (f ), alors la série converge absolument.
ks
18. Soient f et g deux fonctions arithmétiques d'abscisses de convergence nies.
Montrer que si, pour tout s > max(Ac (f ), Ac (g)), Lf (s) = Lg (s), alors f = g .
19. Soient f et g deux fonctions multiplicatives d'abscisses de convergence nies.
Montrer que, pour tout s > max(Ac (f ), Ac (g)),
Lf (s)Lg (s) = Lf ∗g (s)

II. Matrices et endomorphismes de permutation


Dans cette partie n est un entier naturel non nul.
On note Sn le groupe des permutations de [[1, n]]. On notera la composition des permutations de manière multipli-
cative ; par exemple, si γ et σ sont deux permutations de Sn , γ 3 σ 2 = γ ◦ γ ◦ γ ◦ σ ◦ σ .
On dit que deux permutations σ et τ de Sn sont conjuguées s'il existe une permutation ρ ∈ Sn telle que τ = ρσρ−1 .
Pour ` ∈ [[2, n]], on rappelle que, dans Sn , un cycle de longueur ` est une permutation γ ∈ Sn pour laquelle il existe
` éléments deux à deux distincts a1 , . . . , a` de [[1, n]] tels que

si x ∈/ {a1 , . . . , a` }

x

γ(x) = ai+1 si x = ai pour i 6 ` − 1
si x = a`

a1

L'ensemble Supp(γ) = {a1 , . . . , a` } est appelé support du cycle γ et on note γ = a1 a2 · · · a` . On rappelle le




résultat suivant qui pourra être utilisé sans démonstration :


Toute permutation σ ∈ Sn se décompose de manière unique (à. l'odre des facteurs
près) en produit de cycles γ1 , . . . , γr à supports disjoints : σ = γ1 . . . γr .
À toute permutation σ ∈ Sn , on associe la matrice de permutation Pσ = (ai,j ) ∈ Mn (C) où

1 si i = σ(j),
(
ai,j =
0 sinon

3/6
Centrale Mathématiques 1 Calculatrice autorisée 2020

II.A. Similitude de deux matrices de permutation

L'objectif de cette sous partie est de démontrer la propriété (S) suivante :


Les matrices de permutation Pσ et Pτ sont semblables si et seulement si les permu-
tations σ et τ sont conjuguées.
20. Pour toutes permutations ρ, ρ0 ∈ Sn , montrer que Pρρ0 = Pρ Pρ0 . En déduire que si σ et τ dans Sn sont des
permutations conjuguées, alors Pσ et Pτ sont semblables.
21. On considère, dans cette question uniquement, n = 7 et les cycles γ1 = 1 3 7 et γ2 = 2 6 4 . On
 

considère également une permutation ρ ∈ S7 telle que ρ(1) = 2, ρ(3) = 6 et ρ(7) = 4. Vérier que ργ1 ρ−1 = γ2 .
22. Plus généralement, montrer que, dans Sn , deux cycles de même longueur sont conjugués.
Pour σ ∈ Sn et ` ∈ [[2, n]], on note c` (σ) le nombre de cycles de longueur ` dans la décomposition de σ en cycles à
supports disjoints. On note c1 (σ) le nombre de points xes de σ :
c1 (σ) = Card{j ∈ [[1, n]], σ(j) = j}
23. Montrer que σ ∈ Sn et τ ∈ Sn sont conjugués si et seulement si pour tout ` ∈ [[1, n]], c` (σ) = c` (τ ).
La matrice ligne Tσ = c1 (σ) c2 (σ) · · · cn (σ) s'appelle le type cyclique de σ . On vient donc de démontrer que


deux permutations sont conjuguées si et seulement si elles ont le même type cyclique.
Pour tout σ ∈ Sn , on note χσ le polynôme caractéristique de la matrice Pσ : χσ (X) = det(XIn − Pσ ).
24. Soit ` ∈ [[2, n]] et soit γ ∈ S` un cycle de longueur
 `. Montrer que χγ (X) = X − 1.
`

On pourra se ramener au cas γ = 1 2 · · · ` et considérer la matrice


 
0 ... ... ... 0 1
1 0 ... ... 0 0
... .. 
 
. 0

0 1
Γ` =  . . ... ... .. ..  ∈ M` (C)
 
 .. .. . .
 .. ...
 
.

1 0 0
0 ... ... 0 1 0
n
25. Montrer que si σ ∈ Sn alors χσ (X) = (X` − 1)c` (σ) .
Q
`=1
On pourra justier que Pσ est semblable à une matrice diagonale par blocs dont les blocs sont des matrices de
la forme Γ` (` > 1), où Γ` est dénie ci-dessus si ` > 2 et où Γ` = (1) si ` = 1.
26. En raisonnant sur la multiplicité des racines de χσ et de χτ , montrer que si Pσ et Pτ sont semblables, alors,
pour tout q ∈ [[1, n]],
n
X n
X
c` (σ) = c` (τ )
`=1 `=1
q|` q|`

(On somme sur les valeurs de ` multiples de q et dans [[1, n]].)


27. En déduire la propriété (S).
On pourra calculer Tσ D où Tσ est le type cyclique de σ et D la matrice des diviseurs dénie au I.D.

II.B. Endomorphismes de permutation

Dans cette sous-partie, E est un C-espace vectoriel de dimension n > 1. On dit qu'un endomorphisme u de E est un
endomorphisme de permutation s'il existe une base (e1 , . . . , en ) de E et une permutation σ ∈ Sn telle que u(ej ) = eσ(j)
pour tout j ∈ [[1, n]].
On note IdE l'identité de E.
On note Tr(u) la trace d'un endomorphisme u de E et χu son polynôme caractéristique.

4/6
Centrale Mathématiques 1 Calculatrice autorisée 2020

28. Montrer que u est un endomorphisme de permutation si et seulement s'il existe une base dans laquelle sa
matrice est une matrice de permutation.
29. Soit u un endomorphisme de permutation de E. Montrer que u est diagonalisable et que sa trace appartient à
[[0, n]].
30. Soient A, B deux matrices diagonalisables de Mn (C). Montrer que A et B sont semblables si et seulement si
elles ont même polynôme caractéristique.
31. Soit u un endomorphisme de E tel que u2 = IdE . Montrer que u est un endomorphisme de permutation si et
seulement si Tr(u) est un entier naturel.
32. Étudier si l'équivalence de la question précédente subsiste lorsqu'on remplace l'hypothèse u2 = IdE par uk = IdE
pour k = 3, puis pour k = 4.
33. Soit u un endomorphisme de E. Montrer que u est un endomorphisme de permutation si et seulement s'il vérie
les deux conditions suivantes :
n
(a) il existe des entiers naturels c1 , . . . , cn tels que χu = (X` − 1)c` .
Q
`=1
(b) il existe N tel que uN = IdE .
34. Soient u et v deux endomorphismes de E tels que, pour tout k ∈ N, Tr(uk ) = Tr(v k ). Montrer que u et v ont
même polynôme caractéristique.
35. Soit u un endomorphisme diagonalisable de E. Montrer que u est un endomorphisme de permutation si et
seulement s'il existe des entiers naturels c1 , . . . , cn tels que, pour tout k ∈ N,
n
Tr(uk ) =
X
`c`
`=1
`|k

(On somme sur les valeurs de ` divisant k et dans [[1, n]].)

III. Valeurs propres de la matrice de Redheer


On dénit la matrice de Redheer par Hn = (hij )(i,j)∈[[1,n]]2 où

1 si j = 1,


hij = 1 si i divise j et j 6= 1,
0 sinon.

n
On dénit également la fonction de Mertens M, en posant, pour tout n ∈ N∗ , µ(k) où µ est la fonction
X
M(n) =
k=1
de Möbius dénie au I.C.
36. Soient An = (aij )(i,j)∈[[1,n]]2 la matrice de terme général

µ(j) si i = 1,


aij = 1 si i = j,
sinon.

0

et Cn = An Hn . En calculant les coecients de Cn , montrer que det Hn = M(n).


Pour le calcul du terme d'indice (i, j) de Cn , on pourra distinguer le cas i = j = 1, le cas i > 1, j = 1 et le cas i > 1, j > 1.

5/6
Centrale Mathématiques 1 Calculatrice autorisée 2020

On note χn le polynôme caractéristique de Hn , de sorte que χn (λ) = det(λIn − Hn ) pour tout réel λ.
Pour λ réel distinct de 1, on dénit par récurrence la fonction arithmétique b, en posant b(1) = 1 et, pour tout
entier naturel j > 2,
1 X
b(j) = b(d)
λ−1
d|j, d6=j

On dénit également la matrice Bn (λ) = (bij )(i,j)∈[[1,n]]2 de terme général

b(j) si i = 1,


bij = 1 si i = j,
sinon.

0

37. En calculant le produit Bn (λ)(λIn − Hn ), montrer que


n
X
χn (λ) = (λ − 1)n − (λ − 1)n−1 b(j)
j=2

1
Dans toute la suite du problème, on suppose que λ est un réel distinct de 1 et on pose w = .
λ−1
On pose de plus f = (1 + w)δ − w1.
38. Montrer que f ∗ b = δ .
39. En utilisant les notations des séries de Dirichlet données dans la sous-partie I.E, exprimer, pour des valeurs du
réel s à préciser, Lf (s) en fonction de w et L1 (s).
ln(x)
On note log2 la fonction logarithme en base 2, dénie par log2 (x) = pour tout réel x > 0.
ln(2)
40. Montrer que, pour s réel susamment grand,
+∞ blog2 mc
1 X X
=1+ m−s wk Dk (m)
Lf (s)
m=2 k=1

où Dk (m) est le nombre de manières de décomposer l'entier m en un produit de k facteurs supérieurs ou égaux
à 2, l'ordre de ces facteurs étant important.
n
41. Pour n > 1, on pose Sk (n) = Dk (m). Déduire de la question précédente que
X

m=2

blog2 nc
X
n
χn (λ) = (λ − 1) − (λ − 1)n−k−1 Sk (n)
k=1

42. Montrer enn que Hn possède 1 comme valeur propre et que sa multiplicité est exactement
n − blog2 nc − 1

• • • FIN • • •

6/6
Centrale-Supélec 2020 1
MP - Mathématiques 1
On rappelle qu’un produit vide vaut 1 par convention.

I. Quelques résultats utiles


I.A. Propriétés générales de la loi ∗
1. Soit f ∈ A. Soit n ∈ N∗ . Alors (f ∗ δ)(n) = d|n f (d)δ( nd ) = P
f (n) car tous les termes δ( nk ) sont
P
nuls sauf quand d = n où il vaut 1. De même, (δ ∗ f )(n) = d|n δ(d)f ( nd ) = f (n) car tous les
termes sont nuls sauf quand d = 1. Vrai pour tout n ∈ N∗ , donc f ∗ δ = δ ∗ f = f .

La fonction δ est le neutre de la loi ∗.

2. Soit n ∈ N∗ . Si (d1 , d2 ) ∈ Cn alors d1 divise n et d2 = dn1 d’où Cn ⊂ {(d, nd ) : d ∈ Dn }.


Réciproquement, si d ∈ N divise n, alors (d, nd ) ∈ Cn . D’où Cn = {(d, nd ) : d ∈ Dn } et (f ∗ g)(n) =
P n P
d|n f (d)g( d ) = (d1 ,d2 )∈Cn f (d1 )g(d2 ).

X
Pour tout n ∈ N∗ , (f ∗ g)(n) = f (d1 )g(d2 ).
(d1 ,d2 )∈Cn

3. Soient f, g ∈ A et n ∈ N∗ . D’après la question précédente, (f ∗P


P
g)(n) = (d1 ,d2 )∈Cn f (d1 )g(d2 ). Or
(d1 , d2 ) ∈ Cn si et seulement si (d2 , d1 ) ∈ Cn donc (f ∗g)(n) = (d2 ,d1 )∈Cn f (d1 )g(d2 ) = (g ∗f )(n).
Vrai pour tout n ∈ N∗ , donc f ∗ g = g ∗ f . Vrai pour toutes fonctions f, g ∈ A, donc

La loi ∗ est commutative.

4. Soient f, g, h ∈ A et n ∈ N∗ . On a :
X
(f ∗ (g ∗ h))(n) = f (a)(g ∗ h)(b)
(a,b)∈Cn
X X
= f (a) g(c)h(d)
(a,b)∈Cn (c,d)∈Cb
X
= f (a)g(c)h(d)
0
(a,c,d)∈Cn
 
X X
=  f (a)g(c) h(d)
(b,d)∈Cn (a,c)∈Cb
X
= (f ∗ g)(b)h(d)
(b,d)∈Cn

= ((f ∗ g) ∗ h)(n).

En effet, (c, d) ∈ Cb et (a, b) ∈ Cn si seulement si ab = n et cd = b i.e acd = n soit (a, c, d) ∈ Cn0 .


Vrai pour tout n ∈ N∗ donc f ∗ (g ∗ h) = (f ∗ g) ∗ h. Vrai pour tous f, g, h ∈ A donc

La loi ∗ est associative.


1. Vous pouvez envoyez vos remarques ainsi que les irréductibles erreurs et fautes de frappes qui se seront glissées
dans ce document à l’adresse suivante pierre-amaury.monard@laposte.net. L’auteur vous en sera reconnaissant.

1

5. L’ensemble (A, +) est un groupe abélien car (CN , +, ·) est un C-espace vectoriel. La loi de
composition interne δ sur A est associative, commutative et admet un élément neutre. Vérifions
qu’elle par rapport à +.P
est distributive P Soient f, g, h ∈ A et n ∈ N∗ . On a ((f + g) ∗ h)(n) =
P n n n
d|n (f + g)(d)h( d ) = d|n f (d)h( d ) + d|n g(d)h( d ) = (f ∗ h)(n) + (g ∗ h)(n). Vrai pour tout

n ∈ N donc (f + g) ∗ h = f ∗ h + g ∗ h. Donc ∗ est distributive à gauche par rapport à + et
comme ∗ est commutative, elle est aussi distributive à droite. En conclusion,
(A, +, ∗) est un anneau commutatif.

I.B. Groupe des fonctions multiplicatives


6. Soient f, g ∈ M deux fonctions multiplicatives. Commençons par montrer que f (1) = g(1) = 1.
Comme 1 et 1 sont premiers entre eux, on a f (1.1) = f (1)f (1) d’où f (1) = 0 ou f (1) = 1. Or
f (1) 6= 0 par définition d’une fonction multiplicative, donc f (1) = 1. Ceci est vrai pour toute
fonction multiplicative, donc g(1) = 1 aussi.
Soit n > 2. D’après le théorème fondamental de l’arithmétique, il existe p1 , . . . , pr ∈ P des
nombres premiers distincts et α1 , . . . , αr ∈ N∗ des entiers non nuls tels que n = pα1 1 . . . pαr r . On a
alors f (pα1 1 ) = g(pα1 1 ) par hypothèse sur f et g. Si f (pα1 1 . . . pαk k ) = g(pα1 1 . . . pαk k ) pour un certain
αk+1
k ∈ {1, . . . , r − 1} alors comme pα1 1 . . . pαk k et pk+1 sont premiers entre eux, on a
α
f (pα1 1 . . . pk+1
k+1
) = f (pα1 1 . . . pαk k )f (pα1 1 )
= g(pα1 1 . . . pαk k )g(pα1 1 )
α
= g(pα1 1 . . . pk+1
k+1
).
Donc, par récurrence finie sur k = 1, . . . , r on a bien f (pα1 1 . . . pαk k ) = g(pα1 1 . . . pαk k ). En particulier,
pour k = r on tombe bien sur f (n) = g(n). Vrai pour tout n > 2 (et n = 1), donc f = g.
Si f et g multiplicatives coïncident sur les puissances des nombres premiers, alors f = g.
Remarque : si (ap,k ) est une famille d’entiers indexée par P ×N∗ alors il existe une unique fonction
multiplicative f telle que f (pk ) = ap,k . C’est celle définie par f (pα1 1 . . . pαr r ) = ap1 ,α1 . . . apr ,αr .
7. Écrivons n = pα1 1 . . . pαr r et m = q1β1 . . . qsβs la décomposition en facteurs premiers de n et m.
Remarquons que s et r peuvent être éventuellement nuls si n ou m vaut 1 ; ce n’est pas un
problème. Puisque n et m sont premiers entre eux, les facteurs premiers de n et de m sont
disjoints. Autrement dit, les nombres p1 , . . . pr , q1 , . . . qs sont distincts deux à deux.
L’ensemble Dn (resp. Dm ) des diviseurs de n (resp. m) est l’ensemble des nombres de la forme
pγ11 . . . pγr r où γi 6 αi (resp. q1γ1 . . . qsγs et γi 6 βi ) pour tout i. La décomposition en facteurs
premiers de nm étant, pα1 1 . . . pαr r q1β1 . . . qsβs , l’ensemble Dnm des diviseurs de nm est l’ensemble
des entiers de la forme pγ11 . . . pγr r q1δ1 . . . qsδs où γi 6 αi et δi 6 βi . L’application π : Dn × Dm →
Dnm est donc bien définie. L’écriture ` = pγ11 . . . pγr r q1δ1 . . . qsδs fait naturellement apparaître une
décomposition ` = d1 d2 où d1 |n et d2 |m (il suffit de séparer les pi des qj ce qui montre la
surjectivité de π. L’injectivité de π découle de l’unicité de la décomposition en facteur premiers.
(On peut aussi invoquer l’égalité des cardinaux des ensembles de départ et d’arrivée de π).

La fonction π : Dn × Dm → Dnm est bijective si l’on sait dénombrer Dn en fonction de la décomposition en


8. Soient n, m ∈ N∗ premiers entre eux. Alors,
X nm
(f ∗ g)(nm) = f (d)g( )
d
d∈Dnm
X nm
= f (ab)g( )
ab
a∈Dn ,b∈Dm

2
d’après la question précédente,
X X n m
= f (a)f (b)g( )g( )
a b
a∈Dn b∈Dm

n m
car a ∧ b = 1 et a ∧ b = 1 car n et m sont premiers entre eux,

! 
X n X m
= f (a)g( )  f (b)g( )
a b
a∈Dn b∈Dm

= (f ∗ g)(n)(f ∗ g)(m).

Donc f ∗ g est multiplicative.

L’ensemble M des fonctions multiplicatives est stable par ∗.

9. Soit f ∈ A.
Commençons par l’unicité. Supposons qu’une telle fonction multiplicative
∗ k k
Pk g existe.
i
Montrons alors
k−i
que f ∗ g = δ. Soit p ∈ P et k ∈ N . Alors (f ∗ g)(p ) = f (1)g(p ) + 1 f (p )g(p ) = 0. Donc
f ∗ g est multiplicative d’après la question précédente et elle coïncide avec δ sur les puissances
de nombres premiers, donc d’après la question 6, f ∗ g = δ. Comme ∗ est commutative, on as
aussi g ∗ f = δ. Donc f admet un inverse pour la loi ∗ et f −1 = g. Par unicité de l’inverse, g est
unique.
Pour l’existence, nous allons construire g à la main. Commençons par poser g(1) = 1. Soit
p ∈ P un P nombre premier. On a déjà défini g(p0 ). On défini g(pk ) par récurrence en posant
g(pk ) = − k1 f (pi )g(pk−i ) une fois g(p0 ), g(p1 ), . . . , g(pk−1 ) construits. Soit n = pα1 1 . . . pαr r un
entier où les pi sont des nombres premiers distincts. On définit g(n) par g(n) := g(pα1 1 ) . . . g(pαr r ).
Cette définition assure que g est multiplicative d’après la remarque faite à la question 6. De plus
g vérifie bien la l’égalité souhaitée.

Tout élément de M admet un inverse dans M pour la loi ∗.

10. D’après la question 8, ∗ est une LCI sur M. Puisque δ est multiplicative, cette loi possède un
élément neutre. D’après la question précédente,tout élément de M admet un inverse pour la loi
∗. La loi ∗ est donc une loi de composition interne, admettant un élément neutre et telle que tout
élément admet un inverse. Elle est de plus associative et commutative d’après la partie I.

(M, ∗) est un groupe abélien.

I.C. La fonction de Möbius


11. On remarque tout d’abord que µ(1) 6= 0. Soient n, m premiers entre eux. Si n ou m est divisible
par le carré d’un nombre premier, alors nm aussi est divisible par le carré d’un nombre premier
donc µ(n)µ(m) = 0 = µ(nm). Sinon, on écrit n = p1 . . . pr et m = q1 . . . qs où les p, qj sont
des nombres premiers, les pi étant distincts, les qj étant distincts. On a alors µ(n) = (−1)r et
µ(m) = (−1)s y compris si r = 0 ou s = 0. Puisque n et m sont premiers entre eux, les pi sont
distincts des qj , et nm = p1 . . . pr q1 . . . qs est un produit de r + s nombres premiers distincts.
Donc µ(nm) = (−1)r+s = µ(n)µ(m). Vrai pour tout n, m premiers entre eux, donc

µ est multiplicative.

3
12. Soit n ∈ N∗ . Écrivons n = pα1 1 . . . pαr r sa décomposition en facteurs premiers, toujours avec r = 0
si n = 1. Alors,
X n
(µ ∗ 1)(n) = µ(d)1( ).
d
d|n

Puisque µ(d) = 0 dès qu’un facteur carré apparaît dans la décomposition en facteurs premiers
de d, on ne garde dans la somme que les diviseurs de n de la forme p11 . . . prr où i ∈ {0, 1} pour
tout i = 1, . . . , r.
X
= µ(p11 . . . prr )
1 ,...r
X
= (−1)1 +···+r
1 ,...r
X r X
= (−1)|I|
k=0 I⊂{1,...,r},|I|=k

en faisant une disjonction de cas sur k le nombre d’indices i tels que i = 1,


r  
X r
= (−1)k
k
0
= (1 − 1)r
= 0r
= δ0,r

Or r = 0 si et seulement si n = 1. Autrement dit, (µ ∗ 1)(n) = δ(n).

µ ∗ 1 = δ.

13. D’après la façon dont F est définie, on a F = f ∗ 1. Multiplions cette égalité par µ : F ∗ µ =
f ∗ 1 ∗ µ = f ∗ δ = f car µ est l’inverse de 1 pour la loiP∗. Ainsi, on peut « inverser » la formule
et exprimer f en fonction de F : f (n) = (F ∗ µ)(n) = d|n F (d)µ( nd ).

Pour tout n ∈ N∗ , f (n) = (F ∗ µ)(n) = n


P
d|n F (d)µ( d ).

14. Soit n ∈ N∗ . Considérons la liste de rationnels suivantes : n1 , n2 , . . . , nn . Il y a n termes. Chaque


fraction nk s’écrit de manière unique sous la forme dl où l et d sont des entiers naturels premiers
entre eux. De l’égalité nk = dl on tire kd = nl donc d divise nl. Or d ∧ l = 1 donc d divise n d’après
le lemme de Gauss. De plus, nk 6 1 donc l 6 d. Réciproquement, toute fraction de la forme dl avec
d|n, l ∧ d = 1 et l 6 d peut se mettre sous la forme nk en posant simplement k =Pln d ∈ {1, . . . , n}.
Pour chaque diviseur d de n il y a ϕ(d) entiers l 6 d premiers avec d d’où n = d|n ϕ(d).
Ainsi, I = ϕ ∗ 1. D’après la question précédente, ϕ = I ∗ µ.

ϕ = µ ∗ I.

4
I.D. Déterminant de Smith
15. Soient i, j ∈ J1, nK.
n
X
0t
(M D)ij = m0ik djk
k=1
n
X
= g(k)1k|i 1k|j
k=1
X
= g(k)
k|i∧j

car un entier k divise i et j si et seulement si il divise i ∧ j,

= (g ∗ 1)(i ∧ j)

car g ∗ 1 = f

= f (i ∧ j)
= Mij .

M = M 0tD.

16. Prenons le déterminant de l’égalité matricielle que nous venons d’obtenir : det(M ) = det(M 0 ) det(D).
Or M 0 et D sont des matrices triangulaires inférieures car j ne peut diviser i si i < j. Donc
det(M 0 ) = m11 . . . mnn = g(1) . . . g(n) et det(D) = d11 . . . dnn = 1 car dii = 1.

n
Y
det(M ) = g(k).
1

I.E. Séries de Dirichlet


P f (k)
17. Soit s > Ac (f ). Alors il existe t < s tel que kt converge absolument par définition de la
borne inférieure. Mais alors k t < k s pour tout entier naturel k non nul. D’où fk(k)
s 6 fk(k)
t
P f (k)
pour tout k > 1 et la série ks converge absolument par comparaison à une série absolument
convergente.
P f (k)
La série ks converge absolument.

18. Soit t > max(Ac (f ), Ac (g)) un réel que l’on fixe. Par l’absurde, si f 6= g alors on peut po-
ser k0 := min{k ∈ N∗ : f (k) 6= g(k)} puisqu’il s’agit du minimum d’une partie non vide
de N∗ . Soit s > t. Alors Lf (s) = Lg (s) d’où f (k0 ) − g(k0 ) + k0s ∞ f (k)−g(k)
P
k0 +1 ks = 0. Po-
s
P ∞ f (k)−g(k)
sons h(s) := k0 k0 +1 ks et montrons que h(s) → 0 quand s tend vers +∞. On a :
s
P∞ |f (k)|+|g(k)| 1 1 P∞ |f (k)|+|g(k)|
|h(s)| 6 k0 k0 +1 kt . ks−t 6 k0s Ct (k0 +1)s−t où Ct := k0 +1 kt < +∞ est finie
 s
par hypothèse sur t. D’où |h(s)| 6 (k0 + 1)t Ct k0k+1 0
→ 0 quand s → +∞ car k0k+1 0
< 1. Un
passage à la limite dans l’égalité f (k0 ) − g(k0 ) + h(s) = 0 montre que f (k0 ) = g(k0 ) ce qui est
ABSURDE. Donc f = g.

Si Lf (s) = Lg (s) pour s assez grand alors f = g

5
P f (k) P g(k)
19. Soit s > max(Ac (f ), Ac (g)). Les séries ks et ks étant absolument convergente, la série
f (k)g(l) ∗
ak,l où ak,l := ks ls est sommable sur (N )2 :
P
double
+∞ +∞
! !
X f (k) X g(k)
Lf (s)Lg (s) =
ks ks
1 1
X f (k)g(l)
=
k s ls
k,l>1
X X f (k)g(l)
=
(kl)s
n>1 (k,l)∈Cn

puisque les ensembles Cn pour n > 1 forment une partition dénombrable de (N∗ )2 ,
 

X X 1
=  f (k)g(l) s
n
1 (k,l)∈Cn

X (f ∗ g)(n)
=
ns
1
= Lf ∗g (s).

Pour s assez grand, Lf ∗g (s) = Lf (s)Lg (s).

II. Matrices et endomorphismes de permutation


II.A. Similitude de deux matrices de permutation
Pn
Soient σ, τ ∈ Sn des permutations. La matrice Pσ Pτ a pour terme général (Pσ Pτ )ij =
20. P k=1 (Pσ )ik (Pτ )kj =
n
k=1 δi,σ(k) δk,τ (j) = δi,στ (j) = (Pστ )ij . Donc les matrices Pσ Pτ et Pστ sont égales.

Pσ Pτ = Pστ .

En particuliers, Pσ Pσ−1 = Pid = In donc les matrices de permutations sont inversibles et de plus
(Pσ )−1 = Pσ−1 . On a ainsi montré que P : Sn → GLn (R) défini par σ 7→ Pσ est un morphisme
de groupe.
Si σ et τ sont conjugués, il existe γ ∈ Sn tel que σ = γτ γ −1 . D’où, Pσ = Pγ Pτ Pγ −1 et les
matrices Pσ et Pτ sont semblables via la matrice de passage Pγ .

Si σ et τ sont conjugués alors Pσ et Pτ sont semblables.

21. Montrons le résultat suivant : si γ = (a1 . . . ar ) est un r-cycle de Sn et σ ∈ Sn alors σγσ −1 =


(σ(a1 ) σ(a2 ) . . . σ(ar )). En effet, avec la convention ar+1 = a1 , on a σγσ −1 (σ(ai )) = σγ(ai ) =
σ(ai+1 ) et pour x 6∈ {σ(a1 ), . . . , σ(ar )} on a σ −1 (x) 6∈ {a1 , . . . , ar } et donc γσ −1 (x) = σ −1 x d’où
σγσ −1 (x) = x ce qui prouve bien que σγσ −1 = (σ(a1 ) . . . σ(ar )).

Pour γ1 = (1 3 7) et ρ qui envoie 1 sur 2, 3 sur 6 et 7 sur 4 on a ργρ−1 = (2 6 4) = γ2 .

ργ1 ρ−1 = γ2 .

6
22. Soient (a1 . . . ar ) et (b1 . . . br ) deux cycles de même longueur de Sn . D’après le lemme montré
à la question précédente, il suffit de montrer qu’il existe σ ∈ Sn tel que σ(ai ) = bi pour tout
i = 1, . . . , r pour montrer que ces deux cycles sont conjugués car alors σ(a1 . . . ar )σ −1 = (b1 . . . br ).
Une telle permutation existe toujours car les ai sont disjoints et de même pour les bi .
Dans Sn , deux cycles de même longueur sont conjugués.
23. Notons σ = γ1 . . . γs la décomposition en cycles à supports disjoints en incluant les cycles de lon-
gueur 1 (qui correspondent aux points fixes de σ). Alors, pour ρ ∈ Sn , ρσρ−1 = ργ1 . . . γs ρ−1 =
ργ1 ρ−1 ργ2 ρ−1 . . . ργs ρ−1 . Mais d’après le lemme de la question 21, ργi ρ−1 est un cycle de même
longueur que γi . De plus, les cycles ργi ρ−1 sont à supports disjoints par injectivité de ρ. Ainsi, si
τ = ρσρ−1 , alors ργ1 ρ−1 . . . ργs ρ−1 est la décomposition en cycles à supports disjoints de τ . Et
puisque la conjugaison préserve la longueur des cycles, c` (σ) = c` (τ ) pour tout l = 1, . . . , n.

Réciproquement si σ et τ ont le même type cyclique i.e c` (σ) = c` (τ ) pour tout l = 1, . . . , n


alors on peut écrire σ = γ1 . . . γr , τ = δ1 . . . δr les décomposition en cycles à supports disjoints
de σ et τ tel que les cycles γi et δi sont de même longueur ki ∈ J1, nK et k1 + · · · + kr =
n. Cette dernière condition revient à inclure les points fixes dans la décomposition. Écrivons
γi = (ai1 . . . aiki ) et δi = (bi1 . . . biki ). On dispose alors des égalités ensemblistes suivantes :
J1, nK = {a11 , . . . , arkr } = {b11 , . . . brkr }. Soit ρ ∈ Sn l’unique permutation de J1, nK qui envoie
aij sur bij pour tout i = 1, . . . , r et 1 6 j 6 kj . Alors ργi ρ−1 = δi et ρσρ−1 = τ ce qui montre
que σ et τ sont conjugués.
σ et τ sont conjugués si et seulement si ils ont le même type cyclique.

24. Soit γ ∈ S` un cycle de longueur `. Alors γ est conjugué au cycle γ0 = (1 2 . . . `) d’après la


question 22. Les matrices Pγ et Pγ0 sont donc semblables. Le polynôme caractéristique étant un
invariant de similitude, χγ = χγ0 . Il suffit donc de calculer χγ0 . La matrice Pγ0 est la matrice
Γ` donnée par l’énoncé. Il s’agit de la matrice compagnon associée au polynôme X ` − 1 donc
χγ0 = χΓ` = X ` − 1 ce qui conclut.

χγ (X) = X ` − 1.

25. Si ` = 1 alors Γ` = (1) et χ` = X − 1.


Soit σ = γ1 . . . γr la décomposition de σ en cycles à supports disjoints en incluant les points
fixes. Deux cycles à supports disjoints commutant, on peut supposer que les γi sont rangés par
longueur croissante. Quitte à conjuguer σ par la permutation qui « réarrange » les entiers par ordre
croissant, on peut supposer σ = (1 2 . . . , k1 )(k1 +1 . . . k1 +k2 ) . . . (. . . n−1 n) où ki est la longueur
du cycle γi . La matrice Pσ est donc la matrice diagonale par bloc D = Diag(Γk1 , . . . , Γkr ). La
matrice Γ` apparaît autant de fois qu’il Q y a de cycles de longueur ` dans la décomposition en
cycles de σ i.e c` (σ) fois. Donc χσ (X) = n1 χ` (X)c` (σ) .

χσ (X) = n1 (X ` − 1)c` (σ) .


Q

Si Pσ et Pτ sont semblables, elles ont le même polynôme caractéristique. D’où n1 (X ` − 1)c` (σ) =
Q
26. Q
n ` c` (τ ) . Notons T ce polynôme. Soit q ∈ J1, nK. On s’intéresse à la multiplicité de
1 (X − 1)
2πi
ωq := e q en tant que racine de T . Exceptionnellement, on dira que ωq est racine de T de
multiplicité 0 si ωq n’est pas une racine de P .
Si λ ∈ C est de multiplicité P α dans R et β dans S alors il est de multiplicité α + β dans RS.
Donc ωq est de multiplicité n`=1 c` (σ)m` dans T où m` est la multiplicité de ωq dans X ` − 1.
2πi`
Or, ωq est racine de X ` − 1 si et seulement si e q =P1 si et seulement si n` est un entier ie q
divise `. Donc m` = 1q|` . Ainsi, ωP
q est de multiplicité q|` c` (σ) dans T . Mais puisque χσ = χτ ,
ωq est également de multiplicité q|` c` (τ ) dans T d’où l’égalité recherchée.

7
P P
Pour tout q = 1, . . . , n, q|` c` (σ) = q|` c` (τ ).

27. Suivons l’indication de l’énoncé et calculons TPσ D qui est une matrice ligne de longueur n. Soit q ∈
J1, nK. On a : (Tσ D)1,q = n`=1 (Tσ )1,` D`,q = n`=1 c` (σ)1q|` . Ainsi, si Pσ et Pτ sont semblables, les
P
matrices Tσ D et Tτ D sont égales d’après la question précédente. Or D est inversible (triangulaire
avec aucun zéro sur la diagonale) donc Tσ = Tτ . Ainsi, les permutations σ et τ sont de même
type cyclique donc sont conjuguées d’après la question 23. Ceci prouve la réciproque à la question
20.

σ et τ sont conjugués si et seulement si Pσ et Pτ sont semblables.

II.B. Endomorphismes de permutation


28. Soit u ∈ L(E) un endomorphisme de P permutation. Soit B = (e1 , . . . , en ) une base de E et σ ∈ Sn
tels que u(ej ) = eσ(j) . Alors u(ej ) = ni=1 δi,σ(j) ei . La matrice de u dans la base B a pour terme
général δi,σ(j) ...c’est donc Pσ .

Réciproquement, si mat PB (u) = Pσ pour une certaine base B = (e1 , . . . , en ) de E et un certain


σ ∈ Sn . Alors u(ej ) = ni=1 δi,σ(j) ei = eσ(j) pour tout j = 1, . . . , n. L’endomorphisme u est alors
un endomorphisme de permutation.

u est de permutation ssi matB (u) = Pσ pour une certaine base B et σ ∈ Sn .

29. Soit σ ∈ Sn tel que matB (u) = Pσ pour une certaine base B de E. Le groupe Sn étant fini, il
existe un entier k tel que σ k = idJ1,nK . Alors, (Pσ )k = PidJ1,nK = In . Ainsi, le polynôme X k − 1
annule la matrice Pσ . Or X k − 1 est scindé à racine simple sur C. L’endomorphisme u est donc
annulé par un polynôme scindé à racines simples ; il est donc diagonalisable.

L’endomorphisme u est diagonalisable.

De plus, Tr(u) est la somme des coefficients diagonaux de Pσ . La matrice Pσ ne contient que des
0 et des 1 donc sa trace est le nombre de 1 sur sa diagonale ; il s’agit donc d’un entier compris
entre 0 et n.

Tr(u) ∈ J0, nK.

En fait, la trace de u est le nombre de points fixes de σ.


30. Si A et B sont semblables alors elles ont le même polynôme caractéristique.

Réciproquement, supposons que χA = χB . Notons P = r1 (X − λi )mi ce polynôme commun.


Q
Commençons par remarquer que A et B partagent le même spectre à savoir {λ1 , . . . , λr }. Puisque
A est diagonalisable, A est semblable à la matrice diagonale par blocs D = Diag(λ1 Im1 , . . . , λr Imr ).
En effet, pour une matrice diagonalisable, la multiplicité algébrique d’une valeur propre (nombre
de fois où elle apparaît dans une base de diagonalisation) est égale à sa multiplicité algébrique
(multiplicité en tant que racine du polynôme caractéristique). Il en va de même pour B. Donc A
et B sont semblables à une même matrice D donc sont semblables.

A et B diagonalisables sont semblables si et seulement si χA = χB .

31. On a déjà vu à la question 29 que si u est de permutation, alors sa trace est un entier naturel.

8
Réciproquement, supposons que Tr(u) soit un entier naturel. Puisque u est une symétrie, on
dispose de la décomposition E = E1 (u) ⊕ E−1 (u). Dans une base adaptée à cette décomposition
la matrice de u est Diag(In−r , Ir ) où r = dim E−1 (u). La trace de u est alors (n − r) − r.
Puisque Tr(u) est un entier naturel on en déduit r 6 n − r. On pose k = n − r. Soit
(e1 , . . . , er , er+1 , . . . , ek ) une base de E1 (u) et (f1 , . . . , fr ) une base de E−1 (u). On a u(ei ) =
+ei et u(fi ) = −fi . Posons vi := ei + fi et wi = ei − fi pour tout i ∈ J1, rK. On consi-
dère la famille B = (v1 , w1 , . . . , vr , wr , er+1 , . . . , ek ). Montrons que c’est une base de E. Elle
est de cardinal 2r + (k − r) = n. De plus, la famille (e1 , . . . , ek , f1 , . . . , fr ) est une base de
E et chacun de ses vecteurs est combinaison linéaire de vecteurs de la famille B. La famille
B est donc génératrice et c’est une base de E. Dans la base B, la matrice de u est Pσ où
σ = (1 2)(3 4) . . . (2r − 1 2r)(2r + 1) . . . (n). L’endomorphisme u est bien de permutation.

Une symétrie u est de permutation si et seulement si Tr(u) ∈ N.

32. Le sens gauche directe de l’équivalence précédente est toujours vérifiée.


Pour k = 3. L’endomorphisme u est annulé par le polynôme X 3 − 1 scindé à racine simple sur C
donc est diagonalisable et E = E1 (u) ⊕ Ej (u) ⊕ Ej 2 (u). Notons a, b, c les multiplicités de 1, j, j 2 .
Alors Tr(u) = a+bj +cj 2 . Si Tr(u) est un entier naturel, alors en particulier elle est réelle et b = c.
Elle est de plus positive, donc a + bj + bj 2 = a − b > 0. On note r := dim Ej (u) = dim Ej 2 (u)
et k := n − 2r = dim E1 (u). Soient (e1 , . . . , ek ) une base de E1 (u), (f1 , . . . , fr ) une base de
Ej (u) et (g1 , . . . , gr ) une base de Ej 2 (u). On pose vi := e1 + fi + gi , wi := ei + jfi + j 2 gi et
zi := ei + j 2 fi + jgi pour tout i = 1, . . . , r. Alors, on vérifie rapidement que u(vi ) = wi , u(wi ) = zi
et u(zi ) = vi . On considère la famille B = (v1 , w1 , z1 , . . . , vr , wr , zr , er+1 , . . . , ek ) de E. Elle est
de cardinal 3r + (k − r) = n. De plus chaque vecteur ei , fi et gi peut s’écrire comme combinaison
linéaire de vecteurs de B. Donc B engendre E1 (u), Ej (u) et Ej 2 (u) donc E et c’est une base.
Dans la base B, la matrice de u est Pσ où σ = (123)(456) . . . (3r − 2 3r − 1 3r)(3r + 1) . . . (n).
Donc u est un endomorphisme de permutation.
Pour k = 4 la réciproque est fausse pour n = 2 (et n > 2 en adaptant le contre-exemple).  
2 i 0
Considérons l’endomorphisme u de R dont la matrice dans la base canonique est . On
0 −i
a bien u4 = idR2 . De plus, χu (X) = (X − i)(X + i) = X 2 + 1. Or, pour n = 2, il n’existe que deux
permutations : idJ1,2K et (12) dont les polynômes caractéristiques sont respectivement (X − 1)2 et
X 2 − 1. Ainsi, u ne peut être un endomorphisme de permutation. Pour n = 1, u est de la forme
u = λ idE avec λ ∈ N tel que λ4 = 1 donc λ = 1. Donc u = idR2 et est un endomorphisme de
permutation.

Pour k = 4 le sens indirect n’est plus vrai si n > 2.

33. Soit u un Qendomorphisme de permutation. Alors χu = χσ . Or on a vu à la question 25 que


χσ (X) = n1 (X ` − 1)c` (σ) . La condition (a) est vérifiée avec c` = c` (σ). De plus la réponse à
la question 29 a montré l’existence intervenir un polynôme annulateur de u de la forme X k − 1
(prendre k l’ordre de σ dans Sn ). La condition (b) est alors vérifiée avec N = k.

Réciproquement, supposons les conditions (a) et (b) vérifiées. L’endomorphisme u est annulé
par le polynôme X N − 1Pqui est scindé à racines simples dans C donc u est diagonalisable. Le
n
degré de χu vaut n = 1 `c` . Soit σ ∈ Sn une permutation telle que sa décomposition en
cycles à supports disjoints fait intervenir exactementPc` cycles de longueur ` pour tout ` ∈ J1, nK.
L’existence d’un tel σ est assurée par la condition n1 `c` = n. La matrice Pσ etQ la matrice de
u dans une base quelconque de E ont toutes deux pour polynôme caractéristique n1 (X ` − 1)c` .
D’après la question 30 ces deux matrices sont semblables car elles sont de plus diagonalisables.
Il existe une base de E dans laquelle la matrice de u est de la forme Pσ et u est de permutation.

9
u est de permutation si et seulement si il vérifie (a) et (b).

34. Q
Dans C[X] tout polynôme
r n et χ =
Qs est scindém d’après le théorème de D’Alembert-Gauss. Écrivons χu =
1 (X − λ )
i i v 1 (X − µi )i . Nous voulons montrer que r = s, λi = µi et ni = mi
pour tout i = 1, . . . , r. Les endomorphismes u et v sont annulés par leur polynômes caractéris-
tiques d’après le théorème de Cayley-Hamilton qui sont scindés donc u et v sont trigonalisables.
Dans une base de trigonalisation, u est triangulaire supérieure avec comme coefficientsPdiagonaux
λ1 , . . . , λr . La valeur
Ps propre λi apparaît ni fois. La trace de u est donc Tr(u ) = r1 ni λki . De
k k
k
même Tr(v ) = 1 mi µi . k

Pr k
Soit P ∈ C[X]. Par linéarité de la trace, on a Tr(P (u)) = 1 ni P (λi ). Puisque Tr(u ) =
Tr(v k ) pour tout k > 0 alors Tr(P (u)) = Tr(P (v)). Supposons par l’absurde que {λ1 , . . . , λr } = 6
{µ1 , . . . , µs }. Supposons par symétrie que que {µ1 , . . . , µs } 6⊂ {λ1 , . . . , λr }. Il existe alors j ∈
J1, sK tel que µj 6∈ {λ1 , . . . , λr }. Choisissons P ∈ C[X] tel que P (µj ) = 1 et P est nul sur
λ1 , . . . , λr , µ1 , . . . , µj−1 , µj+1 , . . . , µs . Un tel polynôme existe bien grâce à l’hypothèse faite sur
µj (prendre un polynôme interpolateur de Lagrange). Mais alors, Tr(P (u)) = 0 mais Tr(P (v)) =
mj .1 ce qui est contradictoire. Donc {λ1 , . . . , λr } = {µ1 , . . . , µs }. Quitte à renommer les valeurs
propres, on peut supposer λi = µi pour tout i = 1, . . . , r. Prenons maintenant Pi ∈ C[X] tel que
P (λi ) = 1 et P (λj ) = 0 pour j 6= i où i ∈ J1, rK. Alors ni = Tr(Pi (u)) = Tr(P (vi )) = mi ce qui
montre que χu = χv .

Si Tr(uk ) = Tr(v k ) pour tout k > 0, alors χu = χv .

35. Si u est un endomorphisme de permutation. Il existe σ ∈ Sn et une base base B de E dans


laquelle la matrice de u est diagonale par blocs, de blocs PΓ1 , . . . , Γn . Le bloc Γ` apparaît c` (σ)
fois pour tout ` = 1, . . . , n. Pour tout k > 0, Tr(uk ) = n`=1 c` (σ) Tr(Γk` ). Or Γ` est la matrice
de permutation associée à γ0 = (1 2 . . . `) ∈ S` . Donc Tr(Γk` ) est le nombre de points fixes de γ0k .
La permutation γ0k est la permutation identité si ` divise P k et sinon c’est un `-cycle de S` donc
qui a 0 point fixe. Ainsi, Tr(Γ` ) = `1`|k . D’où : Tr(u ) = n`=1,`|k `c` (σ) pour tout k > 0.
k k

Réciproquement, supposons qu’il existe des entiers c1 , . . . , cn tels que Tr(uk ) = n`=1,`|k `c` . En
P

particulier, pour k = 0 on obtient : Tr(u0 ) = n = n`=1 `c` car tout entier divise 0. Donc il existe
P
σ ∈ Sn tel que c` (σ) = c` pour tout ` = 1, . . . , n. Soit B une base de E et v l’endomorphisme de
E défini par v(ei ) = eσ(i) . Alors v est un endomorphisme de permutation et d’après ce qui vient
d’être dit, Tr(v k ) = n`=1,`|k `c` (σ). Donc Tr(uk ) = Tr(v k ) pour tout k > 0 et u et v ont le même
P
polynôme caractéristique d’après la question précédente. Or u est diagonalisable par hypothèse
et v est diagonalisable d’après la question 29 donc les matrices de u et de v dans n’importe quelle
base sont semblables d’après la question 30. Mais alors la matrice de u dans une bonne base est
Pσ et u est de permutation.
n
X
u ∈ L(E) diagonalisable est de permutation ssi Tr(uk ) = `c` pour des entiers c1 , . . . , cn .
`=1,`|k

III. Valeurs propres de la matrice de Redheffer


(Cn )11 = nk=1 a1k hk1 = nk=1 µ(k).1 = M (n).
P P
36. Suivons l’indication
Pn de l’énoncé.
Pn PnPour i > 1,
(C )
Pnn i1 = a h
k=1 ik k1 = δ .h
k=1 i,k k1 = hi1 = 0. Pour i > 1 et j > 1, (C )
n ij = k=1 aik hkj =
k=1 δi,k 1k|j = 1i|j . En particulier, (Cn )ij = 0 dès que i > j. Donc Cn est triangulaire supérieure
et les coefficients sur sa diagonale sont M (n), 1, . . . , 1. D’où det Cn = M (n).
Par ailleurs, det Cn = det An det Hn donc M (n) = det An det Hn = det Hn puisque det An = 1
(An est triangulaire supérieure avec des 1 sur la diagonale).

10
det Hn = M (n).

37. La matrice Bn (λ) est triangulaire avec pour coefficients diagonaux b(1), 1, . . . , 1 donc det Bn (λ) =
b(1) = 1. Donc χn (λ) = det(λIn − H) = det(Bn (λ)(λIn − H)). Posons T = (tij )16i,j6n =
Bn (λ)(λIn − H).
Pour i = j = 1, on a t11 = n1 b1k (λδk1 − hk1 ) = (λ − 1) − n2 b(k).
P P

Pour i > 1, j > 1, on a tij = n1 bik (λδk,j − hk,j ) = λδi,j − 1i|j . En particulier, tij = 0 si i > j > 1
P
et tij = λ − 1 si i = j > 1.
1 et j > 1, on a t1j = nk=1 b1k (λδk,j − hkj ) = nk=1 b1k b(k)(λδk,j − 1k|j ) = (λ −
P P
Pour i =P
1)b(j) − k|j,k6=j b(k) = 0.
Donc la première ligne de T est ((λ − 1) − Pn2 b(k), 0, . . . , 0). Un développement par rapport à la
P
première ligne donne : det T = ((λ − 1) − n2 b(k)) det([T ]11 ) où [T ]11 est la matrice obtenue à
partir de T en rayant la première ligne et la première colonne. Il s’agit d’une matrice triangulaire
supérieure avec les coefficients
Pn λ − 1 sur la diagonale. D’où det([T ]11 ) = (λ − 1)n−1 et det T =
n
(λ − 1) − (λ − 1) n−1
2 b(k).

n
X
Pour λ 6= 1, χn (λ) = (λ − 1)n − (λ − 1)n−1 b(k).
k=2

Commençons par calculer 1 ∗ b. On a (1 ∗ b)(1) = 1(1)b(1) = 1 et pour n > 2, (1 ∗ b)(n) =


38. P
k|n,k6=n b(k) + b(n) = (λ − 1)b(n) + b(n) = λb(n). Ainsi 1 ∗ b = λb + (1 − λ)δ.

On a f ∗ b = ((1 + w)δ − w1) ∗ b = (1 + w)b − wλb − w(1 − λ)δ = [1 − w(λ − 1)]b + w(λ − 1)δ = δ
par définition de w.

f ∗ b = δ.

39. Pour k > 2 et s ∈ R, fk(k)


P f (k)
s = −wks . Donc la série ks converge si et seulement si s > 1. Soit
s > 1. On a Lf (s) = (1 + w)Lδ (s) − wL1 (s). La fonction Lδ est constante égale à 1.

Pour s > 1, Lf (s) = 1 + w − wL1 (s).

40. En admettant que Lb a une abscisse de convergence finie. Soit s > max(1, Λc (b)). La formule
prouvée à la question 19 montre que Lf (s)Lb (s) = Lδ (s) = 1. Donc Lf (s) est non nul et Lf1(s) =
Lb (s) = 1 + +∞
P −s
P+∞ k
m=2 m b(m). Posons c(m) = k=1 w Dk (m) pour tout m > 2.
Commençons par remarquer que pour m > 2 et k > 1, si Dk (m) est non nul, alors il existe une
décomposition de m en k facteurs plus grand que 2 d’où 2k 6 m et k 6 log2 (m). Or k est un entier
Pblog (m)c k
donc k 6 blog2 (m)c. Ainsi, Dk (m) = 0 si k > blog2 (m)c et c(m) = k=12 w Dk (m) pour
tout m > 2 ce qui prouve que c est bien définie (c’est heureux). Pour prouver l’égalité voulue, il
suffit de prouver que b(m) = c(m) pour tout m > 2. Montrons ce résultat par récurrence forte
sur m > 2.
1 P1 k
Pour m = 2. On a b(2) = λ−1 b(1) = w et c(2) = k=1 w D1 (2) = w car log2 (2) = 1 et
D1 (m) = 1 pour tout m > 2.
Supposons le résultat vrai pour tout 2 6 d < m. Alors

11
1 X
b(m) = b(d)
λ−1
d|m,d6=m
 
X
= w c(d) + 1 par hypothèse de récurrence,
d|m,1<d<m

X +∞
X
=w+w wk Dk (d)
d|m,1<d<m k=1
+∞
X X
=w+ wk+1 Dk (d)
k=1 d|m,1<d<m

car les sommes infinies sont des sommes finies donc on peut inverser l’ordre de sommation sans
problème,
+∞
X X
=w+ wk Dk−1 (d)
k=2 d|m,1<d<m

Or, pour tout diviseur 1 < d < m de m, l’écriture m = d. m d donne une factorisation de m en
k entiers plus grand que 2 à partir d’une factorisation de m d en k − 1 entiers plus grand que 2.
Réciproquement si m = n1 . . . nk où les ki sont des entiers plus grand queP 2 alors 1 < n1 m< m
m
et n1 divise m et n2 . . . nk est une factorisation de k1 . Donc Dk (m) = d|m,1<d<m Dk−1 ( d ) =
0
P
d0 |m,1<d0 <m Dk−1 (d ).

+∞
X
=w+ wk Dk (m)
k=2
+∞
X
= wk Dk (m)
k=1
= c(m).
Ceci conclut la récurrence.
P+∞ −s Pblog2 (m)c k
D’où Lf1(s) = 1 + +∞ −s
P
m=2 m c(m) = 1 + m=2 m k=1 w Dk (m).

+∞
X +∞
X
−s
Pour s assez grand, 1
Lf (s) =1+ m wk Dk (m).
m=2 k=1

41. On a
n
X n
X
b(m) = c(m)
2 2
n X
X +∞
= wk Dk (m)
2 1
+∞
X n
X
k
= w Dk (m)
1 2
+∞
X
= wk Sk (n)
1

12
Or, pour k > blog2 (n)c et tout m ∈ J1, nK, on a Dk (m) = 0 donc Sk (n) = 0 d’où n2 b(m) =
P
Pblog2 (n)c k Pblog (n)c
w Sk (n). Donc, χn (λ) = (λ−1)n −(λ−1)n−1 n2 b(j) = (λ−1)n −(λ−1)n−1 1 2
P
1 (λ−
−k
1) Sk (n).

blog2 (n)c
X
Pour tout λ 6= 1, χn (λ) = (λ − 1)n − (λ − 1)n−k−1 Sk (n).
1

42. Soit n > 2. Considérons la fonction g : x 7→ x − 1 − log2 x définie sur R∗+ . La fonction g est
dérivable de dérivée g 0 (x) = 1 − x ln1 1
2 . Donc g est strictement croissante sur [2, +∞) car ln 2 6 2.
Donc g(n) > g(2) = 0 et n − k − 1 est un entier naturel pour tout 1 6 k 6 blog2 (n)c.
Pblog (n)c
Les polynômes χn (X) et (X − 1)n − 1 2 (X − 1)n−k−1 Sk (n) coïncident sur R\{1} qui est
une partie infinie de R donc sont égaux. On écrit χn (X) = (X − 1)n−blog2 (n)c−1 Q où Q = (X −
Pblog (n)c Pblog (n)c−1
1)blog2 (n)c+1 − 1 2 (X −1)blog2 (n)c−k Sk (n). Comme Q(1) = 0− 1 2 0−Sblog2 (n)c (n) 6
blog
Dblog2 (n)c (2 2 (n)c ) = −1, Q(1) est non nul et 1 est racine de χn de multiplicité n−blog2 (n)c−1.

Pour n > 2, 1 est valeur propre de Hn de multiplicité n − blog2 (n)c − 1.

Pour n = 1 le résultat est faux. En effet, 1 est racine de multiplicité 1 de H1 = (1), mais
1 − blog2 (1)c − 1 = 0.

• • • FIN • • •

13
Problème de mathématiques: MP Enoncé

INP-M1-2019

EXERCICE I
+∞
X 1 π2 te−t
On admet que = et on pose, pour t ∈]0, +∞[, f (t) =
n=1
n 2 6 1 − e−t
Q1. Justifier que la fonction f est intégrable sur ]0, +∞[ puis, à l’aide d’un théorème d’intégration terme à terme,
Z +∞
t
calculer l’intégrale t
dt
0 e −1

EXERCICE II
Si X est une variable aléatoire à valeurs dans N de loi de probabilité donnée par : ∀n ∈ N, pn = P (X = n), la fonction
génératrice de X est
+∞
 X
GX (t) = E tX = pn tn
n=0

Q2. Démontrer que l’intervalle ] − 1, 1[ est inclus dans l’ensemble de définition de la fonction GX .

Soient X1 et X2 deux variables aléatoires indépendantes à valeurs dans N .


On pose S = X1 + X2 , démontrer que pour tout t ∈] − 1, 1 [, GS (t) = GX1 (t).GX2 (t)) par deux méthodes
: l’une utilisant le produit de Cauchy de deux séries entières et l’autre utilisant uniquement la définition :

GX (t) = E tX .

On généralise ce résultat, que l’on pourra utiliser dans la question suivante, à n variables aléatoires mutuellement
indépendantes à valeurs dans N (on ne demande pas de preuve de cette récurrence).
Q3. Un sac contient quatre boules : une boule numérotée 0 , deux boules numérotées 1 et une boule numérotée 2.
On effectue n tirages d’une boule avec remise et on note Sn la somme des numéros tirés.
Déterminer pour tout t ∈] − 1, 1[, GSn (t) et en déduire la loi de Sn

PROBLÈME
X xn
Dans ce sujet une série de fonctions La est une série de fonctions an où (an )n>1 est une suite de réels telle
1 − xn
n>1
X
n
que la série entière an x soit de rayon 1.
n>1

Partie I: Propriétés

X xn
Soit une série de fonctions La : an
1 − xn
n>1

Q4. Si x ∈] − 1, 1[ , donner un équivalent de 1 − xn pour n au voisinage de +∞.


X xn
Démontrer que pour tout x ∈] − 1, 1[ , la série an converge absolument.
1 − xn
n>1
Remarque: la série La peut parfois converger en dehors de l’intervalle ]−1, 1[. Donner un exemple de suite (an )n>1
telle que la série La converge en au moins un x0 n’appartenant pas à l’intervalle ] − 1, 1[
X xn
Q5. Démontrer que la série de fonctions an converge uniformément sur tout segment [−b, b] inclus dans
1 − xn
n>1
l’intervalle ] − 1, 1[
+∞
Xxn
Q6. On pose, pour tout x ∈] − 1, 1[, f (x) = an
n=1
1 − xn
Justifier que la fonction f est continue sur l’intervalle ] − 1, 1[ et démontrer ensuite que la fonction f est de classe
C 1 sur l’intervalle ] − 1, 1[. Donner la valeur de f 0 (0)

elamdaoui@gmail.com 1/8 www.elamdaoui.com


Problème de mathématiques: MP Enoncé

INP-M1-2019

Q7. Expression sous forme de série entière


On note A = N ∗ × N∗ .
Lorsque (un,p )(n,p)∈A est une famille sommable de réels, justifier que
!  
+∞ X
X +∞ +∞
X X
un,p =  uk,p  , où In = {(k, p) ∈ A, kp = n}
n=1 p=1 n=1 (k,p)∈In

Démontrer que pour tout x ∈] − 1, 1[ , la famille (an xnp )(n,p)∈A est sommable.
+∞ +∞
X xn X X
En déduire que pour tout x ∈] − 1, 1[, an n
= bn xn où bn = ad (d|n signifiant d divise n)
n=1
1−x n=1 d|n

Partie II: Exemple

Q8. Dans cette question, pour n > 1, an = 1 et on note dn le nombre de diviseurs de n. Exprimer, pour tout

X xn
x ∈] − 1, 1[, f (x) = an comme la somme d’une série entière.
n=1
1 − xn
Q9. Dans cette question, pour n > 1, an = ϕ(n) où ϕ(n) est le nombre d’entiers naturels premiers avec n et inférieurs
à n. X
Justifier que la série entière an xn est de rayon 1
n>1
X
On admet que pour n > 1, n = ϕ(d). Vérifier ce résultat pour n = 12.
d|n
+∞
X xn
Pour x ∈] − 1, 1[ , exprimer ϕ(n) sous la forme d’un quotient de deux polynômes.
n=1
1 − xn
Q10. En utilisant le théorème de la double limite, établir à l’aide du développement en série entière de la fonction
+∞
X (−1)n
x 7→ ln(1 + x) sur l’intervalle ] − 1, 1[ la valeur de la somme
n=1
n
+∞
X xn
Q11. Dans cette question et la suivante, pour n > 1, an = (−1)n et pour tout x ∈] − 1, 1[, f (x) = an .
n=1
1 − xn
f (x)
En utilisant le théorème de la double limite, calculer lim et donner un équivalent de f (x) au voisinage de
x→0 x
0. Retrouver le dernier résultat de la question Q.6
− ln 2
Q12. Démontrer qu’au voisinage de 1, f (x) ∼ .
1−x
1−x 1
On pourra remarquer que pour x ∈]0, 1[, =
1 − xn 1 + x + x2 + . . . + xn−1

elamdaoui@gmail.com 2/8 www.elamdaoui.com


Problème de mathématiques: MP Correction

INP-M1-2019

EXERCICE I
+∞
X 1 π2 te−t
On admet que = et on pose, pour t ∈]0, +∞[, f (t) =
n=1
n 2 6 1 − e−t
te−t
Q1. L’application t 7−→ est continue sur ]0, +∞[
1 − e−t
t + o(t)
— En 0+ : f (t) = = 1 + o(1), donc elle est prolongeable par continuité en 0+ , donc elle est intégrable
t + o(t)
en 0  
−t 1
— En +∞: f (t) ∼ te = o 2 , donc elle est intégrable par la règle de Riemann
t
Donc f est intégrable sur ]0, +∞[.
+∞
x X
Pour tout t > 0, on a ∀x ∈ ]−1, 1[: = xn . Comme e−t ∈ ]0, 1[, alors
1 − x n=1

+∞
te−t X
= te−nt
1 − e−t n=1

Les fonctions fn X: t ∈ ]0, +∞[ 7−→ te−nt sont continues par morceaux sur ]0, +∞[ et, en vertu de l’étude qui
précède, la série fn converge simplement et sa somme f est continue sur ]0, +∞[
n>1
Les fonctions fn sont intégrables sur ]0, +∞[ et par une intégration par parties
Z +∞ Z +∞
1
|fn (t)|dt = te−nt dt =
0 0 n2
X 1
Avec converge, on en déduit, par le théorème d’intégration terme à terme, que
n2
n>1

Z +∞ Z +∞ +∞
t X 1 π
dt = f (t)dt = = 2
0 et − 1 0 n=1
n 2 n

EXERCICE II
Si X est une variable aléatoire à valeurs dans N de loi de probabilité donnée par : ∀n ∈ N, pn = P (X = n), la fonction
génératrice de X est
+∞
 X
GX (t) = E tX = pn tn
n=0
X
Q2. — La série entière P (X = n)tn converge pour t = 1, donc, d’après le lemme d’Abel, elle est de rayon de
n>0
convergence RX > 1, ceci démontre que que l’intervalle ] − 1, 1[ est inclus dans l’ensemble de définition de
la fonction GX .
— Utilisation du produit de Cauchy:
X X
Les deux séries entières P (X1 = n)tn et P (X2 = n)tn ont des rayons de convergence supérieurs
n>0 n>0
ou égaux à 1, alors elles sont absolument convergentes sur ]−1, 1[. En conséquence la série entière, leur
XX n
produit de Cauchy, P (X1 = k)P (X2 = n − k)tn converge absolument convergente sur ]−1, 1[ et est
n>0 k=0
de somme
+∞ X
n +∞
! +∞
!
X X X
n n n
P (X1 = k)P (X2 = n − k)t = P (X1 = n)t P (X2 = n)t = GX1 (t)GX2 (t)
n=0 k=0 n=0 n=0

Or, par indépendance et positivité de X2 , on a:

elamdaoui@gmail.com 3/8 www.elamdaoui.com


Problème de mathématiques: MP Correction

INP-M1-2019

n
X n
X
P (X1 = k)P (X2 = n − k) = P (X1 = k, X2 = n − k) = P (S = n)
k=0 k=0
On obtient
+∞ X
X n +∞
X
n
P (X1 = k)P (X2 = n − k)t = P (S = n)tn = GS (t)
n=0 k=0 n=0

— Utilisation de l’espérance:
Comme X1 et X2 sont indépendantes, alors pour tout t ∈ ]−1, 1[, les deux variables tX1 et tX2 sont
indépendantes par le lemme des coalitions

GS (t) = E tX1 +X2 = E tX1 tX2 = E tX1 E tX2 = GX1 (t) × GX2 (t)
   

— Généralisation:
n
X
Soit X1 , · · · , Xn , n variables aléatoires mutuellement indépendantes à valeurs dans N. On note Sn = Xk ,
k=1
alors
n
Y
∀t ∈ ]−1, 1[ , GSn (t) = GXi (t)
k=1

Q3. Pour k ∈ [[1, n]], on pose Xk la variable aléatoire qui vaut le numéro de la boule tirée au k-ième tirage. Une
1 1
telle variable est de loi: Xk (Ω) = {0, 1, 2}, P (Xk = 0) = P (Xk = 2) = et P (Xk = 1) = . Les variables
4 2
n
X
X1 , · · · , Xn sont indépendantes, de même loi et Sn = Xk . D’après la généralisation précédente
k=1
n
Y
∀t ∈ ]−1, 1[ , GSn (t) = GXi (t) = GX1 (t)n
k=1
 2
1 1 1 1 1
Avec GX1 (t)n = + t + t2 = t+ , soit
4 2 4 2 2
 2n
1 1
∀t ∈ ]−1, 1[ , GSn (t) = t+
2 2
 
1
Donc Sn suit la loi binomiale de paramètre 2n,
2

PROBLÈME

Partie I: Propriétés

Q4. — Si x ∈] − 1, 1[ , on a 1 − xn ∼ 1
n→+∞
n
x X
— Soit x ∈] − 1, 1[ , on a an ∼ |an xn | et la série an xn converge absolument car elle est de
1 − xn n→+∞
n>1
X xn
rayon 1. Donc, par la critère de comparaison des séries à termes positifs, an converge absolument.
1 − xn
n>1
— On définit la suite (an ) par 
a2n = 0
1
a2n+1 =
(2n + 1)2
X X 1 X xn
La série lacunaire an xn = 2
x2n+1 est de rayon de convergence 1 et la série an
(2n + 1) 1 − xn
n>1 n>0 n>1
X −1
converge pour x0 = −1 car converge
2(2n + 1)2
n>0

elamdaoui@gmail.com 4/8 www.elamdaoui.com


Problème de mathématiques: MP Correction

INP-M1-2019

xn bn bn
Q5. Soit b ∈ [0, 1[ et x ∈ [−b, b], on a: an n
6 |an | n
. D’autre part |an | ∼ |an | bn et la
1−x 1−b 1 − bn n→+∞
X X xn
série an bn converge absolument car elle est de rayon 1. Donc la série de fonctions an converge
1 − xn
n>1 n>1
normalement, puis uniformément, sur le segment [−b, b]
+∞
X xn
Q6. On pose, pour tout x ∈] − 1, 1[, f (x) = an
n=1
1 − xn
xn
Continuité: Pour n ∈ N∗ , on pose fn : x ∈ ]−1, 1[ 7−→ an
1 − xn

— Pour tout n ∈ N , l’application fn est continue sur ]−1, 1[
X
— Soit [−b, b] ⊂ ]−1, 1[. D’après la question Q5, la série fn converge uniformément sur [−b, b]
n>1

Par le théorème de la contuinuité de la fonction somme f est continue sur ]−1, 1[


Régularité de C 1 :
— Pour tout n ∈ N∗ , l’application fn est de classe C 1 sur ]−1, 1[ et

nxn−1
∀x ∈ ]−1, 1[ , fn0 (x) = an 2
(1 − xn )

— Soit [−b, b] ⊂ ]−1, 1[


Pour x ∈ [−b, b] et n ∈ N∗ , on a:
n−1
n |x| nbn−1
|fn0 (x)| 6 |an | 2 6 |an | 2
(1 − xn ) (1 − bn )

nbn−1 X
D’autre part |an | 2 ∼ n |an | bn et la série dérivée nan bn converge absolument car elle
(1 − bn ) n→+∞ n>1
X X nxn−1
est de même rayon que an bn . Donc la série de fonctions an 2 converge normalement,
n>1 n>1
(1 − xn )
puis uniformément, sur le segment [−b, b].
Par le théorème de dérivation terme à terme la fonction f est de classe C 1 sur l’intervalle ] − 1, 1[.
+∞
X nxn−1
∀x ∈ ]−1, 1[ , f 0 (x) = an 2
n=1 (1 − xn )

En particulier f 0 (0) = a1
Q7. Expression sous forme de série entière
On note A = N ∗ × N∗ .
— Soit n ∈ N∗ l’élément (1, n) ∈ In , donc In 6= ∅
— Soit m, n ∈ N∗ tels que m 6= n. Si (p, q) ∈ In ∩ Im , alors n = pq = m, donc m = n. Absurde
[
— Pour tout n ∈ N∗ , on a In ⊂ A, donc In ⊂ A. Inversement si (p, q) ∈ A, on pose n = pq, donc il existe
n∈N∗
[ [

n ∈ N tel que (p, q) ∈ In , ainsi A ⊂ In . D’où In = A
n∈N∗ n∈N∗
On conclut que (In )n∈N∗ est une partition de A. Alors par le théorème de sommation par paquets
!  
+∞ X
X +∞ X +∞
X X
un,p = un,p =  uk,p 
n=1 p=1 (n,p)∈A n=1 (k,p)∈In

On montre que pour tout x ∈] − 1, 1[ , la famille (an xnp )(n,p)∈A est sommable.

elamdaoui@gmail.com 5/8 www.elamdaoui.com


Problème de mathématiques: MP Correction

INP-M1-2019

X
— Soit n ∈ N∗ , la série géométrique |an | |xnp | de raison |xn | ∈ [0, 1[ est convergente de somme
p>1
+∞ n
X |x|
|an | |xnp | = |an | n
p=1
1 − |x|
n
|x| n
X
— On a |an | n ∼ |an | |x| et la série an xn converge absolument car elle est de rayon 1. Donc
1 − |x| n→+∞
n>1
n
X |x|
|an | n converge.
1 − |x|
n>1
Donc par le théorème de Fubini, la famille la famille (an xnp )(n,p)∈A est sommable
— Déduction:
+∞ +∞ X
+∞
X xn X X
an = an xnp = an xnp
n=1
1 − xn n=1 p=1 (n,p)∈A
 
+∞
X X
=  ak xkp 
n=1 (k,p)∈In
 
+∞
X X
=  ak  xn
n=1 (k,p)∈In

+∞ +∞
X X X xn X X
Mais ak = ak , donc an n
= bn xn où bn = ak
n=1
1−x n=1
(k,p)∈In k|n k|n

Partie II: Exemple

Q8. D’après la question Q7, pour tout x ∈] − 1, 1[,


∞ +∞
X xn X
f (x) = = bn xn
n=1
1 − xn n=1

X +∞
X
Où bn = 1 = dn . Ainsi f (x) = dn xn
k|n n=1

Q9. Dans cette question, pour n > 1, an = ϕ(n) où ϕ(n) est le nombre d’entiers naturels premiers avec n et inférieurs
à n.      
X X X
— Soit n ∈ N∗ , on a 1 6 ϕ(n) 6 n, donc Rc  nxn  6 Rc  an xn  6 Rc  xn .
n>1 n>1 n>1
     
X X X
Or Rc  nxn  = Rc  nxn  = 1, donc Rc  an xn  = 1
n>1 n>1 n>1

— L’ensemble des diviseurs entiers de 12 est D12 = {1, 2, 3, 4, 6, 12} et par définition

ϕ(1) = ϕ(2) = 1


ϕ(3) = ϕ(4) = ϕ(6) = 2


ϕ(12) = 4

X
On a bien ϕ(d) = 12
d|12
— Soit x ∈] − 1, 1[, d’après la question Q7, on a
+∞ +∞ X +∞
X xn X
n
X
ϕ(n) = ϕ(d)x = nxn
n=1
1 − xn n=1 n=1
d|n

elamdaoui@gmail.com 6/8 www.elamdaoui.com


Problème de mathématiques: MP Correction

INP-M1-2019

+∞ +∞
X X x x
Or nxn = x nxn−1 = 2
, alors f (x) =
n=1 n=1
(1 − x) (1 − x)2
Q10. La fonction ln(1 + x) est développable en série entière sur ] − 1, 1[ et on a :
+∞
X (−1)n−1 n
∀x ∈] − 1, 1[, ln(1 + x) = x
n=1
n

X (−1)n−1
— La série xn converge simplement sur [0, 1[
n
n>1

— Pour tout x ∈ [0, 1[ et n ∈ N∗ , on a


+∞
X (−1)k−1 k xn+1 1
x 6 6 −−−−−→ 0
k n+1 n + 1 n→+∞
k=n+1
 
1
La suite est de réels positifs, indépendante de x, et de limite nulle, donc la suite de fonctions
n+1 n>1
X (−1)n−1
des restes de la série xn converge uniformément vers 0̃
n
n>1

(−1)n−1 n (−1)n−1
— Pour tout n ∈ N∗ , on a x −−−−−→ ∈R
n n→+∞ n
Par le théorème de la double limite
+∞ +∞
X (−1)n−1 n X (−1)n−1
ln(2) = lim x =
x→1−
n=1
n n=1
n

+∞
X (−1)n
Soit = − ln(2)
n=1
n
Q11. Soit b ∈ [0, 1[ et x ∈ [−b, b] \ {0}.
xn−1 bn−1 bn−1 X
— On a: an 6 |an | . D’autre part |an | ∼ |an | bn−1 et la série an bn−1 converge
1 − xn 1 − bn 1 − bn n→+∞
n>1
X xn−1
absolument car elle est de rayon 1. Donc la série de fonctions an converge normalement, puis
1 − xn
n>1
uniformément, sur [−b, b] \ {0}
(
∗ xn−1 a1 si n = 1
— Pour tout n ∈ N , on a an −−−−−→ . Par le théorème de la double limite
1 − xn n→+∞ 0 si n > 1

+∞
f (x) X xn−1
lim = lim an = a1 = −1
x→0 x x→0
n=1
1 − xn

Donc f (x) ∼ −x
x→0
— On conclut que f est dérivable en 0 et f 0 (0) = −1
+∞
X xn
Q12. Pour tout x ∈ [0, 1[, on a (1 − x) f (x) = (1 − x) (−1)n .
n=1
1 − xn
— Soit n > 1 et x ∈ [0, 1[ , on a

xn+1 xn xn (x − 1)
− = 60
1 − xn+1 1 − xn (1 − xn+1 ) (1 − xn )

xn X xn
et n
−−−−−→ 0. Donc la série (−1)n (1 − x) est alternée vérifiant le critère spécial des séries
1 − x n→+∞ 1 − xn
n>1
alternées, alors elle converge

elamdaoui@gmail.com 7/8 www.elamdaoui.com


Problème de mathématiques: MP Correction

INP-M1-2019

— Pour tout x ∈ [0, 1[ et n ∈ N∗ , on a


+∞
X xn xn+1 xn+1
(−1)n (1 − x) 6 (1 − x) = n
1 − xn 1 − xn+1 X
k=n+1 xk
k=0

xn+1
En introduisant l’application ψn définie sur [0, 1[, par ψn (x) = n . Une telle fonction est de classe C 1 ,
X
k
x
k=0
par les théorèmes généraux, et ∀x ∈ [0, 1[:
n
X n
X
(n + 1)xn xk − xn+1 kxk−1
k=0 k=1
ψn0 (x) = !2
n
X
xk
k=0
n
X
(n + 1)xn + (n + 1 − k)xn+k
k=1
= !2 >0
n
X
xk
k=0

1
Donc ψn est croissante sur [0, 1[, avec ψn (0) = 0 et lim− ψn (x) = , on gagne
x→1 n+1
+∞
X xn 1
(−1)n (1 − x) 6
1 − xn n+1
k=n+1
 
1
La suite est de réels positifs, indépendante de x, et de limite nulle, donc la suite de fonctions
n+1 n>1
X xn
des restes de la série (−1)n (1 − x) converge uniformément vers 0̃
1 − xn
n>1

xn n x
n
(−1)n
— Pour tout n ∈ N∗ , on a (−1)n (1 − x) = (−1) −
− −−−→ ∈ R.
1 − xn n−1
X n→+∞ n
k
x
k=0
Par le théorème de la double limite
+∞
X (−1)n
lim− (1 − x)f (x) = = − ln(2)
x→1
n=1
n

− ln 2
Ainsi au voisinage de 1, f (x) ∼ .
1−x

elamdaoui@gmail.com 8/8 www.elamdaoui.com


Enoncé du concours CCP 2019 maths1 - 30 / 4 /2019

EXERCICE I
2
+∞
X 1 π t e −t
On admet que 2
= et on pose, pour t ∈]0, +∞[, f (t ) = .
n=1 n 6 1 − e −t
1) Justifier que la fonction f est intégrable sur ]0, +∞[ puis, Z +∞à l’aide d’un
t
théorème d’intégration terme à terme, calculer l’intégrale t −1
dt.
0 e
EXERCICE II
Si X est une variable aléatoire à valeurs dans N de loi de probabilité donnée
par : ∀n ∈ N, p n = P (X = n), la fonction génératrice de X est G X (t ) = E (t X ) =
+∞
pn t n .
X
n=0
2) Démontrer que l’intervalle ]−1, 1[ est inclus dans l’ensemble de définition
de la fonction G X .
Soient X 1 et X 2 deux variables aléatoires indépendantes à valeurs dans N.
On pose S = X 1 +X 2 , démontrer que pour tout t ∈]−1, 1[, G S (t ) = G X 1 (t )G X 2 (t )
par deux méthodes : l’une utilisant le produit de Cauchy de deux séries en-
tières et l’autre utilisant uniquement la définition : G X (t ) = E (t X ).
On généralise ce résultat, que l’on pourra utiliser dans la question sui-
vante, à n variables aléatoires mutuellement indépendantes à valeurs dans
N ( on ne demande pas de preuve de cette récurrence).
3) Un sac contient quatre boules : une boule numérotée 0, deux boules nu-
mérotées 1 et une boules numérotée 2.
On effectue n tirages d’une boule avec remise et on note S n la somme des
numéros tirés.
déterminer pour tout t ∈] − 1, 1[, G S n (t ) et en déduire la loi de S n .
PROBLEME
X xn
Dans ce sujet une série de fonctions L a est une séries de fonctions an n
Xn≥1 n1 − x
où (a n )n≥1 est une suite de nombres réels telle que la série entières a n x soit
n≥1
de rayon 1.
Partie I : Propriétés
X xn
Soit une une séries de fonctions L a : an
n≥1 1 − xn
4) Si x ∈] − 1, 1[, donner un équivalent de 1 − x n pour n au voisinage de +∞.
X xn
Démontrer que pour tout x ∈] − 1, 1[, la série an converge abso-
n≥1 1 − xn
lument.
Remarque : la série peut parfois converger en dehors de l’intervalle ]−1, 1[.
Donner un exemple de suite (a n )n≥1 telle que L a converge en au mois un
x 0 n’appartient pas à l’intervalle ] − 1, 1[.

1
xn X
5) Démontrer que la série de fonctions an
converge uniformément
n≥1 1 − xn
sur tout segment [−b, b] inclus dans l’intervalle ] − 1, 1[.
+∞
X xn
6) On pose pour tout x ∈] − 1, 1[, f (x) = an .
n=1 1 − xn
Justifier que la fonction f est continue sur ] − 1, 1[ et démontrer ensuite
que la fonction f est de classe C 1 sur l’intervalle ] − 1, 1[. Donner la valeur
de f 0 (0).
7) Expression sous forme de série entière
On note A = N∗ × N∗ .
Lorsque (u n,p )(n,p)∈A est une famille sommable de nombres réels, justifier
que
à ! à !
+∞
X +∞
X +∞
X X
u n,p = u k,p , où I n = {(k, p) ∈ A, kp = n}
n=1 p=1 n=1 (k,p)∈I n

Démontrer que pour tout x ∈] − 1, 1[, la famille (a n x np )(n,p)∈A est som-


mable.
+∞ xn +∞
b n x n où b n =
X X X
En déduire que x ∈] − 1, 1[, an n
= ad
n=1 1−x n=1 d /n
( d /n signifiant d divise n).
Partie II : Exemples
8) Dans cette question, pour n ≥ 1, a n = 1 et on note d n le nombre de divi-
+∞
X xn
seurs de n. Exprimer, pour tout x ∈] − 1, 1[, f (x) = an comme la
n=1 1 − xn
somme d’une série entière.
9) Dans cette question, pour n ≥ 1, a n = ϕ(n) où ϕ(n) est le nombre d’entiers
naturels premiers avec n et inférieurs à n.
a n x n est de rayon 1.
X
Justifier que la série entière
n≥1
ϕ(d ). Vérifier ce résultat pour n = 12.
X
On admet que pour n ≥ 1, n =
d /n
+∞ xn
ϕ(n)
X
Pour x ∈] − 1, 1[, exprimer sous la forme d’un quotient de
n=1 1 − xn
deux polynômes.
10) En utilisant le théorème de la double limite, établir à l’aide du développe-
ment en série entière de la fonction x 7−→ ln(1 + x) sur l’intervalle ] − 1, 1[,
X (−1)n
+∞
la valeur de la somme .
n=1 n
11) Dans cette question et la suivante, pour n ≥ 1, a n = (−1)n et pour tout
+∞
X xn
x ∈] − 1, 1[, f (x) = an .
n=1 1 − xn

2
f (x)
En utilisant le théorème de la double limite, calculer lim et donner
x→0 x
un équivalent de f (x) au voisinage de 0. Retrouver le dernier résultat de la
question Q6.
− ln(2)
12) Démontrer qu’au voisinage de 1, f (x) ∼ .
1−x
1−x 1
On pourra remarquer que pour x ∈] − 1, 1[, n
= .
1−x 1 + x + x + ... + x n−1
2

3
Un corrigé du concours CCP 2019 maths1 - 30 / 4 /2019

EXERCICE I

1) • f est continue sur ]0, +∞[.


• f est prolongeable en 0 et f (t ) = o(1/t 2 ), donc f est intégrable sur
+∞
]0, +∞[.
t e −t
Z +∞ Z +∞ Z +∞ +∞
t X −(n+1)t
• t
dt = −t
dt = te dt.
0 e −1 0 1−e 0 n=0
Considérons la suite de fonctions ( f n )n≥0 définie par f n (t ) = t e −(n+1)t .
• chaque f n est continue et intégrable sur [0, +∞[ car f n (t ) = o(1/t 2 ).
+∞
• la série de fonctions converge simplement sur [0, +∞[ vers g définie par
g (t ) = f (t ) si t ∈]0, +∞[ et g (0) = 0.
g est continue sur ]0, +∞[ et admet une limite finie en 0+ , donc g est CM
sur [0, +∞[.
• par changement de variable u = (n + 1)t , on a :
Z +∞ Z +∞
1 1 1
| f n (t )|d t = 2
ue −u d u = 2
Γ(2) = .
0 (n + 1) 0 (n + 1) (n + 1)2
X 1 X +∞ Z
La série 2
converge donc la série | f n (t )|d t converge. le
n≥0 (n + 1) n≥0 0
Théorème ITT s’applique et :
+∞ +∞ π2
Z +∞ X Z +∞
t X 1
d t = f n (t )d t = = .
0 et − 1 n=0 0 n=0 (n + 1)
2 6
EXERCICE II

∈ N, |p n t n | ≤ p n , or la série
X
2) Soit t ∈] − 1, 1[, alors ∀nX p n converge de
somme 1, donc la série p n t n converge absolument donc convergente.
alors t ∈ D G X , donc ] − 1, 1[⊂ D G X .
Soit t ∈] − 1, 1[, alors G S (t ) = E (t X 1 +X 2 ) = E (t X 1 t X 2 ) = E (t X 1 )E (t X 2 ) car les
variables X 1 et X 2 sont indépendantes donc les variables t X 1 et t X 2 sont
indépendantes aussi.
Donc G S (t ) = G X 1 (t )G X 2 (t ).
Autre méthode :
+∞
G S (t ) = E (t X 1 +X 2 ) = t s P (X 1 + X 2 = s) par la propriété de transfert.
X
s=0
+∞
ts
X X
Alors G S (t ) = P (X 1 = n)P (X 2 = m) Car ces variables X 1 et X 2
s=0 n+m=s
sont indépendantes,
+∞
X X n
alors : G S (t ) = t P (X 1 = n)t m P (X 2 = m)
s=0 n+m=s

4
t n P (X 1 = n) et t m P (X 2 = m) ont un rayon de
X X
Les séries entières
n≥0 m≥0
convergence au mois égal à 1, par application du théorème produit de
cauchy de deux séries entières, il en résulte :
+∞
X n +∞
X n
G S (t ) = t P (X 1 = n) t P (X 2 = n) = G X 1 (t )G X 2 (t ).
n=0 n=0
3) On peut écrire ici S n = X 1 +X 2 +....+X n où chaque X i représente la variable
aléatoire égal au numéro tirée pendant le i-ème tirage. Ces variables sont
indépendantes car le tirage est avec remise, et les variables sont tous à
valeurs dans {0, 1, 2},
Soit i ∈ {1, ..., n} et t ∈] − 1, 1[, alors G X i (t ) = t 0 p 0 + t 1 p 1 + t 2 p 2
1 2 1 1
On a p 0 = , p 1 = = et p 2 = , par application de ce qui précède :
4 4 2 4
2 ¸n
à !
¤n
·
1 t t 1 2n 1 2n
2n
tk.
£ X
G S n (t ) = G X 1 (t ) = + + = n (t + 1) = n k
4 2 4 4 k=0 4
2n
P (S n = k)t k et avec S n (Ω) = {0, 1, ..., (2n)}.
X
Mais G S n (t ) =
k=0
à !
1 2n
Alors ∀k ∈ {0, 1, ..., 2n}, P (S n = k) = n ainsi la loi de S n .
4 k
PROBLEME

4) Soit x ∈] − 1, 1[. On a lim x n = 1, donc 1 − x n ∼ 1.


n→+∞ n→+∞
n
|a n x |
Alors ∀n ∈ N∗ , ∼ |a n x n |, or le rayon de convergence de la sé-
1 − x n n→+∞
a n x n est 1, donc la série a n x n cva et par comparaison, la série
X X
rie
n≥1
X an x n
n
cva.
n≥1 1 − x
1 X 1 2n
Pour la remarque : Si on prend a n = , la série n
(n + 1)2 2
n≥0 (n + 1) 1 − 2
n
1 2 −1 X 1
converge car ∼ et converge.
(n + 1)2 1 − 2n n→+∞ n 2 n2
|a n x n |
5) Soit x ∈ [−b, b], alors ∀n ∈ N∗ , 0 < 1 − b n ≤ 1 − x n , donc ∀n ∈ N∗ , ≤
n n
1 − xn
|a n b | X an b
, la série converge absolument par Q4), donc la série de
1 − bn 1 − b n
X xn
fonctions a n converge normalement donc uniformément sur [−b, b].
1 − xn
xn X
6) Soit f n (x) = a n n
. Les f n sont continues sur ]−1, 1[, la série f n converge
1−x
uniformément sur chaque segment [−b, b] ⊂] − 1, 1[, donc f est continue
sur ] − 1, 1[.
nx n−1
Chaque f n est de classe C 1 sur ] − 1, 1[. ∀x ∈] − 1, 1[, f n0 (x) = a n .
(1 − x n )2

5
Soit b ∈ [0, 1[, alors par le même raisonnement fait en Q)5 ∀x ∈ [−b, b]; | f n0 (x)| ≤
|na n |b n−1 X na n b n−1 na n b n−1
, comme la série converge car un équivalent à
(1 − b n )2 (1 − b n )2 (1 − b n )2
n−1
et que le rayon de convergence de a n x n est
X
quand n → +∞X est na n b
n
égal à celui de na n x .
X 0
Donc la série f n converge
X normalement donc uniformément sur tout
[−b, b] ⊂]−1, 1[ et la série f n déjà converge simplement sur ]−1, 1[, alors
+∞ nx n−1
f est de classe C 1 sur ] − 1, 1[ et ∀x ∈] − 1, 1[, f 0 (x) =
X
an .
n=1 (1 − x n )2
Alors f 0 (0) = a 1 .
7) • Tout revient à montrer que (I n )n∈N∗ forment une partition de A.
[
Il est évident que chaque I n ⊂ A, donc I n ⊂ A.
n∈N∗
[ [
Soit (k, p) ∈ A, il est clair que (k, p) ∈ I kp ⊂ I n , alors I n = A.
n∈N∗ n∈N∗
\
Si on suppose que ∃(k, p) ∈ I n I m , alors kp = n = m, donc I n = I m , donc
(I n )n∈N∗ forment une partition de A.
La famille (u n,p )(n,p)∈A est sommable, par le théorème de sommation par
paquets on a :
à ! à !
+∞
X +∞
X +∞
X +∞
X
u n,p = u k,p
n=1 p=1 n=1 (k,p)∈I n

• Soit x ∈] − 1, 1[ et n ∈ N∗ , la série a n x np converge absolument et


X
p≥1
+∞ |x|n |x|n
|a n ||x|np = |a n |
X X
et la série |a n | converge par Q)4, donc
p=1 1 − |x|n 1 − |x|n
la famille donnée est sommable, en appliquant ce qui précède à u n,p =
a n x np :
X +∞
+∞ +∞
a n x np = a k x kp .
X X X
n=1 p=1 n=1 (k,p)∈I n
+∞ +∞ +∞ +∞
a k x kp = xn xn bn x n .
X X X X X X X
Or ak = ad =
n=1 (k,p)∈I n n=1 (k,p)∈I n n=1 d /n n=1
+∞
X +∞ +∞ n
x
a n x np =
X X
Et on a an série géométrique.
n=1 p=1 n=1 1 − xn
+∞ xn +∞
bn x n .
X X
Donc an =
n=1 1 − xn n=1
X
8) Ici a n = 1, donc le b n de la question 7 est b n = 1 = d n , par application
d /n
de la question 7) on a :
+∞ xn +∞
dn x n .
X X
f (x) = an n
=
n=1 1 − x n=1

6
9) Ici ∀n ∈ N∗ , a n = ϕ(n) = Card{k ∈ [[1, n]] / kX
∧ n = 1}. Donc ∀n ∈ N∗ , 1 ≤
a n ≤ n, par comparaison le rayon de la série a n x n est 1.
On a les diviseurs de 12 sont 1, 2, 3, 4, 6 et 12, or ϕ(1) = 1, ϕ(2) = 1, ϕ(3) = 2,
ϕ(4) = 2, ϕ(6) = 2 et ϕ(12) = 4 l’égalité est donc vraie pour n = 12.
+∞ xn +∞
ϕ(n) bn x n ,
X X
Soit x ∈]−1, 1[. Par application de la question 7), n
=
n=1 1 − x n=1
+∞ xn +∞
n
+∞
ϕ(d ) = n, alors ϕ(n) nx n .
X X X X
avec ici b n = n
= bn x =
d /n n=1 1 − x n=1 n=1
1 +∞
X n +∞ x
nx n =
X
Or ∀x ∈] − 1, 1[, = x , en dérivant on obtient .
1 − x n=0 n=1 (1 − x)2
+∞ xn x
ϕ(n)
X
Alors ∀x ∈] − 1, 1[, n
= c’est ce qui est demandée.
n=1 1 − x (1 − x)2
+∞ xn
(−1)n
X
10) On a ∀x ∈ [0, 1[, − ln(1 + x) = .
n=1 n
x n (−1)n
1 est dans l’adhérence de [0, 1[, pour tout n ∈ N∗ , lim− (−1)n = ∈
x→1 n n
n
x xn
R, pour x ∈ [0, 1[ la série (−1)n
X
est une série alternée qui vérifie lim =
n≥1 n n→+∞ n
µ n¶
x
0 et la suite est décroissante, alors par CSSA :
n n
¯ ¯

∗ ¯
¯ +∞
kx ¯ x n+1 1 1
∀n ∈ N , ¯
X
(−1) ¯≤ ≤ or lim = 0, la conver-
¯k=n+1 k ¯ n +1 n +1 n→+∞ n + 1
xn
(−1)n
X
gence de la série de fonctions est uniforme, le théorème de la
n≥1 n
X (−1)n
+∞
double limite s’applique et on a − ln 2 = .
n=1 n
11) Soit a ∈]0, 1[. On a ∀x ¯∈ [−a, a], ∀n ¯∈ N∗ 0 < 1 − a n ≤ 1 − x n , donc :
¯ x k−1 ¯¯ a k−1
∀x ∈ [−a, a], ∀k ∈ N∗ ¯(−1)k ≤ .
¯
1 − xk ¯ 1 − ak
¯
¯
X a k−1 a k−1
a k−1 et la série a k−1
X
Or la série k
converge car ∼
k≥1 1 − a 1 − ak k→+∞
k−1
k x
X
converge, la convergence de (−1) est donc uniforme sur [−a, a].
k≥1 1 − xk
x k−1
½
k −1 si k = 1
De plus lim (−1) =
x→0 1 − xk0 si k 6= 1
le théorème de la double limite s’applique et on a :

f (x) +∞ x n−1
(−1)n
X
lim = lim = −1
x→0 x x→0 n=1 1 − xn

Un équivalent de f (x) quand x → 0 est −x.

7
f (x) f (x) − f (0)
On a f (0) = 0, donc = , alors f 0 (0) = −1 = a 1 c’est ce qu’on
x x −0
a trouver à la question 6).
+∞ x n−1
12) Toujours a n = (−1)n et f (x) = (−1)n
X
n=1 1 − xn
+∞ (1 − x) +∞ x n−1
(−1)n x n−1 n
X X
Donc (1 − x) f (x) = = (−1) .
n=1 1 − x n n=1 1 + x + x 2 + ... + x n−1
x n−1
Soit ∀n ∈ N∗ ; g n (x) = (−1)n .
1 + x + x 2 + ... + x n−1
le 1 est dans l’adhérence de [0, 1[,
(−1)n
∀n ∈ N∗ , lim− g n (x) = ,
x→1 n
Soient x ∈ [0, 1[ et k ∈ [[0, (n − 1)]], on a x n−1 ≤ x k :
Donc nx n−1 ≤ 1 + x + x 2 + ... + x n−1 ,
x n−1 1
Alors ∀x ∈]0, 1[; ∀n ∈ N∗ ; |g n (x)| ≤ n−1
≤ .
nx µ n ¶
n−1 1
Soit x ∈]0, 1[. Les deux suites (x )n∈N∗ et sont
1 + x + x 2 + ... + x n−1 n∈N∗
décroissantes et elles sont positives, donc la suite (|g n (x)|)n∈N∗ est décrois-
sante comme
¯ elle est¯ positive et tend vers 0, le CSSA s’applique et on a
¯ +∞ ¯ 1
∀n ∈ N; ¯
X
g k (x)¯ ≤ |g n+1 (x)| ≤ −→ 0.
¯ ¯
¯k=n+1 ¯ n + 1 n→+∞
X
Alors la série de fonctions g n converge uniformément sur ]0, 1[, le théo-
n≥1
rème de la double limite s’applique et on a ;
+∞
X X (−1)n
+∞
lim− (1 − x) f (x) = lim− g n (x) = = − ln 2 d’après la question
x→1 n=1 x→1 n=1 n
10).
− ln 2
Alors (1 − x) f (x) ∼ − − ln 2, qui s’écrit f (x) ∼ − .
x→1 x→1 (1 − x)

Pour les remarques sadikoulmeki@yahoo.fr


Omar SADIK CPGE My Driss Fès.

8
E3A PSI 2018 mathématiques 2

Durée 3 h

Dans tout le problème, I est l’intervalle [1; +∞[ .


On note E le R-espace vectoriel des fonctions continues et bornées sur I à valeurs réelles, et E1 = C 1 (I, R) le
R-espace vectoriel de fonctions de classe C 1 sur I à valeurs réelles.
Lorsque V est un endomorphisme de E , on rappelle que V 0 = IdE et que si n est un entier naturel non nul,
n
V =V | ◦ ·{z
· · ◦ V}.
n
Soit a un réel strictement positif .
Pour tout f de E , on considère l’équation différentielle sur I :

y 0 − ay + f = 0 (Eaf )

et on note Saf l’ensemble de ses solutions sur I.

Partie 1
1. Étude de l’équation (Eaf ).
1.1. Soient f ∈ E et z ∈ E1 .
Montrer que z est solution de (Eaf ) si et seulement s’il existe K ∈ R tel que :
 Z x 
ax −at
∀ x ∈ I, z(x) = e K− e f (t) dt
1

1.2. Prouver que s’il existe une solution de (Eaf ) qui soit bornée sur I, alors celle-ci est unique.
Z +∞
1.3. Vérifier que l’intégrale e−at f (t) dt est convergente.
1
Z +∞
1.4. Démontrer que la fonction F : x ∈ I 7→ e ax
e−at f (t) dt est l’unique solution de (Eaf ) bornée sur I.
1

On définit ainsi une application Ua de E dans E qui à toute fonction f de E associe la fonction F = Ua (f ) ainsi
obtenue.
2. Étude de quelques propriétés de Ua .
2.1. Expliciter Ua (f ) lorsque f est la fonction constante égale à 1.
2.2. Vérifier que Ua est un endomorphisme de E .
2.3. a. L’endomorphisme Ua est-il injectif ?
b. Montrer que pour tout f élément de E , Ua (f ) ∈ E1 .
c. L’endomorphisme Ua est-il surjectif ?
2.4. On suppose dans cette question et uniquement dans cette question que a = 1.
Montrer que le sous-espace de E : F = Vect(sin, cos) est stable par U1 . En donner une base B.
Écrire la matrice M de la restriction de U1 à F dans cette base.
Prouver que M = λΩ où λ est un réel positif et Ω une matrice de rotation dont on déterminera l’angle.
3. On revient au cas général.
3.1. Pour r ∈ [0; +∞[ , on note fr la fonction de E définie par : x 7→ e−rx .
Déterminer Ua (fr ).
 
1
3.2. Soit λ ∈ 0; . Le réel λ est-il valeur propre de l’endomorphisme Ua ?
a
3.3. Étudier la convergence simple de la suite de fonctions (Uan (fr ))n∈N sur I.

1
X
3.4. Étudier la convergence simple de la série de fonctions Uan (fr ) sur I et déterminer sa somme lorsqu’elle
n>0
converge.
4. Prouver que l’on a, pour tout élément f de E :
Z +∞
∀ x ∈ I, Ua (f )(x) = e−at f (x + t) dt
0

5. Pour tout entier naturel k, on note gk la fonction de E définie par : gk (x) = e−x xk et on note Gk = Ua (gk ).
Pour tout entier naturel p, on note Fp = Vect(g0 , . . . , gp ).
5.1. Donner une base Bp de Fp .
5.2. Vérifier que Fp est un sous-espace vectoriel de E stable par Ua .
5.3. Calculer le déterminant de la restriction de Ua à Fp .
6. Prouver que l’on a : ∀ f ∈ E , |Ua (f )| 6 Ua (|f |).
7. Soit f dans E à valeurs positives. En est-il de même pour Ua (f ) ?
8. Soit f dans E décroissante. Prouver que aUa (f ) 6 f puis que Ua (f ) est décroissante.
9. On note :
• H l’ensemble des éléments de E de classe C 1 sur I et tels que f 0 est bornée sur I.
• D l’opérateur de dérivation sur H .
Soit f ∈ H .
9.1. Montrer que l’on a : Ua (f 0 ) − aUa (f ) + f = 0.
9.2. En déduire que Ua et D commutent dans H .
+∞
(t − x)n −at
Z
10. Soit f ∈ E . Vérifier que pour tout entier naturel n, Uan+1 (f ) est la fonction x 7→ eax e f (t) dt.
x n!
On pourra procéder par intégration par parties.
11. Soit f ∈ E . On suppose dans cette question et uniquement dans cette question que a > 1.
+∞ 
(t − x)n
X 
−at
11.1. Soient x ∈ I et t un réel supérieur ou égal à x. Calculer e f (t) .
n!
n=0
X
11.2. Démontrer que la série de fonctions Ua (f ) est simplement convergente sur I. On notera S sa somme.
n>1
Z +∞
On pourra utiliser sans démonstration le résultat valable pour tout entier naturel n : un e−u du = n!
0
11.3. Démontrer qu’il existe un réel b > 0 tel que S = Ub (f ).

Partie 2
On admettra que :
Z +∞
si u et v sont deux fonctions continues sur I à valeurs réelles tels que v est à valeurs positives et v(t) dt
1
converge
Z +∞ Z +∞ 
si u = o (v), alors u(t) dt = o v(t) dt
+∞ x x→+∞ x

1. Soient f et g dans E avec g à valeurs positives et f = o (g). Montrer que Ua (f ) = o (Ua (g)).
+∞ +∞
2. Soient f et g dans E , g à valeurs positives telles que f ∼ g. Montrer que Ua (f ) ∼ Ua (g).
+∞ +∞
3. Soit f ∈ E admettant une limite finie en +∞. Montrer que Ua (f ) admet aussi une limite finie en +∞.
On pourra commencer par étudier le cas où lim f (x) = 0
x→+∞

2
4. Pour tout réel strictement positif ω, on note pour toute la suite du problème, hω la fonction de E qui à t ∈ I
1
associe ω et Hω = Ua (hω ).
t
hω (x) ω
4.1. Montrer que l’on a pour tout x ∈ I : Hω (x) = − Hω+1 (x).
a a
hω (x)
4.2. En déduire que : Hω (x) ∼ .
x→+∞ a
Z x −at +∞
e X ak
5. 5.1. Montrer que pour tout x ∈ I : dt = ln x + (−1)k (xk − 1).
1 t k · k!
k=1
+∞
!
ak +∞
e−at
X Z
ax k
5.2. En déduire que l’on a : H1 (x) = e − ln x − (−1) (xk − 1) + dt .
k · k! 1 t
k=1

Partie 3
On reprend les fonctions fr définies à la question 3.1 de la partie 1 avec maintenant r > 0.
Z +∞
1. Montrer que l’intégrale Ua (fr )(t) dt converge.
1
Z +∞
2. Pour les fonctions hω définies à la question 4 de la partie 2, l’intégrale Hω (t) dt est-elle convergente ?
1
Z +∞
3. Soit f ∈ E , à valeurs positives et telle que f (t) dt converge.
Z x 1 Z x
On note ϕ : x ∈ I 7→ f (t) dt, F = Ua (f ) et Φ : x ∈ I 7→ F (t) dt.
1 1

3.1. Vérifier que l’on a pour tout x ∈ I : Φ0 (x) − aΦ(x) + ϕ(x) − F (1) = 0.
3.2. Prouver que ϕ ∈ E .
Z +∞
3.3. En déduire que l’intégrale F (t) dt converge.
1

4. Soit f ∈ E intégrable sur I.


Montrer que Ua (f ) est aussi intégrable sur I.

3
147

CONCOURS ARTS ET MÉTIERS ParisTech - ESTP - POLYTECH

Épreuve de Mathématiques 2 PSI

Durée 3 h

Si, au cours de l’épreuve, un candidat repère ce qui lui semble être une erreur d’énoncé,
d’une part il le signale au chef de salle, d’autre part il le signale sur sa copie et poursuit sa
composition en indiquant les raisons des initiatives qu’il est amené à prendre.

L'usage de

L’usage de calculatrices est interdit.

AVERTISSEMENT

La présentation, la lisibilité, l’orthographe, la qualité de la rédaction, la clarté et la


précision des raisonnements entreront pour une part importante dans
l’appréciation des copies. En particulier, les résultats non justifiés ne seront pas pris
en compte. Les candidats sont invités à encadrer les résultats de leurs calculs.

Tournez la page S.V.P.

Il est interdit aux candidats de signer leur composition ou d’y mettre un signe quelconque pouvant indiquer sa provenance.
I [1, +∞[
E R I E1 = C 1 (I, R) R
1
C I
V E V0 = E n Vn =V
� ◦ ��
... ◦ V�
n

a
f E I

y′ − a y + f = 0 (Eaf )

Saf I

(Eaf )

f ∈E z ∈ E1
( ∫ x )
−at
z (Eaf ) K∈R ∀ x ∈ I z(x) = ax
K− f (t) t
1

(Eaf ) I
∫ +∞
−at
f (t) t
1
∫ +∞
ax −at
F : x ∈ I �→ f (t) t (Eaf ) I
x

Ua E E f E F = Ua (f )

Ua

Ua (f ) f

Ua E

Ua

f E Ua (f ) ∈ E1

Ua

a=1
E F = (sin, cos) U1 B
M U1 F
M = λΩ λ Ω

−rx
r ∈ [0, +∞[ fr E x �→
Ua (fr )
] ]
1
λ ∈ 0, λ Ua
a

(Uan (fr ))n∈N I



Uan (fr ) I
n0

f E
∫ +∞
−at
∀ x ∈ I, Ua (f )(x) = f (x + t) t
0

−x
k gk E gk (x) = xk Gk = Ua (gk )
p Fp = (g0 , ..., gp )

Bp Fp

Fp E Ua

Ua Fp

∀ f ∈ E | Ua (f ) |  Ua ( |f | )

f E Ua (f )

f E a Ua (f )  f Ua (f )

•H E C1 I f′ I
•D H
f ∈H

Ua (f ′ ) − a Ua (f ) + f = 0

Ua D H
∫ +∞
(t − x)n −at
f ∈E n Uan+1 (f ) x → ax
f (t) t
x n!

f ∈E a>1
+∞ (
∑ )
(t − x)n −at
x∈I t x f (t)
n=0
n!

Uan (f ) I S
n1
∫ +∞
−u
n un u = n!
0

b>0 S = Ub (f )

Tournez la page S.V.P.


∫ +∞
u v I v v(t) t
1
∫ +∞ (∫ +∞ )
u = o (v) u(t) t = o v(t) t
+∞ x x→ +∞ x

f g E g f = o (g) Ua (f ) = o (Ua (g))


+∞ +∞

f g E g f ∼ g Ua (f ) ∼ Ua (g)
+∞ +∞

f ∈E +∞ Ua (f ) +∞
lim f (x) = 0
x→+∞

1
ω hω E t∈I

Hω = Ua (hω )
hω (x) ω
x∈I Hω (x) = − Hω+1 (x)
a a
hω (x)
Hω (x) ∼
x→+∞ a

∫ x −at +∞
∑ ak
x∈I t = ln x + (−1)k (xk − 1)
1 t k.k!
k=1
( +∞ ∫ )
∑ ak +∞ −at
k
H1 (x) = ax
− ln x − (−1) (xk − 1) + t
k.k! 1 t
k=1

fr r>0
∫ +∞
Ua (fr )(t) t
1
∫ +∞

I M P R I M E R I E N A T I O N A L E – 18 1147 – D’après documents fournis


hω Hω (t) t
1
∫ +∞
f ∈E f (t) t
1
∫ x ∫ x
φ : x ∈ I → f (t) t F = Ua (f ) Φ : x ∈ I → F (t) t
1 1

x∈I Φ′ (x) − a Φ(x) + φ(x) − F (1) = 0

φ∈E
∫ +∞
F (t) t
1

f ∈E I
Ua (f ) I
E3A PSI 2018 mathématiques 2

corrigé

Partie 1
Dans toute cette partie, on pose, pour tout a ∈ R, fa : x 7→ e−ax (notation introduite en 3.1 du sujet).
1.1. En multipliant (Eaf ) par fa , et puisque fa0 = −afa , on a :

z 0 − az = −f ⇐⇒ fa z 0 − afa z = −fa f ⇐⇒ (fa z)0 = −fa f

donc z est solution de (Eaf ) si et seulement si fa z est une primitive de −fa f .


Rx
Or par le théorème fondamental du calcul intégral, la fonction H : x 7→ − 1 fa f est une primitive de −fa f sur
l’intervalle I, et les primitives de −fa f sur I sont égales entre elles à une constante près.
Donc z est solution de (Eaf ) si et seulement s’il existe K ∈ R tel que fa z = H + K, i.e. z = (H + K)f−a , qui est
le résultat voulu.
1.2. Si z1 et z2 sont deux solutions de (Eaf ), alors par 1.1, il existe K1 , K2 ∈ R tels que z1 = (H + K1 )f−a et
z2 = (H + K2 )f−a , donc tels que z1 − z2 = (K1 − K2 )f−a .
Si de plus z1 et z2 sont bornées sur I, alors z1 − z2 l’est aussi, or f−a ne l’est pas (car on a a > 0, donc fa tend
vers +∞ en +∞), donc nécessairement K1 = K2 , i.e. z1 = z2 .
Ainsi si (Eaf ) admet une solution bornée sur I, alors celle-ci est unique.
1.3. Par hypothèses sur f , la fonction fa f est continue sur I et dominée par fa en +∞. Or fa est intégrable sur I
car a > 0 (intégrales de référence), donc par comparaison, fa f l’est aussi.
R +∞
Autrement dit, l’intégrale 1 e−at f (t) dt converge absolument, donc elle converge.
1.4. • La
R +∞fonction F est bien une solution de (Eaf ) sur I puisqu’elle est de la forme donnée en 1.1, pour K =
1 e−at f (t) dt (par relation de Chasles), qui est bien une constante réelle par 1.3.
• Soit M un majorant de |f | sur I (M existe par hypothèse sur f ). Alors par croissance de l’intégrale, ∀ x ∈ I,
R +∞ R +∞ R +∞  −at +∞ −ax
|F (x)| 6 eax x e−at |f (t)| dt 6 M eax x e−at dt. Or x e−at dt = e−a x = e a .
M
Donc ∀ x ∈ I, |F (x)| 6 a . Ainsi, la fonction F est bornée sur I.
On a montré que F est une solution bornée de (Eaf ) sur I, et par 1.2, elle est unique.
2.1. Par la formule de 1.4 ou de façon évidente au vu de l’équation (Ea1 ), la solution bornée Ua (1) de (Ea1 ) est la
fonction constante égale à a1 .
2.2. • Pour tout f ∈ E , la fonction Ua (f ) est bornée et continue (même dérivable) sur I par construction (cf. 1.1 et
1.4). Donc l’application Ua va de E dans E .
• L’application Ua est linéaire par linéarité de l’intégrale. En effet, pour tous f, g ∈ E et λ ∈ R :
R +∞ R +∞ R +∞ R +∞
∀ x ∈ I, Ua (λf + g)(x) = eax x fa × (λf + g) = eax x (λfa f + fa g) = λeax ( x fa f + x fa g) =
λUa (f )(x) + Ua (g)(x), donc Ua (λf + g) = λUa (f ) + Ua (g).
On a montré que Ua est linéaire de E dans E , i.e. que Ua est un endomorphisme de E .
2.3. a. Soit f ∈ E . Par définition, Ua (f ) est une solution de (Eaf ), donc Ua (f )0 − aUa (f ) + f = 0. Ainsi si Ua (f ) = 0
(fonction nulle sur I), alors Ua (f )0 = 0, et donc f = 0.
Cela montre que Ker(Ua ) = {0}, i.e. que Ua est injectif.
b. Pour tout f ∈ E , Ua (f ) est dérivable sur I puisque c’est une solution sur I de l’équation différentielle d’ordre
un (Eaf ), et Ua (f )0 = aUa (f ) − f est continue comme combinaison linéaire de fonctions qui le sont. Donc
Ua (f ) est de classe C 1 sur I, i.e. Ua (f ) ∈ E1 .
c. On vient de montrer que Im(Ua ) ⊂ E1 . Ainsi, aucun élément de E \ E1 n’a d’antécédent par Ua . Comme
l’ensemble E \ E1 est non vide (il contient par exemple la fonction f définie par f (x) = x sur [1; 2] et f (x) = 2
sur [2; +∞[ , qui est continue et bornée sur I, mais pas dérivable en 2), cela montre que Ua n’est pas surjectif.
2.4. • Pour montrer que F = Vect(sin, cos) est stable par U1 , il suffit, vu la linéarité de U1 , de montrer que U1 (sin)
et U1 (cos) appartiennent à F .
Méthode 1. Avec la formule d’Euler cos(t) + i sin(t) = eit , valable pour tout t ∈ R, on a pour x ∈ I :

1
R +∞ R +∞
U1 (cos)(x) + i U1 (sin)(x) = ex x e−t cos(t) dt + i ex x e−t sin(t) dt
R +∞ R +∞
= ex x e−t eit dt = ex x e(i−1)t dt
 (i−1)t +∞ (i−1)x eix
= ex e i−1 x = ex e 1−i = 1−i = 12 eix (1 + i)
1 i
= 2 (cos(x) − sin(x)) + 2 (cos(x) + sin(x))
On en déduit par unicité des écritures algébriques des nombres complexes que U1 (cos) = 21 (cos − sin) et
U1 (sin) = 21 (cos + sin). D’où le résultat voulu.
Méthode 2. Par intégrations par parties dans lesquelles tous les termes convergent, on a pour x ∈ I :
+∞ R +∞
∗ U1 (sin)(x) = ex − e−t sin(t) x + x e−t cos(t) dt = sin(x) + U1 (cos)(x), et de même,
 

∗ U1 (cos(x) = · · · = cos(x) − U1 (sin)(x).


On en déduit que 2 U1 (sin) = sin + cos et 2 U1 (cos) = cos − sin.
Conclusion. Ainsi U1 (cos) et U1 (sin) appartiennent à F , et donc F est stable par U1 .
• La famille (sin, cos) est évidemment libre (car les fonctions sin et cos ne sont pas proportionnelles) et génératrice
de F (par définition de F ), donc la famille (sin, cos) est une base de F .
• Les calculs du premier point montrent que la matrice, dans la base (sin, cos), de l’endomorphisme de F induit
par U1 , est la matrice :
1 cos( π4 ) − sin( π4 )
   
1 1 −1
M= =√ π π .
2 1 1 2 sin( 4 ) cos( 4 )
Donc M est bien de la forme λΩ où λ = √12 > 0 et Ω est la matrice de la rotation d’angle π4 dans le plan
euclidien orienté usuel.
Remarques (problème d’énoncé).
— En prenant la base (cos, sin), on trouverait un angle de − π4 .
— Pire : en prenant une base plus exotique, comme (sin, 2 cos), ou (sin, sin + cos), la matrice M n’est pas de
la forme λΩ souhaitée ...
R +∞  −(a+r)t +∞ −(a+r)x −rx
3.1. Pour tout x ∈ I, Ua (fr )(x) = eax x e−(a+r)t dt = eax e−(a+r) x = eax e a+r = ea+r = a+r 1
fr (x).
1
Donc Ua (fr ) = a+r fr .
3.2. Tout λ ∈ ]0; a1 ] est de la forme λ = a+r
1
pour r = λ1 − a > 0, et par 3.1, on a alors Ua (fr ) = λfr .
Comme fr 6= 0, cela montre qu’un tel λ est valeur propre de Ua .
1
3.3. Par récurrence immédiate, pour tout n ∈ N, Uan (fr ) = (a+r)n fr .
1
Or a+r > 0 et pour tout x ∈ I, fr (x) 6= 0, on en déduit que :
1
• si a+r < 1, i.e. si a + r > 1, alors la suite (Uan (fr ))n∈N converge simplement vers 0 (fonction nulle),
• si a + r = 1, alors la suite (Uan (fr ))n∈N est constante égale à fr , donc converge simplement vers fr ,
1
• si a+r > 1, i.e. si a + r < 1, alors la suite (Uan (fr ))n∈N diverge.
1
Uan (fr ) converge simplement sur I si et seulement si
P
3.4. Vu 3.3, la série a+r < 1, i.e. a + r > 1.
Le cas échéant, on a alors +∞
P n
P+∞ 1 n 1 a+r
n=0 Ua (fr ) = n=0 ( a+r ) fr = 1− 1 fr = a+r−1 fr .
a+r

4. Soit x ∈ I. Le changement de variable u = t − x est valide dans l’intégrale définissant Ua (f )(x) puisque la
fonction affine t 7→ t − x réalise une bijection de classe C 1 de ]x; +∞[ sur ]0; +∞[ , et donne :
R +∞ R +∞ R +∞
Ua (f )(x) = eax x e−at f (t) dt = eax 0 e−a(x+u) f (x + u) du = 0 e−au f (x + u) du.
C’est le résultat voulu (en remplaçant u par t).
, gp ) est libre. Si (λ0 , . . . , λp ) ∈PRp+1 est tel que pk=0 λk gk = 0 (fonction
5.1. Montrons que la famille Bp = (g0 , . . . P
P
nulle sur I),
Ppalors pour tout x ∈ I, pk=0 λk xk e−x = 0, et donc pk=0 λk xk = 0 puisque e−x 6= 0. Ainsi le
k
polynôme k=0 λk X admet une infinité de racines (car I = [1; +∞[ est infini), donc c’est le polynôme nul, i.e.
λ0 = λ1 = · · · = λp = 0.
Ainsi Bp est libre, et comme elle engendre Fp par définition, c’en est une base.
5.2. Comme en 2.4, pour montrer que Fp = Vect(g0 , . . . , gp ) est stable par Ua , il suffit, vu la linéarité de Ua , de
montrer que pour tout k ∈ [[0; p]], Ua (gk ) ∈ Fp . Or pour tout x ∈ I, on a par la formule de 4 :
R +∞ R +∞
Ua (gk )(x) = 0 e−at gk (x + t) dt = 0 e−at e−(x+t) (x + t)k dt
R +∞
= e−x 0 e−(a+1)t ki=0 ki xi tk−i dt par la formule du binôme,
P 
Pk −x i k
 R +∞ −(a+1)t k−i
= i=0 e x i 0 e t dt par linéarité de l’intégrale.

2
 R +∞
Autrement dit, Ua (gk ) = ki=0 λi,k gi où λi,k = ki 0 e−(a+1)t tk−i dt.
P

Ainsi pour tout k ∈ [[0; p]], Ua (gk ) ∈ Fp = Vect(g0 , . . . , gp ), donc Fp est stable par Ua .
5.3. Les calculs faits en 5.2 montrent que la matrice, dans la base (g0 , . . . , gp ), de l’endomorphisme de Fp induit
R +∞
par Ua , est triangulaire supérieure, et a pour coefficients diagonaux les λk,k = 0 e−(a+1)t dt = a+1 1
, pour
Qp 1
k ∈ [[0; p]]. Son déterminant, qui est le déterminant demandé, est donc k=0 λk,k = (a+1)p+1 .
6. Pour tout f ∈ E , on a |f | ∈ E , de sorte que URa (f ) et Ua (|f |) sont bien définis. Et par croissance de l’intégrale,
+∞ R +∞
on a alors, pour tout x ∈ I, |Ua (f )(x)| = |eax x e−at f (t) dt| 6 eax x e−at |f (t)| dt = Ua (|f |)(x).
Donc |Ua (f )| 6 Ua (|f |).
7. Si f est positive, alors de façon évidente au vu des formules définissant Ua (f ) en 1.4 ou en 4, on a par positivité
de l’intégrale, ∀ x ∈ I, Ua (f )(x) > 0. Donc si f est positive, il en est de même pour Ua (f ).
8. On suppose f décroissante.
• Alors pour tous x ∈ I et t > 0, on a f (x + t) 6 f (x), donc e−at f (x + t) 6 e−at f (x), et donc par croissance de
R +∞
l’intégrale avec la formule vue en 4, Ua (f )(x) 6 f (x) 0 e−at dt = f (x) a .
Puisque a > 0, on a donc ∀ x ∈ I, aUa (f )(x) 6 f (x), i.e. aUa (f ) 6 f .
• Par définition, Ua (f ) est une solution de (Eaf ), donc Ua (f )0 − aUa (f ) + f = 0. Ainsi Ua (f )0 = aUa (f ) − f 6 0
par le point précédent, donc Ua (f ) est décroissante.
9.1. L’hypothèse f ∈ H implique f 0 ∈ E , de sorte que Ua (f 0 ) est bien défini.
Alors par intégration par parties dans laquelle tous les termes convergent, on a pour x ∈ I :
R +∞ +∞ R +∞
Ua (f 0 )(x) = eax x e−at f (t) dt = eax e−at f (t) x + aeax x e−at f (t) dt = −f (x) + aUa (f )(x).


Ainsi Ua (f 0 ) − aUa (f ) + f = 0.
9.2. Comme Ua (f ) est une solution de (Eaf ), on a aussi Ua (f )0 − aUa (f ) + f = 0, et donc vu 9.1, Ua (f )0 = Ua (f 0 ).
Autrement dit, ∀ f ∈ H , D ◦ Ua (f ) = Ua ◦ D(f ), i.e. Ua et D commutent dans H .
R +∞ n
10. On va montrer par récurrence la propriété Pn : ∀ f ∈ E , ∀ x ∈ I, Uan+1 (f )(x) = eax x (t−x) n! e
−at f (t) dt.

• La propriété P0 est vraie par définition de Ua (cf. 1.4).


• Soit n ∈ N tel que Pn est vraie. Fixons f ∈ E et x ∈ I. En appliquant Pn à la fonction Ua (f ), on a :
R +∞ n
Uan+2 (f )(x) = Uan+1 (Uan (f ))(x) = eax x (t−x)
n! e
−at U (f )(t) dt.
a

Or la fonction g : t 7→ e−at Ua (f )(t) est de classe C 1 sur I et admet pour dérivée g 0 : t 7→ −e−at f (t) puisque
Ua (f ) est solution de (Eaf ) (cf. calculs faits en 1.1).
Une intégration par parties donne alors, sous réserve de convergence de l’un des deux termes de droite :
R +∞ (t−x)n −at n+1
−at U (f )(t) +∞ + +∞ (t−x)
n+1
Ua (f )(t) dt = (t−x) −at f (t) dt.
  R
x n! e (n+1)! e a x x (n+1)! e

Or le terme entre crochets converge vers 0 en +∞ par croissances comparées (car Ua (f ) est bornée), et vaut
0 en 0. On en déduit que la dernière intégrale converge, et on a alors, en multipliant par eax :
R +∞ n+1
Uan+2 (f )(x) = eax x (t−x)
(n+1)! e
−at f (t) dt

qui est le formule voulue au rant n + 1. On a montré que si Pn est vraie, alors Pn+1 l’est aussi.
• On conclut par principe de récurrence que la propriété Pn est vraie pour tout n ∈ N, i.e. que :
R +∞ n
∀ f ∈ E , ∀ n ∈ N, Uan+1 (f ) : x 7→ eax x (t−x) n! e
−at f (t) dt.

P (t−x)n
11.1. La série exponentielle n! converge et a pour somme et−x , donc la série proposée converge et a pour somme :
P+∞ (t−x)n −at
n=0 n! e f (t) = et−x e−at f (t) = e−x e(1−a)t f (t).
11.2. Soit x ∈ I. Il s’agit de montrer que la série n>0 Uan+1 (f )(x) converge et de calculer sa somme S(x).
P
R +∞ n
Vu 10, il s’agit de montrer que la série n>0 x (t−x) −at f (t) dt converge et de calculer sa somme e−ax S(x).
P
n! e
On va pour cela utiliser le théorème d’intégration terme à terme :
n
• Pour tout n ∈ N, la fonction fn : t 7→ (t−x)
n! e
−at f (t) est continue (par morceaux) sur [x; +∞[ .

fn converge simplement sur [x; +∞[ , et sa somme F : t 7→ e−x e(1−a)t f (t)


P
• Par 11.1, la série de fonctions
est continue (par morceaux) sur [x; +∞[ .

3
• Soit M un majorant de |f | sur I. Pour tous n ∈ N et t ∈ [x; +∞[ , on a |fn (t)| 6 M n −at , et la fonction
n! t e
n
gn : t 7→ t e −at 1
est intégrable sur [x; +∞[ , puisque dominée en +∞ par t 7→ t2 par croissances comparées.
R +∞ R +∞ n −at R +∞ n −at
Donc fn est intégrable sur [x; +∞[ , et x |fn (t)| dt 6 M n! x t e dt 6 Mn! 0 t e dt.
R +∞ n −at 1
R +∞ n −at 1
R +∞ n −u n!
Or avec l’indication et en posant u = at, 0 t e dt = an 0 (at) e dt = an+1 0 u e du = an+1 .
R +∞ M P 1
On a donc x |fn (t)| dt 6 an+1 . Or la série géométrique an+1
converge puisque a > 1, donc par compa-
R +∞
raison, la série de terme général x |fn (t)| dt converge.
P R +∞ terme à terme s’applique donc et montre que la fonction F est intégrable sur
Le théorème de d’intégration
[x; +∞[ , que la série x fn (t) dt converge, et que :
P+∞ R +∞ R +∞ R +∞
n=0 x fn (t) dt = x F (t) dt = e−x x e(1−a)t f (t) dt.
En multipliant par ea x, on conclut que la série n>0 Uan+1 (f )(x) converge et a pour somme :
P

S(x) = +∞ n+1 (f )(x) = e(a−1)x +∞ e(1−a)t f (t) dt.


P R
n=0 Ua x
11.3. La formule obtenue ci-dessus montre que S = Ub (f ) pour b = a − 1 > 0.
Remarque.P On peut illustrer ce résultat avec l’exemple
P+∞ n traité ena+r3.4. En effet pour f = fr et a + r > 1, on a
montré que +∞ U n (f ) = a+r f , donc S =
n=0 a r a+r−1 r U (f
n=1 a r ) = 1
a+r−1 fr − fr = a+r−1 fr = Ua−1 (fr ) vu 3.1.

Partie 2
1. Il est clair que si f1 , f2 et h ∈ C (I, R) sont telles que f1 = o (f2 ) et h ne s’annulant pas, alors hf1 = o (hf2 ).
+∞ +∞
R +∞ −at R +∞ −at 
Ainsi si f = o (g), alors fa f = o (fa g) et donc par l’encadré, x e f (t) dt = o x e g(t) dt .
+∞ +∞ x→+∞
En multipliant cette relation par eax , on obtient Ua (f )(x) = o (Ua (g)(x)), qui est le résultat voulu.
x→+∞
2. On rappelle que f1 ∼ f2 ⇐⇒ f1 − f2 = o (f2 ).
+∞ +∞
Ainsi si f ∼ g, alors f − g = o (g), donc par 1, Ua (f − g) = o (Ua (g)).
+∞ +∞ +∞
Or par linéarité de Ua (cf. Partie 1, 2.2), on a Ua (f − g) = Ua (f ) − Ua (g), donc Ua (f ) − Ua (g) = o (Ua (g)),
+∞
ce qui signifie Ua (f ) ∼ Ua (g).
+∞
3. • Cas d’une limite nulle.
Si lim f = 0, i.e. si f = o (1), alors par 1, Ua (f ) = o (Ua (1)).
+∞ +∞ +∞
Or les calculs faits en Partie 1, 2.1 ou 3.1, donnent Ua (1) = a1 , donc Ua (f ) = o ( a1 ), i.e. lim Ua (f ) = 0.
+∞ +∞
• Cas d’une limite non nulle.
Si lim f = ` ∈ R∗ , alors en appliquant le point précédent à f − `, ou la question 2 à l’équivalent f ∼ `, on
+∞ +∞
`
obtient en profitant de la linéarité de Ua , lim Ua (f ) = a.
+∞
1
On a montre dans tous les cas que si f converge en +∞, alors Ua (f ) aussi, avec lim Ua (f ) = a lim f.
+∞ +∞
4.1. Soit ω > 0. Les fonctions hω et h0ω = −ωhω+1 appartiennent à E , donc par intégration par parties dans laquelle
tous les termes convergent (la limite en +∞ du terme entre crochets est nulle par croissances comparées), on a
pour x ∈ I :
R +∞
Hω (x) = eax x e−at hω (t) dt
 −at +∞ R +∞
= eax e−a hω (t) x − ωa eax x e−at hω+1 (t) dt
= a1 hω (x) − ωa Hω+1 (x)
qui est l’égalité voulue.
4.2. On a manifestement hω+1 = o (hω ), donc par 1, Hω+1 = o (Hω ).
+∞ +∞
L’égalité de 4.1 donne donc a1 hω = Hω + o (Hω ), i.e. a1 hω ∼ Hω , qui est l’équivalent demandé.
+∞ +∞
R x e−at Rx 1 R x −at R x −at
5.1. Soit x ∈ I. Par linéarité de l’intégrale, 1 t dt = 1 t dt + 1 e t −1 dt = ln(x) + 1 e t −1 dt.
(−at)k e−at −1 (−a)k tk−1
Or pour tout t ∈ R∗ , e−at = +∞ = +∞
P P
k=0 k! , donc t k=1 k! .
k k−1 k k−1
Posons,
P pour tout k ∈ N∗ , gk : t 7→ (−a)k!t . On a manifestement kgk k∞,[1;x] = a xk! , donc la série de fonctions
k>1 gk converge normalement sur [1; x]. On peut donc intervertir somme et intégrale sur le segment [1; x] :

4
Rx e−at −1
R x P+∞ (−a)k tk−1 P+∞ R x (−a)k tk−1 P+∞ (−a)k k
1 t dt = 1 k=1 k! dt = k=1 1 k! dt = k=1 k·k! (x − 1).
Rx e−at P+∞(−a)k k
On a donc bien 1 t dt = ln(x) + k=1 k·k! (x − 1).
R +∞ −at R +∞ e−at
Rx e−at
eax x e t dt = eax

5.2. Immédiat vu 5.1 puisque H1 (x) = 1 t dt − 1 t dt par relation de Chasles.

Partie 3
1
R +∞
1. Par Partie 1, 3.1, Ua (fr ) = a+r fr , donc 1 Ua (fr )(t) dt converge lorsque r > 0 (intégrale de référence).
R +∞ R +∞
2. Par Partie 2, 4.2, on a Hω ∼ a1 hω , donc les deux intégrales 1 Hω (t) dt et 1 hω (t) dt sont de même
+∞
nature (car les fonctions en jeu sont continues et positives sur I = [x; +∞[ ).
R +∞ R +∞
Or 1 hω (t) dt converge ⇐⇒ ω > 1 (intégrale de Riemann), donc 1 Hω (t) dt converge ⇐⇒ ω > 1.
3.1. Par définition, Ua (f ) est une solution de (Eaf ), donc Ua (f )0 − aUa (f ) + f = 0, i.e. avec les notations de cette
question, F 0 − aF + f = 0. En intégrant cette relation entre 1 et x ∈ I, on obtient, puisque Φ0 = F :
F (x) − F (1) − aΦ(x) + ϕ(x) = 0, i.e. Φ0 (x) − F (1) − aΦ(x) + ϕ(x) = 0.
3.2. La fonction ϕ est une primitive de f sur I, R x doncRelle y est continue (même de classe C 1 ). Et puisque f est
+∞
positive, on a pour tout x ∈ I, 0 6 ϕ(x) = 1 f 6 1 f , donc ϕ est bornée sur I. Ainsi, ϕ ∈ E .
Rx
3.3. Par 3.1, on a Φ(x) = 1 F (t) dt = a1 F (x) + ϕ(x) − F (1) .


Or F = Ua (f ) est bornée par construction, et ϕ est bornée par 3.2, donc Φ l’est.
Par ailleurs comme f est positive, F = Ua (f ) l’est aussi par Partie 1, 7, donc Φ est croissante.
Rx
Le théorème de la limite monotone implique alors que Φ(x) = 1 F (t) dt converge quand x → +∞, ce qui est la
R +∞
définition de la convergence de 1 F (t) dt.
R +∞
4. On suppose f intégrable, donc la question 3 s’applique à |f | et montre que l’intégrale 1 Ua (|f |) converge.
R +∞
Or par Partie 1, 6, on a |Ua (f )| 6 Ua (|f |), donc par comparaison, l’intégrale 1 |Ua (f )| converge, autrement
dit Ua (f ) est intégrale sur I.

5
PSI E3A 2 2018

Concours E3A 2018


Épreuve de mathématiques 2 PSI, trois heures
Corrigé

Partie 1
1. 1. Les solutions de l’équation homogène y 0 − ay = 0 sont de la forme x 7→ Keax avec K ∈ R.
De plus, laZ méthode de la variation de la constante permet de démontrer que l’application
x
x 7→ −eax e−at f (t)dt est une solution particulière de l’équation y 0 − ay + f = 0. On en
1
déduit que les solutions de (Eaf ) sur I sont de la forme :
Z x
x 7→ Ke ax
−e ax
e−at f (t)dt, où K ∈ R.
1

Après factorisation par eax , on obtient la forme voulue.


2. Soient z1 et z2 deux solutions de (Eaf ) bornées sur I. Alors z1 − z2 est également bornée ;
or, d’après la question précédente, il existe des réels K1 et K2 tels que :
 Z x   Z x 
ax −at ax −at
∀x ∈ I, z1 (x) = e K1 − e f (t)dt , et z2 (x) = e K2 − e f (t)dt ,
1 1

donc : ∀x ∈ I, z1 (x) − z2 (x) = eax (K1 − K2 ). Comme eax → +∞ pour a > 0, cette
x→+∞
application ne peut être bornée qu’à la condition que K1 − K2 = 0, c’est-à-dire K1 = K2
et donc z1 = z2 . Ainsi il y a au plus une solution de (Eaf ) bornée sur I.
3. L’application t 7→ e−at f (t) est continue en tant que produit d’applications continues : nous
allons démontrer son intégrabilité par relation de comparaison.
L’application f est bornée sur I ; soit, donc, un réel M > 0 tel que |f | 6 M . Alors :

∀t ∈ I, e−at f (t) 6 M e−at .

Comme a > 0, l’application t 7→ e−at est intégrable sur I et donc


Z +∞
t 7→ e−at f (t) également
par comparaison. Ceci implique en particulier que l’intégrale e−at f (t)dt converge.
1
4. L’unicité a déjà été établie dans la question 1.2. Il s’agit donc seulement de vérifier que F est
une solution de (Eaf ) bornée. Pour montrer que F est solution de (Eaf ), on peut se contenter
Z +∞
de vérifier qu’elle est de la forme explicitée dans la question 1.1, avec K = e−at f (t)dt.
1
En effet, d’après la relation de Chasles :
Z +∞ Z 1 Z +∞ 
ax −at ax −at −at
∀x ∈ I, F (x) = e e f (t)dt = e e f (t)dt + e f (t)dt
x x 1
Z +∞ Z x 
ax −at −at
=e e f (t)dt − e f (t)dt .
1 1

1
PSI E3A 2 2018

Montrons à présent que F est bornée : comme dans la question précédente, soit M > 0 un
réel tel que |f | 6 M . Alors :
#+∞
e−at
Z +∞ Z +∞ "
M
∀x ∈ I, |F (x)| 6 eax e−at |f (t)|dt 6 M eax e−at dt = M eax = ,
x x −a x
a
M
donc F est bornée (par la constante a
). Ceci démontre le résultat.
2. 1. L’application 1 est bien sûr continue et bornée (par 1), donc 1 ∈ E et Ua (1) existe.
Soit x ∈ I. Alors :
#+∞
e−at
"
Z +∞
1
Ua (1)(x) = eax e−at dt = eax = ,
x −a x
a

donc Ua (1) est la fonction constante égale à a1 .


2. Soit f ∈ E . Comme f est continue sur I, l’application t 7→ e−at f (t)Zest continue sur I
x
en tant que produit d’applications continues, donc sa primitive x 7→ e−at f (t)dt l’est
+∞ Z
+∞
également (elle est même de classe C1 ). Par conséquent Ua (f ) : x 7→ eax e−at f (t)dt
x
est continue sur I en tant que produit d’applications continues. De plus elle est bornée
d’après la question 1.4 : ceci montre que Ua (f ) ∈ E ; ainsi l’image de Ua est bien incluse
dans E . La linéarité de Ua provient de la linéarité de l’intégrale, donc Ua : E → E est un
endomorphisme.
3. (a) Pour déterminer si Ua est injectif, on étudie son noyau : soit f ∈ ker(Ua ). Alors, pour
tout x ∈ I, on a : Z +∞
e−at f (t)dt = 0
x

(on divise l’égalité Ua (f )(x) = 0 par eax 6= 0). En dérivant cette égalité, on obtient :

∀x ∈ I, −e−ax f (x) = 0.

Or l’exponentielle ne s’annule jamais, donc cela implique f = 0. Ceci démontre que


ker(Ua ) = {0}, c’est-à-dire : Ua est un endomorphisme injectif.
(b) On reprend l’argumentation de la question 2.2. Si f ∈ E , il a été mentionné que
Z +∞
l’application x 7→ e−at f (t)dt est de classe C1 sur I en tant que primitive d’une
x
application continue, et x 7→ eax l’est également, donc leur produit Ua (f ) aussi.
(c) La question précédente démontre que Ua (E ) ⊆ E1 , or E1 6= E puisqu’il √ existe des
applications continues qui ne sont pas de classe C1 sur I : l’application x 7→ x − 1 en
est un exemple (elle n’est pas dérivable en 1). Par conséquent Ua (E ) 6= E et Ua n’est
pas surjectif.

2
PSI E3A 2 2018

4. Tout d’abord, les applications cos et sin sont continues et bornées (par 1), donc appar-
tiennent à E ; cet ensemble étant stable par combinaison linéaire, F = VectR (cos, sin) est
inclus dans E , et il est donc possible de discuter de l’image par U1 de cet espace vectoriel.
Pour montrer que F = VectR (cos, sin) est stable par U1 , il suffit de vérifier que
Z +∞
U1 (cos) ∈ F et U1 (sin) ∈ F . Pour cela, nous avons besoin de calculer e−t cos(t)dt et
Z +∞ x
−t
e sin(t)dt pour tout x ∈ I. Or, pour tout x ∈ I, on a :
x
" #+∞
Z +∞
−t
Z +∞
−t
Z +∞
(i−1)t e(i−1)t
e cos(t)dt + i e sin(t)dt = e dt =
x x x i−1 x
e(i−1)x
=−
i−1
(−i − 1)e(i−1)x
=−
2
i + 1 −x
= e (cos(x) + i sin(x)).
2
En développant et en identifiant parties réelles et imaginaires, on en déduit, pour tout
x∈I :
Z +∞
−t e−x Z +∞
e−x
e cos(t)dt = (cos(x) − sin(x)) , e−t sin(t)dt = (cos(x) + sin(x)) .
x 2 x 2
On en déduit :
1 1
U1 (cos) = (cos − sin) ∈ F , U1 (sin) = (cos + sin) ∈ F ,
2 2
donc F est bien stable par U1 ; si on note V l’endomorphisme induit ! par U1 sur F , alors
1 1 1
la matrice de V relativement à la base (cos, sin) est M = .
2 −1 1
√ !
2 1 1
Si on note Ω = , alors Ω est la matrice d’une rotation planaire de mesure
2 −1 1
π 1
d’angle − (avec la convention que (cos, sin) est une base directe de F ), et on a M = √ Ω.
4 2
−rx −r
3. 1. Soit r > 0. L’application fr : x 7→ e est continue et bornée sur I (par e par exemple),
donc fr ∈ E et Ua (fr ) existe. Alors :
#+∞
e−(a+r)t e−(a+r)x e−rx
Z +∞ "
ax −(a+r)t ax
∀x ∈ I, Ua (fr )(x) = e e dt = e = eax = ,
x −(a + r) x
a+r a+r
1
donc : Ua (fr ) = a+r fr . Comme fr 6= 0, on a montré que fr est un vecteur propre de Ua
1
pour la valeur propre a+r .

3
PSI E3A 2 2018

i i
2. Soit λ ∈ 0, a1 . Comme λ 6 a1 , on a λ1 > a, et il existe donc r > 0 tel que λ = a+r1
: on
1
prend pour cela r = λ − a > 0. Nous avons démontré dans la question précédente que pour
1
tout r > 0, le réel a+r est valeur propre de Ua : donc λ est valeur propre de Ua .
 n
1
3. Soit r > 0. Pour tout n ∈ N, on a Uan (fr ) = a+r
fr d’après la question 3.1. Donc, pour
tout x ∈ I, on a :

n 0 si a + r > 1,
1
 

Uan (fr )(x) = e−rx −→ e−rx si a + r = 1,
a+r n→+∞ 

+∞ si a + r < 1,
donc la suite de fonctions (Uan (fr ))n∈N converge simplement sur I si et seulement si a+r > 1 ;
elle converge vers la fonction identiquement nulle si a + r > 1, et vers fr si a + r = 1.
P n
4. Soit r > 0, et soit x ∈ I. À une constante multiplicative non nulle près, la série Ua (fr )(x)
n>0
P  1 n 1
est la série géométrique a+r
;
elle converge donc si et seulement si < 1, c’est-à-
n>0
P n a+r
dire si et seulement si a + r > 1. Donc la série de fonctions Ua (fr ) converge simplement
n>0
sur I si et seulement si a + r > 1 (il y a même convergence normale). Dans ce cas, on a :
+∞ +∞ n
1 1 −rx a + r

Uan (fr )(x) = e−rx = e−rx
X X
∀x ∈ I, 1 = e ,
n=0 n=0 a+r 1 − a+r a+r−1
+∞
a+r
Uan (fr ) =
P
donc : f.
a+r−1 r
n=0
4. ZSoit f ∈ E , et soit x ∈ I. On fait le changement de variable affine u = t − x dans l’intégrale
+∞
e−at f (t)dt. Alors :
x
Z +∞ Z +∞ Z +∞
Ua (f )(x) = eax e−a(u+x) f (u + x)du = e−au f (u + x)du = e−at f (t + x)dt,
0 0 0

d’où le résultat.
5. 1. Il est tentant de proposer (g0 , . . . , gp ) pour base de Fp ; par définition de cet espace vectoriel,
c’en est une famille génératrice. Montrons donc qu’elle est libre : soit (α0 , . . . , αp ) ∈ Rp+1
p
P
tel que αk gk = 0. Alors :
k=0
p
αk xk e−x = 0.
X
∀x ∈ I,
k=0

En multipliant cette égalité par ex , on obtient :


p
αk xk = 0.
X
∀x ∈ I,
k=0

4
PSI E3A 2 2018

p
Pour tout x ∈ I, le réel x est donc racine du polynôme αk X k ; or I est infini, donc
P
k=0
p
αk X k = 0,
P
ce polynôme admet une infinité de racines et doit être nul, c’est-à-dire :
k=0
ou encore : α0 = · · · = αp = 0. Ceci montre que la famille (g0 , . . . , gp ) est libre en plus
d’engendrer Fp , donc c’est une base de Fp qu’on note désormais Bp .
2. Pour montrer que l’espace vectoriel Fp = VectR (g0 , . . . , gp ) est inclus dans E , il suffit de
montrer que pour tout k ∈ J0, pK on a gk ∈ E . Il est clair que pour tout k ∈ J0, pK,
l’application gk est continue : il reste à montrer qu’elle est bornée sur I.
Soit, donc, k ∈ J0, pK. Notons d’abord que gk −→ 0 par croissances comparées. Alors, par
x→+∞
définition de la limite, il existe x0 ∈ I tel que pour tout x > x0 , on ait |gk (x)| 6 1. De plus,
l’application |gk | étant continue sur le segment [1, x0 ], elle est bornée, disons par un réel m.
En considérant M = max(m, 1), on a : ∀x ∈ I, |gk (t)| 6 M , donc gk est bornée sur I.
(Il est aussi possible d’étudier les variations de gk grâce au signe de gk0 , facile à déterminer
puisque pour tout x ∈ I, on a : gk0 (x) = xk−1 (k − x)e−x . Le maximum de gk est atteint en
k, donc pour tout x ∈ I on a 0 6 gk (x) 6 gk (k) = k k e−k : l’application gk est bornée.)
On a donc bien l’inclusion Fp ⊆ E , et on peut considérer l’action de Ua sur ce sous-espace
vectoriel. Nous allons montrer par récurrence sur k que Ua (gk ) ∈ Fp pour tout k ∈ J0, pK.
(L’idée de procéder par récurrence provient du fait qu’en intégrant par parties, une primitive
de l’exponentielle étant elle-même à peu de choses près, on ramène l’étude de Ua (gk ) à celle
de Ua (gk0 ) ; or gk0 = kgk−1 −gk d’après ce qui précède, faisant le lien entre Ua (gk−1 ) et Ua (gk ).)
Si k = 0, on a Ua (g0 ) = Ua (f1 ) = 1+a 1
g0 ∈ Fp d’après la question 3.1. À présent, soit
k ∈ J0, p − 1K, et supposons que Ua (gk ) ∈ Fp . Alors, en dérivant gk+1 et en intégrant
h −at
i+∞
x 7→ e−ax (on vérifie préalablement que le terme tk+1 e−t e−a est bien défini, et égale
x
−(1+a)x
xk+1 e a grâce aux croissances comparées), on obtient :
−(1+a)x −at
!
k+1 e
Z +∞ 
−t e

ax k k+1
∀x ∈ I, Ua (gk+1 )(x) = e x − (k + 1)t − t e dt
a x −a
1 k+1 −x k + 1 ax Z +∞ k −t −at 1 ax Z +∞ k+1 −t −at
= x e + e t e e dt − e t e e dt,
a a x a x
 
donc : 1 + 1
a
Ua (gk+1 ) = a1 gk+1 + k+1
a
Ua (gk ). Or gk+1 ∈ Fp et, par hypothèse de récurrence,
 
Ua (gk ) ∈ Fp donc 1 + 1
a
Ua (gk+1 ) ∈ Fp . On a 1 + 1
a
6= 0, donc Ua (gk+1 ) ∈ Fp .
Par principe de récurrence, pour tout k ∈ J0, pK, on a Ua (gk ) ∈ Fp . Comme la famille
(g0 , . . . , gp ) engendre Fp , on en déduit que Fp est stable par Ua .
3. Une analyse attentive de la démonstration
  par récurrence de la question précédente montre
que pour tout k ∈ J1, pK, on a 1 + a Ua (gk )− a1 gk ∈ Fk−1 , ou encore Ua (gk )− 1+a
1 1
gk ∈ Fk−1 .
1
On a de plus montré que Ua (g0 ) = 1+a g0 , donc la matrice de l’endomorphisme induit par

5
PSI E3A 2 2018

Ua sur Fp , dans la base Bp = (g0 , . . . , gp ), est triangulaire supérieure et de la forme :


 1


1+a
 .. 
,

 . 
0 1
1+a
p+1
1

et son déterminant égale le produit des coefficients diagonaux, c’est-à-dire : .
1+a
6. Soit f ∈ E . Pour tout x ∈ I on a, d’après la question 4 et l’inégalité triangulaire :
Z +∞ Z +∞ Z +∞
|Ua (f )(x)| = e−at f (x + t)dt 6 |e−at f (x + t)|dt = e−at |f (x + t)|dt = Ua (|f |)(x),
0 0 0

donc : |Ua (f )| 6 Ua (|f |).


7. Soit f ∈ E . Si f > 0, alors pour tout x ∈ I et tout t ∈ [0, +∞[ on a e−at f (x + t) > 0 parce que
l’exponentielle est positive sur R. On en déduit que Ua (f )(x) > 0 en intégrant cette inégalité
sur [0, +∞[, l’intégrale étant croissante.
8. Soit f ∈ E décroissante. Pour tout x ∈ I et tout t ∈ [0, +∞[ on a donc f (x + t) 6 f (x). En
multipliant cette inégalité par e−at > 0, et en intégrant sur [0, +∞[, on obtient :
#+∞
e−at
"
Z +∞
−at
Z +∞
−at 1
∀x ∈ I, Ua (f )(x) = e f (x + t)dt 6 e f (x)dt = f (x) = f (x) × ,
0 0 −a 0
a
donc, a étant un réel strictement positif : aUa (f ) 6 f .
Montrons à présent que Ua (f ) est décroissante : soient x, x0 ∈ I tels que x 6 x0 . L’application f
étant décroissante, pour tout t ∈ [0, +∞[ on a f (x + t) > f (x0 + t). En multipliant cette inégalité
par e−at > 0, et en intégrant sur [0, +∞[, on obtient Ua (f )(x) > Ua (f )(x0 ) : ceci démontre que
Ua (f ) est décroissante sur I.
9. 1. Soit f ∈ H . Alors Ua (f 0 ) est bien défini ; en intégrant par parties (on intègre x 7→ f 0 (x + t)
+∞
en choisissant la primitive x 7→ f (x + t), et on dérive t 7→ e−at ; le terme [e−at f (x + t)]0
est bien défini et égale −f (x)), pour tout x ∈ I on a :
Z +∞ Z +∞
0 −at 0
Ua (f )(x) = e f (x + t)dt = −f (x) + ae−at f (x + t)dt,
0 0
0
donc : Ua (f ) = −f + aUa (f ), ce qu’il fallait démontrer.
2. D’après la question 1.4, Ua (f ) est solution de (Eaf ), donc vérifie :
Ua (f )0 − aUa (f ) + f = 0,
et il vient d’être établi que Ua (f 0 ) − aUa (f ) + f = 0. En comparant ces deux égalités, on
obtient Ua (f 0 ) = Ua (f )0 , ce qu’on peut réécrire ainsi :
Ua (D(f )) = D(Ua (f )).
Ceci vaut pour toute application f ∈ H , donc Ua et D commutent dans H .

6
PSI E3A 2 2018

10. Nous allons montrer par récurrence sur n ∈ N la proposition Pn suivante :


Z +∞
(t − x)n −at
« ∀f ∈ E , ∀x ∈ I, Uan+1 (f )(x) = eax e f (t)dt. »
x n!
On a P0 par définition même de Ua (f ) pour tout f ∈ E . À présent, soit n ∈ N, et supposons
Pn . Soit f ∈ E ; on a Uan+2 (f ) = Uan+1 (Ua (f )) et Ua (f ) ∈ E , donc d’après Pn :
Z +∞
(t − x)n −at
∀x ∈ I, Uan+2 (f )(x) = eax e Ua (f )(t)dt.
x n!
(t − x)n
Intégrons par parties, en intégrant t 7→ et en dérivant t 7→ e−at Ua (f )(t) (notons que
n!
Ua (f ) est bien dérivable d’après la question 2.3.b). Rappelons que pour tout x ∈ I, on a
Z+∞
e−ax Ua (f )(x) = e−at f (t)dt, donc la dérivée de t 7→ e−at Ua (f )(t) est t 7→ −e−at f (t). Si
x
l’on met provisoirement de côté le terme entre crochets, l’intégration par parties donne donc :
" #+∞ 
(t − x)n+1 −at Z +∞
(t − x)n+1 −at
∀x ∈ I, Uan+2 (f )(x) = eax  e Ua (f )(t) + e f (t)dt .
(n + 1)! x x (n + 1)!
#+∞
(t − x)n+1 −at
"
Inspectons de plus près le terme e Ua (f )(t) : nous avons démontré dans la
(n + 1)! x
question 1.4 que Ua (f ) est bornée pour toute application f ∈ E , disons par une constante
M > 0. Donc :
(t − x)n+1 −at M
e Ua (f )(t) 6 (t − x)n+1 e−at −→ 0
(n + 1)! (n + 1)! t→+∞

(t − x)n+1 −at
par croissances comparées : ceci démontre que lim e Ua (f )(t) = 0 d’après le théo-
t→+∞ (n + 1)!
#+∞
(t − x)n+1 −at
"
rème des gendarmes. De plus, ce terme égale 0 quand t = x, donc e Ua (f )(t)
(n + 1)! x
est bien défini et égale 0. On en déduit :
Z +∞
(t − x)n+1 −at
∀x ∈ I, Uan+2 (f )(x) = eax e f (t)dt,
x (n + 1)!
et ceci vaut pour toute application f ∈ E , donc on a Pn+1 .
Ayant démontré l’initialisation et l’hérédité, Pn est vraie pour tout n ∈ N par récurrence.
P xn P xn
+∞
11. 1. Pour tout x ∈ R la série converge, et on a ex = . On en déduit :
n>0 n! n=0 n!

+∞
(t − x)n −at
e f (t) = e−at f (t)et−x .
X
∀x ∈ I, ∀t > x,
n=0 n!

7
PSI E3A 2 2018

2. Soient x ∈ I et n ∈ N \ {0}. D’après la question 10, et en faisant le changement de variable


affine u = t − x, on a :
Z +∞
un−1 −a(u+x) Z +∞
un−1 −au
Uan (f )(x) =e ax
e f (u + x)du = e f (u + x)du.
0 (n − 1)! 0 (n − 1)!
Or, f étant bornée, il existe M > 0 tel que |f | 6 M , et on a alors :
Z +∞
M
|Uan (f )(x)| 6 un−1 e−au du.
(n − 1)! 0
Calculons cette dernière intégrale : on fait le changement de variable linéaire v = au pour
se ramener à l’intégrale dont la valeur nous est donnée dans l’énoncé. On a :
Z +∞  n−1
Z +∞
u dv (n − 1)!
un−1 e−au du = e−v = ,
0 0 a a an
M
donc finalement : ∀n ∈ N\{0}, |Uan (f )(x)| 6 n . Puisque, par hypothèse, on a a > 1, la série
a
P 1 P n
géométrique n
converge. Par comparaison, la série Ua (f )(x) converge absolument,
n>1 a n>1
donc converge.
P n
Ceci vaut pour tout x ∈ I, donc la série de fonctions Ua (f ) converge simplement sur I.
n>1
+∞
(t − x)n −at
Z +∞
3. Soit x ∈ I. Nous allons calculer explicitement S(x) = eax
X
e f (t)dt grâce
n=0 x n!
à la question 11.1 : pour cela, nous avons besoin d’intervertir somme et intégrale. Vérifions
les hypothèses du théorème d’intégration terme à terme :
(t − x)n −at
— pour tout n ∈ N, l’application t 7→ e f (t) est continue par morceaux et
n!
Z +∞
(t − x)n −at
intégrable sur [x, +∞[, et on a e f (t) dt = Uan+1 (|f |)(x) d’après la
x n!
question 10 ;
(t − x)n −at
!
X
— la série de fonctions t 7→ e f (t) converge simplement sur I d’après la
n>0 n!
question 11.1, et sa somme t 7→ e−at f (t)et−x est continue par morceaux sur I ;
X Z +∞ (t − x)n
e−at f (t) dt = Uan+1 (|f |)(x) converge d’après la question
X
— la série
n>0 x n! n>0
11.2.
Les hypothèses du théorème d’intégration terme à terme sont donc vérifiées, et :
+∞ Z +∞ +∞
Z +∞
(t − x)n −at X (t − x)n Z +∞
e−at f (t)dt = e−at et−x f (t)dt.
X
e f (t)dt =
n=0 x n! x n=0 n! x

8
PSI E3A 2 2018

Z +∞
Enfin : S(x) = e(a−1)x e(1−a)t f (t)dt = Ua−1 (f )(x) : ceci vaut pour tout x ∈ I, donc
x
S = Ub (f ) avec b = a − 1. La question 3.4 en est un cas particulier.
Partie 2
Z +∞
1. Si g ∈ E , alors e−at g(t)dt converge d’après la question 1.3 de la partie 1. Ici, par hypothèse,
1
f (t) = o (g(t)), donc e−at f (t) = o (e−at g(t)). D’après le résultat admis, on en déduit :
t→+∞ t→+∞
Z +∞ Z +∞ 
−at −at
e f (t)dt = o e g(t)dt .
x x→+∞ x

Après multiplication par eax , on a Ua (f )(x) = o (Ua (g)(x)), d’où le résultat.


x→+∞
2. Avec les hypothèses de la question, on a f − g = +∞
o (g) et g est à valeurs positives. Donc,
d’après la question précédente, on a Ua (f − g) = o (Ua (g)). L’application Ua étant linéaire, cela
+∞
implique Ua (f ) = Ua (g) + o (Ua (g)), c’est-à-dire : Ua (f ) ∼ Ua (g).
+∞ +∞
3. Soit f ∈ E ayant une limite finie en +∞.
Si lim f = 0, cela signifie précisément que f = o (1) et donc, d’après la question 1 de cette
+∞ +∞
partie, on en déduit Ua (f ) = o (Ua (1)). Or Ua (1) est une application constante d’après la
+∞
question 2 de la partie 1, donc Ua (f ) = o (1), c’est-à-dire : lim Ua (f ) = 0.
+∞ +∞
Si lim f = L 6= 0, alors f ∼ L et donc, d’après la question 1 de cette partie, on en déduit
+∞ +∞
L L
Ua (f ) ∼ Ua (L). Or Ua (L) = LUa (1) = a
d’après la question 2 de la partie 1, donc Ua (f ) ∼ ,
+∞ +∞ a
c’est-à-dire : lim Ua (f ) = La .
+∞
lim f
+∞
Dans tous les cas, Ua (f ) admet une limite finie en +∞ : on a plus précisément lim Ua (f ) = a
.
+∞
4. 1. Soit x ∈ I. Intégrons par parties, en dérivant hω : t 7→ 1

et en intégrant t 7→ e−at ; le terme
#+∞
e−at 1 e−ax
"
est bien défini et égale , donc :
−a tω x axω
Z +∞ −at
e−ax ω Z +∞ e−at ω ax Z +∞ e−at
!
ax e ax 1 1
Hω (x) = e dt = e − dt = − e dt,
x tω axω a x tω+1 a xω a x tω+1
1 ω
donc : Hω (x) = hω (x) − Hω+1 (x).
a a
1
2. Pour tout x ∈ I, l’application t 7→ ω+1 étant décroissante, on a d’après la question 8 de la
t
partie 1 :
1 1
 
0 6 Hω+1 (x) 6 ω+1 = o .
ax x→+∞ xω
1 1
On en déduit : Hω (x) = hω (x) + o (hω (x)), donc : Hω (x) ∼ hω (x).
a x→+∞ x→+∞ a

9
PSI E3A 2 2018

+∞ (−at)k +∞ ak tk
5. 1. Soit x ∈ I. Pour tout t ∈ [1, x], on a e−at = (−1)k
P P
= , et donc :
k=0 k! k=0 k!
Z x +∞
Z x −at
e ka
k k−1
t Z x
dt Z x +∞ ak tk−1
(−1)k
X X
dt = (−1) dt = + dt
1 t 1 k=0 k! 1 t 1 k=1 k!
Z x +∞
ak tk−1
(−1)k
X
= ln(x) + dt.
1 k=1 k!
On aimerait permuter intégrale et somme ; c’est un problème d’interversion. Or la série en-
ak tk−1
(−1)k
X
tière est de rayon de convergence infini : c’est, au terme constant près et à
k>1 k!
multiplication par t près, une série exponentielle (on peut aussi invoquer la règle de D’Alem-
bert). En particulier, elle converge normalement (donc uniformément) sur tout segment in-
clus dans son intervalle ouvert de convergence, qui est R ; elle converge donc uniformément
sur le segment [1, x], ce qui permet d’intégrer terme à terme :
Z x +∞ +∞ +∞
#x +∞
k k−1
ak tk−1 ak tk ak  k
"
XZ x
ka t 
(−1)n (−1)k
X X X
(−1) dt = dt = = x −1 .
1 k=1 k! k=1 1 k! k=1 k! k 1 k=1 k!k
On en déduit : Z x −at +∞
e ak  k 
(−1)k
X
dt = ln(x) + x −1 .
1 t k=1 k!k
2. Soit x ∈ I. Par définition, et d’après la question précédente, on a :
Z +∞ −at Z 1 −at Z +∞ −at !
e e e
H1 (x) = eax dt = eax dt + dt
x t x t 1 t
+∞  Z +∞ e−at
ak  k
!
ax k
X
=e − ln(x) − (−1) x −1 + dt ,
k=1 k!k 1 t
d’où le résultat.

Partie 3
Z +∞ Z +∞
1. On a Ua (fr ) = 1
f
a+r r
d’après la question 3.1 de la première partie. Or fr = e−rt dt
Z +∞ 1 1

converge pour r > 0 (c’est une intégrale de référence), donc Ua (fr ) converge.
1
2. Tout d’abord, Hω est continue sur I d’après la question 2.3.b de la première partie. Ensuite, il a
1 1
été démontré que Hω (x) ∼ hω (x) dans la question 4.2 de la deuxième partie. Or hω : x 7→ ω
x→+∞ a x
est positive sur [1, +∞[ et intégrable sur ce même intervalle si et seulement si ω > 1, donc par
comparaison de fonctions positives l’application Hω est intégrable sur [1, +∞[ si et seulement si
ω > 1.

10
PSI E3A 2 2018

3. 1. Soit x ∈ I. D’après le théorème fondamental de l’analyse, Φ est dérivable et on a Φ0 = F .


Alors : Z x
0
Φ (x) − F (1) = F (x) − F (1) = F 0 (t)dt
1
et comme, d’après la question 1.4 de la première partie, l’application F est solution de
(Eaf ), on a F 0 = −f + aF , donc :
Z x Z x
0
Φ (x) − F (1) = − f (t)dt + a F (t)dt = −ϕ(x) + aΦ(x),
1 1

d’où le résultat.
2. Puisque f est positive et intégrable sur I par hypothèse, pour tout x ∈ I on a :
Z x Z +∞
0 6 ϕ(x) = f6 f.
1 1

Ceci montre que ϕ est bornée, et cette application est continue en tant que primitive de
l’application continue f , donc ϕ ∈ E .
Z +∞
3. Montrer que l’intégrale F (t)dt converge revient précisément à démontrer que la limite
Z x 1
lim Φ(x) = lim F (t)dt existe, et est finie.
x→+∞ x→+∞ 1
Z +∞
Tout d’abord, puisque f est supposée positive, l’application F : x 7→ eax e−at f (t)dt est
Z x x
également positive, et donc l’application Φ : x 7→ F (t)dt est croissante. Par conséquent,
1
Φ admet nécessairement une limite en +∞, soit finie, soit infinie. Montrons par l’absurde
qu’elle ne peut pas être infinie : si tel est le cas, alors l’égalité :

∀x ∈ I, ϕ(x) = F (1) − Φ0 (x) + aΦ(x)

implique, quand x → +∞, que ϕ(x) → +∞, car a 6= 0 et Φ0 = F est bornée d’après la
question 2.2 de la partie 1. Mais c’est impossible, puisque Zϕ ∈ E . Par l’absurde, on en
+∞
déduit que Φ admet une limite finie, et donc que l’intégrale F (t)dt converge.
1
Z +∞
4. Soit f ∈ E une application intégrable ; alors |f | ∈ E est à valeurs positives et l’intégrale |f |
1
converge. D’après les questions 3.1 à 3.3 de cette partie, on en déduit que Ua (|f |) est intégrable sur
I. Or, d’après la question 6 de la partie 1, on a |Ua (f )| 6 Ua (|f |) ; par comparaison, l’application
Ua (f ) est intégrable sur I.

11
CCP 2018 - MP2

Exercice I
On note E l’espace vectoriel des applications continues sur le segment [−1, 1] et à valeurs réelles.
Q.1 Démontrer que l’on définit un produit scalaire sur E en posant pour f et g éléments de E
Z 1
(f |g) = f (t)g(t) dt
−1

Q.2 On note u : t 7→ 1, v : t 7→ t et F = Vect(u, v). Déterminer une base orthonormée de F .


Q.3 Déterminer le projeté orthogonal de la fonction w : t 7→ et sur le sous-espace F et en déduire
la valeur du réel Z 1
2
inf et − (a + bt) dt
(a,b)∈R2 −1

On pourra simplifier les calculs en utilisant le théorème de Pythagore.

Exercice II
Dans cet exercice, n est un entier tel que n ≥ 2.
Q.4 Question préliminaire
Soient un réel 0 < λ < 1 et (Xn )n≥1 une suite de variables aléatoires qui suivent chacune une
loi binomiale de paramètres n et p = nλ .
Justifier que, pour tout entier k ≥ 1,
 
nn−1 n−k+1
lim ... =1
n→+∞ n n n

et déterminer lim P(Xn = k). On convient alors d’approximer pour n > 50, p ≤ 0, 01 et
n→+∞
np < 10 la loi binomiale de paramètres n et p par la loi de Poisson de paramètre λ = np.
Q.5 Un examinateur interroge à l’oral du concours CCP n candidats tous nés en 1998. On suppose
que les dates de naissances des n candidats sont uniformément réparties sur les 365 jours de
l’année 1998. On note Xn la variable aléatoire égale au nombre de candidats qui sont convoqués
le jour de leur anniversaire. Déterminer la loi de la variable Xn et donner son espérance.
Q.6 Dans le cas où l’examinateur interroge 219 candidats, donner une estimation de la probabilité
que deux étudiants soient convoqués le jour de leur anniversaire. Prendre 0, 55 comme valeur
approchée de e−0,6 .

Problème
On note, pour n entier tel que n ≥ 2, Mn (R) l’espace vectoriel des matrices carrées d’ordre n à
coefficients réels. On
 s’intéresse
 dans ce problème, à travers divers exemples, à la réduction de matrices
aA bA
par blocs du type ∈ M2n (R) où A ∈ Mn (R) et a, b, c, d sont quatre réels non tous nuls.
cA dA
On rappelle qu’un produit de matrices par blocs se fait de manière similaire à un produit classique :
 0
A B0 AA0 + BC 0 AB 0 + BD0
   
A B
=
C D C 0 D0 CA0 + DC 0 CB 0 + DD0

chaque matrice bloc étant une matrice de Mn (R).

1
On pourra utiliser ici sans démonstration que si P ∈ GLn (R), A, B ∈ Mn (R) et T ∈ R[X] est un
polynôme, A = P −1 BP entraı̂ne T (A) = P −1 T (B)P .
 
A B
On rapelle que si A, B, C sont des matrices de Mn (R), det = det(A) det(C).
0 C

Questions préliminaires
L’objectif est de démontrer le résultat suivant : “une matrice M ∈ Mn (R) est diagonalisable sur R si
et seulement s’il existe un polynôme P scindé sur R, à racines simples, vérifiant P (M ) = 0”.
Pour cela, on considère une matrice M ∈ Mn (R) et on note u l’endomorphisme de Rn canoniquement
associé à M .
Q.7 On suppose que u est diagonalisable et on note λ1 , . . . , λp (p ≥ 1) les valeurs propres distinctes
de u. Démontrer que le polynôme P = (X − λ1 ) . . . (X − λp ) est annulateur de u.
Q.8 Réciproquement, on suppose que µ1 , . . . , µr sont r nombres réels distincts (r ≥ 1) tels que
Q = (X − µ1 ) . . . (X − µr ) est un polynôme annulateur de u. En utilisant le lemme des noyaux,
démontrer que u est diagonalisable sur R et que le spectre de u est inclus dans l’ensemble
{µ1 , . . . , µr }.
 
a b
Un exemple où la matrice est diagonalisable sur R
c d
 
4 2
Q.9 On suppose que V = . Démontrer que V est diagonalisable sur R et donner une
−3 −1
 
α β
matrice inversible P que l’on notera P = et une matrice diagonale vérifiant V = P DP −1
γ δ
(on précisera P −1 ).
 
αIn βIn
Q.10 Soit A ∈ Mn (R). On pose alors la matrice par blocs Q = . Justifier que la matrice
γIn δIn
 
−1 4A 2A
Q est inversible, donner la matrice Q et démontrer que la matrice ∈ M2n (R)
−3A −A
 
A 0
est semblable à la matrice B = ∈ M2n (R).
0 2A
Q.11 On suppose que la matrice A est diagonalisable sur R, ce qui signifie qu’il existe une matrice R
inversible et une matrice ∆ diagonale telles que A = R∆R−1 . Calculer le produit de matrices
par blocs  −1   
R 0 R 0
B
0 R−1 0 R
 
4A 2A
Que peut-on en déduire pour la matrice ?
−3A −A
Q.12 On
 se propose de démontrer la réciproque du résultat précédent. On suppose que la matrice
4A 2A
est diagonalisable. Soit T un polynôme scindé à racines simples annulateur de
−3A −A
cette matrice,
  calculer T (A). Donner une condition nécessaire et suffisante pour que la matrice
4A 2A
soit diagonalisable.
−3A −A

2
 
a b
Un exemple où la matrice est trigonalisable sur R
c d
 
3 −2
Q.13 Démontrer que la matrice E = est trigonalisable sur R et donner une matrice inversible
2 −1
 
1 −2
P telle que E = P P −1 .
0 1
 
3A −2A
Q.14 Soit A ∈ Mn (R), démontrer que la matrice est semblable à la matrice F =
2A −A
 
A −2A
.
0 A
Q.15 On suppose que la matrice F est diagonalisable sur R. Soit U ∈ R[X] un polynôme annulateur
de F , scindé surR et à racines simples. On note U 0 le polynôme dérivé de U .
U (A) −2AU 0 (A)

Démontrer que ∈ M2n (R) est la matrice nulle.
0 U (A)
Q.16 Vérifier que le polynôme minimal de la matrice A est X. En déduire la valeur de la matrice A.
 
3A −2A
Q.17 Donner une condition nécessaire et suffisante sur la matrice A pour que la matrice
2A −A
soit diagonalisable.
Q.18 On suppose que la matrice F est trigonalisable sur R. Exprimer le polynôme caractéristique de
F en fonction de celui de A. En déduire que F est trigonalisable sur R si et seulement si A est
trigonalisable sur R.
 
3A −2A
Q.19 Donner un exemple de matrice A ∈ Mn (R) telle que la matrice ∈ M4 (R) ne soit
2A −A
pas trigonalisable sur R.

Applications
Q.20 Soit u un endomorphisme de R4 dont la matrice dans la base canonique (e1 , e2 , e3 , e4 ) de R4 est
 
1 3 2 6
2 2 4 4
M = 2 6 1

3
4 4 2 2
Déterminer deux sous-espaces vectoriels de dimension 2 stables par u.
On pourra s’inspirer de la question 10.
Q.21 En adaptant la démarche présentée dans le premier exemple de ce problème, démontrer que la
matrice  
4 0 2 0
0 4 0 2
M = 2 0 4 0

0 2 0 4
est diagonalisable sur R. Déterminer une matrice diagonale D et une matrice inversible P telles
que M = P DP −1 .
Q.22 Utiliser la question 21 pour donner les solutions du système différentiel de fonctions inconnues
x1 , x2 , x3 , x4 de la variable réelle t :



x01 = 4x1 + 2x2
x0 = 4x + 2x

2 2 4
x 0 = 2x + 4x


 3 1 3
x0 = 2x + 4x

4 2 4

3
On ne demande pas de détail.
 
a
b
Q.23 Sachant que la solution ϕ du système différentiel X 0 = M X vérifiant ϕ(0) = 

 c  est la fonction

d
 
a
b
t 7→ etM  tM
 c  où e désigne l’exponentielle de la matrice tM , déterminer la matrice e .
 M

4
CCP 2018 - MP2

Exercice I
Q.1 L’intégrale d’une fonction continue existe sur un segment et (.|.) est bien définie.
- La symétrie provient de la commutativité de la multiplication dans R.
- La linéarité par rapport à la première variable découle essentiellement de la linéarité du
passage à la limite (et de la distributivité de la multiplication sur l’addition).
- Si f ∈ E alors f 2 ≥ 0 et donc (f |f ) ≥ 0. Si cette quantité est nulle, f 2 est une fonction
continue positive d’intégrale nulle et est donc nulle. f l’est donc aussi. Ceci nous donne le
caractère défini positif.

(.|.) est un produit scalaire sur E

Q.2 On pourrait utiliser les formules de Schmidt. Cepedant, il est immédiuat que (u|v) et il nous
suffit de normer les vecteurs pour obtenir une base orthonormée.
r !
1 3
√ u, v est une b.o.n. de F
2 2

Q.3 D’après les règles de calcul en base orthogonale (et en notant p la projection orthogonale sur F )
(w|u) (w|v)
u+
p(w) = v
kuk2 kvk2
R1
Une intégration par partie donne, en posant In = −1 tn et dt,

(−1)n
In = e − − nIn−1
e
On en déduit que
1 2 5
I0 = e − , I1 = , I2 = e −
e e e
et ainsi

e + e−1 3
p(w) = u+ w
2 e
On remarque que
Z 1 2
inf et − (a + bt) dt = inf kw − f k2 = d(w, F )2
(a,b)∈R2 −1 f ∈F

D’après le cours, cette distance est atteinte pour f = p(w) et vaut donc kw − p(w)k2 . En écrivant
que w = (w − p(w)) + p(w) et en remarquant que w − p(w) et p(w) sont orthogonaux, on a alors
aussi (par Pythagore)

e2 − e−2 (w|u)2 (w|v)2


Z 1
2
inf et − (a + bt) dt = kwk2 − kp(w)k2 = − −
(a,b)∈R2 −1 2 kuk2 kvk2
Un calcul au brouillon permet de simplifier cette expression et d’ontenir
Z 1 2 7
inf et − (a + bt) dt = 1 − 2
(a,b)∈R2 −1 e

1
Exercice II
Q.4 Dans le produit proposé il y a k termes, ce qui est un nombre indépendant de n. Chacun des
termes est de limite 1 quand n → +∞. Par théorème d’opération,
 
nn−1 n−k+1
lim ... =1
n→+∞ n n n

Par définition de la loi binomiale, et en posant pn = nλ ,


 
n k
P(Xn = k) = p (1 − pn )n−k
k n
1
= n(n − 1) . . . (n − k + 1)pkn (1 − pn )n−k
k!  
1 nn−1 n−k+1
= ... (npn )k (1 − pn )n−k
k! n n n

(npn )k est égal à λk . (1 − pn )n−k = (1 − pn )−k en ln(1−pn ) tend vers 1 × e−λ (en écrivant que
ln(1 − pn ) ∼ − nλ puis par continuité de exp). Finalement

λk
lim P(Xn = k) = e−λ
n→+∞ k!

Q.5 Pour1 ≤ i ≤ n, on note Bi la variable de Bernouilli valant 1 si le candidat est interrogé le jour
1
de son anniversaire. C’est une variable de Bernoulli de paramètre 365 .
Xn est la somme de Bi . Comme les Bi sont des variables indépendantes, Xn suit une loi binomiale
(dont l’espérance est donnée par le cours ou par linéarité comme somme des espérances des Bi ).

n
Xn ,→ B(n, 1/365), E(Xn ) =
365
1
Q.6 Comme 365 ≤ 0, 01, on est dans le cadre d’approximation précédente et on peut considérer que
219
Xn suit une loi de Poisson de paramère 365 = 35 . La probabilité que Xn soit égal à 2 (c’est une
façon de comprendre l’énoncé) est approché par e−3/5 25
9 1
2! et comme −3/5 = −0, 6,

P(Xn = 2) ≈ 0, 099

On peut aussi comprendre l’énoncé comme “au moins deux étudiants sont convoqués le jours
de leur anniversaire”. Il faut alors estimer P(Xn ≥ 2) = 1 − P(Xn = 0) − P(Xn = 1) =
1 − e−0,6 (1 + 3/5). On obtient

P(Xn = 2) ≈ 0, 12

Problème
Questions préliminaires
Q.7 Par hypothèse, il existe une base de Rn dans laquelle u est représenté par D = diag(d1 , . . . , dn )
où les di sont tous des valeurs propres de u (il suffit de choisir une base de diagonalisation).
P (u) est alors représenté par P (D) = diag(P (d1 ), . . . , P (dn )). Chaque di étant racine de P , on
conclut que P (D) = 0 et donc que P (u) = 0.

P = (X − λ1 ) . . . (X − λp ) est annulateur de u

2
Q.8 Les µi étant deux à deux distincts, les polynômes X − µi sont premiers entre eux deux à deux.
Par lemme des noyaux,
Mr
ker(Q(u)) = ker(u − µi Id)
i=1
Q annulant u, cet espace est égal àRn tout entier. En ne conservant que les µi tels que ker(u −
µi Id) 6= {0} et en concaténant des bases de ces espaces, on obtient une base de Rn dans laquelle
u est représenté par une matrice diagonale dont les coefficients diagonaux font tous partie des
µi . Ainsi,

u est R-diagonalisable et Sp(u) ⊂ {µ1 , . . . , µr }

 
a b
Un exemple où la matrice est diagonalisable sur R
c d
Q.9 On a χV = X 2 − 3X + 2 = (X − 1)(X − 2) et les valeurs propres de V sont donc 1 et 2. Il y a
deux valeurs propres et on est en dimension 2 et ainsi V est diagonalisable à sous-espaces propres
de dimension 1. Comme (2, −3) et (1, −1) sont propres, ils engendrent chacun un sous-espace
propre. On a
     
−1 1 0 2 1 −1 −1 −1
V = P DP avec D = , P = , P =
0 2 −3 −1 3 2

Q.10 En faisant un produit par bloc, on vérifie que Q est inversible d’inverse
 
−1 −In −In
Q =
3In 2In
(il suffit de vérifier que QQ−1 = I2n ). Un produit par blocs montre alors que
   
−1 4A 2A A 0
Q Q=
−3A −A 0 2A

ce qui donne la similitude voulue.


Q.11 On obtien
 −1     −1   
R 0 R 0 R AR 0 ∆ 0
B = =
0 R−1 0 R 0 2R−1 AR 0 2∆

A est semblable à B elle même semblable à une matrice diagonale. Par transitivité de la relation
de similitude,
 
4A 2A
est diagonalisable
−3A −A

Q.12 On a vu que  
4A 2A
= QBQ−1
−3A −A
Appliquons le polynôme T qui annule la matrice de droite :
0 = QT (B)Q−1
En multipliant par Q−1 à gauche et Q à droite, on conclut que T (B) = 0.
On montre par une récurrence immédiate que B k = diag(Ak , (2A)k ) et en combinant linéairement,
T (B) = diag(T (A), T (2A)).
On en déduit alors que

3
T (A) = 0

Ainsi , A est diagonalisable puisqu’elle est annulée par un polynôme scindé simple. Finalement,
 
4A 2A
est diagonalisable ssi A l’est
−3A −A

 
a b
Un exemple où la matrice est trigonalisable sur R
c d
Q.13 On note f l’endomorphisme canoniquement associé à la matrice E. On a

f (1, 1) = (1, 1) et f (−1, 0) = (−3, −2) = −2(1, 1) + (−1, 0)

On peut alors obtenir la matrice de f dans la base ((1, 1), (−1, 0)) et on le traduit matriciellement
par
     
−1 1 −2 1 −1 −1 0 1
P EP = avec P = , P =
0 1 1 0 −1 1
   
In −In 0 In
Q.14 De manière similaire à précédemment, Z = est inversible d’inverse Z −1 =
In 0 −In In
et un calcul par blocs donne
   
−1 3A −2A A −2A
Z Z=
2A −A 0 A

Q.15 Montrons par récurrence que  k


A −2kAk−1

k
F =
0 Ak
- C’est vrai au rang k = 0 car F 0 = I2n .
- Supposons le résultat vrai au rang k. Il suffit alors d’un calcul par bloc pour voir que cela
reste vrai au rang k + 1.
En notant U = dk=0 uk X k , on en déduit que
P

  d
U (A) V (A) X
U (F ) = avec V (A) = −2 kuk Ak−1 = −2AU 0 (A)
0 U (A)
k=1

Comme U (F ) = 0, on en déduit que

U (A) −2AU 0 (A)


 
=0
0 U (A)

Q.16 Ce qui précède montre que U et XU 0 annulent A et sont donc multiples du polynôme minimal
de µA de A (l’ensemble des polynômes annulateurs étant l’idéal engendré par µA ). On en déduit
que µA divise U ∧ XU 0 .
Or, U étant scindé simple, U et U 0 sont premiers entre eux (aucun des diviseurs irréductible de
U ne divise U 0 ) et donc U ∧ XU 0 = U ∧ X.
Ainsi, µA est un diviseur de X. Or deg(µA ) ≥ 1 (un polynôme constant non nul n’annule aucune
matrice) et ainsi µA = X (µA est unitaire). Comme µA annule A, A est nulle.

4
µA = X et A = 0
 
3A −2A
Q.17 Si est diagonalisable alors F (qui lui est semblable) l’est aussi. On vient alors de
2A −A
voir que A = 0.  
3A −2A
Réciproquement, si A = 0 alors est nulle est donc diagonalisable.
2A −A
 
3A −2A
est diagonalisable ssi A = 0
2A −A

Q.18 χF (λ) = det(λI2n − F ) est un déterminant bloc triangulaire. Avec la formule rappelée par
l’énoncé,

χF = χ2A

Si F est trigonalisable alors χF est scindé et tout diviseur de χF l’est donc aussi. Ainsi, χA est
scindé et A est trigonalisable.
Réciproquement, si A est trigonalisable alors χA est scindé et donc χF aussi. F est alors trigo-
nalisable.

F est trigonalisable sur R si et seulement si A l’est


 
0 −1
Q.19 Soit A = diag(M, 0) avec M = . On a alors χA = X n−2 (X 2 + 1) qui n’est pas scindé
1 0
sur R et A n’est donc pas trigonalisable. Avec la question précédente, F ne l’est pas.

Applications
   
1 3 A 2A
Q.20 Si on pose V = , on a M = .
2 2 2A A
 
1 2
La matrice est diagonalisable (symétrique réelle). On vérifie aisément que (1, 1) et (1, −1)
2 1
 
I2 I2
sont vecteurs propres. Comme en Q10, on vérifie que Q = est inversible d’inverse
I2 −I2
 
I I2
Q−1 = 21 2 et que
I2 −I2
 
−1 3V 0
Q MQ =
0 −V
Cette forme diagonale par bloc montre que les sous-espaces engendré par les 2 premiers (resp. 2
derniers) vecteurs de la nouvelle base (celle formée par les colonnes de Q) engendrent un espace
stable par l’endomorphisme u.

Vect((1, 0, 1, 0), (0, 1, 0, 1)) et Vect((1, 0, −1, 0), (0, 1, 0, −1)) sont stables par u
   
4I2 2I2 4 2
Q.21 On a cette fois M = . La matrice est diagonalisable (symétrique réelle) et
2I2 4I2 2 4
on vérifie aisément  et (1, −1) sont vecteurs propres (associés
que (1, 1)   à 6et 2). Comme en Q10,
I I2 I I2
on vérifie que P = 2 est inversible d’inverse P −1 = 21 2 et que
I2 −I2 I2 −I2
 
−1 6I2 0
P MP =
0 2I2

5

x1
x2 
x3 , le système s’écrit
Q.22 En notant U =  

x4

U 0 (t) = M U (t)

Le cours nous apprend que l’ensemble des solutions est un espace vectoriel de dimension 4. Si
X est vecteur propre de M associé à λ, on vérifie que t 7→ eλt X est une solution. La question
précédente donne alors quatre solutions indépendantes qui forment une base de l’ensemble des
solutions. La solution générale est ainsi
       
1 0 1 0
0 6t 1 2t  0  2t  1 
t 7→ c1 e6t 
       
1 + c2 e 0 + c3 e −1 + c4 e  0 
0 1 0 −1

Q.23 La solution telle que ϕ(0) = (a, b, c, d) est associée à des constantes ci telles que
   
c1 a
c2   b 
Pc3  =  c 
  

c4 d

c’est à dire    
c1 a
c2 
  = P −1  b 
 
c3  c
c4 d
On a alors
         
a 1 0 1 0
b 6 0 6 1 2 0  2 1 
eM 
         
c = ϕ(1) = c1 e 1 + c2 e 0 + c3 e −1 + c4 e 0
d 0 1 0 −1
ou encore    6
e2
  
a e 0 0 a
b  0 e 6 0 2
e  −1  b 
eM 
    
= 6 2 P  
 c  e 0 −e 0  c
d 0 e 6 0 −e 2 d

e6 0 e2
 6
e + e2 e6 − e2
  
0 0 0
 0 e6 e2  e6 + e2 e6 − e2 
eM
0  P −1 = 1  0 0

=
e6 0 −e2 6 2 6 2

0  2 e −e
 0 e +e 0 
0 e6 0 −e2 0 e6 − e2 0 e6 + e2

6
E3A PC 2 2017
L'usage de la calculatrice est interdit
AVERTISSEMENT

L'épreuve est constituée d'un problème dont les trois parties sont relativement indépendantes.

La présentation, la lisibilité, l'orthographe, la qualité de la rédaction, la clarté et la précision des raisonnements entreront
pour une part importante dans l'appréciation des copies. En particulier, les résultats non justiés ne seront pas pris en compte.
Les candidats sont invités à encadrer les résultats de leurs calculs.

Partie I
Z 1
I.1) (a) Calculer f (t) = e−t s ds pour t ∈ R, si t = 0 puis t 6= 0.
0
(b) Montrer que f est une application continue sur R et établit une bijection de R sur un intervalle à
préciser.
(c) Montrer que f est développable en série entière sur R et donner son développement
Z x
I.2) Pour x ∈ R, soit S(x) = f (t) dt.
0
(a) Montrer que S est développable en série entière et donner son développement.
+∞ 1
(−1)n+1 1 − e−t
Z
(b) Justier l'égalité :
X
= dt
n=1
n(n!) 0 t
+∞
e−t
Z
I.3) (a) Pour tout x > 0, justier l'existence de R(x) = dt.
x t
1 −t +∞ −t Z +∞
1−e
Z Z
e
(b) On pose γ = S(1) − R(1) = dt − dt. Justier l'égalité : γ = − ln(t) e−t dt
0 t 1 t 0
(c) Montrer que R est de classe C 1 sur R∗+ , donner une relation entre R0 (x) et S 0 (x) pour x > 0 et
justier que : S(x) = R(x) + ln x + γ
n Z n t
xk x
I.4) (a) Pour x > 0 et n ∈ N , soit gn (x) = dt.
X


k=1
k 1 t
+∞ k Z +∞ t
x x
Pour tout x ∈]0, 1[, justier l'existence de g(x) =
X
− dt
k=1
k 1 t
xn
et prouver que, pour tout n ∈ N : 0 6 gn (x) − g(x) 6

n
(b) Prouver que la suite de fonctions (gn )n∈N∗ converge uniformément vers g sur ]0, 1[.
(c) En admettant que lim gn (1) = lim− g(x), montrer que :
n→+∞ x→1
n
!
X 1
γ = S(1) − R(1) = lim − ln(n)
n→+∞
k=1
k
Z +∞ −at
e − e−bt
I.5) Soient a > 0 et b > 0. En utilisant R(ax) − R(bx), calculer dt.
0 t
e−x
I.6) (a) Montrer que, pour tout x > 0, on a : R(x) 6 , puis que lim xR(x) = 0.
x x→+∞
(b) Au moyen d'une intégration par parties, prouver que R est intégrable sur R∗+ et
Z +∞
R(x) dx = 1
0

1
Partie II
Z +∞
II.1) (a) Pour n ∈ N, montrer l'existence de In = tn e−t dt
0
(b) Justier que In+1 = (n + 1)In . En déduire la valeur de In .
II.2) On considère l'espace vectoriel R2 [X] des polynômes réels de degré
Z 6 2. +∞
À tout P ∈ R2 [X], on associe T (P ) tel que : ∀x ∈ R, T (P )(x) = e−t P (x + t) dt.
0
(a) Montrer que T est un endomorphisme de R2 [X] et écrire sa matrice M dans la base B = 1, X, X 2 .


(b) Étudier si M est diagonalisable dans M3 (R).


II.3) Soit n ∈ N et l'espace vectoriel Rn [X] des polynômes réels de degré 6 n. On note D l'endomorphisme
de Rn [X] associant à tout polynôme P son polynôme dérivé P 0 .
n
(a) Soit P ∈ Rn [X] et (x, t) ∈ R . Déterminer des réels b0 (x), . . . , bn (x) tels que P (x + t) = tk bk (x).
X
2

k=0
Indication : On pourra citer et utiliser une formule de Taylor.
Z +∞
(b) À tout P ∈ Rn [X], on associe T (P ) tel que : ∀x ∈ R, T (P )(x) = e−t P (x + t) dt.
0
Montrer que T est un endomorphisme de Rn [X] et déterminer des réels a0 , . . . , an tels que pour
n
tout P ∈ Rn [X] on ait : T (P ) = ak Dk (P ).
X

k=0
(c) Déterminer les éléments propres de T (valeurs propres et vecteurs propres).
II.4) Soit g : R → R, une fonction continue et bornée. Déterminer y ∈ C 1 (R, R) solution de l'équation
diérentielle sur R : y 0 − y + g = 0. Z +∞
Justier que la solution générale est de la forme : y : x 7−→ kex + ex e−t g(t) dt, avec k ∈ R
x
II.5) Soit g : R → R continue et bornée et soit N∞ (g) = sup {|g(t)| , t ∈ R}.
Z +∞
(a) On dénit Tg : R → R par : ∀x ∈ R, Tg (x) = e−t g(x + t) dt.
0
Z +∞
Justier qu'alors Tg (x) = e x
e g(u) du, et que Tg est de classe C 1 sur R en précisant (Tg )0
−u
x
en fonction de Tg et g .
(b) En supposant g non nulle, déterminer s'il existe λ ∈ R tel que Tg = λg .
(c) Montrer qu'en général, Tg est bornée sur R et majorer N∞ (Tg ) au moyen de N∞ (g).
(d) Montrer que si g tend vers 0 en +∞, alors Tg aussi.
Indication : on vériera que si |g(t)| 6 ε pour t > A, alors |Tg (x)| 6 ε pour x > A.
Z +∞
II.6) (a) Pour tout réel A, justier l'existence et calculer e(i−1)t dt.
A
(b) Soit c : t 7−→ cos(t), s : t 7−→ sin(t) et F le sous-espace vectoriel de C (R, R) engendré par (c, s).
Montrer que g 7→ Tg (où Tg déni ci-dessus) dénit un endomorphisme de F et écrire sa matrice
N dans la base (c, s). N est-elle diagonalisable dans M2 (R) ?

2
Partie III

On s'intéresse dans cette partie à l'équation diérentielle : xy 00 + y 0 − (x + 1)y = 1.


III.1) On suppose qu'il existe une solution θ développable en série entière de cette équation diérentielle. On
+∞
note alors θ(x) = an xn pour tout x ∈] − r, r[ où r > 0 est le rayon de convergence et (an )n∈N une
X

n=0
suite réelle.
(a) Déterminer alors une relation entre a1 et a0 , ainsi qu'une relation entre an+2 , an+1 et an pour tout
n ∈ N.
K
(b) Pour une telle suite (an )n∈N , montrer qu'il existe K > 0 telle que : ∀n ∈ N, |an | 6 .
n!
En déduire qu'une telle solution θ existe et que de plus r = +∞.
III.2) On souhaite résoudre ici cette équation diérentielle sur l'intervalle I = R∗+ et l'on note :
n o
S = y ∈ C 2 (I, R) ∀x > 0, xy 00 (x) + y 0 (x) − (x + 1)y(x) = 1

(a) Pour tout y ∈ C 2 (I, R), on pose z(x) = e−x y(x) pour tout x > 0.
Montrer que y ∈ S si et seulement si z vérie :
∀x > 0, xz 00 (x) + (2x + 1)z 0 (x) = e−x (?).

(b) Déterminer les Z ∈ C 1 (I, R) telles que :


∀x > 0, xZ 0 (x) + (2x + 1)Z(x) = 0.

(c) Déterminer les Z ∈ C 1 (I, R) telles que :


∀x > 0, xZ 0 (x) + (2x + 1)Z(x) = e−x .

(d) En déduire l'expression des fonctions


Z z ∈ C (I, R) vériant (?) de III.2.(a), en utilisant la fonction
2
+∞
e−t
R dénie pour x > 0 par R(x) = dt : On utilisera R(x) et R(2x).
x t
(e) Donner alors l'expression de la solution générale y ∈ S .
III.3) (a) Sachant que R(x) = − ln(x) + γ + o(1) quand x → 0 avec x > 0, déterminer les solutions y ∈ S
ayant une limite nie en 0.
Exprimer alors ces solutions en utilisant la fonction S de la partie I et reliée à R par :
S(x) = R(x) + ln(x) + γ pour x > 0 (vu en I.3.)(c)).
(b) Sachant que S est développable en série entière sur R, donner l'expression des solutions f de la
question III. 1) : on exprimera f (x) en fonction de S(x) et S(2x) pour tout x ∈ R.
Comment pourrait-on alors obtenir une expression des suites (an )n∈N de III. 1) ?

3
Corrigé de E3A 2017 PC math 2

Partie I.
 −ts s=1
e 1 − e−t
1) a) Pour t = 0: f (0) = 1. Pour t 6= 0: f (t) = = .
−t s=0 t
b) f est clairement continue sur R∗ . Le développement limité en 0 de e−t = 1 − t + o(t) entraine que
lim f (t) = 1 = f (0). f est donc continue sur R. De plus, f est strictement décroissante sur R puisque,
t→0
e−t
pour s ∈]0, 1], on a t < u ⇒ e−su < e−st ⇒ f (u) < f (t). Enfin, lim f (t) = lim = +∞ et
t→−∞ t→−∞ −t
lim f (t) = 0.
t→+∞
f est donc une bijection de R sur ]0, +∞[.
X (−t)n 1 − e−t X (−1)n−1 tn−1
c) e−t = avec un rayon de convergence infini donc f (t) = = pour
n! t n!
n>0 n>1
t 6= 0, égalité qui s’étend à t = 0 puisque f (0) = 1. Le rayon de convergence est infini.
2) a) Puisque le rayon de convergence est infini on peut intégrer terme à terme ce développement en série
X (−1)n−1 xn
entière sur [0, x] pour tout x réel et obtenir: S(x) =
n(n!)
n>1
1 1
X (−1)n−1 1 − e−t
Z Z
b) Pour x = 1 on obtient S(1) = = f (t)dt = dt.
n(n!) 0 0 t
n>1

e−t
3) a) R(x) est défini pour tout x > 0 puisque t 7→ est continue sur [x, +∞[ et que, au voisinage de
t
e−t
+∞, = o(e−t ) qui est intégrable sur [x, +∞[.
t
Z 1 1 Z 1
e−t − 1 1 − e−t
Z
1
ln(t)e−t dt = ln(t)(e−t − 1) 0 −

b) En intégrant par parties: − dt = dt =
0 0 t 0 t
S(1). Cette intégration par parties est légitime puisque l’intégrale S(1) existe, que ln(1) = 0 et qu’en
0 on a ln(t)(e−t − 1) ∼ −t ln(t) qui a pour limite 0.
Z +∞ Z +∞ −t Z +∞ −t
−t
 −t +∞
 e e
En intégrant par parties: − ln(t)e dt = ln(t)e 1 − dt = − dt =
1 1 t 1 t
−R(1). Cette intégration par parties est légitime puisque l’intégrale R(1) existe, que ln(1) = 0
et que ln(t)e−t a pour limite 0 en +∞.
Z +∞
On a bien obtenu γ = S(1) − R(1) = − ln(t)e−t dt.
0
Z x −t
e e−x
c) Par la relation de Chasles: R(x) = R(1) − dt est de classe C 1 sur R∗+ et R0 (x) = − .
1 t x
Z x
1 − e−x
De S(x) = f (t)dt on déduit S 0 (x) = f (x) = pour x 6= 0.
0 x
1
On en déduit S 0 (x) = R0 (x) + pour x > 0. Les fonctions S et x 7→ R(x) + ln(x) + γ ont des dérivées
x
égales sur l’intervalle ]0, +∞]. Comme elles prennent la même valeur pour x = 1 (S(1) = R(1) + γ)
elles sont égales sur l’intervalle ]0, +∞[.
xk
4) a) La série entière de terme général a un rayon de convergence égal à 1 (le terme général tend vers
k
0 pour 0 < x < 1 et tend vers l’infini pour x > 1) donc la série converge pour x ∈]0, 1[.
xt xt
La fonction t 7→ est continue sur [1, +∞[ et 0 6 6 et ln(x) qui est intégrable sur [1, +∞[ puisque
t t
ln(x) < 0. g(x) est donc bien défini pour x ∈]0, 1[.
Z +∞ t +∞
x X xk
gn (x) − g(x) = dt − .
n t k
k=n+1
1
Les fonctions t 7→ et t → 7 xt = et ln(x) sont décroissantes et positives sur ]1, +∞[ (ln(x) < 0) donc
t Z k+1 t Z N +1 t
xt xk+1 x xk x
t 7→ l’est aussi. On peut donc écrire 6 dt 6 d’où en ajoutant, dt 6
t k+1 k t k n+1 t
N Z N t Z +∞ t +∞ Z +∞ t
X xk x x X xk x
6 dt puis en faisant tendre N vers l’infini: dt 6 6 dt.
k n t n+1 t k n t
k=n+1 k=n+1

1
+∞ +∞ Z n+1 t
xt xk xn
Z X x
On en déduit 0 6 gn (x) − g(x) = dt − 6 dt 6 .
n t k n t n
k=n+1
1 1
b) On en déduit pour tout x ∈]0, 1[: 0 6 gn (x) − g(x) 6 donc N∞ (gn − g) 6 qui a pour limite 0
n n
donc la suite gn converge uniformément vers g sur ]0, 1[.
X xn
c) En utilisant le développement en série entière ln(1 + x) = (−1)n−1 valable pour |x| < 1 on
n
n>1
X xn
obtient = − ln(1 − x) pour x ∈]0, 1[.
n
n>1
Z +∞ t Z +∞ t ln(x) Z +∞ −u
x e e
D’autre part dt = dt = du = R(− ln(x)) car t 7→ u = −t ln(x) est
1 t 1 t − ln(x) u
une bijection de classe C 1 de [1, +∞[ sur [− ln(x), +∞[ quand x ∈]0, 1[.
On en déduit pour x ∈]0, 1[: g(x) = − ln(1−x)−R(− ln(x)) = − ln(1−x)−S(− ln(x))+ln(− ln(x))+γ
en utilisant le I.3) c).
 
− ln(x)
Par suite g(x) = ln − S(− ln(x)) + γ.
1−x
ln(x)
De lim = ln0 (1) = 1 et de la continuité de S en 0 on obtient lim g(x) = ln(1)−S(0)+γ = γ. En
x→1 x − 1 x→1−
n n
X 1 X 1
admettant lim gn (1) = lim− g(x) = γ on obtient avec gn (1) = −ln(n): lim −ln(n) = γ.
n→∞ x→1 k n→∞ k
k=1 k=1

+∞ +∞
e−t e−au
Z Z
5) R(ax) = dt = du puisque u 7→ au est une bijection de classe C 1 de [x, +∞[ sur
ax t x u
Z +∞ −au
e
[ax, +∞[ (pour x > 0 et a > 0). De même R(bx) = du. On obtient donc R(ax) − R(bx) =
x u
Z +∞ −au
e − e−bu
du. D’autre part en utilisant le I.3) c):
x u
R(ax) − R(bx) = S(ax) − S(bx) − ln(ax) + ln(bx) = S(ax) − S(bx) + ln(b) − ln(a).
Z +∞ −au
e − e−bu
Par continuité de S en 0 on déduit en faisant tendre x vers 0: du = ln(b) − ln(a).
0 u
Z +∞ −t Z +∞ −t
e e 1 e−x
6) a) Pour x > 0 on a: R(x) = dt 6 dt = [−e−t ]+∞x = .
x t x x x x
Par suite 0 6 xR(x) 6 e−x d’où lim xR(x) = 0.
x→+∞

b) Avec le I.3) c): R(x) = − ln(x) + S(x) − γ qui donne R(x) ∼ − ln(x) quand x tend vers 0. On en
déduit lim xR(x) = lim −x ln(x) = 0.
x→0 x→0
Z +∞ Z +∞ Z +∞
0
On peut donc intégrer par parties: +∞
R(x)dx = [xR(x)]0 − xR (x)dx = e−x dx
0 Z +∞ 0 0

puisque xR0 (x) = −e−x (montré au I.3)c)). On obtient ensuite R(x)dx = [−e−x ]+∞
0 = 1.
0

Partie II
 
n −t n −t 1
1) a) In existe puisque t 7→ t e est continue et t e =o en +∞.
t2
Z +∞
b) On intègre par parties: In+1 = [−tn+1 e−t ]+∞
0 + (n+1)tn e−t dt = (n+1)In car lim tn+1 e−t = 0.
0 t→+∞

On montre par récurrence sur n que In = n!. C’est vérifié pour n = 0 puisque I0 = [−e−t ]+∞
0 = 1.
Supposons In = n! pour un entier n ∈ N; on en déduit In+1 = (n + 1)In = (n + 1)!.
La proposition est donc vraie pour tout n ∈ N.
2) a) Puisque P (x + t) est une combinaison linéaire des tk on déduit que T (P )(x) est bien défini comme
combinaison linéaire des Ik .
T est une application linéaire puisque T (λP +Q)(x) = λT (P )(x)+T (Q)(x) par linéarité de l’intégrale.

2
Z +∞
On calcule T (1)(x) = I0 = 1, T (X)(x) = e−t (x + t)dt = xI0 + I1 = 1 + x donc T (X) = 1 + X.
0
Z +∞
2
T (X )(x) = e−t (x + t)2 dt = x2 I0 + 2xI1 + I2 = 2 + 2x + x2 donc T (X 2 ) = 2 + 2X + X 2 .
0
T est donc bien un endomorphisme de R2 [X].
 
1 1 2
Sa matrice dans la base canonique de R2 [X] est M =  0 1 2 .
0 0 1
b) M est triangulaire et a donc comme unique valeur propre 1 (d’ordre 3). Si elle était diagonalisable elle
serait semblable à la matrice diagonale ayant des 1 sur la diagonale, c’est-à-dire la matrice identité
et on aurait alors M = I3 ce qui est faux: M n’est donc pas diagonalisable.
3) a) Appliquons la formule de Taylor pour les polynômes. Pour un polynôme de degré au plus n on a:
n n
X P (k) (x) k X P (k) (x)
P (x + t) = t donc P (x + t) = tk bk (x) en posant bk (x) = .
k! k!
k=0 k=0
b) T est une application linéaire puisque T (λP +Q)(x) = λT (P )(x)+T (Q)(x) par linéarité de l’intégrale.
P (x + t) étant combinaison linéaire des tk (0 6 k 6 n) on obtient que T (P )(x) est combinaison
linéaire des Ik (0 6 k 6 n):
n n n
X X P (k) (x) X (k)
T (P )(x) = Ik bk (x) = Ik = P (x) puisque Ik = k!.
k!
k=0 k=0 k=0
n
X
On obtient donc T (P ) = P (k) qui appartient bien à Rn [X]. On a ak = 1 pour k ∈ [[0, n]].
k=0

c) Le calcul de T (X ) = X + kX k−1 + ... + k! donne la matrice de T dans la base canonique de Rn [X]:


k k

1 1 2 ... n!
 
 0 1 2 ... n! 
M =  0 0 1 ... n!/2 .
 
... ... ... ... ...
 
0 0 0 ... 1
M est triangulaire et a donc comme unique valeur propre 1 (d’ordre n + 1). Le sous-espace propre
associé à 1 est l’ensemble des polynômes P qui vérifient T (P ) = P , c’est-à-dire P 0 +P 00 +...+P (n) = 0.
C’est équivalent à P 0 = 0 puisque si P 0 n’était pas nul, P 0 + P 00 + ... + P (n) aurait le degré de P 0 .
Le sous-espace propre associé à 1 est donc l’ensemble des polynômes constants.
d −x
4) En la multipliant par e−x l’équation différentielle devient (e y) = −e−x g(x) d’où e−x y(x) − y(0) =
Z x dx
− e−t g(t)dt.
0
Puisque g est continue et bornée (par M ), on a |g(x)e−x | 6 M e−x qui est intégrable sur R+ . On peut
Z +∞ Z +∞ Z +∞
donc écrire e−x y(x) − y(0) = − e−t g(t)dt + e−t g(t)dt d’où y(x) = kex + ex e−t g(t)dt en
Z +∞ 0 x x
−t
posant k = y(0) − e g(t)dt.
0

5) a) Tg est bien définie puisque g est continue et bornée donc |g(x + t)e−t | 6 M e−t qui est intégrable sur
R+ . Le changement de variable défini par t 7→ u = t + x est une bijection de classe C 1 de [0, +∞[ sur
Z +∞ Z +∞ Z +∞
−t −u+x
[x, +∞[ donc on peut écrire Tg (x) = e g(x + t)dt = e g(u)du = ex
e−u g(u)du.
0 x x
Z +∞ Z +∞ Z x
−u −u
Puisque e g(u)du = e g(u)du − e−u g(u)du on obtient par dérivation:
x 0 0
Tg0 (x) = Tg (x) + ex (−e−x g(x)) d’où (Tg )0 = Tg − g.
b) D’après ce qui précède, Tg = λg entraine λg 0 = (λ − 1)g. Si λ = 0 on obtient g = 0 qui est exclu
λ−1
par l’énoncé. Si λ 6= 0, on obtient g(x) = ke λ x avec k ∈ R∗ . Comme g doit être bornée sur R, la
seule possibilité est λ = 1 et g = k constante. Réciproquement si g = k constante on obtient bien
Tg (x) = k donc Tg = g.
Si g est constante non nulle, seul λ = 1 convient. Sinon il n’existe pas de λ tel que Tg = λg.
c) Puisque g est bornée on a pour tout x ∈ R: |g(x)| 6 N∞ (g). On en déduit:
Z +∞
|Tg (x)| 6 e−t N∞ (g)dt = N∞ (g). On a donc N∞ (Tg ) 6 N∞ (g).
0

3
d) Si g tend vers 0 en +∞, pour tout ε > 0 il existe A ∈ R+ tel que |g(x)| 6 ε pour x > A. On en
Z +∞
déduit pour x > A: |Tg (x)| 6 ε e−t dt = ε puisque |g(x + t)| 6 ε pour t > 0. On a bien montré
0
que Tg tend vers 0 en +∞.

6) a) L’application t 7→ e(i−1)t est continue et intégrable sur [A, +∞[ puisque |e(i−1)t | = e−t est intégrable
sur R+ donc sur [A, +∞[ pour tout A ∈ R.
Z +∞  (i−1)t +∞
e e(i−1)A
e(i−1)t dt = = puisque |e(i−1)t | = e−t tend vers 0 en +∞.
A i − 1 A 1 − i
b) L’application g 7→ Tg est clairement linéaire.
Z +∞
eix
Pour g(t) = eit on calcule Tg (x) = ex e−u eiu du = avec le calcul du II.6)a).
x 1−i
1 1 i
On a donc Tg (x) = (cos(x) + i sin(x))(1 + i) = (cos(x) − sin(x)) + (cos(x) + sin(x)).
2 2 2
Par linéarité de T on en déduit:
1 1
Tc (x) = Re(Tg (x)) = (cos(x) − sin(x)) et Ts (x) = Im(Tg (x)) = (cos(x) + sin(x)).
2 2
1 1
On a donc Tc = (c − s) et Ts = (c + s). L’application t 7→ Tg est bien un endomorphisme de F et
2 2  
1 1 1
sa matrice dans la base (c, s) est N = .
2 −1 1
N
 n’est pas
 diagonalisable
 dans M2 (R) puisque son polynôme caractéristique égal à χN (x) =
1 1 1 1
−x − x + = x2 − x + n’a pas de racine réelle (∆ = 1 − 2 = −1 < 0).
2 2 4 2

Partie III

1) a) Pour x ∈] − r, r[ on peut dériver terme à terme et obtenir:


X X
θ0 (x) = nan xn−1 et θ00 (x) = n(n − 1)an xn−2 .
n>0 n>0
En reportant dans l’équation différentielle on obtient:
X X X X X X
n(n−1)an xn−1 + nan xn−1 − an xn − an xn+1 = 1 ou encore n2 an xn−1 − an xn −
n>0 n>0 n>0 n>0 n>0 n>0
X
an xn+1 = 1. Posons n = n0 + 2 dans la première somme et n = n0 + 1 dans la deuxième:
n>0
X 0 X 0 X
(n0 + 2)2 an0 +2 xn +1 − an0 +1 xn +1 − an xn+1 = 1 d’où en regroupant:
n0 >−2 n0 >−1 n>0
X
2 n+1
a1 − a0 + ((n + 2) an+2 − an+1 − an )x = 1.
n>0
Par unicité des coefficients d’un développement en série entière on en déduit: a1 − a0 = 1 et pour
n > 0: (n + 2)2 an+2 − an+1 − an = 0.
b) Soit K = max(|a0 |, |a1 |). Montrons par récurrence double sur n que pour tout n on a n!|an | 6 K.
C’est vérifié pour n = 0 et n = 1. Supposons la propriété vraie pour n et n + 1. On en déduit:
(n + 1)! (n + 1)! 1
(n + 2)!|an+2 | = |an+1 + an | 6 (|an+1 | + |an |) 6 (K + (n + 1)K) = K en
n+2 n+2 n+2
utilisant l’hypothèse de récurrence pour n et n + 1. On a donc bien montré que pour tout n > 0:
K
|an | 6 .
n!
Soit une série entière associée à une suite (an ) vérifiant a1 − a0 = 1 et pour n > 0: (n + 2)2 an+2 −
K
an+1 −an = 0. Puisque pour tout n > 0 elle vérifie |an | 6 on en déduit que le rayon de convergence
n!
est infini, donc la somme de la série vérifie l’équation différentielle sur R.
2) a) De z(x) = e−x y(x) on déduit z 0 (x) = −e−x y(x)+e−x y 0 (x) et z 00 (x) = e−x y(x)−2e−x y 0 (x)+e−x y 00 (x).
On en déduit xz 00 (x) + (2x + 1)z 0 (x) = e−x (xy(x) − 2xy 0 (x) + xy 00 (x)) + (2x + 1)(−y(x) + y 0 (x)) =
e−x (xy 00 (x) + y 0 (x) − (x + 1)y(x)).
Par suite y est dans S si et seulement si xz 00 (x) + (2x + 1)z 0 (x) = e−x .

4
2x + 1 2x + 1
b) Pour x > 0 l’équation s’écrit Z 0 (x) + Z(x) = 0. Une primitive de est 2x + ln(x).
x x
En multipliant l’équation par e2x+ln(x) = xe2x elle se réécrit: (xe2x Z(x))0 = xe2x Z 0 (x) + (2x +
λ
1)e2x Z(x) = 0 d’où les solutions Z(x) = e−2x .
x
c) Le même calcul qu’au b) donne ici (xe2x Z(x))0 = ex qui s’intègre en (xe2x Z(x)) = ex + λ d’où
1
Z(x) = (e−x + λe−2x ).
x
On peut aussi utiliser le III.2)b) et appliquer la méthode de variation de la constante.
Z x −t Z x −2t
e e
d) Il suffit d’intégrer Z pour obtenir z(x) = dt + λ dt + C. En introduisant R(x) =
1 t 1 t
Z +∞ −t Z x −t Z +∞ −t Z +∞ −t Z +∞ −2u
e e e e e
dt = − dt + dt et de même R(2x) = dt = du =
xZ t 1 t 1 t 2x t x u
x −2t Z +∞ −2t
e e
− dt + dt on obtient z(x) = −R(x) − λR(2x) + µ où λ et µ sont des réels quel-
1 t 1 t
conques.
e) La solution générale y ∈ S s’en déduit: y(x) = ex z(x) = −ex R(x) − λex R(2x) + µex où λ et µ sont
des réels quelconques.
3) a) En reportant R(x) = − ln(x) − γ + o(1) et de même pour R(2x) dans le III.2)e) on obtient:
y(x) = ex (ln(x) + γ + λ(ln(2x) + γ) + µ + o(1)) = ex (ln(x)(1 + λ) + γ + λ(ln(2) + γ) + µ + o(1)).
Cette expression n’a une limite finie en 0 que pour λ = −1 puisque ln(x) tend vers −∞.
On a alors avec R(x) = S(x) − ln(x) − γ:
y(x) = ex (R(2x) − R(x) + µ) = ex (S(2x) − S(x) + µ0 ) pour x > 0 en posant µ0 = − ln(2) + µ.
b) En posant comme au III.2)a) z(x) = e−x y(x) = S(2x) − S(x) + µ0 on calcule pour x 6= 0:
1 − e−2x 1 − e−x e−x − e−2x
z 0 (x) = 2S 0 (2x) − S 0 (x) = 2 − =
2x x x
00 e−x − e−2x −e−x + 2e−2x
z (x) = − +
x2 x
e−x − e−2x e−x − e−2x
D’où xz 00 (x) + (2x + 1)z 0 (x) = − + −e−x + 2e−2x + 2(e−x − e−2x ) + = e−x .
x x
C’est vrai aussi pour x = 0 donc z vérifie l’équation (*) pour tout x ∈ R et par suite y(x) =
ex (S(2x) − S(x) + µ0 ) vérifie l’équation xy 00 + y 0 − (x + 1)y = 1. Comme S est développable en série
entière on en déduit par application du produit de Cauchy que y est développable en série entière
avec un rayon de convergence infini. Au III.1) on a trouvé que l’équation xy 00 + y 0 − (x + 1)y = 1
possède une unique solution possédant un développement en série entière si on impose la condition
initiale y(0) = a0 . On peut donc écrire θ(x) = ex (S(2x) − S(x) + a0 ) pour tout x ∈ R.
L’application du produit de Cauchy permet d’en déduire une expression de an .

Complément: calculons une expression de an . On peut se limiter au cas où a0 = 0.


X X (−1)n−1 (2n − 1)xn X xn
De θ(x) = an xn , S(2x) − S(x) = et ex = on déduit:
n(n!) n!
n>1 n>1 n>0
n n
(−1)k−1 (2k − 1)
  k
1 k−1 n (2 − 1)
X X
an = d’où n!an = (−1) .
k(k!) (n − k)! k k
k=1 k=1
n Z 2 Z 2 n Z 2
(1 − t)n − 1
 
k−1 n
X X
On peut simplifier en écrivant: n!an = (−t) dt = dt = (1 −
k (1 − t) − 1
k=1 1 1 k=1 1
n  2 X n
k−1
X (1 − t)k (−1)k−1
t) dt = − = .
k 1 k
k=1 k=1
n
1 X (−1)k−1
On obtient finalement an = .
n! k
k=1
On peut aussi retrouver cette expression en utilisant la relation (n+2)2 an+2 −an+1 −an = 0 et a1 = 1.
On pose bn = n!an qui donne (n + 2)bn+2 − bn+1 − (n + 1)bn = 0 ou encore (n + 2)(bn+2 − bn+1 ) =
n
X (−1)k−1
−(n + 1)(bn+1 − bn ) qui avec b1 = 1 et b0 = 0 donne n(bn − bn−1 ) = (−1)n−1 d’où bn = .
k
k=1

5
SESSION 2016 MPMA102  

 
 
 
EPREUVE SPECIFIQUE - FILIERE MP  
____________________  
 
MATHEMATIQUES 1

Mardi 3 mai : 14 h - 18 h  
____________________  

N.B. : le candidat attachera la plus grande importance à la clarté, à la précision et à la concision de


la rédaction. Si un candidat est amené à repérer ce qui peut lui sembler être une erreur d’énoncé, il le
signalera sur sa copie et devra poursuivre sa composition en expliquant les raisons des initiatives
qu’il a été amené à prendre.  
 
___________________________________________________________________________________  
 
 
 
  Les calculatrices sont interdites
 
 
 
 
 
 
  Le sujet est composé de deux exercices et d’un problème tous indépendants.
 
 
 
 
 
 
 
 
 
 
 
 
 
 
 
 
 
 

1/4
EXERCICE I

On considère l’équation différentielle (E) : x2 y + (x2 − x) y + 2 y = 0.

I.1. Existe-t-il des solutions non nulles de l’équation (E) développables en série entière sur un
intervalle ]−r, r[ (r > 0) de R?

EXERCICE II
 
i+ j
II.1. Démontrer que la famille est sommable et calculer sa somme.
2i+ j (i, j)∈N²

II.2. Soit X et Y deux variables aléatoires sur un même espace probabilisé à valeurs dans N.
On suppose que la loi conjointe du couple (X,Y ) vérifie :
i+ j
pour tout (i, j) ∈ N², P(X = i,Y = j)= P [(X = i) ∩ (Y = j)] = i+ j+3 .
2
II.2.a. Vérifier que la relation ci-dessus définit bien une loi conjointe.

II.2.b. Démontrer que les variables aléatoires X et Y suivent une même loi.

II.2.c. Les variables aléatoires X et Y sont-elles indépendantes?

PROBLÈME : Fonction Digamma

Partie préliminaire

III.1.

III.1.a. Soit x ∈ ]0, + ∞[, démontrer que la fonction t → e−t t x−1 est intégrable sur ]0, + ∞[.
 +∞
III.1.b. On note, pour tout x ∈ ]0, + ∞[, Γ(x) = e−t t x−1 dt (fonction Gamma d’Euler).
0

Démontrer que pour tout x ∈ ]0, + ∞[, Γ(x) > 0.

III.1.c. Démontrer que la fonction Γ est dérivable sur ]0, + ∞[ puis exprimer Γ (x) sous forme
d’intégrale.
 n
1 1
III.2. Pour tout entier n ≥ 2, on pose un = dt − .
n−1 t n

III.2.a. Utiliser un théorème du cours pour justifier simplement que la série ∑ un converge.
n≥2

n
1
III.2.b. Pour tout entier n ≥ 1, on pose Hn = ∑ k − ln(n).
k=1
Démontrer que la suite (Hn )n≥1 converge.

2/4
La limite de la suite (Hn )n≥1 sera notée γ dans tout le sujet (γ est appelée la constante d’Euler). Dans
Γ (x)
la suite de ce problème, on définit pour tout x ∈ ]0, + ∞[, ψ(x) = appelée fonction Digamma.
Γ(x)

Expression de la fonction Digamma à l’aide d’une série

III.3. Pour x ∈ ]0, + ∞[ et pour tout entier


 nt ≥ n1, on définit la fonction fn sur ]0, + ∞[ telle que :
pour tout t ∈ ]0,n], fn (t) = 1 − t x−1 et pour tout t ∈ ]n, + ∞[ , fn (t) = 0.
n
III.3.a. Démontrer que pour tout x < 1, ln(1 − x) ≤ −x.

En déduire que pour tout entier n ≥ 1 , pour tout x ∈ ]0, + ∞[ et tout t ∈ ]0, + ∞[ , 0 ≤ fn (t) ≤ e−t t x−1 .

III.3.b. En utilisant le théorème de convergence dominée, démontrer que pour tout x ∈ ]0, + ∞[,
n t n x−1
Γ(x) = lim 1− t dt.
n→+∞ 0 n
 1
III.4. On pose, pour n entier naturel et pour x ∈ ]0, + ∞[, In (x) = (1 − u)n ux−1 du.
0

III.4.a. Après avoir justifié l’existence de l’intégrale In (x), déterminer, pour x > 0 et pour n ≥ 1,
une relation entre In (x) et In−1 (x + 1).

III.4.b. En déduire, pour n entier naturel et pour x ∈ ]0, + ∞[ une expression de In (x).
n! nx
III.4.c. Démontrer que, pour tout x ∈ ]0, + ∞[, Γ(x) = lim (formule de Gauss).
n→+∞ n
∏ (x + k)
k=0
n
1
III.5. Pour tout entier n ≥ 1, on note toujours Hn = ∑ k − ln(n).
k=1
1 n  x n 
x  −x 
En remarquant que pour n ≥ 1 et x ∈ ]0, + ∞[, x ∏ 1 + = e x Hn ∏ 1 + e k , démontrer
n k=1 k k=1 k
n 
1 x  −x 
que pour tout x ∈ ]0, + ∞[, = x e lim ∏ 1 +
γx
e k (formule de Weierstrass).
Γ(x) n→+∞
k=1 k

III.6.
  x x
III.6.a. En déduire que la série ∑ ln 1 +
k

k
converge simplement sur ]0, + ∞[ .
k≥1

+∞ 
 x x
III.6.b. On pose, pour tout x ∈ ]0, + ∞[, g(x) = ∑ ln 1 +
k
− . Démontrer que l’application
k
k=1
1
g est de classe C sur ]0, + ∞[ et exprimer g (x) comme somme d’une série de fonctions.


+∞  
−1 1 1
III.6.c. En déduire que, pour tout x ∈ ]0, + ∞[, ψ(x) = −γ + ∑ − . On rappelle
x k=1 k k+x
Γ (x)
que pour tout x ∈ ]0, + ∞[, ψ(x) = .
Γ(x)

3/4
III.7.
 +∞
III.7.a. Que vaut ψ(1)? En déduire la valeur de l’intégrale e−t ln(t) dt.
0

III.7.b. Calculer, pour tout x ∈ ]0, + ∞[, ψ(x + 1) − ψ(x) puis démontrer que, pour tout entier
n−1
1
n ≥ 2, ψ(n) = −γ + ∑ .
k=1 k

1 1
III.7.c. On pose, pour tout (x,y) ∈ ]0, + ∞[2 et k entier naturel, jk (y) = − .
k+y+1 k+y+x

Démontrer que la série ∑ jk converge uniformément sur ]0, + ∞[.


k≥0

En déduire lim (ψ(x + n) − ψ(1 + n)).


n→+∞

III.8. Déterminer l’ensemble des applications f définies sur ]0, + ∞[ et à valeurs réelles vérifiant
les trois conditions :

• f (1) = −γ ,
1
• pour tout x ∈ ]0, + ∞[, f (x + 1) = f (x) + ,
x
• pour tout x ∈ ]0, + ∞[, lim ( f (x + n) − f (1 + n)) = 0.
n→+∞

Autour de la fonction Digamma

III.9. Une urne contient n boules numérotées de 1 à n.

On effectue un premier tirage d’un boule dans l’urne et on adopte le protocole suivant :
si on a tiré la boule numéro k, on la remet alors dans l’urne
avec k nouvelles boules toutes numérotées k.
Par exemple, si on a tiré la boule numéro 3, on remet quatre boules de numéro 3 dans l’urne (la boule
tirée plus 3 nouvelles boules numéro 3).

On effectue ensuite un deuxième tirage d’une boule.


I M P R I M E R I E N A T I O N A L E – 16 1213 – D’après documents fournis
On note X (respectivement Y ) la variable aléatoire égale au numéro de la boule choisie au premier
tirage (respectivement au deuxième tirage).

III.9.a. Déterminer la loi de la variable aléatoire X ainsi que son espérance E(X).

III.9.b. Déterminer
 la loi de la variable aléatoire
 Y et vérifier que pour tout entier naturel non
1 k
nul k, P(Y = k) = ψ(2n + 1) − ψ(n + 1) + .
n n+k
III.9.c. Calculer l’espérance E(Y ). On pourra utiliser, sans démonstration, que
n
k2 1−n
∑ n(n + k) = 2 + n (ψ(2n + 1) − ψ(n + 1)).
k=1

Fin de l’énoncé

4/4
CCP 2016 - Filière MP
Corrigé de l’épreuve Mathématiques I
Nicolas Basbois & Damien Broizat
Institut Stanislas, Cannes - Lycée Jules Ferry

EXERCICE I
I.1. Supposons que l’équation différentielle (E) possède une solution développable en série
+∞
X
entière sur ] − r; r[ (avec r > 0), notée y : x 7→ an xn . En dérivant deux fois cette
n=0
série entière terme à terme sur son intervalle ouvert de convergence, on obtient pour tout
x ∈] − r; r[ :
+∞
X +∞
X +∞
X +∞
X +∞
X
(x2 −x)y 0 (x) = (x2 −x) nan xn−1 = nan xn+1 − nan xn = (n−1)an−1 xn − nan xn ,
n=1 n=0 n=0 n=1 n=0
ainsi que
+∞
X +∞
X +∞
X
2 00 2 n−2 n
x y (x) = x n(n − 1)an x = n(n − 1)an x = n(n − 1)an xn .
n=2 n=2 n=0
En sommant ces développements en série entière, il vient, pour tout x ∈] − r; r[ :
+∞
X +∞
X +∞
X +∞
X
n n n
x2 y 00 (x) + (x2 − x)y 0 (x) + 2y(x) = n(n − 1)an x + (n − 1)an−1 x − nan x + 2an xn
n=0 n=1 n=0 n=0

+∞
X
(n2 − 2n + 2)an + (n − 1)an−1 xn + 2a0 .

=
n=1
Puisque yest solution de (E), on obtient par unicité du développement en série entière les
2a0 = 0
relations .
∀n ≥ 1, (n2 − 2n + 2)an + (n − 1)an−1 = 0

2 2 a0 = 0
Puisque n −2n+2 = 1+(n−1) 6= 0, ces relations se réécrivent ∀n ≥ 1, a = 1−n a ,
n 1+(n−1)2 n−1
ce qui entraı̂ne la nullité de la suite (an )n∈N par une récurrence immédiate.
En conclusion, on a montré qu’une telle solution est nécessairement la fonction nulle.
Il n’existe donc pas de solution non nulle de (E) qui soit développable en série entière au
voisinage de 0.

EXERCICE II
On utilisera dans cet exercice les relations :
+∞ +∞ +∞
!
1 X 1 d X X
∀x ∈] − 1; 1[, = xn , 2
= x n
= nxn−1 ,
1−x (1 − x) dx
n=0 n=0 n=1
la seconde étant obtenue par dérivation de la somme d’une série entière sur son intervalle
ouvert de convergence.
De ces relations, on déduit (en évaluant en x = 21 ) :
+∞ +∞ +∞ +∞
X 1 X n 1X n 1X n 1 1
= 2, = = = × = 2.
2n 2 n 2 2n−1 2 2 n−1 2 (1 − 12 )2
n=0 n=0 n=0 n=1
Corrigé CCP MP 2016 - Math I 2/10
i+j
II.1. Notons ui,j = i+j pour tout couple (i, j) ∈ N2 .
2
On a :
• ui,j = uj,i ≥ 0 pour tout (i, j) ∈ N2 ;
X
• pour tout i ∈ N, la série ui,j converge. En effet, on a (sous réserve de convergence
j≥0
de chacune des séries utilisées) :
+∞ +∞ +∞ +∞ +∞
X X i X j i X 1 1 X j
ui,j = + = +
2i+j 2i+j 2i 2j 2i 2j
j=0 j=0 j=0 j=0 j=0

+∞
X i+1
et on reconnaı̂t là des séries convergentes. Au passage, on obtient ui,j = = 4ui,1
2i−1
j=0
(en utilisant les calculs du préambule) ;
 
X X +∞ +∞
P
• la série  ui,j  converge, car pour tout i ∈ N, ui,j = 4ui,1 par ce qui précède
i≥0 j=0 j=0
P P
et parce que ui,1 converge (et elle a même somme que u1,i par symétrie). On
i≥0   i≥0
+∞ X
X +∞ +∞
X +∞
X
obtient concrètement :  ui,j =
 4ui,1 = 4 u1,i = 4 × 4 × u1,1 = 16u1,1 .
i=0 j=0 i=0 i=0

On en déduit, par le théorème de sommation par paquets pour les familles à termes positifs,
que la famille (ui,j )(i,j)∈N2 est sommable, et sa somme vaut :
 
+∞ X+∞
X X 16 × 2
ui,j =  ui,j  = 16 × u1,1 = = 8.
2
22
(i,j)∈N i=0 j=0

II.2.
II.2.a. Les relations données définissent bien une loi de probabilité sur l’univers dénombrable
N2 , puisque :
i+j ui,j
• ∀(i, j) ∈ N2 , i+j+3 = ≥ 0;
2 8
X i+j 1 X
• = ui,j = 1.
2i+j+3 8
(i,j)∈N2 (i,j)∈N2
| {z }
=8
II.2.b. Pour tout i ∈ N, on a la décomposition d’événement :
+∞
[ 
(X = i) = (X = i) ∩ (Y = j) ,
j=0

et cette réunion est disjointe, donc


+∞ +∞
X  X ui,j
P (X = i) = P (X = i) ∩ (Y = j) = .
8
j=0 j=0

De même, on a
+∞ +∞
X  X uj,i
P (Y = i) = P (X = j) ∩ (Y = i) = = P (X = i),
8
j=0 j=0

Nicolas Basbois & Damien Broizat Institut Stanislas, Cannes - Lycée Jules Ferry
Corrigé CCP MP 2016 - Math I 3/10

puisque ui,j = uj,i . Les variables aléatoires X et Y suivent donc la même loi, donnée par
+∞
X uk,l 4uk,1 1 k+1
∀k ∈ N, P (X = k) = P (Y = k) = = = uk,1 = k+2 .
8 8 2 2
l=0

II.2.c. On a d’après l’énoncé :


 0+0
P (X = 0) ∩ (Y = 0) = 0+0+3 = 0.
2
0+1 0+1 1

Pourtant P (X = 0) × P (Y = 0) = 20+2 × 20+2 = 16 6= P (X = 0) ∩ (Y = 0) , donc les
variables X et Y ne sont pas indépendantes.

PROBLÈME : Fonction Digamma.


Partie préliminaire
III.1.
a. Soit x > 0. La fonction hx : t 7→ e−t tx−1 est continue sur ]0, +∞[ par produit de fonctions
continues, les fonctions exponentielle et puissances étant bien continues sur ]0, +∞[.
On a hx (t) ∼ tx−1 = t1−x 1
avec 1 − x < 1 et t2 e−t tx−1 = tx+1 e−t −→ 0 par croissance
t→0+ t→+∞
1

comparée, d’où hx (t) = o t2
.
t→+∞
Ainsi, par comparaison de fonctions positives et critère de Riemann en 0 et en +∞,
hx : t 7→ e−t tx−1 est intégrable sur ]0, +∞[.
Z +∞
On peut ainsi définir la fameuse fonction Gamma d’Euler Γ : x 7→ e−t tx−1 dt,
0
sur ]0, +∞[.
b. Soit x > 0. La fonction hx définie dans la question précédente est Rcontinue et stricte-
+∞
ment positive sur ]0, +∞[. La positivité de l’intégrale nous donne 0 hx (t)dt ≥ 0 et
R +∞
la continuité de hx implique qu’on ne pourrait avoir 0 hx (t)dt = 0 que si hx était
identiquementR nulle sur ]0, +∞[, ce qui n’est pas le cas.
+∞
Ainsi Γ(x) = 0 hx (t)dt > 0, et ce pour tout x > 0.
 ∗
R+ × R∗+ −→ R
c. On définit h : .
(x, t) 7−→ hx (t) = e−t tx−1
– Pour tout t > 0, x 7→ h(x, t) est de classe C 1 (et même C ∞ en fait) sur R∗+ . On a donc
∂h ∂h
l’existence de sur tout (R∗+ )2 et, pour tout t > 0, la continuité de x 7→ (x, t) sur

∂x ∂x
R+ .
∂h
Notons d’ailleurs qu’on a, pour tout (x, t) ∈ (R∗+ )2 , (x, t) = ln(t)e−t tx−1 .
∂x
∂h
– Pour tout x > 0, t 7→ (x, t) est continue (donc continue par morceaux) sur R∗+ .
∂x ∗
– Soit [a, b] un segment de R+ . On a donc 0 < a ≤ b.
| ln(t)|e−t ta−1 si t ≤ 1

∗ ∂h
∀(x, t) ∈ [a, b] × R+ , (x, t) ≤ .
∂x ln(t)e−t tb−1 si t > 1
| ln(t)|e−t ta−1 si t ≤ 1


Notons donc ϕ la fonction définie sur R+ par ϕ(t) = . Cette
ln(t)e−t tb−1 si t > 1
fonction est continue par morceaux (et même continue en fait).
De plus, pour t > 1, on a t2 ϕ(t) = t1+b ln(t)e−t , donc t2 ϕ(t) −→ 0 par croissance
t→+∞
1− a2 a
1
= t 2 | ln(t)|e−t −→

comparée, d’où ϕ(t) = o t2 . Et, pour t ∈]0, 1], on a t ϕ(t)
t→+∞   t→0+
1
0 (toujours par croissance comparée, car a > 0), donc ϕ(t) = o 1− a , avec 1 − a2 <
t→0+ t 2

Nicolas Basbois & Damien Broizat Institut Stanislas, Cannes - Lycée Jules Ferry
Corrigé CCP MP 2016 - Math I 4/10

1.
Donc ϕ est intégrable sur ]0, +∞[.
On en déduit l’hypothèse de domination sur tous les segments de ]0, +∞[.
Cela prouve finalement que Γ est de classe C 1 sur ]0, +∞[, donc dérivable, avec :
Z +∞ Z +∞
0 ∂h
∀x > 0, Γ (x) = (x, t)dt = ln(t)e−t tx−1 dt.
0 ∂x 0
Z n
1 1
III.2. Pour tout entier n ≥ 2, on pose un = dt − .
n−1 t n

[1, +∞[ −→ R
a. Notons f : . Comme la fonction f est continue (donc continue par
t 7−→ 1t
morceaux), décroissanteet à valeurs positives, un théorème du cours indique que la série
P R n P
n−1 f (t)dt − f (n) converge, c’est-à-dire que un converge.
n≥2 n≥2
n
 
P 1
b. Pour tout entier n ≥ 1, on pose Hn = k − ln(n).
k=1
n Rn n
P dt P 1
Pour n ≥ 2, on a uk = 1 t − k par relation de Chasles, d’où
k=2 k=2
n n
P P 1
uk = ln(n) + 1 − k = 1 − Hn .
k=2  n k=1 
P
Comme la suite uk converge par la question précédente, il s’ensuit que la suite
k=2 n≥2
(Hn )n≥1 converge.

On note dans la suite γ = lim Hn , et on définit la fonction Digamma ψ, pour x ∈


n→+∞
Γ0 (x)
]0, +∞[, par ψ(x) = Γ(x) .

Expression de la fonction Digamma à l’aide d’une série


III.3. Pour x ∈]0, +∞[ et pour tout entier n ≥ 1, on définit la fonction fn sur ]0, +∞[ par :
n
1 − nt tx−1 si t ∈]0, n]

fn : t 7→ .
0 si t > n
a. On peut établir l’inégalité souhaitée par simple étude de la fonction x 7→ ln(1 − x) + x sur
] − ∞, 1[, ou bien par un argument de convexité : en effet la fonction ln est notoirement
concave sur R∗+ , donc son graphe est au-dessous de chacune de ses tangentes. Comme la
tangente en x = 1 a pour équation y = x − 1, on en déduit : ∀x ∈ R∗+ , ln(x) ≤ x − 1. Il
vient ensuite, via deux changements de variable successifs : ∀x > −1, ln(1 + x) ≤ x, puis
∀x < 1, ln(1 − x) ≤ −x.

Ensuite, soit n ≥ 1 (et, normalement, x > 0 est déjà fixé aussi dès l’énoncé de la question
III.3.). La fonction fn est positive par définition.
t
De plus, pour tout t ∈]0, n[, fn (t) = en ln(1− n ) tx−1 , avec ln 1 − nt ≤ − nt par la question


précédente, vu qu’on a bien nt < 1 pour t ∈]0, n[. On en déduit, par croissance de l’expo-
t
nentielle et produit par une quantité positive : f (t) ≤ en×(− n ) tx−1 = e−t tx−1 . Enfin f
n n
est nulle sur [n, +∞[, tandis que la fonction t 7→ e−t tx−1 y est positive, d’où finalement
l’encadrement :
∀t > 0, 0 ≤ fn (t) ≤ e−t tx−1 .
b. Comme demandé, on applique le théorème de convergence dominée :
– Pour tout n ≥ 1, fn est continue par morceaux sur R∗+ .
Nicolas Basbois & Damien Broizat Institut Stanislas, Cannes - Lycée Jules Ferry
Corrigé CCP MP 2016 - Math I 5/10

– Soit t > 0. Il existe N ∈ N tel que N ≥ t, par exemple N = btc + 1. Alors, pour
n n t
tout n ≥ N , t ∈]0, n], et donc fn (t) = 1 − nt tx−1 . Or, 1 − nt = en ln(1− n ) , et
n t 1
ln 1 − nt = − nt + o n1 , donc 1 − nt = en(− n +o( n )) = e−t+o(1) −→ e−t par
 
n→+∞
continuité de l’exponentielle. Donc fn (t) −→ e−t tx−1 .
n→+∞
On a ainsi prouvé que (fn )n≥1 converge simplement sur R∗+ vers la fonction t 7→ e−t tx−1 .
– De plus, pour tout n ≥ 1 et pour tout t > 0, |fn (t)| ≤ e−t tx−1 par la question
précédente, et on a prouvé dans la première question du problème que la fonction
t 7→ e−t tx−1 est (continue bien sûr et) intégrable sur R∗+ .
Z +∞ Z +∞
Donc, par le théorème de convergence dominée, fn (t)dt −→ e−t tx−1 dt.
0 n→+∞ 0
Comme fn est nulle sur [n, +∞[, cela donne finalement :
t n x−1
Z n 
1− t dt −→ Γ(x),
0 n n→+∞

et ce raisonnement a bien été mené pour tout x > 0.


Z 1
III.4. Pour tout entier naturel n et tout x > 0, on pose In (x) = (1 − u)n ux−1 du.
0
a. Soient n ∈ N∗ et x > 0.
La fonction α : u 7→ (1 − u)n ux−1 est bien définie et continue sur ]0, 1].
1
De plus, α(u) ∼ ux−1 = u1−x , avec 1 − x < 1, donc α est intégrable sur ]0, 1] par
u→0+
comparaison de fonctions positives et critère de Riemann.
Cela assure la bonne définition de In (x).
x
On définit maintenant sur ]0, 1] les fonctions α1 : u 7→ (1 − u)n et α2 : u 7→ ux . Ces
fonctions sont de classe C 1 , et on a α1 (u)α2 (u) qui admet une limite finie pour u −→ 0+ ,
en l’occurrence 0. On en déduit, par intégration par parties :
Z 1 Z 1
0
In (x) = α1 (u)α2 (u)du = α1 (1)α2 (1) − lim α1 (u)α2 (u) − α10 (u)α2 (u)du
0 u→0+ 0
Z 1
n n
=0−0+ (1 − u)n−1 ux du = In−1 (x + 1).
x 0 x
b. Soit x > 0. R  x 1
1
On a I0 (x) = 0 ux−1 du = ux 0 = x1 .
Soit n ≥ 1. On a, par une récurrence immédiate,
In (x) = nx In−1 (x + 1) = nx × n−1 n!
x+1 In−2 (x + 2) = x(x+1)···(x+n−1) I0 (x + n) =
n!
x(x+1)···(x+n) .

c. La fonction t 7→ nt réalise une bijection strictement croissante et de classe C 1 de ]0, n] sur


]0, 1]. Via le changement de variable u = nt , on obtient donc :
t n x−1
Z n  Z 1 Z 1
1− t dt = (1 − u)n (nu)x−1 ndu = nx (1 − u)n ux−1 du = nx In (x).
0 n 0 0
Le résultat de la question 3.b. se réécrit ainsi : Γ(x) = lim nx In (x). Et le calcul de la
n→+∞
question précédente permet de conclure :
n! n!nx
Γ(x) = lim nx × = lim n .
n→+∞ x(x + 1) · · · (x + n) n→+∞ Q
(x + k)
k=0

Cette relation est appelée formule de Gauss (selon l’énoncé, mais n’est-ce pas plutôt la
formule dite d’Euler dans la littérature ?).
III.5. Soient n ∈ N∗ et x > 0.
L’indication donnée (fallait-il la prouver ?) est immédiate en remarquant qu’on a
Nicolas Basbois & Damien Broizat Institut Stanislas, Cannes - Lycée Jules Ferry
Corrigé CCP MP 2016 - Math I 6/10
n
P 1  n

x k x
exHn e−x ln(n) 1
Q
=e k=1 = e k × nx .
k=1
Ensuite, d’après la formule de Gauss établie à la question précédente, on a :
n
Q n
Q
(x + k) (k + x) n
1 k=0 x k=1 x Y x
= lim = lim × n = lim 1 + .
Γ(x) n→+∞ n!nx n→+∞ nx Q n→+∞ nx k
k k=1
k=1
Grâce à l’indication fournie, on réécrit :
n h
1 xHn
Y x −x i
= lim xe 1+ e k .
Γ(x) n→+∞ k
k=1

Or Hn −→ γ donc, par continuité de l’exponentielle, exHn −→ exγ et, finalement, par


n→+∞ n→+∞
produit de limites,
n h
1 Y x −x i
= xeγx lim 1+ e k .
Γ(x) n→+∞ k
k=1
Cette formule est appelée formule de Weierstrass.
III.6.
a. On note qu’on pourrait répondre directement à la question à l’aide d’un DL d’ordre 2.
Si l’on veut rester dans les clous du sujet, on commence par réécrire la formule précédente :
n h
Y x −x i 1
1+ e k −→ .
k n→+∞ Γ(x)xeγx
k=1
Par continuité de ln, on en déduit :
n h
!  
Y x −x i 1
ln 1+ e k −→ ln , i. e.
k n→+∞ Γ(x)xeγx
k=1
n h 
X x xi
−→ − ln Γ(x)xeγx .

ln 1 + −
k k n→+∞
k=1
ln 1 + xk − xk converge. Ceci ayant été
P  
En particulier, on a prouvé que la série
k≥1
démontré pour tout x > 0, on a établi la convergence
 x simple de la série de fonctions
P x
gk sur ]0, +∞[, où l’on pose gk : x 7→ ln 1 + k − k .
k≥1
+∞
P
b. On note g = gk sur ]0, +∞[.
k=1
P
Outre la convergence de gk vers g établie à la question précédente, on a :
k≥1
– Les fonctions gk sont toutes de classe C 1 sur ]0, +∞[.
– Pour tout k ≥ 1, pour tout x > 0, gk0 (x) = k+x 1
− k1 = − k(k+x)
x
.

Soit [a, b] un segment de R+ . On a donc 0 < a ≤ b. Alors pour tout k ≥ 1 et tout
x ∈ [a, b], |gk0 (x)| ≤ kb2 et, comme
P b
k2
converge, on a établi la convergence normale,
P 0 k≥1
donc uniforme, de gk sur [a, b].
k≥1
+∞ +∞
P 
On en déduit que g est de classe C 1 , avec : ∀x > 0, g 0 (x) = gk0 (x) = 1 1
P
k+x − k .
k=1 k=1
c. Par la question 6.a., on a, pour tout x > 0,

g(x) = − ln Γ(x)xeγx = − ln Γ(x) − ln(x) − γx.


 

Nicolas Basbois & Damien Broizat Institut Stanislas, Cannes - Lycée Jules Ferry
Corrigé CCP MP 2016 - Math I 7/10

Dérivant cette relation sur R∗+ , on obtient :


Γ0 (x) 1
g 0 (x) = − − − γ,
Γ(x) x
0
c’est-à-dire, vu que ψ = ΓΓ , ψ(x) = −g 0 (x) − x1 − γ.
+∞
P 1  +∞P 1 
Comme −g 0 (x) = − k+x − 1
k = − k+x + 1
k , on a finalement établi :
k=1 k=1
+∞  
1 X 1 1
∀x > 0, ψ(x) = − − γ + − .
x k k+x
k=1

III.7.
+∞
P 
1 1
a. Posant x = 1 dans la formule précédente, on trouve : ψ(1) = −1 − γ + k − k+1 ,
k=1
R +∞ψ(1) = −1 − γ + 1 = −γ.
d’où, par télescopage,
De plus Γ(1) = 0 e−t dt = lim [−e−t ]X 0 = lim 1 − e−X = 1 donc, vu que
X→+∞ X→+∞
0
ψ(1) = ΓΓ(1)
(1)
, on obtient Γ0 (1) = −γ.
Mais
R +∞ en reprenant l’expression obtenue à la question 1.c., on constate que Γ0 (1) =
−t
e ln(t)dt, d’où finalement :
0
Z +∞
e−t ln(t)dt = −γ.
0

b. D’après la formule de la question 6.c., on a, pour tout x > 0,


+∞   X +∞  
1 1 X 1 1 1 1
ψ(x + 1) − ψ(x) = − + + − − −
x+1 x k k+x+1 k k+x
k=1 k=1

+∞  
1 1 X 1 1 1 1
= − + − − +
x x+1 k k+x+1 k k+x
k=1
par somme de séries convergentes. Et donc :
+∞   X+∞  
1 1 X 1 1 1 1 1
ψ(x + 1) − ψ(x) = − + − = − = .
x x+1 k+x k+x+1 k+x k+x+1 x
k=1 k=0

Remarque. On aurait aussi pu procéder ainsi :


Γ0 (x + 1) Γ0 (x)
  
d Γ(x + 1)
ψ(x + 1) − ψ(x) = − = ln .
Γ(x + 1) Γ(x) dx Γ(x)
Or, il est bien connu que Γ(x + 1) = xΓ(x) (il suffit d’intégrer par parties), donc
d 1
ψ(x + 1) − ψ(x) = (ln(x)) = .
dx x
En particulier, pour tout k ∈ N∗ , ψ(k + 1) − ψ(k) = k1 .
Il s’ensuit, pour tout entier n ≥ 2,
n−1
X n−1
X1

ψ(n) = ψ(1) + ψ(k + 1) − ψ(k) = −γ + .
k
k=1 k=1

R∗+ −→

R
c. Soit x > 0 fixé. Pour tout k ∈ N, on définit jk : 1 1 .
y 7−→ k+y+1 − k+y+x
Cette notation est discutable : il aurait peut-être été préférable de noter jk,x , pour insister
sur le fait que l’on travaille à x > 0 fixé, et que la convergence uniforme étudiée ici ne
Nicolas Basbois & Damien Broizat Institut Stanislas, Cannes - Lycée Jules Ferry
Corrigé CCP MP 2016 - Math I 8/10

porte que sur la variable y.


k+y+x−k−y−1 x−1
On peut réécrire jk (y) = (k+y+1)(k+y+x) = (k+y+1)(k+y+x) donc,

|x − 1|
∀y > 0, |jk (y)| ≤ .
(k + 1)(k + x)
P |x−1| |x−1| |x−1|
Comme est une série convergente, vu que (k+1)(k+x)
(k+1)(k+x) ∼ 2 , on a la
k≥0 k→+∞ k
P
convergence normale, donc uniforme, de jk sur ]0, +∞[.
k≥0
Ensuite, reprenant la formule de 6.c., on a, pour tout n ∈ N∗ ,
+∞   X +∞  
1 1 X 1 1 1 1
ψ(x + n) − ψ(1 + n) = − + + − − − ,
x+n n k k+x+n k k+1+n
k=1 k=1

et selon le même principe de calcul qu’à la question précédente, on aboutit à :


+∞   X +∞
X 1 1
ψ(x + n) − ψ(1 + n) = − = jk (n).
k+1+n k+x+n
k=0 k=0

Or, pour tout k ∈ N, jk (n) −→ 0 donc, par le théorème de la double limite (qui
n→+∞
s’applique ici car la série de fonctions étudiée converge uniformément sur un voisinage de
+∞),
+∞
 X
lim ψ(x + n) − ψ(1 + n) = lim jk (n) = 0.
n→+∞ n→+∞
k=0

III.8. Par analyse-synthèse :


– Analyse : Soit f solution. On va montrer que f vérifie la formule de ψ établie en 6.c.,
à savoir :
+∞  
1 X 1 1
∀x > 0, f (x) = − − γ + −
x k k+x
k=1
1
Puisque t = f (t + 1) − f (t) pour tout t > 0, on a
+∞   +∞
X 1 1 X 
− = f (k + 1) − f (k) − f (k + x + 1) + f (k + x)
k k+x
k=1 k=1

n n
!
X  X 
= lim f (k + 1) − f (k) + f (k + x) − f (k + x + 1)
n→+∞
k=1 k=1
 

= lim f (n + 1) − f (1) +f (1 + x) − f (n + x + 1)


 
n→+∞ |{z}
=−γ

 1
= f (x + 1) + γ − lim f (x + 1 + n) − f (1 + n) = f (x) + + γ,
n→+∞
| {z } x
=0

ce qui montre bien la relation voulue, et donc f = ψ.


– Synthèse : La seule solution éventuelle au problème est donc ψ. Mais on a prouvé
en 7.a., 7.b. et 7.c. que ψ satisfait les trois conditions voulues, donc finalement ψ est
solution, et c’est la seule.

Nicolas Basbois & Damien Broizat Institut Stanislas, Cannes - Lycée Jules Ferry
Corrigé CCP MP 2016 - Math I 9/10

Autour de la fonction Digamma


III.9. Soit n ∈ N∗ .
a. On suppose les boules indiscernables, ce qui implique qu’à tout moment de l’expérience,
chaque boule de l’urne a la même probabilité d’être tirée, peu importe son numéro (cette
hypothèse n’était pas faite par l’énoncé – est-ce un oubli ou un acte volontaire de la part
du concepteur du sujet ? – mais elle est éminemment raisonnable).
Avec cette hypothèse, X suit la loi uniforme sur {1, . . . , n}. On a donc, pour tout
k ∈ {1, . . . , n}, P (X = k) = n1 .
n n
kP (X = k) = n1 k = n(n+1) = n+1
P P
Il s’ensuit E(X) = 2n 2 .
k=1 k=1
n
P n(n+1)
Les correcteurs acceptent-t-ils la formule k= 2 sans preuve ? S’il faut la redémontrer,
k=1
on peut utiliser la jolie méthode du jeune Gauss, ou une bête récurrence.
b. Vu l’expérience, Y prend ses valeurs dans {1, . . . , n}.
Soit k ∈ {1, . . . , n}.
On utilise la formule des probabilités totales, avec le système complet d’événements
{(X = 1), (X = 2), . . . , (X = n)} :
n n
X 1X
P (Y = k) = P(X=j) (Y = k) × P (X = j) = P(X=j) (Y = k).
n
j=1 j=1

On calcule cette somme en distinguant selon les valeurs de j (j = k ou j 6= k). En effet,


pour j = k, le premier tirage aura amené k boules numérotées k en plus dans l’urne, tandis
que pour j 6= k, le premier tirage n’aura pas amené de boule numérotée k supplémentaire
dans l’urne. Ainsi :
   
1 X 1 k+1 X 1 
P (Y = k) = P(X=k) (Y = k) + P(X=j) (Y = k) =  + ,
n n k+n j+n
1≤j≤n, j6=k 1≤j≤n, j6=k
 
n
1 k X 1
= + .
n k+n j+n
j=1
2n n 2n n
P 1 P 1 P 1 P 1
Or, par 7.b., ψ(2n + 1) − ψ(n + 1) = k − k = k = j+n , d’où finalement :
k=1 k=1 k=n+1 j=1
 
1 k
∀k ∈ {1, . . . , n}, P (Y = k) = + ψ(2n + 1) − ψ(n + 1) .
n k+n
Et il faut corriger ce que demandait l’énoncé, c’est-à-dire prouver cette relation pour tout
k ∈ N∗ , alors qu’elle n’est valable que pour k ∈ {1, . . . , n}.
n n  
P P k k
c. On a E(Y ) = kP (X = k) = n k+n + ψ(2n + 1) − ψ(n + 1) , donc :
k=1 k=1
n
X k2 n+1 
E(Y ) = + ψ(2n + 1) − ψ(n + 1) .
n(n + k) 2
k=1

Utilisant l’indication fournie,


1−n  n+1 
E(Y ) = + n ψ(2n + 1) − ψ(n + 1) + ψ(2n + 1) − ψ(n + 1)
2 2
1 − n 3n + 1 
= + ψ(2n + 1) − ψ(n + 1) .
2 2
Et on est un peu perplexe devant ce résultat : était-ce ce à quoi l’énoncé voulait arriver ?
Nicolas Basbois & Damien Broizat Institut Stanislas, Cannes - Lycée Jules Ferry
Corrigé CCP MP 2016 - Math I 10/10

Remarque. Il n’était pas demandé de démontrer l’indication fournie, mais elle n’avait
rien d’extraordinaire :
n n  n n 
k2
 
X X k k n+1 Xn+k−n n+1 X n
= − = − = − 1−
n(n + k) n n+k 2 n+k 2 n+k
k=1 k=1 k=1 k=1
n 2n
n+1 X 1 1−n X 1 1−n 
= −n+n = +n = + n ψ(2n + 1) − ψ(n + 1) .
2 n+k 2 k 2
k=1 k=n+1

Nicolas Basbois & Damien Broizat Institut Stanislas, Cannes - Lycée Jules Ferry
Mines 2016 - Math1 PSI

Préliminaire
1. Montrer que, pour tout x ∈] − 1, 1[,
+∞ 2k
1 X
√ = k
xk
1 − x k=0 4k

Identité de Karamata
On considère dans cette partie une suite réelle (ak )k∈N telle que, pour tout réel x ∈] − 1, 1[, la série de
terme général ak xk converge absolument. Pour tout réel x ∈] − 1, 1[, on note f (x) la somme de cette
série et l’on suppose que √ √
lim 1 − xf (x) = π
x→1−

2. Pour tout p ∈ N, déterminer


+∞
√ X
lim 1−x ak x(p+1)k
x→1−
k=0

3. Pour tout p ∈ N, justifier la convergence de l’intégrale


Z +∞ −(p+1)t
e
√ dt
0 t
et calculer sa valeur. En déduire l’égalité :
+∞
√ +∞
e−(p+1)t
X Z
(p+1)k
lim 1−x ak x = √ dt
x→1−
k=0 0 t
R +∞ −t
e√ √
On admettra que 0 t
dt = π.

4. Montrer que pour toute application polynomiale réelle Q, on a


+∞
√ +∞
e−t Q(e−t )
X Z
k k
lim 1−x ak x Q(x ) = √ dt
x→1−
k=0 0 t

Soit h la fonction définie, pour tout x ∈ [0, 1], par :

0 si x ∈ [0, e−1 [

h(x) = 1
x si x ∈ [e−1 , 1]

5. Justifier la convergence de
+∞
e−t
Z
√ h(e−t ) dt
0 t
et donner sa valeur.

6. Soit x ∈ [0, 1[. Justifier la convergence de la série de terme général ak xk h(xk ).


On admet l’égalité (dite de Karamata) :

√ +∞
e−t h(e−t )
X Z
k k
lim 1−x ak x h(x ) = √ dt
x→1−
k=0 0 t

1
1
7. En utilisant ce résultat pour x = e− n , en déduire que
n
X √
ak ∼ 2 n
n→+∞
k=0

Théorème taubérien
On considère une suite (an )n∈N décroissante de réels positifs et, pour tout entier naturel n, on pose
Sn = nk=0 ak . On fait l’hypothèse que
P


Sn ∼ 2 n
n→+∞

On va montrer qu’alors
1
an ∼ √
n→+∞ n
On notera [x] la partie entière d’un réel x.

8. Soit α, β un couple de nombres réels vérifiant : 0 < α < 1 < β. Pour tout entier naturel n tel
que n − [αn] et n − [βn] soient non nuls, justifier l’encadrement

S[βn] − Sn Sn − S[αn]
≤ an ≤
[βn] − n n − [αn]

9. Soit γ un réel strictement positif. Déterminer les limites des suites de termes généraux

n S[γn]
et √
[γn] n

10. Soit ε un réel strictement positif. Montrer que, pour tout entier naturel n assez grand, on a :
√ √
2( β − 1) √ 2(1 − α)
− ε ≤ nan ≤ +ε
β−1 1−α

11. En déduire que lim nan = 1.
n→+∞

Marche aléatoire

On considère Ω = ZN l’ensemble des suites indexées par N∗ à valeurs dans Z.
On définit les applications coordonnées, pour tout i ≥ 1,

Xi : ω = (ω1 , ω2 , . . . ) ∈ Ω 7→ ωi ∈ Z

On admet que l’on peut construire une tribu B et une mesure de probabilité P sur Ω, de sorte que les
Xi soient des variables aléatoires, indépendantes et de même loi donnée par
1
P(X1 = 1) = P(X1 = −1) =
2
On définit la suite de variables aléatoires (Sn , n ≥ 0) par
n
X
S0 (ω) = 0, Sn (ω) = Xi (ω)
i=1

2
On définit enfin la variable aléatoire T par

T : Ω → N ∗ ∗
 = N ∪ {+∞}
+∞ si Sn (ω) 6= 0, ∀n ≥ 1
ω →
inf{n ≥ 1, Sn (ω) = 0} sinon

Pour tout entier naturel n, on note En = {T > n}, pour n ≥ 1, Ann = {Sn = 0} et pour k ∈
{0, . . . , n − 1},
n
\
Ank = {Sk = 0} ∩ {Si 6= 0}
i=k+1
Sn
p p
S2 =2
p p @p p p -
p
@p p n
S1 =1
6 S3 =1
T =6

Figure 1 - Ici ω commence par (1, 1, −1, 1, −1, −1, −1, 1, 1, −1).
ω appartient à A66 et A88 ainsi qu’à A10 , A20 , . . . ,A50 , A76 etc.

12. Montrer pour tout 1 ≤ k < n, pour tout (i1 , . . . , in−k ) ∈ {−1, 1}n−k ,

P(Xk+1 = i1 , . . . , Xn = in−k ) = P(X1 = i1 , . . . , Xn−k = in−k )

13. Montrer pour tout 1 ≤ k < n, pour tout (j1 , . . . , jn−k ) ∈ Zn−k que

P(Sk+1 − Sk = j1 , . . . , Sn − Sk = jn−k ) = P(S1 = j1 , . . . , Sn−k = jn−k )

Indication : on pourra considérer l’application


 
n−k
X
θ : (z1 , . . . , zn−k ) ∈ Zn−k 7→ z1 , z1 + z2 , . . . , zj  ∈ Zn−k
j=1

14. En déduire que pour tout k ∈ {0, . . . , n}

P(Ank ) = P(Sk = 0)P(En−k )

15. Montrer l’égalité


n
X
1= P(Sk = 0)P(En−k )
k=0

16. Pour tout réel x de ]0, 1[, établir l’égalité :


∞ ∞
! !
1 X
n
X
n
= P(Sn = 0)x P(En )x
1−x
n=0 n=0

17. Pour tout entier naturel n, calculer P(Sn = 0).


Indication : on discutera suivant la parité de n.

18. En déduire que, pour tout x ∈]0, 1[, on a


+∞ r
X
n 1+x
P(En )x =
1−x
n=0

19. A l’aide des résultats obtenus dans les parties précédentes déterminer, quand l’entier naturel n
tend vers l’infini, un équivalent de P(En ).

3
20. Montrer que l’on a : P(T = +∞) = 0.

21. Pour tout réel x ∈ [0, 1], prouver l’égalité :

p ∞
X
1− 1 − x2 = P(T = n)xn
n=1

22. En déduire que, pour tout n ∈ N∗ ,


2n

1 n
P(T = 2n) =
2n − 1 4n

4
Mines 2016 - Math1 PSI

Préliminaire
1. Montrer que, pour tout x ∈] − 1, 1[,
+∞ 2k
1 X
√ = k
xk
1 − x k=0 4k

Identité de Karamata
On considère dans cette partie une suite réelle (ak )k∈N telle que, pour tout réel x ∈] − 1, 1[, la série de
terme général ak xk converge absolument. Pour tout réel x ∈] − 1, 1[, on note f (x) la somme de cette
série et l’on suppose que √ √
lim 1 − xf (x) = π
x→1−

2. Pour tout p ∈ N, déterminer


+∞
√ X
lim 1−x ak x(p+1)k
x→1−
k=0

3. Pour tout p ∈ N, justifier la convergence de l’intégrale


Z +∞ −(p+1)t
e
√ dt
0 t
et calculer sa valeur. En déduire l’égalité :
+∞
√ +∞
e−(p+1)t
X Z
(p+1)k
lim 1−x ak x = √ dt
x→1−
k=0 0 t
R +∞ −t
e√ √
On admettra que 0 t
dt = π.

4. Montrer que pour toute application polynomiale réelle Q, on a


+∞
√ +∞
e−t Q(e−t )
X Z
k k
lim 1−x ak x Q(x ) = √ dt
x→1−
k=0 0 t

Soit h la fonction définie, pour tout x ∈ [0, 1], par :

0 si x ∈ [0, e−1 [

h(x) = 1
x si x ∈ [e−1 , 1]

5. Justifier la convergence de
+∞
e−t
Z
√ h(e−t ) dt
0 t
et donner sa valeur.

6. Soit x ∈ [0, 1[. Justifier la convergence de la série de terme général ak xk h(xk ).


On admet l’égalité (dite de Karamata) :

√ +∞
e−t h(e−t )
X Z
k k
lim 1−x ak x h(x ) = √ dt
x→1−
k=0 0 t

1
1
7. En utilisant ce résultat pour x = e− n , en déduire que
n
X √
ak ∼ 2 n
n→+∞
k=0

Théorème taubérien
On considère une suite (an )n∈N décroissante de réels positifs et, pour tout entier naturel n, on pose
Sn = nk=0 ak . On fait l’hypothèse que
P


Sn ∼ 2 n
n→+∞

On va montrer qu’alors
1
an ∼ √
n→+∞ n
On notera [x] la partie entière d’un réel x.

8. Soit α, β un couple de nombres réels vérifiant : 0 < α < 1 < β. Pour tout entier naturel n tel
que n − [αn] et n − [βn] soient non nuls, justifier l’encadrement

S[βn] − Sn Sn − S[αn]
≤ an ≤
[βn] − n n − [αn]

9. Soit γ un réel strictement positif. Déterminer les limites des suites de termes généraux

n S[γn]
et √
[γn] n

10. Soit ε un réel strictement positif. Montrer que, pour tout entier naturel n assez grand, on a :
√ √
2( β − 1) √ 2(1 − α)
− ε ≤ nan ≤ +ε
β−1 1−α

11. En déduire que lim nan = 1.
n→+∞

Marche aléatoire

On considère Ω = ZN l’ensemble des suites indexées par N∗ à valeurs dans Z.
On définit les applications coordonnées, pour tout i ≥ 1,

Xi : ω = (ω1 , ω2 , . . . ) ∈ Ω 7→ ωi ∈ Z

On admet que l’on peut construire une tribu B et une mesure de probabilité P sur Ω, de sorte que les
Xi soient des variables aléatoires, indépendantes et de même loi donnée par
1
P(X1 = 1) = P(X1 = −1) =
2
On définit la suite de variables aléatoires (Sn , n ≥ 0) par
n
X
S0 (ω) = 0, Sn (ω) = Xi (ω)
i=1

2
On définit enfin la variable aléatoire T par

T : Ω → N ∗ ∗
 = N ∪ {+∞}
+∞ si Sn (ω) 6= 0, ∀n ≥ 1
ω →
inf{n ≥ 1, Sn (ω) = 0} sinon

Pour tout entier naturel n, on note En = {T > n}, pour n ≥ 1, Ann = {Sn = 0} et pour k ∈
{0, . . . , n − 1},
n
\
Ank = {Sk = 0} ∩ {Si 6= 0}
i=k+1
Sn
p p
S2 =2
p p @p p p -
p
@p p n
S1 =1
6 S3 =1
T =6

Figure 1 - Ici ω commence par (1, 1, −1, 1, −1, −1, −1, 1, 1, −1).
ω appartient à A66 et A88 ainsi qu’à A10 , A20 , . . . ,A50 , A76 etc.

12. Montrer pour tout 1 ≤ k < n, pour tout (i1 , . . . , in−k ) ∈ {−1, 1}n−k ,

P(Xk+1 = i1 , . . . , Xn = in−k ) = P(X1 = i1 , . . . , Xn−k = in−k )

13. Montrer pour tout 1 ≤ k < n, pour tout (j1 , . . . , jn−k ) ∈ Zn−k que

P(Sk+1 − Sk = j1 , . . . , Sn − Sk = jn−k ) = P(S1 = j1 , . . . , Sn−k = jn−k )

Indication : on pourra considérer l’application


 
n−k
X
θ : (z1 , . . . , zn−k ) ∈ Zn−k 7→ z1 , z1 + z2 , . . . , zj  ∈ Zn−k
j=1

14. En déduire que pour tout k ∈ {0, . . . , n}

P(Ank ) = P(Sk = 0)P(En−k )

15. Montrer l’égalité


n
X
1= P(Sk = 0)P(En−k )
k=0

16. Pour tout réel x de ]0, 1[, établir l’égalité :


∞ ∞
! !
1 X
n
X
n
= P(Sn = 0)x P(En )x
1−x
n=0 n=0

17. Pour tout entier naturel n, calculer P(Sn = 0).


Indication : on discutera suivant la parité de n.

18. En déduire que, pour tout x ∈]0, 1[, on a


+∞ r
X
n 1+x
P(En )x =
1−x
n=0

19. A l’aide des résultats obtenus dans les parties précédentes déterminer, quand l’entier naturel n
tend vers l’infini, un équivalent de P(En ).

3
20. Montrer que l’on a : P(T = +∞) = 0.

21. Pour tout réel x ∈ [0, 1], prouver l’égalité :

p ∞
X
1− 1 − x2 = P(T = n)xn
n=1

22. En déduire que, pour tout n ∈ N∗ ,


2n

1 n
P(T = 2n) =
2n − 1 4n

4
Mines 2016 - Math1 PSI
Un corrigé

Préliminaire
1. Le cours nous apprend que pour tout réel α, on a
+∞
α
X α(α − 1) . . . (α − k + 1)
∀x ∈] − 1, 1[, (1 + x) = 1 + xk
k!
k=1

En choisissant α = −1/2 et en substituant −x à x, on a donc



1 X 1 −1 − k 2k + 1
∀x ∈] − 1, 1[, √ = 1+ ak xk avec a1 = , ∀k ≥ 1 : ak+1 = − 2 ak = ak
1−x 2 k + 1 2(k + 1)
k=1

On montre par récurrence que


2k

k
∀k ≥ 1, ak =
4k
(21) 2
- Initialisation : c’est vrai pour k = 1 car 4 = 4 = 12 .
- Hérédité : supposons le résultat vrai pour un rang k ≥ 1. On a alors
2k

k 2k + 1 (2k)! 2k + 1 (2k + 2)! 1 (2(k + 1))!
ak+1 = k = k 2
= k 2 2
= k+1
4 2(k + 1) 4 (k!) 2(k + 1) 4 (k!) 4(k + 1) 4 ((k + 1)!)2

ce qui montre le résultat au rang k + 1.

La formule étant encore valable au rang k = 0 (a0 = 1), on a ainsi


+∞ 2k
1 X
∀x ∈] − 1, 1[, √ = k
k
xk
1 − x k=0 4

Identité de Karamata
2. On remarque que pour tout x ∈] − 1, 1[ et p ∈ N, on a xp ∈] − 1, 1[ et
+∞
√ X √
1−x ak x(p+1)k = 1 − xf (xp+1 )
k=0

1−x p
= √ 1 − xp+1 f (xp+1 )
1−x p+1
1 p
= √ p
1 − xp+1 f (xp+1 )
1 + x + ··· + x
Pp
puisque ap+1 − bp+1 = (a − b) k=0 a
k bp−k . On en déduit que
+∞

r
X
(p+1)k π
lim 1−x ak x =
x→1− p+1
k=0

e−(p+1)t
3. Soit p ∈ N. gp : t 7→ √
t
est continue sur R+∗ et on a des problèmes d’intégrabilité en 0 et
+∞.

1
1
- Au voisinage de 0, gp (t) ∼ √
t
est intégrable (fonction de Riemann).
- Au voisinage de +∞, gp (t) = o(1/t2 ) par croissances comparées (car p + 1 > 0) et est donc
intégrable.
gp est intégrable sur R+ et l’intégrale proposée converge donc a fortiori. Le changement de
variable u = (p + 1)t (licite car t 7→ (p + 1)t est de classe C 1 sur R+∗ à dérivée ne s’annulant
pas) donne
Z +∞ −(p+1)t Z +∞ −u r
e 1 e π
√ dt = √ √ du =
0 t p+1 0 u p+1
Avec la question précédente, on a donc immédiatement
+∞ Z +∞ −(p+1)t
√ X
(p+1)k e
lim 1 − x ak x = √ dt
x→1 −
k=0 0 t
Pd i.
4. Soit Q ∈ R[X]. Il existe un entier d et des scalaires c0 , . . . , cd tels que Q = i=0 ci X La
question précédente donne
+∞ Z +∞ −t
√ X
k i k e ci (e−t )i
∀i ∈ [|0, d|], lim 1 − x ak ci (x ) x = √ dt
x→1−
k=0 0 t

En sommant ces relations (linéarité du passage à la limite et du passage à l’intégrale, pas de


souci pour intervertir les symboles car on effectue une somme FINIE) on obtient que
+∞
√ +∞
e−t Q(e−t )
X Z
k k
lim 1−x ak x Q(x ) = √ dt
x→1−
k=0 0 t

−t
5. La fonction g : t 7→ e√t h(e−t ) est continue par morceaux sur R+∗ (il y a un unique problème
de continuité en 1 où la fonction a une limite finie à droite et gauche) et on a des problèmes
d’intégrabilité en 0 et +∞.
−t
h étant majorée en module par e, |g(t)| ≤ e√t et h est donc intégrable sur R+ (le majorant l’est).
Par définition de h Z +∞ −t
e
Z 1
dt h √ i1
−t
√ h(e ) dt = √ = 2 t =2
0 t 0 t 0

6. Pour x ∈ [0, 1[ fixé, on a xk qui tend vers 0 quand k → +∞. La suite (ak xk h(xk )) est ainsi
nulle à partir d’un certain rang. La série associé est donc convergente (les sommes partielles
stationnent à partir d’un certain rang).
7. Soit n ∈ N∗ . Par définition de h, on a
p +∞
X p n
X
1 − e−1/n ak e−k/n h(e−k/n ) = 1 − e−1/n ak
k=0 k=0

Quand n → +∞, e−1/n → 1− et l’égalité de Karamata donne


n Z +∞ −t
p X e
lim 1−e −1/n ak = √ h(e−t ) dt = 2
n→+∞
k=0 0 t

Comme 1 − e−x ∼0 x, on en déduit que


n
X 2 √
ak ∼ p ∼2 n
k=0 1 − e−1/n

2
Théorème taubérien
8. Comme n ≥ [αn], on a
n
X
Sn − S[αn] = ak ≥ (n − [αn])an
k=[αn]+1

l’inégalité provenant de la décroissance de (ak ). Si n − [αn] est non nul, c’est une quantité > 0
et on peut diviser pour obtenir
Sn − S[αn]
an ≤
n − [αn]
De même, comme [βn] ≥ n, on a
[βn]
X
S[βn] − Sn = ak ≤ ([βn] − n)an+1 ≤ ([βn] − n)an
k=n+1

et quand [βn] − n > 0,


S[βn] − Sn
≤ an
[βn] − n

9. Comme [x] ≤ x ≤ [x] + 1, on a [γn] ≤ γn ≤ [γn] + 1. Pour γ > 0, [γn] > 0 pour n assez grand
(et tend vers +∞) et on peut diviser par [γn] pour en déduire
n 1
1≤γ ≤1+
[γn] [γn]

Par théorème d’encadrement (et comme [γn] → +∞) on a donc


n 1
lim =
n→+∞ [γn] γ

Comme [γn] → +∞, l’hypothèse fait sur la suite (Sn ) indique que
r
S[γn] [γn]
√ ∼2
n n

et ainsi
S[γn] √
lim √ =2 γ
n→+∞ n

10. Pour γ > 0 différent de 1, on écrit que


√ Sn − S[γn] S[γn]
 
1 S
n = √n − √
n − [γn] 1 − [γn]
n
n n

2(1− γ)
et les questions précédentes montre que cette quantité tend vers 1−γ .
√ √
2(1− α)
L’encadrement de la question 8 montre que nan est majoré par un terme de limite 1−α et

2(1− β)
minoré par un terme de limite 1−β . Par définition des limites,
√ √
2(1 − β) √ 2(1 − α)
∀ε > 0, ∃ n0 / ∀n ≥ n0 , − ε ≤ nan ≤ +ε
1−β 1−α

3

11. Soit ε > 0. Comme 2(1−
1−x
x)
= 2√
1+ x
est de limite 1 en 1, il existe α < 1 < β tels que
√ √
2(1 − β) 2(1 − α)
1−ε≤ et ≤1+ε
1−β 1−α
Pour ces α et β, la question précédente donne un rang n0 et
√ √
2(1 − β) √ 2(1 − α)
∀n ≥ n0 , − 2ε ≤ nan ≤ + 2ε
1−β 1−α

Par définition, des limites, on a donc nan → 1 et donc
1
an ∼ √
n

Marche aléatoire
12. Par indépendance des Xi , on a
n
Y
P(Xk+1 = i1 , . . . , Xn = in−k ) = P(Xj = ij−k )
j=k+1

Comme les Xi ont toutes la même loi, P(Xj = ij−k ) = P(Xj−k = j − k) et donc (en utilisant
encore l’indépendance)
n−k
Y
P(Xk+1 = i1 , . . . , Xn = in−k ) = P(Xj = ij ) = P(X1 = i1 , . . . , Xn−k = in−k )
j=1

13. Par définition des Sn , on a


  

 Sk+1 − Sk = j1 
 Xk+1 = j1 
 Xk+1 = j1
 Sk+2 − Sk = j2
  Xk+1 + Xk+2 = j2
  Xk+2 = j2 − j1

.. ⇐⇒ .. ⇐⇒ ..


 . 

 . 

 .
Sn − Sk = jn−k Xk+1 + · · · + Xn = jn−k Xn = jn−k − jn−k−1
  

On peut alors utiliser la question précédente pour en déduire que

P(Sk+1 − Sk = j1 , . . . , Sn − Sk = jn−k ) = P(X1 = j1 , X2 = j2 − j1 , . . . , Xn−k = jn−k − jn−k−1 )

On fait alors le trajt dans l’autre sens et on remarque que


 

 X1 = j1 
 S1 = j1
 X2 = j2 − j1
 
 S2 = j2
.. ⇐⇒ ..


 . 

 .
Xn−k = jn−k − jn−k−1 Sn−k = jn−k
 

ce qui donne finalement

P(Sk+1 − Sk = j1 , . . . , Sn − Sk = jn−k ) = P(S1 = j1 , . . . , Sn−k = jn−k )

14. On remarque que Ank = (Sk = 0) ∩ (Sk+1 6= 0) ∩ . . . (Sn 6= 0) et donc

P(Ank ) = P(Sk = 0)PSk =0 (Sk+1 6= 0, . . . , Sn 6= 0)

4
De façon évidente,

PSk =0 (Sk+1 6= 0, . . . , Sn 6= 0) = PSk =0 (Sk+1 − Sk 6= 0, . . . , Sn − Sk 6= 0)

On remarque que les variables Sk+1 − Sk , . . . , Sn − Sk ne dépendent que de Xk+1 , . . . , Xn et


sont donc indépendantes de Sk qui ne dépend que de X1 , . . . , Xk (et puisque les Xi sont, elles,
indépendantes). On a donc en fait

PSk =0 (Sk+1 6= 0, . . . , Sn 6= 0) = P(Sk+1 − Sk 6= 0, . . . , Sn − Sk 6= 0)

Remarquons que
 
[
P(Sk+1 − Sk 6= 0, . . . , Sn − Sk 6= 0) = P  (Sk+1 − Sk = j1 , . . . , Sn − Sk = jn−k )
j1 ,...,jn−k 6=0
X
= P (Sk+1 − Sk = j1 , . . . , Sn − Sk = jn−k )
j1 ,...,jn−k 6=0

la dernière égalité provenant de l’incompatiblité des événements. La question précédente donne


alors X
P(Sk+1 − Sk 6= 0, . . . , Sn − Sk 6= 0) = P (S1 = j1 , . . . , Sn−k = jn−k )
j1 ,...,jn−k 6=0

et en refaisant le chemin dans l’autre sens

P(Sk+1 − Sk 6= 0, . . . , Sn − Sk 6= 0) = P(S1 6= 0, . . . , Sn−k 6= 0)) = P(En−k )

Finalement, on trouve que


P(Ank ) = P(Sk = 0)P(En−k )

15. Soit n ∈ N. Soit ω ∈ Ω ; comme S0 (ω) = 0, il existe un plus grand k ∈ [|0, n|] tel que Sk (ω) = 0
et on a alors ω ∈ Ank . La réunion des parties An0 , . . . , Ann est donc égale à Ω. Ces parties étant
disjointes, on a donc
Xn Xn
1= P(Ank ) = P(Sk = 0)P(En−k )
k=0 k=0

(P(Sn = 0)xn ) et (P(En )xn ) étant des séries entières de rayon de convergence au moins égal
P P
16.
à 1 (|P(En )xn | ≤ 1 si |x| ≤ 1). On peut donc utiliser le théorème sur le produit de Cauchy et
écrire que pour x ∈] − 1, 1[

! ∞ ! ∞ n
X X X X
n n
P(Sn = 0)x P(En )x = un xn avec un = P(Sk = 0)P(En−k )
n=0 n=0 n=0 k=0

Avec la question précédente, on a donc



! ∞ ! +∞
X
n
X
n
X 1
P(Sn = 0)x P(En )x = xn =
1−x
n=0 n=0 n=0

17. Sn (ω) est la somme de n quantités valant 1 ou −1 et ne peut donc être nul que s’il y a autant
de 1 que de −1, c’est à dire que si n est pair. On a donc

∀n ∈ N, P(S2n+1 = 0) = 0

5
Supposons maintenant n pair et écrivons que n = 2p. La valeur de Sn (ω) ne dépend que des
valeurs de ω1 , . . . , ω2p . Il y a 22p = 4p choix pour ces valeurs et on a
αp
P(S2p = 0) =
4p
où αp est le nombre de uplets (ω1 , . . . , ω2p ) contenant autant de 1 que de −1. Choisir un tel
2p

uplet, c’est choisir la position des 1 et il y a p tels choix. On a donc

2p

p
P(S2p = 0) =
4p

18. On en déduit que pour x ∈] − 1, 1[,


+∞ +∞ 2p

X
n
X p 1
P(Sn = 0)x = x2p = √
4p 1 − x2
n=0 p=0

et donc que
+∞ √ r
X
n 1 − x2 1+x
P(En )x = =
1−x 1−x
n=0

19. Considérons la fonction f définie par


r r
π 1+x
∀x ∈] − 1, 1[, f (x) =
2 1−x
√ √
Comme 1 − xf (x) → π quand x → 1− , on peut utiliser la partie B (et la question précédente)
pour obtenir
n r
X π √
P(En ) ∼ 2 n
2
k=0

Par ailleurs P(T > n) = P(En ) est le terme général d’une suite décroissante ((T > n) ⊂ (T >
n − 1)) et le théorème taubérien indique que
r
π 1
P(En ) ∼ √
2 n

On a donc r
2
P(En ) ∼
πn
20. On a \ \
(T = +∞) = (T > n) = En
n∈N n∈N

Par continuité décroissante des probabilités (appliqué avec la suite décroissante d’ensemble de
terme général E0 ∩ · · · ∩ En = En ), on en déduit que

P(T = ∞) = lim P(En ) = 0


n→+∞

6
21. On a (T = n) = (T > n − 1) \ (T > n) et comme (T > n) ⊂ (T > n − 1), on a P(T = n) =
P(T > n − 1) − P(T > n) et
n
X n
X
k
P(T = k)x = (P(T > k − 1) − P(T > k))xk
k=0 k=0
n
X n
X
= P(T > −1) + x P(T > k − 1)xk−1 − P(T > k)xk
k=1 k=0
n−1
X n
X
= 1+x P(Ek )xk − P(Ek )xk
k=0 k=0

Pour x ∈] − 1, 1[, les termes du membre de droite admettent une limite quand n → +∞ et, avec
l’expression des sommes,
+∞ r
X
k 1+x p
P(T = k)x = 1 + (x − 1) = 1 − 1 − x2
1−x
k=0

La relation reste vraie pour x = 0 car (T = n)n∈N est un système complet d’événements (et se
lit 1 = 1).

22. On commence par chercher le DSE de h : x 7→ 1 − 1 − x2 . Pour cela on remarque que (pour
x ∈] − 1, 1[)
∞ 2k ∞ 2k
x X X
h0 (x) = √ =x k
x2k = k
x2k+1
1 − x2 4k 4k
k=0 k=0

Par théorème de primitivation des séries entières,


∞ ∞ 2(j−1)

2k

j−1
X X
h(x) = h(0) + k
x2k+2 = x2j
4k (2k + 2) j−1
4 .2j
k=0 k=1

Par unicité des DSE, on en déduit que


2(j−1)

j−1
P(T = 2j) = j−1
4 .2j
Il nous reste à remarquer que

j2
 
2(j − 1) (2j − 2)! (2j)!
= =
j−1 ((j − 1)!)2 (j!)2 2j(2j − 1)

pour en déduire que


2j

j
P(T = 2j) =
4j (2j − 1)

7
Mines 2016 - Math1 PSI
Un corrigé

Préliminaire
1. Le cours nous apprend que pour tout réel α, on a
+∞
α
X α(α − 1) . . . (α − k + 1)
∀x ∈] − 1, 1[, (1 + x) = 1 + xk
k!
k=1

En choisissant α = −1/2 et en substituant −x à x, on a donc



1 X 1 −1 − k 2k + 1
∀x ∈] − 1, 1[, √ = 1+ ak xk avec a1 = , ∀k ≥ 1 : ak+1 = − 2 ak = ak
1−x 2 k + 1 2(k + 1)
k=1

On montre par récurrence que


2k

k
∀k ≥ 1, ak =
4k
(21) 2
- Initialisation : c’est vrai pour k = 1 car 4 = 4 = 12 .
- Hérédité : supposons le résultat vrai pour un rang k ≥ 1. On a alors
2k

k 2k + 1 (2k)! 2k + 1 (2k + 2)! 1 (2(k + 1))!
ak+1 = k = k 2
= k 2 2
= k+1
4 2(k + 1) 4 (k!) 2(k + 1) 4 (k!) 4(k + 1) 4 ((k + 1)!)2

ce qui montre le résultat au rang k + 1.

La formule étant encore valable au rang k = 0 (a0 = 1), on a ainsi


+∞ 2k
1 X
∀x ∈] − 1, 1[, √ = k
k
xk
1 − x k=0 4

Identité de Karamata
2. On remarque que pour tout x ∈] − 1, 1[ et p ∈ N, on a xp ∈] − 1, 1[ et
+∞
√ X √
1−x ak x(p+1)k = 1 − xf (xp+1 )
k=0

1−x p
= √ 1 − xp+1 f (xp+1 )
1−x p+1
1 p
= √ p
1 − xp+1 f (xp+1 )
1 + x + ··· + x
Pp
puisque ap+1 − bp+1 = (a − b) k=0 a
k bp−k . On en déduit que
+∞

r
X
(p+1)k π
lim 1−x ak x =
x→1− p+1
k=0

e−(p+1)t
3. Soit p ∈ N. gp : t 7→ √
t
est continue sur R+∗ et on a des problèmes d’intégrabilité en 0 et
+∞.

1
1
- Au voisinage de 0, gp (t) ∼ √
t
est intégrable (fonction de Riemann).
- Au voisinage de +∞, gp (t) = o(1/t2 ) par croissances comparées (car p + 1 > 0) et est donc
intégrable.
gp est intégrable sur R+ et l’intégrale proposée converge donc a fortiori. Le changement de
variable u = (p + 1)t (licite car t 7→ (p + 1)t est de classe C 1 sur R+∗ à dérivée ne s’annulant
pas) donne
Z +∞ −(p+1)t Z +∞ −u r
e 1 e π
√ dt = √ √ du =
0 t p+1 0 u p+1
Avec la question précédente, on a donc immédiatement
+∞ Z +∞ −(p+1)t
√ X
(p+1)k e
lim 1 − x ak x = √ dt
x→1 −
k=0 0 t
Pd i.
4. Soit Q ∈ R[X]. Il existe un entier d et des scalaires c0 , . . . , cd tels que Q = i=0 ci X La
question précédente donne
+∞ Z +∞ −t
√ X
k i k e ci (e−t )i
∀i ∈ [|0, d|], lim 1 − x ak ci (x ) x = √ dt
x→1−
k=0 0 t

En sommant ces relations (linéarité du passage à la limite et du passage à l’intégrale, pas de


souci pour intervertir les symboles car on effectue une somme FINIE) on obtient que
+∞
√ +∞
e−t Q(e−t )
X Z
k k
lim 1−x ak x Q(x ) = √ dt
x→1−
k=0 0 t

−t
5. La fonction g : t 7→ e√t h(e−t ) est continue par morceaux sur R+∗ (il y a un unique problème
de continuité en 1 où la fonction a une limite finie à droite et gauche) et on a des problèmes
d’intégrabilité en 0 et +∞.
−t
h étant majorée en module par e, |g(t)| ≤ e√t et h est donc intégrable sur R+ (le majorant l’est).
Par définition de h Z +∞ −t
e
Z 1
dt h √ i1
−t
√ h(e ) dt = √ = 2 t =2
0 t 0 t 0

6. Pour x ∈ [0, 1[ fixé, on a xk qui tend vers 0 quand k → +∞. La suite (ak xk h(xk )) est ainsi
nulle à partir d’un certain rang. La série associé est donc convergente (les sommes partielles
stationnent à partir d’un certain rang).
7. Soit n ∈ N∗ . Par définition de h, on a
p +∞
X p n
X
1 − e−1/n ak e−k/n h(e−k/n ) = 1 − e−1/n ak
k=0 k=0

Quand n → +∞, e−1/n → 1− et l’égalité de Karamata donne


n Z +∞ −t
p X e
lim 1−e −1/n ak = √ h(e−t ) dt = 2
n→+∞
k=0 0 t

Comme 1 − e−x ∼0 x, on en déduit que


n
X 2 √
ak ∼ p ∼2 n
k=0 1 − e−1/n

2
Théorème taubérien
8. Comme n ≥ [αn], on a
n
X
Sn − S[αn] = ak ≥ (n − [αn])an
k=[αn]+1

l’inégalité provenant de la décroissance de (ak ). Si n − [αn] est non nul, c’est une quantité > 0
et on peut diviser pour obtenir
Sn − S[αn]
an ≤
n − [αn]
De même, comme [βn] ≥ n, on a
[βn]
X
S[βn] − Sn = ak ≤ ([βn] − n)an+1 ≤ ([βn] − n)an
k=n+1

et quand [βn] − n > 0,


S[βn] − Sn
≤ an
[βn] − n

9. Comme [x] ≤ x ≤ [x] + 1, on a [γn] ≤ γn ≤ [γn] + 1. Pour γ > 0, [γn] > 0 pour n assez grand
(et tend vers +∞) et on peut diviser par [γn] pour en déduire
n 1
1≤γ ≤1+
[γn] [γn]

Par théorème d’encadrement (et comme [γn] → +∞) on a donc


n 1
lim =
n→+∞ [γn] γ

Comme [γn] → +∞, l’hypothèse fait sur la suite (Sn ) indique que
r
S[γn] [γn]
√ ∼2
n n

et ainsi
S[γn] √
lim √ =2 γ
n→+∞ n

10. Pour γ > 0 différent de 1, on écrit que


√ Sn − S[γn] S[γn]
 
1 S
n = √n − √
n − [γn] 1 − [γn]
n
n n

2(1− γ)
et les questions précédentes montre que cette quantité tend vers 1−γ .
√ √
2(1− α)
L’encadrement de la question 8 montre que nan est majoré par un terme de limite 1−α et

2(1− β)
minoré par un terme de limite 1−β . Par définition des limites,
√ √
2(1 − β) √ 2(1 − α)
∀ε > 0, ∃ n0 / ∀n ≥ n0 , − ε ≤ nan ≤ +ε
1−β 1−α

3

11. Soit ε > 0. Comme 2(1−
1−x
x)
= 2√
1+ x
est de limite 1 en 1, il existe α < 1 < β tels que
√ √
2(1 − β) 2(1 − α)
1−ε≤ et ≤1+ε
1−β 1−α
Pour ces α et β, la question précédente donne un rang n0 et
√ √
2(1 − β) √ 2(1 − α)
∀n ≥ n0 , − 2ε ≤ nan ≤ + 2ε
1−β 1−α

Par définition, des limites, on a donc nan → 1 et donc
1
an ∼ √
n

Marche aléatoire
12. Par indépendance des Xi , on a
n
Y
P(Xk+1 = i1 , . . . , Xn = in−k ) = P(Xj = ij−k )
j=k+1

Comme les Xi ont toutes la même loi, P(Xj = ij−k ) = P(Xj−k = j − k) et donc (en utilisant
encore l’indépendance)
n−k
Y
P(Xk+1 = i1 , . . . , Xn = in−k ) = P(Xj = ij ) = P(X1 = i1 , . . . , Xn−k = in−k )
j=1

13. Par définition des Sn , on a


  

 Sk+1 − Sk = j1 
 Xk+1 = j1 
 Xk+1 = j1
 Sk+2 − Sk = j2
  Xk+1 + Xk+2 = j2
  Xk+2 = j2 − j1

.. ⇐⇒ .. ⇐⇒ ..


 . 

 . 

 .
Sn − Sk = jn−k Xk+1 + · · · + Xn = jn−k Xn = jn−k − jn−k−1
  

On peut alors utiliser la question précédente pour en déduire que

P(Sk+1 − Sk = j1 , . . . , Sn − Sk = jn−k ) = P(X1 = j1 , X2 = j2 − j1 , . . . , Xn−k = jn−k − jn−k−1 )

On fait alors le trajt dans l’autre sens et on remarque que


 

 X1 = j1 
 S1 = j1
 X2 = j2 − j1
 
 S2 = j2
.. ⇐⇒ ..


 . 

 .
Xn−k = jn−k − jn−k−1 Sn−k = jn−k
 

ce qui donne finalement

P(Sk+1 − Sk = j1 , . . . , Sn − Sk = jn−k ) = P(S1 = j1 , . . . , Sn−k = jn−k )

14. On remarque que Ank = (Sk = 0) ∩ (Sk+1 6= 0) ∩ . . . (Sn 6= 0) et donc

P(Ank ) = P(Sk = 0)PSk =0 (Sk+1 6= 0, . . . , Sn 6= 0)

4
De façon évidente,

PSk =0 (Sk+1 6= 0, . . . , Sn 6= 0) = PSk =0 (Sk+1 − Sk 6= 0, . . . , Sn − Sk 6= 0)

On remarque que les variables Sk+1 − Sk , . . . , Sn − Sk ne dépendent que de Xk+1 , . . . , Xn et


sont donc indépendantes de Sk qui ne dépend que de X1 , . . . , Xk (et puisque les Xi sont, elles,
indépendantes). On a donc en fait

PSk =0 (Sk+1 6= 0, . . . , Sn 6= 0) = P(Sk+1 − Sk 6= 0, . . . , Sn − Sk 6= 0)

Remarquons que
 
[
P(Sk+1 − Sk 6= 0, . . . , Sn − Sk 6= 0) = P  (Sk+1 − Sk = j1 , . . . , Sn − Sk = jn−k )
j1 ,...,jn−k 6=0
X
= P (Sk+1 − Sk = j1 , . . . , Sn − Sk = jn−k )
j1 ,...,jn−k 6=0

la dernière égalité provenant de l’incompatiblité des événements. La question précédente donne


alors X
P(Sk+1 − Sk 6= 0, . . . , Sn − Sk 6= 0) = P (S1 = j1 , . . . , Sn−k = jn−k )
j1 ,...,jn−k 6=0

et en refaisant le chemin dans l’autre sens

P(Sk+1 − Sk 6= 0, . . . , Sn − Sk 6= 0) = P(S1 6= 0, . . . , Sn−k 6= 0)) = P(En−k )

Finalement, on trouve que


P(Ank ) = P(Sk = 0)P(En−k )

15. Soit n ∈ N. Soit ω ∈ Ω ; comme S0 (ω) = 0, il existe un plus grand k ∈ [|0, n|] tel que Sk (ω) = 0
et on a alors ω ∈ Ank . La réunion des parties An0 , . . . , Ann est donc égale à Ω. Ces parties étant
disjointes, on a donc
Xn Xn
1= P(Ank ) = P(Sk = 0)P(En−k )
k=0 k=0

(P(Sn = 0)xn ) et (P(En )xn ) étant des séries entières de rayon de convergence au moins égal
P P
16.
à 1 (|P(En )xn | ≤ 1 si |x| ≤ 1). On peut donc utiliser le théorème sur le produit de Cauchy et
écrire que pour x ∈] − 1, 1[

! ∞ ! ∞ n
X X X X
n n
P(Sn = 0)x P(En )x = un xn avec un = P(Sk = 0)P(En−k )
n=0 n=0 n=0 k=0

Avec la question précédente, on a donc



! ∞ ! +∞
X
n
X
n
X 1
P(Sn = 0)x P(En )x = xn =
1−x
n=0 n=0 n=0

17. Sn (ω) est la somme de n quantités valant 1 ou −1 et ne peut donc être nul que s’il y a autant
de 1 que de −1, c’est à dire que si n est pair. On a donc

∀n ∈ N, P(S2n+1 = 0) = 0

5
Supposons maintenant n pair et écrivons que n = 2p. La valeur de Sn (ω) ne dépend que des
valeurs de ω1 , . . . , ω2p . Il y a 22p = 4p choix pour ces valeurs et on a
αp
P(S2p = 0) =
4p
où αp est le nombre de uplets (ω1 , . . . , ω2p ) contenant autant de 1 que de −1. Choisir un tel
2p

uplet, c’est choisir la position des 1 et il y a p tels choix. On a donc

2p

p
P(S2p = 0) =
4p

18. On en déduit que pour x ∈] − 1, 1[,


+∞ +∞ 2p

X
n
X p 1
P(Sn = 0)x = x2p = √
4p 1 − x2
n=0 p=0

et donc que
+∞ √ r
X
n 1 − x2 1+x
P(En )x = =
1−x 1−x
n=0

19. Considérons la fonction f définie par


r r
π 1+x
∀x ∈] − 1, 1[, f (x) =
2 1−x
√ √
Comme 1 − xf (x) → π quand x → 1− , on peut utiliser la partie B (et la question précédente)
pour obtenir
n r
X π √
P(En ) ∼ 2 n
2
k=0

Par ailleurs P(T > n) = P(En ) est le terme général d’une suite décroissante ((T > n) ⊂ (T >
n − 1)) et le théorème taubérien indique que
r
π 1
P(En ) ∼ √
2 n

On a donc r
2
P(En ) ∼
πn
20. On a \ \
(T = +∞) = (T > n) = En
n∈N n∈N

Par continuité décroissante des probabilités (appliqué avec la suite décroissante d’ensemble de
terme général E0 ∩ · · · ∩ En = En ), on en déduit que

P(T = ∞) = lim P(En ) = 0


n→+∞

6
21. On a (T = n) = (T > n − 1) \ (T > n) et comme (T > n) ⊂ (T > n − 1), on a P(T = n) =
P(T > n − 1) − P(T > n) et
n
X n
X
k
P(T = k)x = (P(T > k − 1) − P(T > k))xk
k=0 k=0
n
X n
X
= P(T > −1) + x P(T > k − 1)xk−1 − P(T > k)xk
k=1 k=0
n−1
X n
X
= 1+x P(Ek )xk − P(Ek )xk
k=0 k=0

Pour x ∈] − 1, 1[, les termes du membre de droite admettent une limite quand n → +∞ et, avec
l’expression des sommes,
+∞ r
X
k 1+x p
P(T = k)x = 1 + (x − 1) = 1 − 1 − x2
1−x
k=0

La relation reste vraie pour x = 0 car (T = n)n∈N est un système complet d’événements (et se
lit 1 = 1).

22. On commence par chercher le DSE de h : x 7→ 1 − 1 − x2 . Pour cela on remarque que (pour
x ∈] − 1, 1[)
∞ 2k ∞ 2k
x X X
h0 (x) = √ =x k
x2k = k
x2k+1
1 − x2 4k 4k
k=0 k=0

Par théorème de primitivation des séries entières,


∞ ∞ 2(j−1)

2k

j−1
X X
h(x) = h(0) + k
x2k+2 = x2j
4k (2k + 2) j−1
4 .2j
k=0 k=1

Par unicité des DSE, on en déduit que


2(j−1)

j−1
P(T = 2j) = j−1
4 .2j
Il nous reste à remarquer que

j2
 
2(j − 1) (2j − 2)! (2j)!
= =
j−1 ((j − 1)!)2 (j!)2 2j(2j − 1)

pour en déduire que


2j

j
P(T = 2j) =
4j (2j − 1)

7
Mines-Ponts MP Mathématiques 2 2016

Théorème taubérien de Hardy-Littlewood-Karamata

Dans tout le problème, I désigne l'intervalle ]0, +∞[.

A Une intégrale à paramètre

Pour tout x ∈ R on pose, sous réserve d'existence,


+∞
e−u +∞
e−u
Z Z
F(x) = √ du et K = √ du
0 u(u + x) 0 u
e−u
1. Démontrer que ψ : u 7→ √ est intégrale sur I.
u
2. Déterminer les valeurs de x pour lesquelles F(x) est dénie.
3. Montrer que la fonction F est de classe C 1 sur I et exprimer F0 (x) sous forme intégrale.
4. En déduire que pour tout x ∈ I, xF0 (x) − (x − 12 )F(x) = −K.
√ −x
5. Pour tout x ∈ I, on pose
Z G(x) = xe F(x). Montrer qu'il existe une constante réelle C telle que pour tout
x
e−t
x ∈ I, G(x) = C − K · √ dt.
0 t
6. Déterminer les limites de G en 0 et +∞, et en déduire la valeur de K.

B Étude de deux séries de fonctions

+∞ −nx +∞
e √ −nx
Dans toute cette partie, on pose f (x) = et g(x) = .
X X
√ ne
n=1
n n=0
7. Montrer que f et g sont dénies et continues sur I.
e−ux
Z +∞ Z +∞ −ux
e
8. Montrer que pour tout x ∈ I, √ du 6 f (x) 6 √ du.
1 u 0 u
En déduire un équivalent de f (x) lorsque x → 0.
n
!
1 √
9. Montrer que la suite converge.
X
√ −2 n
k=1
k
n>1
n
!
1
10. Démontrer que pour tout x > 0, la série e−nx converge et exprimer sa somme h(x) en fonction
X X

n>1 k=1 k
de f (x) pour tout x ∈ I.

π
11. En déduire un équivalent de h(x) lorsque x → 0. Montrer alors que g(x) est équivalent à 3/2 lorsque x → 0.
2x

C Séries de fonctions associées à des ensembles d'entiers

À tout ensemble A ⊆ N on associe la suite (an ) dénie par

1 si n ∈ A,

an =
0 sinon.

1/3
Mines-Ponts MP Mathématiques 2 2016

+∞
Soit IA l'ensemble des réels x > 0 pour lesquels la série an e−nx converge. On pose fA (x) = an e−nx pour tout
X X

n>0 n=0
x ∈ IA . Enn, sous réserve d'existence, on pose Φ(A) = lim xfA (x) et on note S l'ensemble des parties A ⊆ N pour
x→0
lesquelles Φ(A) existe.
12. Quel est l'ensemble Ia si A est ni ? Si A est inni, montrer que l'on peut extraire une suite (bn ) de la suite
(an ) telle que pour tout n ∈ N, bn = 1. Déterminer IA dans ce cas.
13. Soit A ∈ S et (an ) la suite associée. Pour tout entier
X naturel n, on note A(n) l'ensemble des éléments de A
qui sont 6 n. Vérier que pour tout x > 0 la série Card(A(n))e−nx converge et que
n>0

+∞
X fA (x)
Card(A(n))e−nx =
1 − e−x
n=0

Dans la question suivante, A = A1 désigne l'ensemble des carrés d'entiers naturels non nuls.
+∞
fA1 (x) √
14. Montrer que si x > 0, b nce−nx où b·c désigne la partie entière.
X
−x
=
1−e
n=0
+∞
√ fA1 (x)
En déduire un encadrement de , puis un équivalent de fA1 en 0. Prouver alors que
X
ne−nx −
1 − e−x
n=0
A1 ∈ S et donner Φ(A1 ).
Dans la question suivante, A = A2 désigne l'ensemble constitués des entiers qui sont la sommes des carrés de deux
entiers naturels non nuls. On admet que A2 appartient à S, et on désire majorer Φ(A2 ).
Soit v(n) le nombre de couple d'entiers naturels non nuls (p, q) pour lesquels n = p2 + q 2 .
15. Montrer que pour tout réel x > 0, la série v(n)e−nx converge et établir que
X

n>0

+∞
X
v(n)e−nx = (fA1 (x))2 .
n=0

Montrer alors que pour tout x > 0, fA2 (x) 6 (fA1 (x))2 . En déduire un majorant de Φ(A2 ).

D Étude de deux séries de fonctions

Soit (αn )n>0 une suite de nombres réels positifs tels que pour tout réel x > 0, la série αn e−nx converge. On
X

n>0
suppose que
+∞
!
X
lim x αn e−nx = ` ∈ [0, +∞[.
x→0
n=0

On note F l'espace vectoriel des fonctions de [0, 1] dans R, E le sous-espace de F des fonctions continues par morceaux
et E0 le sous-espace de E des fonctions continues sur [0, 1]. On munit E de la norme k k∞ dénie par la formule
kψk∞ = sup |ψ(t)|.
t∈[0,1]
Si ψ ∈ E, on note L(ψ) l'application qui à x > 0 associe
+∞
X
(L(ψ))(x) = αn e−nx ψ(e−nx ).
n=0

2/3
Mines-Ponts MP Mathématiques 2 2016

16. Montrer que L(ψ) est bien dénie pour tout ψ ∈ E et que l'application L est une application linéaire de E
dans F. Vérier que pour tous ψ1 , ψ2 dans E1 , ψ1 6 ψ2 entraîne L(ψ1 ) 6 L(ψ2 ).
On note E1 l'ensemble des ψ ∈ E pour lesquels lim x(L(ψ))(x) existe et si ψ ∈ E1 , on pose
x→0

∆(ψ) = lim x(L(ψ))(x)


x→0

17. Vérier que E1 est un sous espace vectoriel de E et que l'application ∆ est une forme linéaire continue de
(E1 , k k∞ ).
18. Montrer que pour tout p ∈ N, ep : t ∈ [0, 1] 7→ tp appartient à E1 et calculer ∆(ep ). En déduire que E0 ⊆ E1
et calculer ∆(ψ) pour tout ψ ∈ E0 .
Pour tous a, b ∈ [0, 1] tel que a < b, on note 1[a,b] : [0, 1] → {0, 1} la fonction dénie par

1 si x ∈ [a, b]

1[a,b] (x) =
0 sinon.

Soit a ∈]0, 1[ et ε ∈]0, min(a, 1 − a)[. On note

si x ∈ [0, a − ε]

 1

a−x
g− (x) = si x ∈]a − ε, a[
 ε
si x ∈ [a, 1]

0

et
si x ∈ [0, a]

 1

a+ε−x
g+ (x) = si x ∈]a, a + ε[
ε
si x ∈ [a + ε, 1].

0

19. Vérier que g− et g+ appartiennent à E0 et calculer ∆(g− ) et ∆(g+ ). Montrer alors que 1[0,a] ∈ E1 et calculer
∆(1[0,a] ). En déduire que E1 = E et donner ∆(ψ) pour tout ψ ∈ E.
On considère maintenant la fonction ψ dénie sur [0, 1] par la formule :

si x ∈ [0, 1e [
(
0
ψ(x) = 1
si x ∈ [ 1e , 1].
x
20. Calculer (L(ψ))( N1 ) pour tout entier N > 0 et en déduire la limite
N
1 X
lim αk
N→+∞ N
k=0

(théorème taubérien).
On rappelle que v(n) est le nombre de couples d'entiers naturels non nuls (p, q) tels que n = p2 + q 2 .
n
1 1X
21. Si A ∈ S, que vaut lim Card (A(n)) ? Déterminer alors lim v(k).
n→+∞ n n→+∞ n
k=1
Fin du problème

3/3
Mines-Ponts MP Un corrigé de Mathématiques 2 2016

Théorème taubérien de Hardy-Littlewood-Karamata

A Une intégrale à paramètre

e−u
1. La fonction ψ : u 7→ √ est continue sur I par théorèmes généraux.
u
1 1
On a ψ(u) ∼ 1/2 or la fonction u 7→ 1/2 est intégrable sur ]0, 1] car 12 < 1
u→0 u u
Donc par comparaison à une fonction positive, ψ est intégrable sur ]0,1] 
1
De plus par croissance comparée u2 ψ(u) −→ 0 donc ψ(u) = o
u→+∞ u→+∞ u2
1
or la fonction u 7→ 2 est intégrable sur [1, +∞[ car 2 > 1
u
Donc par comparaison à une fonction positive, ψ est intégrable sur [1, +∞[
e−u
Ainsi ψ : u 7→ √ est intégrale sur I En particulier, on en déduit l'existence de K.
u
2. Analyse : Soit x ∈ R tel que F(x) existe.
e−u
Par les limitations du programme, la fonction u 7→ √ est dénie (et continue par morceaux) sur I
u(u + x)
donc x > 0
e−u
De plus a fonction u 7→ √ étant positive sur I, elle y est intégrable.
u(u + x)
Si on avait x = 0, on aurait √u(u+x)
e−u 1
∼ u3/2 et par équivalence entre fonctions positives u 7→ 1
u3/2
serait
u→0
intégrable sur ]0, 1] et donc on aurait 3/2 < 1 Absurde
donc x > 0
Synthèse : Soit x > 0.
e−u
La fonction u 7→ √ est continue sur I
u(u + x)
e−u e−u
 
1 1
De plus √ ∼ et √ = o
u(u + x) u→0 xu1/2 u(u + x) u→+∞ u2
e−u
On peut conclure comme en 1 que u 7→ √ est intégrable sur I
u(u + x)
Conclusion : l'ensemble les valeurs de x pour lesquelles F(x) est dénie est I =]0, +∞[
e−u
3. On pose f : (x, u) ∈ I2 7−→ √
u(u + x)
(i) Soit u ∈ I.
e−u ∂f −e−u
La fonction x 7−→ √ est de classe C 1 sur I et admet comme dérivée x 7−→ (x, u) = √
u(u + x) ∂x u(u + x)2
e −u
(ii) Soit x ∈ I. La fonction u 7−→ f (x, u) = √ est continue et intégrable sur I d'après la question
u(u + x)
précédente.
∂f −e−u
(iii) Soit x ∈ I. La fonction u 7−→ (x, u) = √ est continue sur I
∂x u(u + x)2
(iv) Soit a < b dans I. On a l'hypothèse de domination :
∂f e−u
∀x ∈ [a, b], ∀t ∈ I, (x, u) 6 √
∂x u(u + a)2

1/9
Mines-Ponts MP Un corrigé de Mathématiques 2 2016

e−u
et la fonction u 7→ √ est continue et intégrable sur ]0, +∞[ (l'intégrabilité étant analogue aux
u(u + a)2
précédentes)
En conclusion avec (i), (ii), (iii) et (iv), le théorème de Leibniz s'applique :
+∞
−e−u
Z
la fonction F est de classe C 1 sur I et F0 : x 7−→ √ du
0 u(u + x)2
4. Soit x ∈ I. Z +∞ −u √
−e−u (x + u)
+∞
e−u u −e−u
Z Z +∞ Z +∞
e u
On a xF0 (x)
= √ du + √ du = √ du + du
0 u(u + x) 2
0 u(u + x) 2
0 u(u + x) 0 (u + x)2
Z +∞ −u √
e u
donc xF0 (x) = −F(x) + du
0 (u + x)2
On va eectuer une intégration par parties (sous réserve d'existence) avec des fonctions de classe C 1 .
Z +∞ −u √  −u √ u→+∞ Z +∞  √ 
e u −e u e−u e−u u
du = + √ − du
0 (u + x)2 u + x u=0 0 2 u(u + x) u+x
Le terme entre crochets

est nul par croissance comparée, ce√qui valide l'intégration par √parties
e−u u e−u e−u u
Z +∞ Z +∞ Z +∞ Z +∞ −u
1 e u
ainsi du = √ du − du = F(x) − du
0 (u + x)2 0 2 u(u + x)0 u+x 2 0 u+x
On a bien le droit de couper l'intégrale en 2 car on a reconnu F(x)
Z +∞ −u √
e u
donc xF (x) = − 2 F(x) −
0 1
du
0 u+x
Z +∞ −u √
−xe−u
Z +∞
e u
donc xF0 (x) − (x − 21 )F(x) = √ du − du
0 u(u + x) 0 u + x
−(x + u)e−u
Z +∞
donc xF (x) − (x − 2 )F(x) =
0 1 √ du après mise au même dénominateur
0 u(u + x)
On en déduit pour tout x ∈ I, xF0 (x) − (x − 21 )F(x) = −K.
5. La fonction G est de classe C 1 par produit
√ √
et on a G0 (x) = 2√1 x e−x F(x) − xe−x F(x) + xe−x F0 (x) =
−x
e√
xF0 (x) − (x − 12 )F(x)

x
e−x
donc G0 (x) = −K √
x
e−x
La fonction x 7→ K √ étant continue sur ]0, +∞[ et intégrable au voisinage de 0
x
Z x −t
e e−x
alors la fonction x 7→ K · √ dt est de classe C 1 et de dérivée x 7→ K √
0 t x
Z x −t
e
donc la fonction x 7→ G(x) + K · √ dt est constante sur l'intervalle I
0 t
x
e−t
Z
Ainsi il existe une constante réelle C telle que pour tout x ∈ I, G(x) = C − K · √ dt
0 t
6. Soit x > 0. La fonction u 7→ ux est C 1 , strictement croissante et bijective de I vers I
On eectue dans G(x) sous forme intégrale le changement de variable t = ux ; u = tx ; du = xdt
xe−tx
+∞
e−tx
Z Z +∞
donc G(x) = e−x √ dt = e−x √ dt
0 t(tx + x) 0 t(t + 1)
On considère une suite (xn ) à valeurs dans I qui converge vers 0
e−txn 1
On pose fn : t 7→ √ et f : t 7→ √
t(t + 1) t(t + 1)

2/9
Mines-Ponts MP Un corrigé de Mathématiques 2 2016

(i) Les fonctions fn sont continues sur I


(ii) La suite (fn )n converge simplement vers f sur I
(iii) La fonction f est continue sur I
1 1
(iv) De plus f (t) ∼ 1/2 et f (t) ∼ 3/2 ainsi comme en 1., f est intégrable sur I
t→0 t t→+∞ t
et on a l'hypothèse de domination :
∀n ∈ N, ∀t ∈ I, |fn (t)| 6 f (t)
Z Z
Avec (i), (ii), (iii) et (iv), le théorème de convergence dominée s'applique et on a lim fn = f
n→+∞ I I
+∞
e−tx +∞
Z Z
1
Donc par caractérisation séquentielle de la limite lim √ dt = √ dt
x→0 0 t(t + 1) 0 t(t + 1)
Remarque : On aurait pu utiliser directement l'extension du théorème de convergence dominé
Z +∞
1
donc par produit lim G(x) = dt √
x→0 0 t(t + 1)

On la fonction t 7→ t est de classe C 1 , strictement

croissante et bijective de I vers I
on eectue le changement de variables
Z +∞ Z +∞
v = t ; t = v 2 ; dt = 2vdv
1 1
et donc √ dt = 2 2
dv = 2 [arctan(v)]v→+∞
v=0 =π
0 t(t + 1) 0 v + 1
donc lim G(x) = π
x→0
+∞
e−tx +∞
Z Z
1
de plus pour x > 0, on a 0 6 G(x) = e−x √ dt 6 e−x √ dt
0 t(t + 1) 0 t(t + 1)
donc par théorème d'encadrement lim G(x) = 0
x→+∞
x
e−t
Z
La fonction t 7→ étant continue et intégrable sur I, on a lim
−t
e√
t
√ dt = 0
x→0 0 t
e−t x
Z
comme G : x 7→ C − K · √ dt on a donc C = π
0 t
Z x −t Z +∞ −t
e e
or lim √ dt = √ dt = K
x→+∞ 0 t 0 t

donc 0 = lim G(x) = π − K de plus on a K > 0 par positivité de l'intégrale donc K = π
2
x→+∞

B Étude de deux séries de fonctions



7. (i) Pour n > 1, la fonction x 7→ ne−nx est continue
√ sur I √
(ii) Soit a < b dans I. On a : ∀x ∈ I, ∀n ∈ N∗ , | ne−nx | 6 ne−na
2√ √
 
1 X 1
or n ne −na
−→ 0 par croissance comparée donc ne −na
= o et converge
n→+∞ n→+∞ n2 n2
n>1
X√
donc par comparaison à une série à termes positifs la série ne −na
converge
n>1

Ainsi la série de fonctions ne−nx converge normalement sur tout segment de I
X 
x 7→
n>1

(iii) Ainsi converge simplement vers g sur I
X
−nx

x 7→ ne
n>1
Avec (i), (ii) et (iii), on vient de montrer que g est dénie et continue sur I
de manière analogue f est dénie et continue sur I

3/9
Mines-Ponts MP Un corrigé de Mathématiques 2 2016

e−ux
8. Soit x ∈ I. On pose l : u 7→ √ .
u
1
Cette fonction est le produit des deux fonctions positives et décroissantes sur I : u 7→ √ et u 7→ e−ux
u
donc la fonction l est décroissante sur I, et comme en 1., la fonction l est continue et intégrable sur I
Z n+1 Z n
donc pour n > 1, on a l(u)du 6 l(n) 6 l(u)du
n n−1
N
e−ux e−nx
Z N+1 Z N −ux
e
En sommant on obtient pour N > 1,
X
√ du 6 √ 6 √ du
1 u n=1
n 0 u
Z +∞ −ux Z +∞ −ux
e e
Puis par passage à la limite quand N → +∞ : √ du 6 f (x) 6 √ du
1 u 0 u
On eectue le changement de variable C 1 bijectif, strictement croissant : ux = t, xdu = dt
+∞
e−t e−t +∞
Z Z
on obtient √ dt 6 f (x) 6√ dt
x xt 0 xt
√ +∞
e−t √
Z
donc par théorème d'encadrement lim xf (x) = √ dt = K = π
x→0 0 t
r
π
On en déduit l'équivalent f (x) ∼
x→0 x
n+1 n
! !
1 √ 1 √ 1 √ √
9. Soit n ∈ N∗ . On a
X X
√ −2 n+1 − √ −2 n = √ + 2( n − n + 1)
k=1
k k=1
k n+1
n+1
! n
! √ √
X 1 √ 1 √ 1 2 n− n+1
donc
X
√ −2 n+1 − √ −2 n = √ −√ √ = p 60
k=1
k k=1
k n+1 n+ n+1 n(n + 1) + n + 1
n
!
1 √
donc la suite est décroissante
X
√ −2 n
k=1
k
n>1
n
1
Z n+1
1 √ √
En utilisant une comparaison série intégrale comme en 8. on a
X
√ > √ dt = 2 n + 1 − 2 1
k=1
k 1 t
n
1 √ √ √
donc
X
√ − 2 n > 2 n + 1 − 2 − 2 n > −2
k=1
k
n
!
1 √
Ainsi la suite et minorée par -2 (et décroissante)
X
√ −2 n
k=1
k
n>1
n
!
X 1 √
d'où la suite √ −2 n converge
k=1
k
n>1
n
!
1
10. Soit x > 0. On a pour n ∈ N∗ , 0 6
X
√ e−nx 6 ne−nx
k=1
k
n
!
1
or n3 e−nx −→ 0 ce qui prouve que
X
e−nx = o 1

√ n 2
n→+∞ k n→+∞
k=1
n
!
1
On peut donc conclure comme en 7. que la série e−nx converge
X X

n>1 k=1 k

4/9
Mines-Ponts MP Un corrigé de Mathématiques 2 2016

On considère les séries de termes généraux ak = et bk = e−kx géométrique de raison e−x ∈]0, 1[
−kx
e√
k
+∞ +∞
b1 1
ces séries sont absolument convergentes de sommes ak = f (x) et
X X
bk = −x
= x
1−e e −1
k=1 k=1
n n
e−kx −(n−k)x
On eectue le produit de Cauchy de ces séries absolument convergentes : cn =
X X
ak bn−k = √ e
k=1 k=1
k
+∞ +∞ +∞
! !
f (x)
donc h(x) = Ainsi h(x) =
pour tout x ∈ I
X X X
cn = an bn
ex − 1
n=1 n=1 n=1

π
11. Quand x → 0, on a e − 1 ∼ x donc avec 8., on a h(x) ∼ 3/2
x
x→0 x
+∞ n +∞ X n
! ! !
X X 1 √ 1 √
On a 2g(x) + 0 + = h(x) donc g(x) = 2 h(x) +
X
−nx 1 −nx
√ −2 n e √ −2 n e
n=1 k=1 k n=1 k=1 k
n
1 √
Toute suite convergente étant bornée, le 9. nous fournit M > 0 tel que ∀n ∈ N∗ ,
X
√ −2 n 6M
k=1
k
+∞ X n +∞
!
1 √ M M
Ainsi
X X
√ − 2 n e−nx 6 M e−nx = x ∼
k e −1 x
n=1 k=1 n=1
+∞ X n
!
1 √
donc √ − 2 n e−nx = o (h(x)) donc g(x) ∼ 21 h(x)
X

n=1 k=1 k x→0 x→0



π
Ainsi g(x) est équivalent à 3/2 lorsque x → 0
2x

C Séries de fonctions associées à des ensembles d'entiers

12. Si A est nie alors fA : x 7→ e−nx est bien dénie sur R+ donc si A est ni, alors IA = [0, +∞[
X

n∈A
On suppose désormais que A est inni.
On dénit ϕ par récurrence par ϕ(0) = min A et ϕ(n + 1) = min (A \ {ϕ(k) / 0 6 k 6 n})
Par construction la suite ϕ est strictement croissante à valeurs dans A donc telle que ∀n ∈ N, aϕ(n) = 1
on peut extraire une suite (bn ) = aϕ(n) de la suite (an ) telle que pour tout n ∈ N, bn = 1


Soit x = 0, la suite (an e−nx ) ne converge pas vers 0 avec la suite extraite bn e−ϕ(n)x = (1)n>0


donc la série an e−nx diverge grossièrement.


X

n>0
Si x > 0, on a |an e−nx | 6 e−nx ce qui donne la convergence de la série
X
an e−nx
n>0

Ainsi si A est inni, alors IA =]0, +∞[


+∞ +∞
1
13. Soit x > 0, on a : fA (x) = ak e−kx et
X X
= e−kx
1 − e−x
k=0 k=0
n
On remarque que pour n ∈ N,on a Card(A(n)) = ak , on peut donc faire le produit de Cauchy de ces deux
X

k=0
+∞
fA (x)
séries absolument convergentes pour obtenir :
X
Card(A(n))e−nx =
1 − e−x
n=0

5/9
Mines-Ponts MP Un corrigé de Mathématiques 2 2016


14. Soit n ∈ N. On a A1 (n) = k2 / k ∈ N∗ et k2 6 n = k2 / k ∈ N∗ et k 6 n
 
√ √
donc A1 (n) = k2 / 1 6 k 6 b nc de cardinal b nc


X √ +∞
fA1 (x)
Soit x > 0. À l'aide de la question précédente −x
= b nce−nx
1−e
n=0

X√ +∞ +∞
√ √ fA1 (x) 1
Pour n ∈ N, on a n − b nc ∈ [0, 1] donc 0 6
X
ne−nx − −x
6 e−nx =
1−e 1 − e−x
n=0 n=0
donc (1 − e−x )g(x) − 1 6 fA1 (x) 6 (1 − e )g(x) car 1 − e−x
−x >0

or d'après 11., (1 − e−x )g(x) équivaut à √π quand x → 0
2 x

2 xfA1 (x)
donc √ tend vers 1 par théorème d'encadrement
π
√ √
π xπ
Ainsi fA1 (x) ∼ √ donc xfA1 (x) ∼ donc A1 ∈ S et Φ(A1 ) = 0
x→0 2 x x→0 2
15. Soit x > 0. On note la suite (an ) associée à l'ensemble A = A1
Soit n ∈ N∗ . On a v(n) = card (α, β) ∈ A21 / α + β = n = card ({(k, n − k) / k ∈ A1 et n − k ∈ A1 })
 
n−1 n 0
donc v(n) = ak an−k car a0 = 0 et aussi v(0) = 0 =
X X X
ak an−k = ak a0−k
k=1 k=0 k=0
On eectue ensuite le produit de Cauchy de la série ak e−kx absolument convergente par elle-même
X

k>0
+∞
pour obtenir : la série v(n)e−nx converge et
X X
v(n)e−nx = (fA1 (x))2
n>0 n=0

Pour n ∈ N. On note le terme de la suite (an ) associée à l'ensemble A2


a(2) (n)
On a a(2) (n) 6 v(n) ainsi pour tout x > 0, fA2 (x) 6 (fA1 (x))2
√ π
donc xfA2 (x) 6 ( xfA1 (x))2 d'où Φ(A2 ) 6
4

D Étude de deux séries de fonctions

16. Soit ψ1 , ψ2 ∈ E. Soit λ ∈ R.


Soit x > 0.
On a |αn e−nx ψ1 (e−nx )| 6 kψ1 k∞ αn e−nx
donc la série αn e−nx ψ1 (e−nx ) converge par comparaison entre séries à termes positifs
X

n>0
donc L(ψ1 )(x) existe dans R
+∞ +∞ +∞
On a L(λψ1 + ψ2 )(x) =
X X X
αn e−nx λψ1 (e−nx ) + ψ2 (e−nx ) = λ αn e−nx ψ1 (e−nx ) + αn e−nx ψ2 (e−nx )

n=0 n=0 n=0
donc L(λψ1 + ψ2 )(x) = λL(ψ1 )(x) + L(ψ2 )(x)
ainsi L(λψ1 + ψ2 ) = λL(ψ1 ) + L(ψ2 )
donc L est bien dénie sur E et l'application L est une application linéaire de E vers RI = F(]0, +∞[, R)
Erreur d'énoncé ? Selon lequel, l'espace d'arrivée serait R[0,1] = F([0, 1], R)) !
On suppose que ψ1 6 ψ2

6/9
Mines-Ponts MP Un corrigé de Mathématiques 2 2016

On a pour tout x > 0 et pour tout n ∈ N, αn e−nx ψ1 (e−nx ) 6 αn e−nx ψ2 (e−nx ) car αn e−nx > 0
donc L(ψ1 )(x) 6 L(ψ2 )(x) par comparaison de séries
donc pour tous ψ1 , ψ2 dans E, ψ1 6 ψ2 entraîne L(ψ1 ) 6 L(ψ2 )
17. On a bien E1 ⊂ E (i) et E1 6= ∅ (ii) car θ : x ∈ [0, 1] 7→ 0 vérie θ ∈ E et lim x(L(θ))(x) = 0
x→0
Soit ψ1 , ψ2 ∈ E1 et λ ∈ R.
Pour x > 0, on a x(L(λψ1 + ψ2 ))(x) = λx(L(ψ1 ))(x) + x(L(ψ2 ))(x)
donc par combinaison linéaire de limites on a lim x(L(λψ1 + ψ2 ))(x) = λ lim x(L(ψ1 ))(x) + lim x(L(ψ2 ))(x)
x→0 x→0 x→0
ceci prouve que λψ1 + ψ2 ∈ E1 donc E1 est stable par combinaison linéaire (iii)
Avec (i), (ii) et (iii), E1 est un sous espace vectoriel de E
De plus ∆(λψ1 + ψ2 ) = λ∆(ψ1 ) + ∆(ψ2 ) et ∆ : E1 −→ R donc ∆ est une forme linéaire de E1 .
+∞
De plus |x(L(ψ1 ))(x)| 6 kψ1 k∞ x
X
αn e−nx
n=0
Par passage à la limite en 0, on a |∆(ψ1 )| 6 `kψ1 k∞
d'où l'application ∆ est une forme linéaire continue de (E1 , k k∞ )
18. Soit p ∈ N. On a ep ∈ E car continue par morceaux sur [0, 1].
+∞
Soit x > 0. On a L(ep )(x) =
X
αn e−n(p+1)x
n=0
+∞
X
[(p + 1)x] αn e−n[(p+1)x]
donc xL(ep )(x) = n=0
et (p + 1)x > 0
p+1
Z 1
1
par composition de limites, on a ∆(ep ) = et ep ∈ E1 . On remarque que ∆(ep ) = ` ep
p+1 0
Z 1
Donc par combinaison linéaire , pour toute fonction polynomiale P, on a ∆(P) = ` P
0
Soit ψ ∈ E0 . Le théorème de Stone-Weierstrass nous fournit une suite de fonction polynomiale (Pk ) qui
converge uniformément vers ψ sur [0, 1]
Soit x > 0. Soit k ∈ N. On a |xL(ψ)(x) − xL(Pk )(x)| = x |L(ψ − Pk )(x)|
comme −kψ − Pk k∞ e0 6 ψ − Pk 6 kψ − Pk k∞ e0 ,
on a −kψ − Pk k∞ L(e0 ) 6 L(ψ − Pk ) 6 kψ − Pk k∞ L(e0 ) en utilisant 16.
Ainsi |xL(ψ)(x) − xL(Pk )(x)| 6 kψ − Pk k∞ xL(e0 )(x)
La fonction x 7→ xL(e0 )(x) est continue sur ]0, 1] et admet comme limite ` en 0
donc x 7→ xL(e0 )(x) est prolongeable par continuité sur le segment [0, 1]
donc le théorème des bornes atteintes nous fournit un majorant M > 0
donc ∀x ∈]0, 1], |xL(ψ)(x) − xL(Pk )(x)| 6 kψ − Pk k∞ M or lim kψ − Pk k∞ M = 0
k→+∞
donc la suite de fonction (x 7→ xL(Pk )(x))k>0 converge uniformément sur ]0, 1] vers x 7→ xL(ψ)(x)
En notant δk = lim xL(Pk )(x) = ∆(Pk ), le théorème de la double limite nous donne alors que la suite (δk )
x→0
converge vers un certain L ∈ R et L = lim xL(ψ)(x).
x→0
Ainsi ψ ∈ E1 . On en déduit que E0 ⊆ E1
Z 1 Z 1
La fonction : ψ ∈ E0 7→ ` ψ est une forme linéaire continue de (E0 , k k∞ ) car ∀ψ ∈ E0 , ` ψ 6 `kψk∞
0 0

7/9
Mines-Ponts MP Un corrigé de Mathématiques 2 2016

Z 1
Les applications ∆ et ψ 7→ ` ψ sont continues sur E et coïncident sur la partie des dense des fonctions
0
Z 1
polynomiales donc pour tout ψ ∈ E0 , on a ∆(ψ) = ` ψ
0

19. La fonction g− est continue en tous points de [0, 1] \ {a − ε, a}


De plus lim − g( x) = lim − g( x) = g( a − ε) = 1 de même en a
x→(a−ε) x→(a−ε)

donc g− et g+ (analogue) sont continues sur [0, 1] ainsi g− et g+ ∈ E0


Z 1 Z a−ε Z a Z 1 
On a ∆(g− ) = ` g− = ` g− + g− + g−
0 0 a−ε a
Z a−ε Z a Z 1
ε·1
on a g− = a − ε et g− = (aire d'un triangle) et g− = 0
0 a−ε 2 a
Z a Z a+ε Z 1  Z a Z a+ε Z 1
ε·1
et ∆(g+ ) = ` g+ + g+ + g+ et g+ = a et g+ = et g+ = 0
0 a a+ε 0 a 2 a+ε
 ε  ε
donc ∆(g− ) = ` a − et ∆(g+ ) = ` a +
2 2
On a 1[0,a] ∈ E et g− 6 1[0,a] 6 g+
donc pour tout x > 0, xL(g− )(x) 6 xL(1[0,a] )(x) 6 xL(g+ )(x)
 ε  ε ε
On a lim xL(g− )(x) = ` a − ceci nous fournit α1 > 0 tel que ∀x ∈]0, α1 ], xL(g− )(x) > ` a − −`
x→0 2 2 2
donc ∀x ∈]0, α1 ], xL(g− )(x) > ` (a − ε)
de même on peut trouver α2 > 0, on a ∀x ∈]0, α2 ], xL(g+ )(x) 6 ` (a + ε)
donc en prenant α = min(α1 , α2 ) on a ∀x ∈]0, α], xL(1[0,a] )(x) − `a 6 `ε
On vient de montrer que lim xL(1[0,a] )(x) = `a car `ε est aussi petit que l'on veut
x→0
Z 1
ainsi 1[0,a] ∈ E1 et ∆(1[0,a] ) = `a = ` 1[0,a]
0
Z 1
Pour 1[0,a[ , les calcul sont identiques ce qui donne : 1[0,a[ ∈ E1 et ∆(1[0,a[ ) = `a = ` 1[0,a[
0
Pour α ∈ [0, 1], on note δα = 1{α} .
Z 1
On a donc δa = 1[0,a] − 1[0,a[ ainsi par linéarité δa ∈ E1 et ∆(δa ) = 0 = ` δa
0
Z 1
On remarque que L(δ0 ) : x 7→ 0 donc on a encore : δ0 ∈ E1 et ∆(δ0 ) = 0 = ` δ0
0
Z 1
En ce qui concerne 1[a,b] = 1[0,b] − 1[0,a[ , on a 1[a,b] ∈ E1 et ∆(1[a,b] ) = `(b − a) = ` 1[a,b]
0
C'est analogue pour 1]a,b] , 1]a,b[ et 1[a,b[ et cela reste valable même si a = 0
On sait que E1 ⊂ E. Soit maintenant ψ ∈ E.
On peut écrire ψ = ϕ + E où ϕ est continue sur [0, 1] et E est une fonction en escalier
On peut écrire E = λi 1Ji où (λi )i∈I est une famille nie de réels
X

i∈I
et (Ji )i∈I est une famille nie d'intervalles de [0, 1] (éventuellement singleton)
or ϕ ∈ E1 d'après 18 et les 1Ji ∈ E1 d'après ce qui précède
Z 1 Z 1
et on a ∆(ϕ) = ` ϕ et ∆(1Ji ) = ` 1 Ji
0 0
Comme ∆ est linéaire sur le sous-espace vectoriel E1 ,

8/9
Mines-Ponts MP Un corrigé de Mathématiques 2 2016

Z 1
on en déduit ψ ∈ E1 et ∆(ψ) = ∆(ϕ) +
X
λi ∆(1Ji ) = ` (ϕ + E)
i∈I 0
Z 1
On en déduit que E1 = E et ∆(ψ) = ` ψ pour tout ψ ∈ E
0

+∞ N N
20. On a (L(ψ))( N1 ) =
X X X
αn e−n/N ψ(e−n/N ) = αn e−n/N ψ(e−n/N ) = αn e−n/N en/N
n=0 n=0 n=0
N
donc (L(ψ))( N1 ) =
X
αk
k=0
Z 1 Z 1
On a lim x(L(ψ))(x) = ∆(ψ) = ` ψ=` ψ = `(ln(1) − ln(1/e))
x→0 0 1/e
N +∞
!
1 X
donc par composition de limites lim
X
αk = ` = lim x αn e−nx
N→+∞ N x→0
k=0 n=0

n
21. En reprenant les notations de la partie C. On a Card (A(n)) =
X
ak
k=0
+∞
!
comme A ∈ S, lim x
X
an e−nx = lim xfA (x) = Φ(A)
x→0 x→0
n=0
On peut appliquer donc le résultat précédent à la suite (an )
+∞
!
1
donc lim
X
Card (A(n)) = lim x an e−nx = lim xfA (x)
n→+∞ n x→0 x→0
n=0
1
Si A ∈ S, alors lim Card (A(n)) = Φ(A)
n→+∞ n

Pour tout x > 0, la série v(n)e−nx converge ayant pour somme (fA1 (x))2 et pour tout n ∈ N, v(n) > 0
X

n>0
π
de plus lim x (fA1 (x)) =
2
d'après 14 et 15
x→0 4
n
1X π
On peut donc appliquer les résultats de cette partie et alors lim v(k) =
n→+∞ n 4
k=1

9/9
Mathématiques 1

2016
PC
4 heures Calculatrices autorisées
On utilise la fonction Gamma d’Euler Γ (partie I) pour calculer, en partie II, une intégrale dépendant d’un
paramètre. En partie III, en liaison avec des variables aléatoires suivant une loi de Poisson, on détermine
l’équivalent, quand 𝑛 → +∞, de sommes dépendant d’un paramètre entier 𝑛. Les trois parties sont largement
indépendantes.

I Autour de la fonction Gamma d’Euler


+∞

Pour 𝑥 ∈ ℝ, on pose, lorsque cela a un sens, Γ(𝑥) = ∫ 𝑡u�−1 e−u� d𝑡.


0

I.A –
I.A.1) Quel est le domaine de définition 𝒟 de la fonction Γ ?
I.A.2) Pour tout 𝑥 ∈ 𝒟, exprimer Γ(𝑥 + 1) en fonction de 𝑥 et de Γ(𝑥).
En déduire, pour tout 𝑥 ∈ 𝒟 et tout 𝑛 ∈ ℕ∗ , une expression de Γ(𝑥 + 𝑛) en fonction de 𝑥, 𝑛 et Γ(𝑥), ainsi que
la valeur de Γ(𝑛) pour tout 𝑛 ⩾ 1.
+∞ +∞

I.A.3) Montrer l’existence des deux intégrales ∫ e −u�2


d𝑡 et ∫ e−u� d𝑡 et les exprimer à l’aide de Γ.
4

0 0

I.B –
I.B.1) Soit 𝑎 et 𝑏 deux réels tels que 0 < 𝑎 < 𝑏. Montrer que, pour tout 𝑡 > 0 et tout 𝑥 ∈ [𝑎, 𝑏],

𝑡u� ⩽ max(𝑡u� , 𝑡u� ) ⩽ 𝑡u� + 𝑡u�

I.B.2) Montrer que Γ est de classe 𝒞∞ sur 𝒟.


Soit 𝑘 ∈ ℕ∗ et 𝑥 ∈ 𝒟. Exprimer Γ(u�) (𝑥), dérivée 𝑘-ième de Γ au point 𝑥, sous forme d’une intégrale.

I.C –
I.C.1) Montrer que Γ′ s’annule en un unique réel 𝜉 dont on déterminera la partie entière.
I.C.2) En déduire les variations de Γ sur 𝒟. Préciser en particulier les limites de Γ en 0 et en +∞. Préciser
également les limites de Γ′ en 0 et en +∞. Esquisser le graphe de Γ.

II Une transformée de Fourier


+∞

Pour 𝑥 ∈ ℝ, on pose 𝐹 (𝑥) = ∫ e−u� 𝑡−3/4 eiu�u� d𝑡, où i désigne le nombre complexe de module 1 et d’argument 𝜋/2.
0

ℝ→ℂ
II.A – Montrer que la fonction 𝐹 : est définie et de classe 𝒞∞ sur ℝ.
𝑥 ↦ 𝐹 (𝑥)
Soit 𝑘 un entier naturel non nul et soit 𝑥 un réel. Donner une expression intégrale de 𝐹 (u�) (𝑥), dérivée 𝑘-ième de
𝐹 en 𝑥. Préciser 𝐹 (0).

II.B –
II.B.1) Montrer qu’au voisinage de 𝑥 = 0, la fonction 𝐹 peut s’écrire sous la forme
+∞
(i𝑥)u�
𝐹 (𝑥) = ∑ 𝑐u� (𝑆)
u�=0
𝑛!

où 𝑐u� est la valeur de Gamma en un point à préciser. On exprimera 𝑐u� en fonction de 𝑛 et de 𝑐0 .

2016-02-10 19:46:40 Page 1/3


Quel est le rayon de convergence de la série entière qui apparaît au second membre de (𝑆) ?

II.B.2) On admet que Γ(𝑥) ∼ 2𝜋 𝑥(u�−1/2) e−u� .
u�→+∞
Étudier si la série du second membre de (𝑆) converge absolument lorsque |𝑥| = 𝑅.
II.B.3) Soit 𝑅(𝑥) la partie réelle et 𝐼(𝑥) la partie imaginaire de 𝐹 (𝑥).
Déterminer, au voisinage de 0, le développement limité de 𝑅(𝑥) à l’ordre 3 et de 𝐼(𝑥) à l’ordre 4.

II.C –
II.C.1) Prouver que 𝐹 vérifie sur ℝ une équation différentielle de la forme 𝐹 ′ + 𝐴 𝐹 = 0, où 𝐴 est une fonction
à préciser.
II.C.2) En déduire une expression de 𝐹 (𝑥).
1 i
On pourra commencer par dériver la fonction 𝑥 ↦ − ln(1 + 𝑥2 ) + arctan 𝑥.
8 4

III Autour de la loi de Poisson


Dans cette partie, 𝜆 désigne un réel strictement positif.
On rappelle qu’une variable aléatoire 𝑋, à valeurs dans ℕ, suit la loi de Poisson 𝒫(𝜆) de paramètre 𝜆 si, pour
tout 𝑛 ∈ ℕ :
u�
P(𝑋 = 𝑛) = 𝜆 e−u�
𝑛!
Pour tout sous-ensemble 𝐴 de ℝ, P(𝑋 ∈ 𝐴) désigne la probabilité de l’événement 𝑋 −1 (𝐴).

On note 𝐺u� (𝑡) = E(𝑡u� ) = ∑ P(𝑋 = 𝑘)𝑡u� (série génératrice de la variable aléatoire 𝑋).
u�=0

III.A – Soit 𝑋 une variable aléatoire qui suit la loi de Poisson 𝒫(𝜆).
III.A.1) Déterminer 𝐺u� (𝑡).
III.A.2) Calculer l’espérance E(𝑋), la variance 𝑉 (𝑋) et l’écart type de 𝑋.
III.A.3) Soit 𝜇 un réel strictement positif. Soit 𝑌 une variable aléatoire suivant la loi de Poisson 𝒫(𝜇) et telle
que 𝑋 et 𝑌 soient indépendantes. Déterminer la loi de 𝑋 + 𝑌 .

III.B – Soit (𝑋u� )u�⩾1 une suite de variables aléatoires mutuellement indépendantes, de loi 𝒫(𝜆). On rappelle
que, quels que soient les entiers 1 ⩽ 𝑖1 < 𝑖2 < ⋯ < 𝑖u� et les intervalles 𝐼1 , 𝐼2 ,…𝐼u� de ℝ
u�=u�
P(𝑋u�1 ∈ 𝐼1 , 𝑋u�2 ∈ 𝐼2 , …, 𝑋u�u� ∈ 𝐼u� ) = ∏ P(𝑋u�u� ∈ 𝐼u� )
u�=1

III.B.1) Pour tout entier 𝑛 ⩾ 1, déterminer la loi de 𝑆u� = 𝑋1 + 𝑋2 + ⋯ + 𝑋u� .


𝑆u� − 𝑛𝜆
III.B.2) Déterminer l’espérance et l’écart type des variables aléatoires 𝑆u� et 𝑇u� = √ .
𝑛𝜆
III.B.3) Montrer que, pour tout 𝜀 > 0, il existe un réel 𝑐(𝜀) tel que, si 𝑐 ⩾ 𝑐(𝜀) et 𝑛 ∈ ℕ∗ , on a P(|𝑇u� | ⩾ 𝑐) ⩽ 𝜀.

III.C – Dans cette sous-partie, on fixe deux réels 𝑎 et 𝑏 tels que 𝑎 < 𝑏.

Pour tout entier 𝑛 ⩾ 1 tel que 𝑎 + 𝑛𝜆 > 0, on pose
√ √
𝐼u� = {𝑘 ∈ ℕ| 𝑛𝜆 + 𝑎 𝑛𝜆 ⩽ 𝑘 ⩽ 𝑛𝜆 + 𝑏 𝑛𝜆}
𝑘 − 𝑛𝜆
Pour 𝑘 ∈ ℤ, on pose 𝑥u�,u� = √ .
𝑛𝜆
On considère enfin la fonction 𝑓 : ℝ → ℝ définie par 𝑓(𝑥) = e− 2 u� pour tout 𝑥 ∈ ℝ.
1 2

III.C.1) Montrer qu’il existe un réel 𝑀 > 0 tel que 𝑓 soit une fonction 𝑀 -lipschitzienne.
III.C.2)
u�+ℎ
ℎ2
a) Montrer que, si 𝑥, ℎ ∈ ℝ et ℎ > 0, alors |ℎ𝑓(𝑥) − ∫ 𝑓(𝑡) d𝑡| ⩽ 𝑀 .
2
u�

2016-02-10 19:46:40 Page 2/3


b) En déduire, lorsque 𝐼u� est non vide, une majoration de
u�u�+1,u�

∣ √1 ∑ 𝑓(𝑥u�,u� ) − ∫ 𝑓(𝑡) d𝑡∣


𝑛𝜆 u�∈u�u�
u�u�,u�

où 𝑝 est le plus petit élément de 𝐼u� et 𝑞 est le plus grand.


c) Montrer que
u�

lim √1 ∑ 𝑓(𝑥u�,u� ) = ∫ 𝑓(𝑥) d𝑥


u�→+∞ 𝑛𝜆 u�∈u�u�
u�

𝑥u�,u� √ √ u�
III.C.3) Pour tout 𝑘 ∈ 𝐼u� , on note 𝑦u�,u� = (1 − 𝑛𝜆) exp(𝑥u�,u� 𝑛𝜆).
𝑘
Soit 𝜀 > 0. Démontrer l’existence d’un entier 𝑁 (𝜀) tel que, pour tout 𝑛 ⩾ 𝑁 (𝜀) et tout 𝑘 ∈ 𝐼u� , les inégalités
suivantes soient satisfaites :
1−𝜀 1 (𝑛𝜆)u� 1+𝜀 1
a) √ √ 𝑦u�,u� ⩽ e−u�u� ⩽ √ √ 𝑦u�,u� ;
2𝜋 𝑛𝜆 𝑘! 2𝜋 𝑛𝜆
√ 𝑛 u�
On utilisera la formule de Stirling 𝑛! ∼ 2𝜋𝑛 ( ) .
u�→+∞ e
b) (1 − 𝜀)𝑓(𝑥u�,u� ) ⩽ 𝑦u�,u� ⩽ (1 + 𝜀)𝑓(𝑥u�,u� ).
(𝑛𝜆)u� −u�
III.C.4) Exprimer, sous forme d’intégrale, lim ∑ e .
u�→+∞
u�∈u�u�
𝑘!
III.C.5) Comparer P(𝑎 ⩽ 𝑇u� ⩽ 𝑏) et ∑ P(𝑆u� = 𝑘), où 𝑆u� et 𝑇u� sont définies en III.B.
u�∈u�u�

III.C.6) Déterminer les limites, quand 𝑛 → +∞, de

P(𝑇u� ⩾ 𝑎), P(𝑇u� = 𝑎), P(𝑇u� > 𝑎) et P(𝑇u� ⩽ 𝑏)

III.D –
+∞

III.D.1) Déduire de la question III.C.6) la valeur de ∫ 𝑓(𝑥) d𝑥.


−∞
III.D.2) Déterminer un équivalent, lorsque 𝑛 → +∞, de
⌊u�u�⌋ +∞
(𝑛𝜆)u� (𝑛𝜆)u�
𝐴u� = ∑ et 𝐵u� = ∑
u�=0
𝑘! 𝑘!
⌊u�u�⌋

où ⌊𝑡⌋ désigne la partie entière du réel 𝑡.


On interprétera e−u�u� 𝐴u� comme la probabilité d’un événement lié à 𝑆u� et donc à 𝑇u� .
u� +∞
(𝑛𝜆)u� (𝑛𝜆)u�
III.D.3) Pour 𝜆 ≠ 1, on note 𝐶u� = ∑ et 𝐷u� = ∑ .
u�=0
𝑘! u�=u�+1
𝑘!
Déterminer lim e−u�u� 𝐶u� si 𝜆 < 1 et lim e−u�u� 𝐷u� si 𝜆 > 1.
u�→+∞ u�→+∞

III.E – On suppose 𝜆 < 1.


u�u�

III.E.1) Déterminer lim ⎛ ⎜(𝑛𝜆)−u� ∫ (𝑛𝜆 − 𝑡)u� eu� d𝑡⎞


⎟.
u�→+∞
⎝ 0 ⎠
III.E.2) En utilisant la formule de Taylor avec reste intégral, en déduire un équivalent de 𝐷u� quand 𝑛 → +∞.

III.F – Si 𝜆 > 1, déterminer un équivalent de 𝐶u� lorsque 𝑛 → +∞.


0
1
Considérer l’intégrale ∫ (𝑟 − 𝑡)u� eu� d𝑡 et choisir convenablement le réel 𝑟.
𝑛!
−∞

• • • FIN • • •

2016-02-10 19:46:40 Page 3/3


Corrigé de Centrale 2016 PC math 1

I Autour de la fonction Gamma d’Euler


Z 1
1
I.A.1) f (t) = tx−1 e−t ∼ donc f (t)dt existe si et seulement si x > 0.
t→0 t1−x 0
Z +∞
Puisque lim tx+1 e−t = 0, f (t) = o( t12 ) donc f (t)dt existe pour tout x.
x→+∞ t→+∞ 1
Le domaine de définition de Γ est donc D =]0, +∞[.
Z +∞
I.A.2) On intègre par parties pour x > 0: Γ(x + 1) = [−e−t tx ]+∞
0 +x tx−1 e−t dt = xΓ(x) puisque l’expression
0
entre crochets a pour limite 0 en 0 et en +∞.
n−1
Y
On en déduit par récurrence, pour n > 1 et x > 0: Γ(x + n) = Γ(x) (x + k).
k=0
Z +∞
Pour x = 1 on obtient avec Γ(1) = e−t dt = 1, Γ(n + 1) = n! donc Γ(n) = (n − 1)! pour n > 1.
0

I.A.3) Dans la première intégrale on pose t = u1/2 (bijection de classe C 1 de ]0, +∞[ dans lui-même):
Z +∞ Z +∞
−t2 1 1
e dt = e−u u−1/2 du = Γ(1/2) = Γ(3/2).
0 0 2 2
Dans la seconde intégrale on pose t = u1/4 (bijection de classe C 1 de ]0, +∞[ dans lui-même):
Z +∞ Z +∞
4 1 1
e−t dt = e−u u−3/4 du = Γ(1/4) = Γ(5/4).
0 0 4 4

I.B.1) Pour t > 0 fixé et x variant entre a et b, ex ln t est compris entre ea ln t et eb ln t donc tx 6 max(ta , tb ) 6 ta + tb .
∂f k
I.B.2) Pour x > 0 et t > 0 posons f (x, t) = tx−1 e−t = e(x−1) ln t−t . On calcule (x, t) = (ln t)k tx−1 e−t .
∂xk
Pour x > 0 fixé: |(ln t)k tx−1 e−t | = o( t12 ) puisque lim tx+1 (ln t)k e−t = 0.
t→+∞ t→+∞
1
D’autre part |(ln t)k tx−1 e−t | ∼ | ln t|k tx/2 1
= o( t1−x/2 ) qui est intégrable sur ]0, 1] puisque x > 0. On
t→0 t1−x/2 t→0
∂f k
en déduit que t 7→ (x, t) est intégrable sur ]0, +∞[.
∂xk
On peut maintenant appliquer le théorème de dérivation sous le signe intégral:
– Pour tout x ∈]0, +∞[, t 7→ f (x, t) est continue et intégrable sur ]0, +∞[
– Pour tout t ∈]0, +∞[, x 7→ f (x, t) est de classe C ∞ sur ]0, +∞[
∂f k
– Pour tout x ∈]0, +∞[ et pour tout k ∈ N∗ , t 7→ (x, t) est continue sur ]0, +∞[
∂xk
– Pour tout k ∈ N∗ et pour tout segment [a, b] ⊂]0, +∞[ il existe ϕ continue et intégrable sur ]0, +∞[ telle
∂f k ∂f k ∂f k
k
(x, t) 6 ϕ(t): en appliquant le I.B.1 on peut prendre ϕ(t) = k
(a, t) + (b, t) .
∂x ∂x ∂xk
Z +∞
On en conclut pour x > 0: Γ(k) (x) = (ln t)k tx−1 e−t dt.
0

I.C.1) Puisque (ln t)2 > 0 pour t 6= 1, on a Γ00 (x) > 0 et donc Γ0 est strictement croissante sur ]0, +∞[.
Avec Γ(n) = (n − 1)! pour n ∈ N∗ on déduit que Γ(1) = Γ(2) = 1. On peut appliquer le théorème de Rolle à Γ
sur [1, 2] puisqu’elle est de classe C 1 et que Γ(1) = Γ(2). On en déduit que Γ0 s’annule sur ]1, 2[, une seule fois
puisque Γ0 est strictement croissante. Il existe un unique ξ tel que Γ0 (ξ) = 0 et sa partie entière est égale à 1.
I.C.2) Pour 0 < x < ξ, Γ0 (x) < 0 donc Γ est strictement décroissante. Pour x > ξ, Γ0 (x) > 0 donc Γ est strictement
croissante.
1
De Γ(x + 1) = xΓ(x) et de Γ(1) = 1 on déduit par continuité de Γ en 1 que Γ(x) ∼ au voisinage de 0+ et par
x
suite Γ a pour limite +∞ en 0+ .
Puisque Γ est croissante pour x > 2 et que Γ(n) = (n − 1)! pour n ∈ N∗ on déduit que Γ a pour limite +∞ en
+∞.

1
De Γ(x + 1) = xΓ(x) on déduit Γ0 (x + 1) = Γ(x) + xΓ0 (x). Par continuité de Γ0 en 1 et avec l’équivalent obtenu
1
pour Γ(x) en 0+ on déduit que Γ0 (x) ∼ − 2 , donc Γ0 a pour limite −∞ en 0+ .
x→0 + x
Pour x > ξ on a Γ0 (x) > 0 et par suite Γ0 (x + 1) = Γ(x) + xΓ0 (x) > Γ(x): on en déduit que Γ0 a pour limite +∞
en +∞.
La courbe représentative de Γ a pour asymptote la droite d’équation x = 0. Quand x tend vers +∞ la croissance
vers +∞ est très rapide puisque n! croı̂t très vite vers +∞.

II Une transformée de Fourier


∂g k
II.A Pour x ∈ R et t > 0 posons g(x, t) = e−t t−3/4 eixt . On calcule (x, t) = (it)k e−t t−3/4 eixt .
∂xk
∂g k
Pour x fixé et k ∈ N, t 7→ (x, t) = e−t tk−3/4 est intégrable sur ]0, +∞[ puisque Γ(k + 1/4) existe.
∂xk
On peut appliquer le théorème de dérivation sous le signe intégral en dominant la dérivée k-ième par ϕ(t) =
Z +∞
−t k−3/4
e t ∞ (k)
. F est donc de classe C et F (x) = i k
e−t tk−3/4 eixt dt.
0
F (0) = Γ(1/4).
+∞ ∞
(ixt)n
Z X
II.B.1) En utilisant le développement en série entière de eitx on obtient: F (x) = e−t t−3/4 dt.
0 n=0
n!
P
Appliquons le théorème d’intégration terme à terme pour la série de fonction ( fn ) définie par fn (t) =
(ixt)n
e−t t−3/4 (x étant fixé):
n!
|x|n −t n−3/4
– fn est continue et intégrable sur ]0, +∞[ puisque |fn (t)| = e t et que Γ(n + 1/4) existe.
P n!
– La série ( fn ) converge pour tout t > 0.
Z +∞
– Si on choisit |x| < 1, la série de terme général un = |fn (t)|dt converge.
0
|x|n +∞ −t n−3/4 |x|n
Z
En effet, un = e t dt = Γ(n + 1/4). Pour n > 2, par croissance de la fonction Γ, on
n! 0 n!
n
|x|
obtient un 6 Γ(n + 1) = |x|n qui est le terme général d’une série géométrique convergente.
n!

X (ix)n
On obtient donc pour |x| < 1 en intégrant terme à terme: F (x) = cn avec cn = Γ(n + 1/4).
n=0
n!
n−1
Y
Avec le résultat du I.A.2) on déduit: cn = c0 (k + 1/4) avec c0 = Γ(1/4).
k=0
|x|n (ix)n |x|n
La croissance de la fonction Γ pour x > n > 2 entraı̂ne que Γ(n) 6 cn 6 Γ(n + 1) et par suite
n! n! n!
|x|n (ix)n
6 cn 6 |x|n . On en déduit que le rayon de convergence est égal à 1.
n n!
II.B.2) L’inégalité que l’on vient de montrer entraı̂ne qu’il n’y a pas convergence absolue pour |x| = 1 puisque la série
( n1 ) diverge.
P

II.B.3) Le développement en série entière de F (x) donne son développement limité en 0 à l’ordre 3:
2 3
F (x) = c0 + c1 ix + c2 ( −x −ix 3
2 ) + c3 ( 6 ) + o(x ).
On en déduit avec c1 = 41 c0 , c2 = 5
16 c0 et c3 = 45
64 c0 :
5 2
R(x) = c0 (1 − 32 x ) + o(x3 ) et I(x) = c0 ( x4 − 15 3
128 x ) + o(x4 ) (on obtient l’ordre 4 pour I(x) puisque c’est une
fonction impaire).
+∞ +∞ Z +∞
e(ix−1)t
Z 
i
II.C.1) Intégrons par parties: F 0 (x) = i t1/4 e(ix−1)t dt = it1/4 − t−3/4 e(ix−1)t dt =
0 (ix − 1) 0 4(ix − 1) 0
i
− F (x) puisque les limites en 0 et en +∞ de l’expression entre crochets sont nulles. On a donc bien
4(ix − 1)
i 1
F 0 + AF = 0 en posant A(x) = = .
4(ix − 1) 4(x + i)

2
x−i 1 i
II.C.2) On obtient A(x) = dont une primitive est G(x) = ln(1 + x2 ) − arctan x.
4(x2 + 1) 8 4
On en déduit que (F eG )0 = (F 0 + F G0 )eG = 0 d’où F (x) = Ce−G(x) avec C = F (0) = Γ(1/4).
i
On obtient donc F (x) = Γ(1/4)(1 + x2 )−1/8 e 4 arctan x .

III Autour de la loi de Poisson



X (λt)k
III.A.1) GX (t) = e−λ = eλ(t−1) .
k!
k=0

III.A.2) E(X) = G0X (1) = λ.


V(X) = G00X (1) + G0X (1) − (G0X (1))2 = λ2 + λ − λ2 = λ.

σ(X) = λ.
III.A.3) Puisque X et Y sont indépendantes on a GX+Y (t) = GX (t)GY (t) = e(λ+µ)(t−1) donc X + Y a pour loi P(λ + µ).
III.B.1) On montre par récurrence que Sn a pour loi P(nλ).
C’est vrai pour n = 1 puisque S1 = X1 .
Supposons, pour un entier n, que Sn a pour loi P(nλ). Sn = X1 + ... + Xn et Xn+1 sont indépendantes donc le
III.A.3) montre que Sn+1 = Sn + Xn+1 a pour loi P(nλ + λ) = P((n + 1)λ).
Le résultat est donc vrai pour tout n > 1.

III.B.2) E(Sn ) = nλ et σ(Sn ) = nλ.
1
E(Tn ) = √ (nλ − nλ) = 0.

1
σ(Tn ) = √ σ(Sn − λ) = 1.

III.B.3) Puisque Tn possède une variance on peut lui appliquer l’inégalité de Bienaymé-Tchebychev:
V(Tn ) 1 1
P(|Tn − E(Tn )| > c) 6 2
donc P(|Tn | > c) 6 2 . En choisissant c > c(ε) = √ on obtient P(|Tn | > c) 6 ε.
c c ε
1 2 1 2
III.C.1) f 0 (x) = −xe− 2 x et f 00 (x) = (x2 − 1)e− 2 x . Pour x > 0, f 0 (x) 6 0 et f 0 possède un minimum égal à f 0 (1) =
−e−1/2 . Puisque f 0 est impaire on en déduit que pour tout x ∈ R on a |f 0 (x)| 6 M = e−1/2 . Cela entraı̂ne que
f est M -lipschitzienne.
Z x+h
III.C.2) a) Pour x fixé posons g(h) = hf (x) − f (t)dt. |g 0 (h)| = |f (x) − f (x + h)| 6 M h pour h > 0. On en
x
Z h Z h Z h
h2
déduit |g(h)| = |g(h) − g(0)| = g 0 (t)dt 6 |g 0 (t)|dt 6 M tdt = M .
0 0 0 2
q q
Z xq+1,n Z xk+1,n !
1 X X f (xk,n ) X 1 M (q + 1 − p)
b) √ f (xk,n ) − f (t)dt = √ − f (t)dt 6 M = en
nλ k∈In xp,n nλ xk,n 2λn 2λn
k=p k=p
1
appliquant le a) pour x = xk,n et h = √ (car xk+1,n = xk,n + h).

√ 1
c) D’une part on a p − 1 < nλ + a nλ 6 p donc a 6 xp,n < a + √ . Par suite lim xp,n = a.
nλ n→+∞
√ 1
De même, q 6 nλ + b nλ < q + 1 donc b − √ < xq,n 6 b. Par suite lim xq,n = b.
nλ n→+∞
Z xq+1,n Z b
On en déduit puisque f est continue: lim f (t)dt = f (x)dx.
n→+∞ xp,n a
√ M (q + 1 − p)
D’autre part 0 6 q − p 6 (b − a) nλ donc lim = 0. On en déduit avec le b):
n→+∞ 2λn
Z xq+1,n Z b
1 X
lim √ f (xk,n ) = lim f (t)dt = f (x)dx.
n→+∞ nλ k∈In n→+∞ x a
p,n

k



III.C.3) a) Par définition, xk,n nλ = k − nλ donc yk,n = ek−nλ .
k
√ √ √ r
2πnλ −nλ (nλ)k 2πnλk k 2πkk k nλ
On en déduit e = = .
yk,n k! ek k! ek k! k

3
a k b k
Puisque k ∈ In , on a 1 + √ 6 6 1+ √ donc a pour limite 1 quand n tend vers +∞. Cela
nλ nλ nλ nλ
entraı̂ne que k tend vers +∞ quand n tend vers +∞.

2πkk k
D’autre part l’équivalent de Stirling entraı̂ne que tend vers 1 quand k tend vers +∞. Par suite,
√ ek k!
2πnλ −nλ (nλ)k
e tend vers 1 quand n tend vers +∞. Il est donc compris entre 1 − ε et 1 + ε pour
yk,n k!
n > N1 (ε) ce qui démontre le résultat demandé.
k
b) Pour k ∈ In on a a 6 xk,n 6 b donc xk,n est borné. De plus on a montré que a pour limite 1 quand n

xk,n √ nλ 1
tend vers +∞ donc nλ = xk,n √ tend vers 0 quand n tend vers +∞.
k k nλ
xk,n √
On peut donc utiliser le développement limité ln(1 + t) = t − 12 t2 + o(t2 ) avec t = nλ. On obtient:
k
xk,n √ √ 1
ln(yk,n ) − ln f (xk,n ) = k ln(1 − nλ) + xk,n nλ + x2k,n
k 2
xk,n √ 1 xk,n √ xk,n √ √
 
2 1
= k − nλ − ( nλ) + o(( nλ)2 ) + xk,n nλ + x2k,n
k 2 k k 2
 
1 2 nλ nλ
= xk,n 1 − + o( ) .
2 k k

k
Cette expression a pour limite 0 quand n tend vers +∞ puisque xk,n est borné et a pour limite 1.

yk,n
On en déduit que a pour limite 1 et on obtient l’inégalité demandée pour n > N2 (ε).
f (xk,n )
III.C.4) On déduit de la question précédente que:
(1 − ε)2 1 X X (nλ)k (1 + ε)2 1 X
√ √ f (xk,n ) 6 e−nλ 6 √ √ f (xk,n ).
2π nλ k∈In k! 2π nλ k∈In
k∈In
X (nλ)k Z b
1
Avec le III.C.2)c) on déduit lim e−nλ = √ f (x)dx.
n→+∞ k! 2π a
k∈I n

√ √ X
III.C.5) P(a 6 Tn 6 b) = P(nλ + a nλ 6 Sn 6 nλ + b nλ) = P(Sn = k) puisque Sn ne prend que des valeurs
k∈In
entières.
X (nλ)k Z b
1
III.C.6) Puisque Sn a pour loi P(nλ), P(a 6 Tn 6 b) = e−nλ donc lim P(a 6 Tn 6 b) = √ f (x)dx.
k! n→+∞ 2π a
k∈In

Pour c > a on a P(Tn > a) = P(a 6 Tn 6 c) + P(Tn > c). Soit ε > 0. Avec la question III.B.3) on peut choisir
c1 tel que pour c > c1 on ait P(Tn > c) 6 P(|Tn | > c) 6 ε. D’autre part, puisque f (x) = o( x12 ), f est
Z +∞ Zx→+∞
c
1
intégrable sur R+ , donc on peut choisir c2 tel que pour c > c2 on ait √ f (x)dx − f (x)dx 6 ε.
2π a a
Z +∞ Z c
1 1
Pour c > max(c1 , c2 ) on a P(Tn > a) − √ f (x)dx 6 2ε + |P(a 6 Tn 6 c) − √ f (x)dx| 6 3ε pour
2π a
Z +∞ 2π a
1
n > n0 . Par suite lim P(Tn > a) = √ f (x)dx.
n→+∞ 2π a
Z a+ε
1
Pour tout ε > 0, P(Tn = a) 6 P(a 6 Tn 6 a + ε) qui tend vers √ f (x)dx quand n tend vers +∞.
2π a
Comme cette intégrale peut être arbitrairement proche de 0, on en déduit que lim P(Tn = a) = 0.
n→+∞
Z +∞
1
lim P(Tn > a) = lim P(Tn > a) = √ f (x)dx.
n→+∞ n→+∞ 2π a
Z +∞
1
lim P(Tn 6 b) = 1 − lim P(Tn > b) = 1 − √ f (x)dx.
n→+∞ n→+∞ 2π b
III.D.1) Avec le III.B.3) on a pour b 6 −c(ε): P(Tn 6 b) 6 P(Tn > |b|) 6 ε. On en déduit avec lim P(Tn 6 b) =
n→+∞
1
Z +∞ Z +∞ √
1− √ f (x)dx que f (x)dx = 2π.
2π b −∞

4
bnλc
X
III.D.2) e−nλ An = P(Sn = k) = P(Sn 6 nλ) puisque Sn ne prend que des valeurs entières.
k=0
Z +∞
−nλ 1 1
On a donc e An = P(Tn 6 0) qui tend vers 1 − √ f (x)dx = par parité de la fonction f .
2π 0 2
1
On a donc An ∼ enλ .
2
(nλ)k
e−nλ (An + Bn ) = 1 + e−nλ avec k = bnλc. Comme k 6 nλ < k + 1 on a:
k!
(nλ)k (k + 1)k 1
e−nλ 6 e−k ∼ (1 + k1 )k √ qui tend vers 0 puisque k tend vers +∞ quand n tend vers +∞ et
k! k! 2πk
1
(1 + k1 )k a pour limite e. Par suite Bn ∼ enλ .
2
n
X 1 − λ√
III.D.3) e−nλ Cn = P(Sn = k) = P(Sn 6 n) = P(Tn 6 √ n).
k=0
λ
Z b
1 1 − λ√
Pour tout ε > 0 il existe b tel que √ f (x)dx > 1 − ε. Puisque 1 − λ > 0, on a √ n > b pour n > n1 .
2π −∞ λ
Z b
1
On a alors e−nλ Cn > P(Tn 6 b) qui tend vers √ f (x)dx > 1 − ε. On en déduit, puisque que e−nλ Cn 6 1
2π −∞
(c’est une probabilité), que e−nλ Cn a pour limite 1 si λ < 1.
+∞
X 1 − λ√
e−nλ Dn = P(Sn = k) = P(Sn > n) = P(Tn > √ n). Pour tout ε > 0 il existe a < 0 tel que
k=n+1
λ
Z ∞
1 1 − λ√
√ f (x)dx > 1 − ε. Puisque 1 − λ < 0, on a √ n 6 a pour n > n2 . On a alors e−nλ Dn > P(Tn > a)
2π a Z ∞ λ
1
qui tend vers √ f (x)dx > 1 − ε. On en déduit, puisque que e−nλ Dn 6 1 (c’est une probabilité), que
2π a
e−nλ Dn a pour limite 1 si λ > 1.
Z nλ Z nλ  n
−n n t t
III.E.1) (nλ) (nλ − t) e dt = 1− et dt.
0 0 nλ
 n
t
Définissons fn (t) = 1 − et si t < nλ et fn (t) = 0 si t > nλ et utilisons le théorème de convergence

Z +∞
dominée pour calculer la limite de fn (t)dt.
0
Chaque fonction fn est continue sur R+ (la limite à gauche en t = nλ de fn (t) est égale à 0).
t 1
Pour n > t on a fn (t) = et+n ln(1− nλ ) qui a pour limite f (t) = et(1− λ ) quand n tend vers +∞, puisque
t
n ln(1 − nλ ) ∼ − λt . La suite (fn ) converge donc simplement vers la fonction f qui est continue sur R+ .
1
La majoration connue ln(1 + x) 6 x entraı̂ne pour t < nλ que fn (t) 6 et(1− λ ) = f (t). C’est aussi vérifié pour
t > nλ puisque fn (t) = 0. La fonction f est intégrable sur [0, +∞[ puisque 1 − λ1 < 0.
Le théorème de convergence dominée s’applique et donc:
Z nλ ! Z " 1
#+∞
+∞ t(1− λ )
e λ
lim (nλ)−n (nλ − t)n et dt = f (t)dt = 1 = .
n→+∞ 0 0 (1 − λ ) 1 − λ
0

III.E.2) Appliquons la formule de Taylor avec reste intégral à l’ordre n pour la fonction exp sur l’intervalle [0, nλ]:
n Z nλ
X (nλ)k (nλ − t)n t
enλ = + e dt. On en déduit avec le résultat du III.E.1):
k! 0 n!
k=0
n Z nλ
X (nλ)k (nλ − t)n t λ (nλ)n
Dn = enλ − = e dt ∼ quand λ < 1.
k! 0 n! 1 − λ n!
k=0

0 0 Z 0
(r − t)n t (r − t)n t (r − t)n−1 t
Z 
III.F Intégrons par parties: e dt = e + e dt. C’est légitime: les intégrales
−∞ n! n! −∞ −∞ (n − 1)!
existent car (r − t)n et = o( t12 ) en −∞ et l’expression entre crochets a une limite en 0 et en −∞. On obtient
Z 0 Z 0
(r − t)n t rn (r − t)n−1 t
e dt = + e dt. On continue à intégrer par parties et on montre par récurrence
−∞ n! n! −∞ (n − 1)!
sur k que:

5
0 Z 0
(r − t)n t rn rn−k+1 (r − t)n−k t
Z
e dt = + ... + + e dt.
−∞ n! n! (n − k + 1)! −∞ (n − k)!
Z 0 Z 0
(r − t)n t rn r1
On obtient finalement pour k = n: e dt = + ... + + et dt qui est égal à Cn si on choisit
−∞ n! n! 1! −∞
r = nλ.
n
(nλ)n 0
Z 
t
On a donc Cn = 1− et dt.
n! −∞ nλ
Z 0
Appliquons à nouveau le théorème de convergence dominée pour calculer la limite de gn (t)dt avec gn (t) =
 n −∞
t
1− et :

Chaque fonction gn est continue sur R− .
t 1
Puisque t 6 0 on peut écrire gn (t) = et+n ln(1− nλ ) qui a pour limite f (t) = et(1− λ ) quand n tend vers +∞
(comme à la question III.E.1) avec f qui est continue sur R− .
1
La majoration connue ln(1+x) 6 x entraı̂ne que gn (t) 6 et(1− λ ) = f (t). La fonction f est intégrable sur ]−∞, 0]
puisque 1 − λ1 > 0.
Le théorème de convergence dominée s’applique et donc:
Z 0 Z 0 " 1
#0
et(1− λ ) λ
lim gn (t)dt = f (t)dt = 1 = .
n→+∞ −∞ −∞ (1 − λ ) λ−1
−∞
n
λ (nλ)
On en déduit que Cn ∼ quand λ > 1.
λ − 1 n!

6
Mines-Ponts MP Mathématiques 1 2015

Opérateur de Volterra et équations diérentielles


L'objectif de ce problème est l'étude d'un opérateur de Volterra appliqué notamment à la résolution de certaines
équations diérentielles.
π
On considère l'espace vectoriel E des fonctions réelles dénies et continues sur l'intervalle [0, ], muni du produit
2
scalaire déni pour tous f, g dans E par : Z π
2
hf, gi = f (t)g(t)dt.
0

On note kf k = hf, f i la norme associée à ce produit scalaire. Un endomorphisme V de E est dit symétrique
p

déni positif si pour tous f, g dans E, on a : hV(f ), gi = hf, V(gi et si de plus hV(f ), f i > 0 pour tout f ∈ E non
nul.

Les parties A et B sont mutuellement indépendantes.


A. Opérateur de Volterra
On note V et V∗ les endomorphismes de E dénis par les formules :
Z x
V(f )(x) = f (t)dt
0
Z π/2

V (f )(x) = f (t)dt
x
π
pour tous f ∈ E et x ∈ [0, ].
2
1) En observant que V(f ) et −V∗ (f ) sont des primitives de f , montrer que pour tous f, g dans E, on a :
hV(f ), gi = hf, V∗ (g)i.
2) Montrer que l'endomorphisme V∗ ◦ V est symétrique déni positif. En déduire que ses valeurs propres sont
strictement positives.
Soit λ une valeur propre de V∗ ◦ V et fλ un vecteur propre associé à λ.
1
3) Montrer que fλ est de classe C 2 et est solution de l'équation diérentielle : y 00 + y = 0 avec les conditions
λ
y( π2 ) = 0 et y 0 (0) = 0.
4) En déduire que λ est une valeur propre de V∗ ◦ V si et seulement s'il existe n ∈ N tel que :
1
λ= . Préciser alors les vecteurs propres associés.
(2n + 1)2

B. Théorème d'approximation de Weierstrass


Soit n un entier strictement positif, x ∈ [0, 1] et f : [0, 1] → R une fonction continue. On note X1 , X2 , . . . , Xn des
variables aléatoires mutuellement indépendantes et distribuées selon la même loi de Bernoulli de paramètre x. On
S
note également : Sn = X1 + X2 + · · · + Xn ; Zn = n et Bn (f )(x) = E(f (Zn )).
n
5) Rappeler sans démonstration, la loi de Sn . En déduire, avec démonstration, les valeurs de l'éspèrance et de la
variance de Sn en fonction de n et de x.
6) En utilisant l'inégalité de Bienaymé-Tchebychev, montrer que pour tout α > 0 :

1/3
Mines-Ponts MP Mathématiques 1 2015

 
X n k 1
x (1 − x)n−k 6
k 4nα2
06k6n
k
|n −x|>α

7) Montrer que
n    
X n k n−k k
Bn (f )(x) − f (x) = x (1 − x) f ( ) − f (x)
k n
k=0
et en déduire que la suite (Bn (f ))n∈N converge uniformément vers f sur [0, 1]. On pourra utiliser le résultat
de la question précèdente ainsi que le théorème de Heine.
On a donc établi le théorème de Weierstrass sur le segment [0, 1] : toute fonction continue sur [0, 1] y est limite
uniforme d'une suite de polynômes. On en déduit aisément, et on l'admet, le théorème d'approximation de Weierstrass
sur un segment quelconque [a, b].

C. Développement de V∗ ◦ V(f ) en série trigonométrique


On considère maintenant l'espace vectoriel G des fonctions réelles dénies et continues sur l'intervalle [0, π], muni
du produit scalaire déni pour tous f, g ∈ G par :
Z π
hf, giG = f (t)g(t)dt.
0

On note : kf kG = hf, f iG la norme associée à ce produit scalaire.


p

Pour n ∈ N, on dénit la fonction cn ∈ G par la formule cn (t) = cos(nt) et on note Fn = vect(c0 , . . . , cn ) le sous
espace vectoriel de G engendré par (c0 , . . . , cn ). On note également PFn la projection orthogonale de G sur Fn .
8) Montrer que si p est un polynôme de degré n ∈ N, la fonction t 7−→ p(cos(t)) dénie sur [0, π] appartient à
Fn .
9) Trouver une suite (αn )n∈N de nombres réels strictement positifs telle que la suite (αn cn )n∈N soit orthonormée.
Déduire du théorème d'approximation de Weierstrass que la suite orthonormée (αn cn )n∈N est totale.
10) Soit f ∈ G, montrer que kf − PFn (f )kG tend vers 0 lorsque n tend vers l'inni. Si de plus la suite (PFn (f ))n∈N
converge uniformément sur [0, π] vers une fonction g , montrer que g = f .
π
Pour tout x ∈ [0, ], on dénit la fonction gx sur [0, π] par la formule :
2
 π π
 − max(x, t) si 0 6 t 6
gx (t) = 2 π 2
 −gx (π − t) si 6 t 6 π
2
11) Soit n ∈ N. Déterminer les coordonnées de PFn (gx ) sur la base (c0 , c1 , . . . , cn ) de Fn . En déduire que pour
tout t ∈ [0, π/2] :
+∞
π 4 X cos((2n + 1)x)
− max(x, t) = cos((2n + 1)t).
2 π (2n + 1)2
n=0
π
12) Montrer que pour tous f ∈ E et x ∈ [0, ] :
2
Z π π
2

V∗ ◦ V(f )(x) = − max(x, t) f (t)dt
0 2
et en déduire la suite des coecients (an (f ))n∈N pour laquelle on a :
+∞
X

V ◦ V(f )(x) = an (f ) cos((2n + 1)x).
n=0

2/3
Mines-Ponts MP Mathématiques 1 2015

D. Équations diérentielles du type Sturm-Liouville


Soit h ∈ E, λ ∈ R et l'équation diérentielle :
y 00 + λy + h = 0

(S)
y( π2 ) = 0 et y 0 (0) = 0

2
On dénit ϕn ∈ E pour tout n ∈ N par la formule : ϕn (t) = √ cos((2n + 1)t).
π
1
13) Montrer que pour tout f ∈ E et n ∈ N, hV∗ ◦ V(f ), ϕn i = hf, ϕn i.
(2n + 1)2
14) Montrer que g est solution de l'équation diérentielle (S) si et seulement si
g = λ · V∗ ◦ V(g) + V∗ ◦ V(h) et que dans ce cas, on a les formules suivantes pour tout n ∈ N :
 
λ 1
1− hg, ϕn i = hh, ϕn i
(2n + 1)2 (2n + 1)2
et
+∞
X
g= hg, ϕn iϕn
n=0

15) On suppose dans cette question que λ n'est pas égale au carré d'un entier impair. Montrer que la série :
X 1
hh, ϕn iϕn
(2n + 1)2 −λ

est normalement convergente. Exhiber alors une solution de S.


On suppose maintenant qu'il existe p ∈ N tel que : λ = (2p + 1)2 .
16) Montrer que si hh, ϕp i = 0 alors (S) a une innité de solutions, puis exhiber l'une d'entre elles. Que peut-on
dire si hh, ϕp i =
6 0?

Fin du problème

3/3
Mines-Ponts MP Mathématiques 1 2015

Opérateur de Volterra et équations diérentielles


L'objectif de ce problème est l'étude d'un opérateur de Volterra appliqué notamment à la résolution de certaines
équations diérentielles.
π
On considère l'espace vectoriel E des fonctions réelles dénies et continues sur l'intervalle [0, ], muni du produit
2
scalaire déni pour tous f, g dans E par : Z π
2
hf, gi = f (t)g(t)dt.
0

On note kf k = hf, f i la norme associée à ce produit scalaire. Un endomorphisme V de E est dit symétrique
p

déni positif si pour tous f, g dans E, on a : hV(f ), gi = hf, V(gi et si de plus hV(f ), f i > 0 pour tout f ∈ E non
nul.

Les parties A et B sont mutuellement indépendantes.


A. Opérateur de Volterra
On note V et V∗ les endomorphismes de E dénis par les formules :
Z x
V(f )(x) = f (t)dt
0
Z π/2

V (f )(x) = f (t)dt
x
π
pour tous f ∈ E et x ∈ [0, ].
2
1) En observant que V(f ) et −V∗ (f ) sont des primitives de f , montrer que pour tous f, g dans E, on a :
hV(f ), gi = hf, V∗ (g)i.
2) Montrer que l'endomorphisme V∗ ◦ V est symétrique déni positif. En déduire que ses valeurs propres sont
strictement positives.
Soit λ une valeur propre de V∗ ◦ V et fλ un vecteur propre associé à λ.
1
3) Montrer que fλ est de classe C 2 et est solution de l'équation diérentielle : y 00 + y = 0 avec les conditions
λ
y( π2 ) = 0 et y 0 (0) = 0.
4) En déduire que λ est une valeur propre de V∗ ◦ V si et seulement s'il existe n ∈ N tel que :
1
λ= . Préciser alors les vecteurs propres associés.
(2n + 1)2

B. Théorème d'approximation de Weierstrass


Soit n un entier strictement positif, x ∈ [0, 1] et f : [0, 1] → R une fonction continue. On note X1 , X2 , . . . , Xn des
variables aléatoires mutuellement indépendantes et distribuées selon la même loi de Bernoulli de paramètre x. On
S
note également : Sn = X1 + X2 + · · · + Xn ; Zn = n et Bn (f )(x) = E(f (Zn )).
n
5) Rappeler sans démonstration, la loi de Sn . En déduire, avec démonstration, les valeurs de l'éspèrance et de la
variance de Sn en fonction de n et de x.
6) En utilisant l'inégalité de Bienaymé-Tchebychev, montrer que pour tout α > 0 :

1/3
Mines-Ponts MP Mathématiques 1 2015

 
X n k 1
x (1 − x)n−k 6
k 4nα2
06k6n
k
|n −x|>α

7) Montrer que
n    
X n k n−k k
Bn (f )(x) − f (x) = x (1 − x) f ( ) − f (x)
k n
k=0
et en déduire que la suite (Bn (f ))n∈N converge uniformément vers f sur [0, 1]. On pourra utiliser le résultat
de la question précèdente ainsi que le théorème de Heine.
On a donc établi le théorème de Weierstrass sur le segment [0, 1] : toute fonction continue sur [0, 1] y est limite
uniforme d'une suite de polynômes. On en déduit aisément, et on l'admet, le théorème d'approximation de Weierstrass
sur un segment quelconque [a, b].

C. Développement de V∗ ◦ V(f ) en série trigonométrique


On considère maintenant l'espace vectoriel G des fonctions réelles dénies et continues sur l'intervalle [0, π], muni
du produit scalaire déni pour tous f, g ∈ G par :
Z π
hf, giG = f (t)g(t)dt.
0

On note : kf kG = hf, f iG la norme associée à ce produit scalaire.


p

Pour n ∈ N, on dénit la fonction cn ∈ G par la formule cn (t) = cos(nt) et on note Fn = vect(c0 , . . . , cn ) le sous
espace vectoriel de G engendré par (c0 , . . . , cn ). On note également PFn la projection orthogonale de G sur Fn .
8) Montrer que si p est un polynôme de degré n ∈ N, la fonction t 7−→ p(cos(t)) dénie sur [0, π] appartient à
Fn .
9) Trouver une suite (αn )n∈N de nombres réels strictement positifs telle que la suite (αn cn )n∈N soit orthonormée.
Déduire du théorème d'approximation de Weierstrass que la suite orthonormée (αn cn )n∈N est totale.
10) Soit f ∈ G, montrer que kf − PFn (f )kG tend vers 0 lorsque n tend vers l'inni. Si de plus la suite (PFn (f ))n∈N
converge uniformément sur [0, π] vers une fonction g , montrer que g = f .
π
Pour tout x ∈ [0, ], on dénit la fonction gx sur [0, π] par la formule :
2
 π π
 − max(x, t) si 0 6 t 6
gx (t) = 2 π 2
 −gx (π − t) si 6 t 6 π
2
11) Soit n ∈ N. Déterminer les coordonnées de PFn (gx ) sur la base (c0 , c1 , . . . , cn ) de Fn . En déduire que pour
tout t ∈ [0, π/2] :
+∞
π 4 X cos((2n + 1)x)
− max(x, t) = cos((2n + 1)t).
2 π (2n + 1)2
n=0
π
12) Montrer que pour tous f ∈ E et x ∈ [0, ] :
2
Z π π
2

V∗ ◦ V(f )(x) = − max(x, t) f (t)dt
0 2
et en déduire la suite des coecients (an (f ))n∈N pour laquelle on a :
+∞
X

V ◦ V(f )(x) = an (f ) cos((2n + 1)x).
n=0

2/3
Mines-Ponts MP Mathématiques 1 2015

D. Équations diérentielles du type Sturm-Liouville


Soit h ∈ E, λ ∈ R et l'équation diérentielle :
y 00 + λy + h = 0

(S)
y( π2 ) = 0 et y 0 (0) = 0

2
On dénit ϕn ∈ E pour tout n ∈ N par la formule : ϕn (t) = √ cos((2n + 1)t).
π
1
13) Montrer que pour tout f ∈ E et n ∈ N, hV∗ ◦ V(f ), ϕn i = hf, ϕn i.
(2n + 1)2
14) Montrer que g est solution de l'équation diérentielle (S) si et seulement si
g = λ · V∗ ◦ V(g) + V∗ ◦ V(h) et que dans ce cas, on a les formules suivantes pour tout n ∈ N :
 
λ 1
1− hg, ϕn i = hh, ϕn i
(2n + 1)2 (2n + 1)2
et
+∞
X
g= hg, ϕn iϕn
n=0

15) On suppose dans cette question que λ n'est pas égale au carré d'un entier impair. Montrer que la série :
X 1
hh, ϕn iϕn
(2n + 1)2 −λ

est normalement convergente. Exhiber alors une solution de S.


On suppose maintenant qu'il existe p ∈ N tel que : λ = (2p + 1)2 .
16) Montrer que si hh, ϕp i = 0 alors (S) a une innité de solutions, puis exhiber l'une d'entre elles. Que peut-on
dire si hh, ϕp i =
6 0?

Fin du problème

3/3
Mines-Ponts MP Un corrigé de Mathématiques 1 2015

Opérateur de Volterra et équations diérentielles

A. Opérateur de Volterra
1) Soit f et g dans E.
D'après le théorème fondamentale de l'analyse, V(f ) et −V (f ) sont des primitives f sur [0, π/2] car f y est ∗

continue
De plus on a V(f ) + V (f ) : x 7→ π/2, d'après la relation de Chasles et V(f )(0) = 0 = V (f )(π/2)
∗ ∗

On utilise le théorème d'intégration par parties avec les fonctions de classe C : V(f ) et −V (g). 1 ∗

On a : hV(f ), gi = V(f )(x)g(x)dx = V(f )(x)(−V (g)(x)) + f (x)V (g)(x)dx


Z π/2 Z π/2 π/2
∗ ∗
0 0
On a bien hV(f ), gi = hf, V (g)i
0

2) V ◦ V est bien un endomorphisme par composition.


Soit f et g dans E. On utilise deux fois la question précédente.


On a hV ◦ V(f ), gi = hV(f ), V(g)i = hf, V ◦ V(g)i
∗ ∗

On va montrer que : hV ◦ V(f ), f i = 0 =⇒ f = 0 ∗

On suppose que : hV ◦ V(f ), f i = 0 donc 0 = hV(f ), V(f )i = kV(f )k


E
∗ 2

donc V(f ) = 0 et en dérivant f = V(f ) = 0 0

ainsi l'endomorphisme V ◦ V est symétrique déni positif


E E

Soit λ une valeur propre de V ◦ V et f un vecteur propre associé à λ ∗

On a hV ◦ V(f ), f i = kV(f )k
λ
∗ 2

or V(f ) 6= 0 car f 6= 0 et que V(f ) = f


λ λ λ
0

donc hV ◦ V(f ), f i > 0


λ E λ E λ λ

et on a hV ◦ V(f ), f i = hλf , f i = λkf k


λ λ
∗ 2

Comme kf k > 0, on en déduit que les valeurs propres de V ◦ V sont strictement positives
λ λ λ λ λ
2 ∗
λ

3) On a f = V ◦ V(f ) = V (V(f ))
λ
1
λ

λ
1
λ

λ

À l'aide des observations du 1), on a f = V(f ) et f = f ce qui prouve que f est de classe C
0 1
λ
00 1
λ λ λ
2

De plus f (π/2) = V (V(f ))(π/2) = 0 et f (0) = V(f )(0)


λ λ λ
1 ∗ 0 1
λ λ λ λ λ λ

donc f est solution de l'équation diérentielle : y + λ1 y = 0 avec les conditions : y(π/2) = 0 et y (0) = 0
λ
00 0

4) =⇒ : Soit λ est une valeur propre de V ◦ V et f un vecteur propre associé à λ. ∗

Par résolution de l'équationx diérentiellex car λ > 0, on peut trouver A et B ∈ R tels que
λ

f : x 7→ A cos( √ ) + B sin( √ ) et on a f : x 7→ − √ sin( √ ) + √ cos( √ )


A x B x 0
λ λ
λ  λ λ λ λ λ
on a 0 = f (π/2) = A cos 2√λ + B sin 2√λ et 0 = f (0) =

π π 0 B
λ √
λ

donc B = 0 donc on a f : x 7→ A cos √xλ


 
λ

Ainsi f ∈ vect x 7→ A cos √λ


  
x
λ

et A cos 2√λ = f (π/2) = 0


 
π
λ

Comme f 6= 0 , on a A 6= 0 donc cos 2√λ = 0


 
π
λ E

1/11
Mines-Ponts MP Un corrigé de Mathématiques 1 2015

ce qui nous fournit k ∈ Z tel que 2√π λ = π2 + kπ = (2k +2 1)π


donc 2√λ = (2k 2+ 1)
donc il existe n ∈ N tel que : λ = (2n +1 1) en prenant n ∈ N tel que 2n + 1 = |2k + 1|
2

=⇒ : Soit λ ∈ R tel qu'il existe n ∈ N tel que : λ = et f : x 7→ cos( ) = cos((2n + 1)x)


1
√x
2 λ
(2n + 1)
Soit x ∈ [0, ]. Z π
2

On a V(f )(x) = cos((2n + 1)t)dt =  sin((2n


x
+ 1)t)  sin((2n + 1)x) x
= 0
0 2n + 1 2n + 1

et (V ◦ V) (f )(x) = sin((2n
Z π/2
∗ + 1)t)  cos((2n + 1)t)  cos((2n + 1)x) cos((2n + 1)π/2)
π/2
dt = − = − 2 x 2 2
2n + 1 (2n + 1) (2n + 1) (2n + 1)
Ainsi et (V ◦ V) (f )(x) = λf (x)
x

donc (V ◦ V) (f ) = λf et f 6= 0

Conclusion : On en déduit que


E

λ est une valeur propre de V ◦ V si et seulement s'il existe n ∈ N tel que : λ =


∗ 1
2
(2n + 1)

Dans ce cas
 
E1/(2n+1)2 (V∗ ◦ V) = vect x 7−→ cos((2n + 1)x)

B. Théorème d'approximation de Weierstrass


suit la loi binomiale de paramètres et de loi donnée par
 
n k
5) Sn n x ∀k ∈ [[0, n]], P(Sn = k) = x (1 − x)n−k
k

On utilise pour tout ,


0 < k 6 n k nk = k
 n!
(n − k)!k!
=n
(n − 1)!
(n − 1 − (k − 1))!(k − 1)!
= n n−1
k−1


On a
n n   n  
X X n k n−k
X n−1 k
E(Sn ) = kP(Sn = k) = 0 + k x (1 − x) =n x (1 − x)n−k
k k−1
k=0 k=1 k=1

ainsi par changement d'indice :


n−1
X n − 1
E(Sn ) = nx xj (1 − x)n−1−j j =k−1
j
j=0

et si n > 2, on a par la formule du transfert


n
X
E(S2n − Sn ) = (k 2 − k)P(Sn = k)
k=0

donc
n   n  
X n k n−k
X n−2 k
E(S2n − Sn ) = k(k − 1) x (1 − x) = n(n − 1) x (1 − x)n−k
k k−2
k=2 k=2

donc valable même si


n−2
X n
E(S2n − Sn ) = n(n − 1)x2 xj (1 − x)n−2−j = n(n − 1)x2 n=1
j
donc E(S ) = E(S − S ) + E(S ) = n(n − 1)x + nx
j=0
2 2 2

donc V(S ) = E(S ) − E(S ) = n x − nx + nx − (nx)


n n n n
2 2 2 2 2 2
n n
d'où E(S ) = nx et V(S ) = nx(1 − x) (comme prévu)
n
n n

6) D'après Irénée-Jules Bienaymé et Pafnouti Tchebychev, on a

V(Sn )
P (|Sn − E(Sn )| > nα) 6
(nα)2

2/11
Mines-Ponts MP Un corrigé de Mathématiques 1 2015

or V(S
(nα)
)
=
nnx(1 − x)
2 n α
=
x(1 − x)
2 2nα 2

Une étude classique de variations nous donne ∀x ∈ [0, 1], 0 6 x(1 − x) 6 1/4.
donc P (|S − E(S )| > nα) 6 4nα1
n n 2

or (|S − E(S )| > nα) = (|S − nx| > nα) = − x > α = [ (S = k)


n n n
Sn
n n
06k6n
k
|n −x|>α

donc P (|S n − E(Sn )| > nα) =


X
P(Sn = k) par réunion disjointe
06k6n
k
|n −x|>α

donc on a bien
 
X n k 1
x (1 − x)n−k 6
k 4nα2
06k6n
k
|n −x|>α

On a B (f )(x) = E(f (Z )) = E(f (S /n)) = X f ( nk )P(S selon la formule du transfert


n
7) n n n n = k)
k=0

donc
 
n k k
Bn (f )(x) = x (1 − x)n−k f ( )
k n

et
n
X n  
xk (1 − x)n−k f (x) = (x + 1 − x)n f (x) = f (x)
k
k=0

En faisant la soustraction :
n    
X n k n−k k
Bn (f )(x) − f (x) = x (1 − x) f ( ) − f (x)
k n
Soit ε > 0.
k=0

La fonction f est continue sur le segment [0, 1] donc y est bornée uniformément continue d'après le théorème
de Heine. On note kf k la norme innie de f sur [0, 1].
L'uniforme continuité de f nous fournit α > 0 tel que : ∀y, z ∈ [0, 1], |y − z| < α =⇒ |f (x) − f (y)| 6 donc

ε
2

selon l'inégalité triangulaire


     
X n k n−k k X n k k
x (1 − x) f ( ) − f (x) 6 x (1 − x)n−k f ( ) − f (x)
k n k n
06k6n 06k6n
k k
|n −x|<α |n −x|<α
 
X n k ε
6 x (1 − x)n−k
k 2
06k6n
k
|n −x|<α
n  
X n k ε
6 x (1 − x)n−k
k 2
k=0

   
X n k n−k k ε
x (1 − x) f ( ) − f (x) 6
k n 2
06k6n
k
|n −x|<α

3/11
Mines-Ponts MP Un corrigé de Mathématiques 1 2015

d'autre part, on a

inégalité triangulaire
       
X n k n−k k X n k n−k k
x (1 − x) f ( ) − f (x) 6 x (1 − x) f ( ) + |f (x)|
k n k n
06k6n 06k6n
k k
|n −x|>α |n −x|>α
 
X n k
6 2kf k∞ x (1 − x)n−k
k
06k6n
k
|n −x|>α

selon 6
   
X n k n−k k kf k∞
x (1 − x) f ( ) − f (x) 6
k n 2nα2
06k6n
k
|n −x|>α

or lim kf2nαk = 0 Ceci nous fournit N ∈ N, tel que ∀n ∈ N, n > N =⇒



2
kf k∞
2nα2
6 ε
2

On vient de montrer que


n→+∞

∀ε > 0, ∃N ∈ N, ∀n ∈ N, n > N =⇒ (∀x ∈ [0, 1], |Bn (f )(x) − f (x)| 6 ε)

ou encore : kB (f ) − f k −→ 0
n ∞ n→+∞

on en déduit que la suite (B (f )) n n∈N converge uniformément vers f sur [0, 1]


C. Développement de V∗ ◦ V(f ) en série trigonométrique
Soit p un polynôme de degré n que l'on écrit p(X) = X a X et on note q de sorte que : la
n
k
8) k k : t 7→ cosk (t)

fonction t 7−→ p(cos(t)) appartient à vect(q , . . . , q ) n=0

Soit k ∈ [[0, n]]. Soit t ∈ [0, π]. Par la formule du binôme on a :


0 n

k n  
eit + e−it
 X n  
1 X n (2j−n)it 1 1 X n (2j−n)it
qk (t) = = k e = k e(2k−n)it + k e +r
2 2 j 2 k 2 j
j=0 062j<n n<2j62n

où r = si pair
n
(
(n/2 )
n
0
2k
sinon
donc avec le changement d'indice :
p = n − kj 2j − n = 2n − 2p − n = −(2p − n)
     
1 X n (2k−n)it 1 X n 1 X n
qk (t) = k e + k e−(2p−n)it + r = k−1 cos((2k − n)t) + r
2 k 2 n−p 2 k
06j<n/2 06p<n/2 06k<n/2

donc q ∈ vect(c , c , . . . , c ) ⊂ vect(c , c , . . . , c )


k 0 1 k 0 1 p

Par combinaison linéaire, la fonction t 7−→ p(cos(t)) dénie sur [0, π] appartient à F n

4/11
Mines-Ponts MP Un corrigé de Mathématiques 1 2015

9) Soit p et k ∈ N. Soit t ∈ [0, π]. On a c (t)c (t) = cos((p + k)t) +2 cos((p − k)t)
p k

donc si k 6= p alors p +Z k > 0


on a donc hc , c i =
π   t=π
cos((p + k)t) + cos((p − k)t) sin((p + k)t) sin((p − k)t)
p k G dt = + =0
0 2 2(p + k) 2(p − k) t=0

et kc k = hc , c i = cos(2pt)
Z π
2 +1
p p p G dt
2 0

si p = 0, on a kc k = 1 = π
Z π
2
0
0

et si p 6= 0, on a kc k = 4p + 2 = π2
  t=π
sin(2pt)2 t
p
t=0
si n = 0 √1
En prenant α = q sinon , on a (α ) ∈]0, +∞[ et (α c ) est orthonormée
(
π N
n 2 n n∈N n n n∈N
π

Soit f ∈ G. On va montrer que f ∈ vect(α c ) , adhérence dans G muni de la norme euclidienne k · k .


On remarque que Arccos est la bijection réciproque de la restriction : t ∈ [0, π] 7→ cos(t) ∈ [−1, 1]
n n n∈N G

On note g = f ◦ Arccos qui est continue sur le segment [−1, 1] par composition car Arccos est continue
Le théorème théorème de Weierstrass, nous fournit une suite (g ) de fonctions polynomiales qui converge
uniformément vers g sur [−1, 1].
k k∈N

Je note N la norme innie sur [−1, 1] de sorte que : N (g − g ) −→ 0


∞ ∞ k
k→+∞

Soit k ∈ N. On pose f : [0, π]t 7−−→



R
k
→ g (cos(t))
Il existe alors N ∈ N tel que f ∈ F donc f ∈ vect(α c ) car les α 6= 0
k

Pour tout t ∈ [0, π], on a |f (t) − f (t)| = |f (Arccos(cos t)) − f (Arccos(cos t))| = |g(cos(t)) − g (cos(t))|
k N k n n n∈N n

donc on a |f (t) − fZ (t)| 6 N (g − g ) Z


k k k
k ∞ k

donc kf − f k = (f (t) − f (t)) dt 6 N (g − g ) dt 6 πN (g − g )


π π
2 2 2 2
k G k ∞ k ∞ k

donc kf − f k 6 √πN (g − g )
0 0

donc kf − f k −→ 0 par théorème d'encadrement


k G ∞ k
k G

on a trouvé une suite de vect(α c ) qui converge vers f pour G muni de de la norme associé au produit
k→+∞

scalaire h·, ·i
n n n∈N

Ce qui donne bien la densité de vect(α c ) dans G


G
n n n∈N
Ainsi la suite orthonormée (α c ) est totale dans G n n n∈N

10) On remarque que la suite des sous espaces (F ) est croissante pour l'inclusion
Ainsi pour n ∈ N, comme F et F sont de dimensions nies, on a
n
n n+1

kf − PFn (f )kG = inf kf − gkG > inf kf − gkG = f − PFn+1 (f ) G


>0
g∈Fn g∈Fn+1

Donc la suite (kf − P (f )k ) est décroissante et positive


Soit ε > 0.
Fn G n∈N

Il existe une suite (f ) à valeurs dans vect(α c ) qui converge vers f pour la norme k · k d'après la
question précédente.
k n n n∈N G

Donc il existe m ∈ N tel que kf − f k 6 ε


Comme f ∈ vect(α c ) , ceci nous fournit N ∈ N tel que f ∈ vect(α c ) donc
m G
m n n n∈N m n n 06n6N

5/11
Mines-Ponts MP Un corrigé de Mathématiques 1 2015

kf − PFN (f )kG 6 kf − fm kG
Par décroissance et positivité, on a : ∀n > N, 0 6 kf − P Fn (f )kG 6ε
On vient de prouver
∀ε > 0, ∃N ∈ N, ∀n ∈ N, n > N =⇒ 0 6 kf − PFn (f )kG 6 ε

On a bien kf − P (f )k tend vers 0 lorsque n tend vers l'inni


Fn
On suppose de plus que la suite (P (f )) converge uniformément sur [0, π] vers une fonction g.
G

Comme de plus, chaque fonction P (f ) est continue sur [0, π], la fonction g est continue sur [0, π] par
Fn n∈N

théorème.
Fn

Donc g ∈ G et en utilisant l'inégalité de 9 avec la notation N , on a ∞

kg − PFn (f )kG 6 N∞ (g − PFn (f ))

donc par théorème d'encadrement : kg − P (f )k −→ 0 Fn G n→∞

donc la suite de G, P (f ) converge vers g pour la norme k · k




Par unicité de la limite f = g


Fn G

Si de plus la suite (P (f )) converge uniformément sur [0, π] vers une fonction g, alors g = f
Fn n∈N

11) Soit x ∈ [0, π/2]. Vérions que g est correctement dénie


x
 si π2 < t 6 π, on a 0 6 π − t < π2 et donc g (t) est correctement déni
 d'un côté g (π/2) = π/2 − π/2 = 0 d'un autre côté g (π/2) = −g (π/2) = 0
x

De plus, on remarque que :


x x x

 π − x − t − |x − t| si 0 6 t 6 π

si π2 6 t 6 π
g (t) = x 2 2
 −g (π − t) x

Ceci prouve la continuité de g en tout point de [0, π] \ {π/2} et la continuité à gauche et à droite en π/2.
On a bien g ∈ G Il n'est pas certain que cela soit à vérier !
x

Comme (α c ) est une base orthonormée de F , on a


x
k k 06k6n n

n n
X 1 2X
PFn (gx ) = hαk ck , gx iG αk ck = hc0 , gx iG c0 + hck , gx iG ck
π π
k=0 k=1

On a hc , g i = g (t) cos(kt)dt + g (t) cos(kt)dt = g (t) cos(kt)dt − g (π − t) cos(kt)dt


Z π/2 Z π Z π/2 Z π
k x G x x x x

dans la deuxième Zintégrale on eectue le Zchangement de variables de classe C : u = π −t ; t = π −u ; du = −dt


0 π/2 0 π/2
1

donc hc , g i = g (t) cos(kt)dt + g (u) cos(k(π − u))du


π/2 0
k x G x x
0 π/2

Ainsi hc , g i = g (t) cos(kt)dt − (−1)


Z π/2 Z π/2
k
k x G x g (u) cos(ku)du x

si k est pair, hc , g i = 0 Z
0 0
k x G

si k est impair, hc , g i = 2 g (t) cos(kt)dt


π/2
k x G x

donc 0

6/11
Mines-Ponts MP Un corrigé de Mathématiques 1 2015

n
! !
Z π/2 Z π/2
4 X 4 X
PFn (gx ) = gx (t) cos(kt)dt ck = gx (t) cos((2k + 1)t)dt c2k+1
π 0 π 0
k=1 062k+16n
k impair

et g (t) cos((2k + 1)t)dt = (π/2 − x) cos((2k + 1)t)dt + (π/2 − t) cos((2k + 1)t)dt


Z π/2 Z x Z π/2
x
0 0 x

or (π/2 − x) cos((2k + 1)t)dt = (π/2 − x) 2k + 1


Z x   t=x
sin((2k + 1)t) sin((2k + 1)x)
= (π/2 − x)
2k + 1
et à l'aide d'une intégration par parties avec des fonctions C :
0 t=0
1

Z π/2  t=π/2 Z π/2


sin((2k + 1)t) sin((2k + 1)t)
(π/2 − t) cos((2k + 1)t)dt = (π/2 − t) + dt
x 2k + 1 t=x x 2k + 1

donc π/2
− cos((2k + 1)t) t=π/2
Z  
sin((2k + 1)x)
(π/2 − t) cos((2k + 1)t)dt = 0 − (π/2 − x) +
x 2k + 1 (2k + 1)2
Ainsi en sommant :
t=x

Z π/2
cos((2k + 1)x) cos((2k + 1)π/2) cos((2k + 1)x)
(π/2 − t) cos((2k + 1)t)dt = − =
0 (2k + 1)2 (2k + 1)2 (2k + 1)2

b(n−1)/2c
donc PFn (gx ) =
X 4 cos((2k + 1)x)
π(2k + 1)2
c2k+1 coordonnées de P Fn (gx ) sur la base (c , c , . . . , c )
0 1 n
k=0

On a ∀k ∈ N, ∀t ∈ [0, π], 4 cos((2k + 1)x)


π(2k + 1)
c 2 2k+1 (t) 6
4
π(2k + 1)2
et la série X π(2k4+ 1) converge par 2

comparaison à une série (positive) de Riemann


k>0

donc la série de fonctions X 4 cos((2k + 1)x)


π(2k + 1)
c 2 2k+1 converge normalement sur [0, π]
k>0

donc la suite des sommes partielles converge uniformément sur [0, π]


m
!
X 4 cos((2k + 1)x)
c 2 2k+1
π(2k + 1)
donc la suite (P (g )) converge uniformément sur [0, π] vers une fonction h
k=0 m>0

d'après la question précédente, h = g


Fn x n>0

ainsi la suite (P (g )) converge simplement sur [0, π] vers la fonction g


x
Fn x n>0 x

On en déduit que : pour tout t ∈ [0, π/2] : π2 − max(x, t) = g (t) = π4 X cos((2n


+∞
+ 1)x)
cos((2n + 1)t) x 2
(2n + 1) n=0

12) Soit f ∈ E et x ∈ [0, π2 ]. On eectue une intégration par parties :


On a V ◦ V(f )(x) = V(f )(t)dt = [(t − π/2)V(f )(t)] − (t − π/2)f (t)dt
Z π/2 Z π/2
∗ t=π/2
t=x
x x

donc V ◦ V(f )(x) = 0 − (x − π/2)V(f )(x) − (t − π/2)f (t)dt = 2 f (t)dt − xf (t)dt −


Z Z Z Z π/2 π/2 x π/2
∗ π
tf (t)dt
x 0 0 x

d'où
Z π π
2


V ◦ V(f )(x) = − max(x, t) f (t)dt
0 2

7/11
Mines-Ponts MP Un corrigé de Mathématiques 1 2015

Ainsi Z π Z π +∞

2 2 4 X cos((2n + 1)x)
V ◦ V(f )(x) = gx (t)f (t)dt = cos((2n + 1)t)f (t)dt
0 0 π
n=0
(2n + 1)2

Les fonctions notées f : t 7→ cos((2n + 1)x)


(2n + 1)
cos((2n + 1)t)f (t) sont continues sur [0, π/2]
n 2

On note N (f ) la norme innie de f sur le segment [0, π] qui y est continue


donc on a

N (f ) ∞
∀n ∈ N, ∀x ∈ [0, π//2], |fn (t)| 6
(2n + 1)2

or la série X (2nN +(f1)) converge par à une comparaison à une série de Riemann (à termes positifs)

2
n>0

donc la série de fonctions X f converge normalement sur [0, π/2] ; on peut donc intervertir somme et inté-
n

grale : Z
n>0

+∞ π
∗ 4X 2 cos((2n + 1)x)
V ◦ V(f )(x) = cos((2n + 1)t)f (t)dt
π
n=0 0
(2n + 1)2
donc +∞ Z π
!
∗ 4X 2 cos((2n + 1)x)
V ◦ V(f )(x) = cos((2n + 1)t)f (t)dt
π
n=0 0 (2n + 1)2

En prenant ,
Z π
4 2
an (f ) = cos((2n + 1)t)f (t)dt
π(2n + 1)2 0

on a :
+∞
X
∀x ∈ [0, π/2], V∗ ◦ V(f )(x) = an (f ) cos((2n + 1)x)
n=0

D. Équations diérentielles du type Sturm-Liouville


13) Soit f ∈ E et n ∈ N. On a ϕ ∈ E.
donc d'après 2), hV ◦ V(f ), ϕ i = hf, V ◦ V(ϕ )i et d'après 4), ϕ ∈ E
n
∗ ∗
n n n 1/(2n+1)2 (V∗ ◦ V)

donc hV ◦ V(f ), ϕ i = (2n +1 1) hf, ϕ i.



n 2 n

14) Soit g ∈ E.
⇒ : On suppose que g est solution de l'équation diérentielle (S).
Alors g est deux fois dérivable et g = −λg − h donc g ∈ E 00 00

ainsi V(g ) = −λ · V(g) − V(h) car V ∈ L(E)


00

or V(g ) est la primitive de g sur s'annulant en 0


00 00

Comme g (0) = 0, on a g = V(g ).


0 0 00

donc V (g ) = −λ · V ◦ V(g) − V ◦ V(h) car V ∈ L(E)


∗ 0 ∗ ∗ ∗

or V (g ) est la primitive de −g s'annulant en π/2 et −g(π/2) = 0


∗ 0 0

donc g = λ · V ◦ V(g) + V ◦ V(h) ∗ ∗

⇐ : On suppose que g = λ · V  ◦ V(g) + V ◦ V(h) ∗ ∗

donc g = V λ · V(g) + V(h) ∗

donc g(π/2) = 0
En dérivant, g = − λ · V(g) + V(h) = V(−λg − h)
0

8/11
Mines-Ponts MP Un corrigé de Mathématiques 1 2015

donc g (0) = 0 et en dérivant g = −λg − h


0 00

donc g est solution de l'équation diérentielle (S)


On a bien g est solution de l'équation diérentielle (S) si et seulement si g = λ · V ∗ ◦ V(g) + V∗ ◦ V(h)
Soit n ∈ N. On suppose g est solution de l'équation diérentielle (S).
donc hg, ϕ i = hV ◦ V(λg + h), ϕ i = (2n +1 1) hλg + h, ϕ i d'après 13.
n

n 2 n

On a bien la formule : 1 − (2n + 1) hg, ϕ i = (2n +1 1) hh, ϕ i


 
λ
2 n 2 n

Soit x ∈ [0, ]. on a : pour f ∈ E, a (f ) = π(2n4+ 1) hf, ϕ i. Ainsi



π π
2 n 2 2 n

+∞
X hf, ϕn i
V∗ ◦ V(f )(x) = ϕn (x)
(2n + 1)2
n=0

On a λg + h ∈ E donc +∞
X hλg + h, ϕn i
g(x) = V∗ ◦ V(λg + h)(x) = ϕn (x)
(2n + 1)2
n=0

Or hλg(2n++h,1)ϕ i = λ (2nhg,+ϕ 1)i


n
2
n
2
+
hh, ϕn i
(2n + 1)2
= hg, ϕn i

ce qui permet de conclure à l'aide de la formule que g = hg, ϕ iϕ


+∞
X
n n

Remarque : On a égalité au sens de la convergence simple; vue la question suivante c'est sans doute le sens
n=0

de cette question.
Cependant on peut facilement montrer que l'on a convergence normale à l'aide de la formule de 12); ce qui
entraîne la convergence uniforme; ce qui permet de d'établir la convergence au sens de la norme k · k.
15) Soit n ∈ N. Soit x ∈ [0, ]. On a en utilisant Cauchy-Schwarz
π
2

1 2 2khkkϕn k
2 hh, ϕn iϕn (x) 6 √ 2 |hh, ϕn i| 6 √ 2 = αn
(2n + 1) −λ π|(2n + 1) −λ| π|(2n + 1) −λ|
s
On a 4π √
donc
Z π/2 r
αn = O n12

kϕn k = 2
ϕn 6 6 2
0 π2 n→∞

donc par comparaison entre séries à termes positifs, la série


X
αn converge et
n>0

π 1
∀x ∈ [0, ], hh, ϕn iϕn (x) 6 αn
2 (2n + 1)2 −λ

Ainsi la série : X
(2n +
1
1) 2
−λ
hh, ϕn iϕn est normalement convergente sur 0, π
2

n>0

On note alors g = (2n +11) −λ hh, ϕ iϕ


+∞
X
2 n n

La fonction g est continue sur [0, π/2] car les ϕ le sont également et qu'il y a convergence normale
n=0

donc g ∈ E.
n

9/11
Mines-Ponts MP Un corrigé de Mathématiques 1 2015

Pour montrer que g est solution de (S), il sut d'établir que g = λV ∗ ◦ V(g) + V∗ ◦ V(h) en servant de la
caractérisation de 14). On a :
+∞ +∞

X
∗ hg, ϕn i X hh, ϕn i
λV ◦ V(g) + V ◦ V(h) = λ 2
ϕn + ϕn
(2n + 1) (2n + 1)2
n=0 n=0

Soit , on a
+∞
!
Z π/2
1 X
n∈N hg, ϕn i = hh, ϕm iϕm (t)ϕn (t) dt
0 m=0
(2m + 1)2 −λ

On a 4
∀m ∈ N, ∀t ∈ [0, π/2], |hh, ϕm iϕm (t)ϕn (t)| 6 |hh, ϕm i|
π
Là encore la série de fonctions continues X
converge normalement

1
t 7→ hh, ϕm iϕm (t)ϕn (t)
(2m + 1)2 −λ
sur le segment , ce qui permet l'échange série/intégrale :
m>0
[0, π/2]
+∞ π/2
hh, ϕm i
X Z
hg, ϕn i = ϕm (t)ϕn (t)dt
m=0
(2m + 1)2 −λ 0

or
π/2
4 π/2 cos(2(m − n)t) + cos(2(m + n + 1)t)
Z Z
ϕm (t)ϕn (t)dt = dt
0 π 0 2

si , alors 4 sin(2(m − n)t) sin(2(m + n + 1)t) t=π/2


Z π/2  
m 6= n ϕm (t)ϕn (t)dt = + =0
π 4(m − n) 4(m + n + 1)
Dans ce cas
0 t=0
hϕm , ϕn i = 0

si , alors
Z π/2
4 π/2 1 + cos(2(m + n + 1)t)
Z
π
m=n ϕm (t)ϕn (t)dt = dt = + 0
π 0 2 4
ainsi
0
hϕn , ϕn i = 1
On remarque que la famille (ϕn )n∈N est une famille orthonormée de E
Ainsi hg, ϕn i =
hh, ϕn i
(2n + 1)2 −λ

Ainsi
+∞ +∞
X 1 hh, ϕn i X hh, ϕn i
λ · V∗ ◦ V(g) + V∗ ◦ V(h) = λ 2 2 ϕ n + ϕn = g
(2n + 1) (2n + 1) −λ (2n + 1)2
n=0 n=0

Ainsi d'après 14), X (2n +11) −λ hh, ϕ iϕ est solution de S


+∞

2 n n
n=0

16) Par l'absurde supposons hh, ϕ i = 6 0 et qu'il existe une solution notée g
À l'aide la première formule de 14, on a hh, ϕ i = ((2p + 1) − λ)hg, ϕ i = 0
p
2

Absurde
p n

si hh, ϕ i =6 0, alors (S) n'a pas de solution


p

On suppose désormais que hh, ϕ i = 0


Il existe un unique p ∈ N tel que : λ = (2p + 1)
p
2

On pose g = X (2n +11) −λ hh, ϕ iϕ


+∞

2 n n
n=0

On montre de la même manière qu'en 15 qu'il y a convergence normale puis que g est solution de S ce qui
n6=p

change un petit peu, c'est le calcul de hg, ϕ i = 0 = hV ◦ V(g), ϕ i = hh, ϕ i


p

p p

10/11
Mines-Ponts MP Un corrigé de Mathématiques 1 2015

Donc est solution de S si hh, ϕ i = 0 et λ = (2p + 1) où p ∈ N


+∞
X 1 2
g= hh, ϕn iϕn p
n=0
(2n + 1)2 −λ
n6=p

On considère l'équation homogène associée à (S) :


y 00 + (2p + 1)2 y = 0

(SH)
et
y( π2 ) = 0 y 0 (0) = 0

Il est claire que ϕ est solution de (SH) donc que la droite vectorielle vect(ϕ ) est inclus dans l'ensemble des
solutions de (SH)
p p

Donc pour chaque µ ∈ R, la fonction de E : g + µϕ est une solution de (S)


p

si hh, ϕ i = 0 et λ = (2p + 1) alors (S) admet une innité de solutions car ϕ 6= 0


p
2
p E

Fin du problème

11/11
Mines-Ponts MP Un corrigé de Mathématiques 1 2015

Opérateur de Volterra et équations diérentielles

A. Opérateur de Volterra
1) Soit f et g dans E.
D'après le théorème fondamentale de l'analyse, V(f ) et −V (f ) sont des primitives f sur [0, π/2] car f y est ∗

continue
De plus on a V(f ) + V (f ) : x 7→ π/2, d'après la relation de Chasles et V(f )(0) = 0 = V (f )(π/2)
∗ ∗

On utilise le théorème d'intégration par parties avec les fonctions de classe C : V(f ) et −V (g). 1 ∗

On a : hV(f ), gi = V(f )(x)g(x)dx = V(f )(x)(−V (g)(x)) + f (x)V (g)(x)dx


Z π/2 Z π/2 π/2
∗ ∗
0 0
On a bien hV(f ), gi = hf, V (g)i
0

2) V ◦ V est bien un endomorphisme par composition.


Soit f et g dans E. On utilise deux fois la question précédente.


On a hV ◦ V(f ), gi = hV(f ), V(g)i = hf, V ◦ V(g)i
∗ ∗

On va montrer que : hV ◦ V(f ), f i = 0 =⇒ f = 0 ∗

On suppose que : hV ◦ V(f ), f i = 0 donc 0 = hV(f ), V(f )i = kV(f )k


E
∗ 2

donc V(f ) = 0 et en dérivant f = V(f ) = 0 0

ainsi l'endomorphisme V ◦ V est symétrique déni positif


E E

Soit λ une valeur propre de V ◦ V et f un vecteur propre associé à λ ∗

On a hV ◦ V(f ), f i = kV(f )k
λ
∗ 2

or V(f ) 6= 0 car f 6= 0 et que V(f ) = f


λ λ λ
0

donc hV ◦ V(f ), f i > 0


λ E λ E λ λ

et on a hV ◦ V(f ), f i = hλf , f i = λkf k


λ λ
∗ 2

Comme kf k > 0, on en déduit que les valeurs propres de V ◦ V sont strictement positives
λ λ λ λ λ
2 ∗
λ

3) On a f = V ◦ V(f ) = V (V(f ))
λ
1
λ

λ
1
λ

λ

À l'aide des observations du 1), on a f = V(f ) et f = f ce qui prouve que f est de classe C
0 1
λ
00 1
λ λ λ
2

De plus f (π/2) = V (V(f ))(π/2) = 0 et f (0) = V(f )(0)


λ λ λ
1 ∗ 0 1
λ λ λ λ λ λ

donc f est solution de l'équation diérentielle : y + λ1 y = 0 avec les conditions : y(π/2) = 0 et y (0) = 0
λ
00 0

4) =⇒ : Soit λ est une valeur propre de V ◦ V et f un vecteur propre associé à λ. ∗

Par résolution de l'équationx diérentiellex car λ > 0, on peut trouver A et B ∈ R tels que
λ

f : x 7→ A cos( √ ) + B sin( √ ) et on a f : x 7→ − √ sin( √ ) + √ cos( √ )


A x B x 0
λ λ
λ  λ λ λ λ λ
on a 0 = f (π/2) = A cos 2√λ + B sin 2√λ et 0 = f (0) =

π π 0 B
λ √
λ

donc B = 0 donc on a f : x 7→ A cos √xλ


 
λ

Ainsi f ∈ vect x 7→ A cos √λ


  
x
λ

et A cos 2√λ = f (π/2) = 0


 
π
λ

Comme f 6= 0 , on a A 6= 0 donc cos 2√λ = 0


 
π
λ E

1/11
Mines-Ponts MP Un corrigé de Mathématiques 1 2015

ce qui nous fournit k ∈ Z tel que 2√π λ = π2 + kπ = (2k +2 1)π


donc 2√λ = (2k 2+ 1)
donc il existe n ∈ N tel que : λ = (2n +1 1) en prenant n ∈ N tel que 2n + 1 = |2k + 1|
2

=⇒ : Soit λ ∈ R tel qu'il existe n ∈ N tel que : λ = et f : x 7→ cos( ) = cos((2n + 1)x)


1
√x
2 λ
(2n + 1)
Soit x ∈ [0, ]. Z π
2

On a V(f )(x) = cos((2n + 1)t)dt =  sin((2n


x
+ 1)t)  sin((2n + 1)x) x
= 0
0 2n + 1 2n + 1

et (V ◦ V) (f )(x) = sin((2n
Z π/2
∗ + 1)t)  cos((2n + 1)t)  cos((2n + 1)x) cos((2n + 1)π/2)
π/2
dt = − = − 2 x 2 2
2n + 1 (2n + 1) (2n + 1) (2n + 1)
Ainsi et (V ◦ V) (f )(x) = λf (x)
x

donc (V ◦ V) (f ) = λf et f 6= 0

Conclusion : On en déduit que


E

λ est une valeur propre de V ◦ V si et seulement s'il existe n ∈ N tel que : λ =


∗ 1
2
(2n + 1)

Dans ce cas
 
E1/(2n+1)2 (V∗ ◦ V) = vect x 7−→ cos((2n + 1)x)

B. Théorème d'approximation de Weierstrass


suit la loi binomiale de paramètres et de loi donnée par
 
n k
5) Sn n x ∀k ∈ [[0, n]], P(Sn = k) = x (1 − x)n−k
k

On utilise pour tout ,


0 < k 6 n k nk = k
 n!
(n − k)!k!
=n
(n − 1)!
(n − 1 − (k − 1))!(k − 1)!
= n n−1
k−1


On a
n n   n  
X X n k n−k
X n−1 k
E(Sn ) = kP(Sn = k) = 0 + k x (1 − x) =n x (1 − x)n−k
k k−1
k=0 k=1 k=1

ainsi par changement d'indice :


n−1
X n − 1
E(Sn ) = nx xj (1 − x)n−1−j j =k−1
j
j=0

et si n > 2, on a par la formule du transfert


n
X
E(S2n − Sn ) = (k 2 − k)P(Sn = k)
k=0

donc
n   n  
X n k n−k
X n−2 k
E(S2n − Sn ) = k(k − 1) x (1 − x) = n(n − 1) x (1 − x)n−k
k k−2
k=2 k=2

donc valable même si


n−2
X n
E(S2n − Sn ) = n(n − 1)x2 xj (1 − x)n−2−j = n(n − 1)x2 n=1
j
donc E(S ) = E(S − S ) + E(S ) = n(n − 1)x + nx
j=0
2 2 2

donc V(S ) = E(S ) − E(S ) = n x − nx + nx − (nx)


n n n n
2 2 2 2 2 2
n n
d'où E(S ) = nx et V(S ) = nx(1 − x) (comme prévu)
n
n n

6) D'après Irénée-Jules Bienaymé et Pafnouti Tchebychev, on a

V(Sn )
P (|Sn − E(Sn )| > nα) 6
(nα)2

2/11
Mines-Ponts MP Un corrigé de Mathématiques 1 2015

or V(S
(nα)
)
=
nnx(1 − x)
2 n α
=
x(1 − x)
2 2nα 2

Une étude classique de variations nous donne ∀x ∈ [0, 1], 0 6 x(1 − x) 6 1/4.
donc P (|S − E(S )| > nα) 6 4nα1
n n 2

or (|S − E(S )| > nα) = (|S − nx| > nα) = − x > α = [ (S = k)


n n n
Sn
n n
06k6n
k
|n −x|>α

donc P (|S n − E(Sn )| > nα) =


X
P(Sn = k) par réunion disjointe
06k6n
k
|n −x|>α

donc on a bien
 
X n k 1
x (1 − x)n−k 6
k 4nα2
06k6n
k
|n −x|>α

On a B (f )(x) = E(f (Z )) = E(f (S /n)) = X f ( nk )P(S selon la formule du transfert


n
7) n n n n = k)
k=0

donc
 
n k k
Bn (f )(x) = x (1 − x)n−k f ( )
k n

et
n
X n  
xk (1 − x)n−k f (x) = (x + 1 − x)n f (x) = f (x)
k
k=0

En faisant la soustraction :
n    
X n k n−k k
Bn (f )(x) − f (x) = x (1 − x) f ( ) − f (x)
k n
Soit ε > 0.
k=0

La fonction f est continue sur le segment [0, 1] donc y est bornée uniformément continue d'après le théorème
de Heine. On note kf k la norme innie de f sur [0, 1].
L'uniforme continuité de f nous fournit α > 0 tel que : ∀y, z ∈ [0, 1], |y − z| < α =⇒ |f (x) − f (y)| 6 donc

ε
2

selon l'inégalité triangulaire


     
X n k n−k k X n k k
x (1 − x) f ( ) − f (x) 6 x (1 − x)n−k f ( ) − f (x)
k n k n
06k6n 06k6n
k k
|n −x|<α |n −x|<α
 
X n k ε
6 x (1 − x)n−k
k 2
06k6n
k
|n −x|<α
n  
X n k ε
6 x (1 − x)n−k
k 2
k=0

   
X n k n−k k ε
x (1 − x) f ( ) − f (x) 6
k n 2
06k6n
k
|n −x|<α

3/11
Mines-Ponts MP Un corrigé de Mathématiques 1 2015

d'autre part, on a

inégalité triangulaire
       
X n k n−k k X n k n−k k
x (1 − x) f ( ) − f (x) 6 x (1 − x) f ( ) + |f (x)|
k n k n
06k6n 06k6n
k k
|n −x|>α |n −x|>α
 
X n k
6 2kf k∞ x (1 − x)n−k
k
06k6n
k
|n −x|>α

selon 6
   
X n k n−k k kf k∞
x (1 − x) f ( ) − f (x) 6
k n 2nα2
06k6n
k
|n −x|>α

or lim kf2nαk = 0 Ceci nous fournit N ∈ N, tel que ∀n ∈ N, n > N =⇒



2
kf k∞
2nα2
6 ε
2

On vient de montrer que


n→+∞

∀ε > 0, ∃N ∈ N, ∀n ∈ N, n > N =⇒ (∀x ∈ [0, 1], |Bn (f )(x) − f (x)| 6 ε)

ou encore : kB (f ) − f k −→ 0
n ∞ n→+∞

on en déduit que la suite (B (f )) n n∈N converge uniformément vers f sur [0, 1]


C. Développement de V∗ ◦ V(f ) en série trigonométrique
Soit p un polynôme de degré n que l'on écrit p(X) = X a X et on note q de sorte que : la
n
k
8) k k : t 7→ cosk (t)

fonction t 7−→ p(cos(t)) appartient à vect(q , . . . , q ) n=0

Soit k ∈ [[0, n]]. Soit t ∈ [0, π]. Par la formule du binôme on a :


0 n

k n  
eit + e−it
 X n  
1 X n (2j−n)it 1 1 X n (2j−n)it
qk (t) = = k e = k e(2k−n)it + k e +r
2 2 j 2 k 2 j
j=0 062j<n n<2j62n

où r = si pair
n
(
(n/2 )
n
0
2k
sinon
donc avec le changement d'indice :
p = n − kj 2j − n = 2n − 2p − n = −(2p − n)
     
1 X n (2k−n)it 1 X n 1 X n
qk (t) = k e + k e−(2p−n)it + r = k−1 cos((2k − n)t) + r
2 k 2 n−p 2 k
06j<n/2 06p<n/2 06k<n/2

donc q ∈ vect(c , c , . . . , c ) ⊂ vect(c , c , . . . , c )


k 0 1 k 0 1 p

Par combinaison linéaire, la fonction t 7−→ p(cos(t)) dénie sur [0, π] appartient à F n

4/11
Mines-Ponts MP Un corrigé de Mathématiques 1 2015

9) Soit p et k ∈ N. Soit t ∈ [0, π]. On a c (t)c (t) = cos((p + k)t) +2 cos((p − k)t)
p k

donc si k 6= p alors p +Z k > 0


on a donc hc , c i =
π   t=π
cos((p + k)t) + cos((p − k)t) sin((p + k)t) sin((p − k)t)
p k G dt = + =0
0 2 2(p + k) 2(p − k) t=0

et kc k = hc , c i = cos(2pt)
Z π
2 +1
p p p G dt
2 0

si p = 0, on a kc k = 1 = π
Z π
2
0
0

et si p 6= 0, on a kc k = 4p + 2 = π2
  t=π
sin(2pt)2 t
p
t=0
si n = 0 √1
En prenant α = q sinon , on a (α ) ∈]0, +∞[ et (α c ) est orthonormée
(
π N
n 2 n n∈N n n n∈N
π

Soit f ∈ G. On va montrer que f ∈ vect(α c ) , adhérence dans G muni de la norme euclidienne k · k .


On remarque que Arccos est la bijection réciproque de la restriction : t ∈ [0, π] 7→ cos(t) ∈ [−1, 1]
n n n∈N G

On note g = f ◦ Arccos qui est continue sur le segment [−1, 1] par composition car Arccos est continue
Le théorème théorème de Weierstrass, nous fournit une suite (g ) de fonctions polynomiales qui converge
uniformément vers g sur [−1, 1].
k k∈N

Je note N la norme innie sur [−1, 1] de sorte que : N (g − g ) −→ 0


∞ ∞ k
k→+∞

Soit k ∈ N. On pose f : [0, π]t 7−−→



R
k
→ g (cos(t))
Il existe alors N ∈ N tel que f ∈ F donc f ∈ vect(α c ) car les α 6= 0
k

Pour tout t ∈ [0, π], on a |f (t) − f (t)| = |f (Arccos(cos t)) − f (Arccos(cos t))| = |g(cos(t)) − g (cos(t))|
k N k n n n∈N n

donc on a |f (t) − fZ (t)| 6 N (g − g ) Z


k k k
k ∞ k

donc kf − f k = (f (t) − f (t)) dt 6 N (g − g ) dt 6 πN (g − g )


π π
2 2 2 2
k G k ∞ k ∞ k

donc kf − f k 6 √πN (g − g )
0 0

donc kf − f k −→ 0 par théorème d'encadrement


k G ∞ k
k G

on a trouvé une suite de vect(α c ) qui converge vers f pour G muni de de la norme associé au produit
k→+∞

scalaire h·, ·i
n n n∈N

Ce qui donne bien la densité de vect(α c ) dans G


G
n n n∈N
Ainsi la suite orthonormée (α c ) est totale dans G n n n∈N

10) On remarque que la suite des sous espaces (F ) est croissante pour l'inclusion
Ainsi pour n ∈ N, comme F et F sont de dimensions nies, on a
n
n n+1

kf − PFn (f )kG = inf kf − gkG > inf kf − gkG = f − PFn+1 (f ) G


>0
g∈Fn g∈Fn+1

Donc la suite (kf − P (f )k ) est décroissante et positive


Soit ε > 0.
Fn G n∈N

Il existe une suite (f ) à valeurs dans vect(α c ) qui converge vers f pour la norme k · k d'après la
question précédente.
k n n n∈N G

Donc il existe m ∈ N tel que kf − f k 6 ε


Comme f ∈ vect(α c ) , ceci nous fournit N ∈ N tel que f ∈ vect(α c ) donc
m G
m n n n∈N m n n 06n6N

5/11
Mines-Ponts MP Un corrigé de Mathématiques 1 2015

kf − PFN (f )kG 6 kf − fm kG
Par décroissance et positivité, on a : ∀n > N, 0 6 kf − P Fn (f )kG 6ε
On vient de prouver
∀ε > 0, ∃N ∈ N, ∀n ∈ N, n > N =⇒ 0 6 kf − PFn (f )kG 6 ε

On a bien kf − P (f )k tend vers 0 lorsque n tend vers l'inni


Fn
On suppose de plus que la suite (P (f )) converge uniformément sur [0, π] vers une fonction g.
G

Comme de plus, chaque fonction P (f ) est continue sur [0, π], la fonction g est continue sur [0, π] par
Fn n∈N

théorème.
Fn

Donc g ∈ G et en utilisant l'inégalité de 9 avec la notation N , on a ∞

kg − PFn (f )kG 6 N∞ (g − PFn (f ))

donc par théorème d'encadrement : kg − P (f )k −→ 0 Fn G n→∞

donc la suite de G, P (f ) converge vers g pour la norme k · k




Par unicité de la limite f = g


Fn G

Si de plus la suite (P (f )) converge uniformément sur [0, π] vers une fonction g, alors g = f
Fn n∈N

11) Soit x ∈ [0, π/2]. Vérions que g est correctement dénie


x
 si π2 < t 6 π, on a 0 6 π − t < π2 et donc g (t) est correctement déni
 d'un côté g (π/2) = π/2 − π/2 = 0 d'un autre côté g (π/2) = −g (π/2) = 0
x

De plus, on remarque que :


x x x

 π − x − t − |x − t| si 0 6 t 6 π

si π2 6 t 6 π
g (t) = x 2 2
 −g (π − t) x

Ceci prouve la continuité de g en tout point de [0, π] \ {π/2} et la continuité à gauche et à droite en π/2.
On a bien g ∈ G Il n'est pas certain que cela soit à vérier !
x

Comme (α c ) est une base orthonormée de F , on a


x
k k 06k6n n

n n
X 1 2X
PFn (gx ) = hαk ck , gx iG αk ck = hc0 , gx iG c0 + hck , gx iG ck
π π
k=0 k=1

On a hc , g i = g (t) cos(kt)dt + g (t) cos(kt)dt = g (t) cos(kt)dt − g (π − t) cos(kt)dt


Z π/2 Z π Z π/2 Z π
k x G x x x x

dans la deuxième Zintégrale on eectue le Zchangement de variables de classe C : u = π −t ; t = π −u ; du = −dt


0 π/2 0 π/2
1

donc hc , g i = g (t) cos(kt)dt + g (u) cos(k(π − u))du


π/2 0
k x G x x
0 π/2

Ainsi hc , g i = g (t) cos(kt)dt − (−1)


Z π/2 Z π/2
k
k x G x g (u) cos(ku)du x

si k est pair, hc , g i = 0 Z
0 0
k x G

si k est impair, hc , g i = 2 g (t) cos(kt)dt


π/2
k x G x

donc 0

6/11
Mines-Ponts MP Un corrigé de Mathématiques 1 2015

n
! !
Z π/2 Z π/2
4 X 4 X
PFn (gx ) = gx (t) cos(kt)dt ck = gx (t) cos((2k + 1)t)dt c2k+1
π 0 π 0
k=1 062k+16n
k impair

et g (t) cos((2k + 1)t)dt = (π/2 − x) cos((2k + 1)t)dt + (π/2 − t) cos((2k + 1)t)dt


Z π/2 Z x Z π/2
x
0 0 x

or (π/2 − x) cos((2k + 1)t)dt = (π/2 − x) 2k + 1


Z x   t=x
sin((2k + 1)t) sin((2k + 1)x)
= (π/2 − x)
2k + 1
et à l'aide d'une intégration par parties avec des fonctions C :
0 t=0
1

Z π/2  t=π/2 Z π/2


sin((2k + 1)t) sin((2k + 1)t)
(π/2 − t) cos((2k + 1)t)dt = (π/2 − t) + dt
x 2k + 1 t=x x 2k + 1

donc π/2
− cos((2k + 1)t) t=π/2
Z  
sin((2k + 1)x)
(π/2 − t) cos((2k + 1)t)dt = 0 − (π/2 − x) +
x 2k + 1 (2k + 1)2
Ainsi en sommant :
t=x

Z π/2
cos((2k + 1)x) cos((2k + 1)π/2) cos((2k + 1)x)
(π/2 − t) cos((2k + 1)t)dt = − =
0 (2k + 1)2 (2k + 1)2 (2k + 1)2

b(n−1)/2c
donc PFn (gx ) =
X 4 cos((2k + 1)x)
π(2k + 1)2
c2k+1 coordonnées de P Fn (gx ) sur la base (c , c , . . . , c )
0 1 n
k=0

On a ∀k ∈ N, ∀t ∈ [0, π], 4 cos((2k + 1)x)


π(2k + 1)
c 2 2k+1 (t) 6
4
π(2k + 1)2
et la série X π(2k4+ 1) converge par 2

comparaison à une série (positive) de Riemann


k>0

donc la série de fonctions X 4 cos((2k + 1)x)


π(2k + 1)
c 2 2k+1 converge normalement sur [0, π]
k>0

donc la suite des sommes partielles converge uniformément sur [0, π]


m
!
X 4 cos((2k + 1)x)
c 2 2k+1
π(2k + 1)
donc la suite (P (g )) converge uniformément sur [0, π] vers une fonction h
k=0 m>0

d'après la question précédente, h = g


Fn x n>0

ainsi la suite (P (g )) converge simplement sur [0, π] vers la fonction g


x
Fn x n>0 x

On en déduit que : pour tout t ∈ [0, π/2] : π2 − max(x, t) = g (t) = π4 X cos((2n


+∞
+ 1)x)
cos((2n + 1)t) x 2
(2n + 1) n=0

12) Soit f ∈ E et x ∈ [0, π2 ]. On eectue une intégration par parties :


On a V ◦ V(f )(x) = V(f )(t)dt = [(t − π/2)V(f )(t)] − (t − π/2)f (t)dt
Z π/2 Z π/2
∗ t=π/2
t=x
x x

donc V ◦ V(f )(x) = 0 − (x − π/2)V(f )(x) − (t − π/2)f (t)dt = 2 f (t)dt − xf (t)dt −


Z Z Z Z π/2 π/2 x π/2
∗ π
tf (t)dt
x 0 0 x

d'où
Z π π
2


V ◦ V(f )(x) = − max(x, t) f (t)dt
0 2

7/11
Mines-Ponts MP Un corrigé de Mathématiques 1 2015

Ainsi Z π Z π +∞

2 2 4 X cos((2n + 1)x)
V ◦ V(f )(x) = gx (t)f (t)dt = cos((2n + 1)t)f (t)dt
0 0 π
n=0
(2n + 1)2

Les fonctions notées f : t 7→ cos((2n + 1)x)


(2n + 1)
cos((2n + 1)t)f (t) sont continues sur [0, π/2]
n 2

On note N (f ) la norme innie de f sur le segment [0, π] qui y est continue


donc on a

N (f ) ∞
∀n ∈ N, ∀x ∈ [0, π//2], |fn (t)| 6
(2n + 1)2

or la série X (2nN +(f1)) converge par à une comparaison à une série de Riemann (à termes positifs)

2
n>0

donc la série de fonctions X f converge normalement sur [0, π/2] ; on peut donc intervertir somme et inté-
n

grale : Z
n>0

+∞ π
∗ 4X 2 cos((2n + 1)x)
V ◦ V(f )(x) = cos((2n + 1)t)f (t)dt
π
n=0 0
(2n + 1)2
donc +∞ Z π
!
∗ 4X 2 cos((2n + 1)x)
V ◦ V(f )(x) = cos((2n + 1)t)f (t)dt
π
n=0 0 (2n + 1)2

En prenant ,
Z π
4 2
an (f ) = cos((2n + 1)t)f (t)dt
π(2n + 1)2 0

on a :
+∞
X
∀x ∈ [0, π/2], V∗ ◦ V(f )(x) = an (f ) cos((2n + 1)x)
n=0

D. Équations diérentielles du type Sturm-Liouville


13) Soit f ∈ E et n ∈ N. On a ϕ ∈ E.
donc d'après 2), hV ◦ V(f ), ϕ i = hf, V ◦ V(ϕ )i et d'après 4), ϕ ∈ E
n
∗ ∗
n n n 1/(2n+1)2 (V∗ ◦ V)

donc hV ◦ V(f ), ϕ i = (2n +1 1) hf, ϕ i.



n 2 n

14) Soit g ∈ E.
⇒ : On suppose que g est solution de l'équation diérentielle (S).
Alors g est deux fois dérivable et g = −λg − h donc g ∈ E 00 00

ainsi V(g ) = −λ · V(g) − V(h) car V ∈ L(E)


00

or V(g ) est la primitive de g sur s'annulant en 0


00 00

Comme g (0) = 0, on a g = V(g ).


0 0 00

donc V (g ) = −λ · V ◦ V(g) − V ◦ V(h) car V ∈ L(E)


∗ 0 ∗ ∗ ∗

or V (g ) est la primitive de −g s'annulant en π/2 et −g(π/2) = 0


∗ 0 0

donc g = λ · V ◦ V(g) + V ◦ V(h) ∗ ∗

⇐ : On suppose que g = λ · V  ◦ V(g) + V ◦ V(h) ∗ ∗

donc g = V λ · V(g) + V(h) ∗

donc g(π/2) = 0
En dérivant, g = − λ · V(g) + V(h) = V(−λg − h)
0

8/11
Mines-Ponts MP Un corrigé de Mathématiques 1 2015

donc g (0) = 0 et en dérivant g = −λg − h


0 00

donc g est solution de l'équation diérentielle (S)


On a bien g est solution de l'équation diérentielle (S) si et seulement si g = λ · V ∗ ◦ V(g) + V∗ ◦ V(h)
Soit n ∈ N. On suppose g est solution de l'équation diérentielle (S).
donc hg, ϕ i = hV ◦ V(λg + h), ϕ i = (2n +1 1) hλg + h, ϕ i d'après 13.
n

n 2 n

On a bien la formule : 1 − (2n + 1) hg, ϕ i = (2n +1 1) hh, ϕ i


 
λ
2 n 2 n

Soit x ∈ [0, ]. on a : pour f ∈ E, a (f ) = π(2n4+ 1) hf, ϕ i. Ainsi



π π
2 n 2 2 n

+∞
X hf, ϕn i
V∗ ◦ V(f )(x) = ϕn (x)
(2n + 1)2
n=0

On a λg + h ∈ E donc +∞
X hλg + h, ϕn i
g(x) = V∗ ◦ V(λg + h)(x) = ϕn (x)
(2n + 1)2
n=0

Or hλg(2n++h,1)ϕ i = λ (2nhg,+ϕ 1)i


n
2
n
2
+
hh, ϕn i
(2n + 1)2
= hg, ϕn i

ce qui permet de conclure à l'aide de la formule que g = hg, ϕ iϕ


+∞
X
n n

Remarque : On a égalité au sens de la convergence simple; vue la question suivante c'est sans doute le sens
n=0

de cette question.
Cependant on peut facilement montrer que l'on a convergence normale à l'aide de la formule de 12); ce qui
entraîne la convergence uniforme; ce qui permet de d'établir la convergence au sens de la norme k · k.
15) Soit n ∈ N. Soit x ∈ [0, ]. On a en utilisant Cauchy-Schwarz
π
2

1 2 2khkkϕn k
2 hh, ϕn iϕn (x) 6 √ 2 |hh, ϕn i| 6 √ 2 = αn
(2n + 1) −λ π|(2n + 1) −λ| π|(2n + 1) −λ|
s
On a 4π √
donc
Z π/2 r
αn = O n12

kϕn k = 2
ϕn 6 6 2
0 π2 n→∞

donc par comparaison entre séries à termes positifs, la série


X
αn converge et
n>0

π 1
∀x ∈ [0, ], hh, ϕn iϕn (x) 6 αn
2 (2n + 1)2 −λ

Ainsi la série : X
(2n +
1
1) 2
−λ
hh, ϕn iϕn est normalement convergente sur 0, π
2

n>0

On note alors g = (2n +11) −λ hh, ϕ iϕ


+∞
X
2 n n

La fonction g est continue sur [0, π/2] car les ϕ le sont également et qu'il y a convergence normale
n=0

donc g ∈ E.
n

9/11
Mines-Ponts MP Un corrigé de Mathématiques 1 2015

Pour montrer que g est solution de (S), il sut d'établir que g = λV ∗ ◦ V(g) + V∗ ◦ V(h) en servant de la
caractérisation de 14). On a :
+∞ +∞

X
∗ hg, ϕn i X hh, ϕn i
λV ◦ V(g) + V ◦ V(h) = λ 2
ϕn + ϕn
(2n + 1) (2n + 1)2
n=0 n=0

Soit , on a
+∞
!
Z π/2
1 X
n∈N hg, ϕn i = hh, ϕm iϕm (t)ϕn (t) dt
0 m=0
(2m + 1)2 −λ

On a 4
∀m ∈ N, ∀t ∈ [0, π/2], |hh, ϕm iϕm (t)ϕn (t)| 6 |hh, ϕm i|
π
Là encore la série de fonctions continues X
converge normalement

1
t 7→ hh, ϕm iϕm (t)ϕn (t)
(2m + 1)2 −λ
sur le segment , ce qui permet l'échange série/intégrale :
m>0
[0, π/2]
+∞ π/2
hh, ϕm i
X Z
hg, ϕn i = ϕm (t)ϕn (t)dt
m=0
(2m + 1)2 −λ 0

or
π/2
4 π/2 cos(2(m − n)t) + cos(2(m + n + 1)t)
Z Z
ϕm (t)ϕn (t)dt = dt
0 π 0 2

si , alors 4 sin(2(m − n)t) sin(2(m + n + 1)t) t=π/2


Z π/2  
m 6= n ϕm (t)ϕn (t)dt = + =0
π 4(m − n) 4(m + n + 1)
Dans ce cas
0 t=0
hϕm , ϕn i = 0

si , alors
Z π/2
4 π/2 1 + cos(2(m + n + 1)t)
Z
π
m=n ϕm (t)ϕn (t)dt = dt = + 0
π 0 2 4
ainsi
0
hϕn , ϕn i = 1
On remarque que la famille (ϕn )n∈N est une famille orthonormée de E
Ainsi hg, ϕn i =
hh, ϕn i
(2n + 1)2 −λ

Ainsi
+∞ +∞
X 1 hh, ϕn i X hh, ϕn i
λ · V∗ ◦ V(g) + V∗ ◦ V(h) = λ 2 2 ϕ n + ϕn = g
(2n + 1) (2n + 1) −λ (2n + 1)2
n=0 n=0

Ainsi d'après 14), X (2n +11) −λ hh, ϕ iϕ est solution de S


+∞

2 n n
n=0

16) Par l'absurde supposons hh, ϕ i = 6 0 et qu'il existe une solution notée g
À l'aide la première formule de 14, on a hh, ϕ i = ((2p + 1) − λ)hg, ϕ i = 0
p
2

Absurde
p n

si hh, ϕ i =6 0, alors (S) n'a pas de solution


p

On suppose désormais que hh, ϕ i = 0


Il existe un unique p ∈ N tel que : λ = (2p + 1)
p
2

On pose g = X (2n +11) −λ hh, ϕ iϕ


+∞

2 n n
n=0

On montre de la même manière qu'en 15 qu'il y a convergence normale puis que g est solution de S ce qui
n6=p

change un petit peu, c'est le calcul de hg, ϕ i = 0 = hV ◦ V(g), ϕ i = hh, ϕ i


p

p p

10/11
Mines-Ponts MP Un corrigé de Mathématiques 1 2015

Donc est solution de S si hh, ϕ i = 0 et λ = (2p + 1) où p ∈ N


+∞
X 1 2
g= hh, ϕn iϕn p
n=0
(2n + 1)2 −λ
n6=p

On considère l'équation homogène associée à (S) :


y 00 + (2p + 1)2 y = 0

(SH)
et
y( π2 ) = 0 y 0 (0) = 0

Il est claire que ϕ est solution de (SH) donc que la droite vectorielle vect(ϕ ) est inclus dans l'ensemble des
solutions de (SH)
p p

Donc pour chaque µ ∈ R, la fonction de E : g + µϕ est une solution de (S)


p

si hh, ϕ i = 0 et λ = (2p + 1) alors (S) admet une innité de solutions car ϕ 6= 0


p
2
p E

Fin du problème

11/11
CENTRALE MP 2015 - MATHS 2
Autour des sommes d’Euler
n
X 1 1 1
Dans tout le problème, on note pour tout entier n ≥ 1, Hn = =1+ + ··· + ·
k=1
k 2 n
+∞
X 1
On note ζ la fonction définie pour x > 1 par ζ(x) = ·
n=1
nx
Le but du problème est d’étudier des séries faisant intervenir la suite (Hn ) et notamment d’obtenir
+∞
X Hn
une relation due à Euler qui exprime, pour r entier naturel supérieur ou égal à 2, à
n=1
(n + 1)r
l’aide de la valeur de la fonction ζ en certains points entiers.

I. Représentation intégrale de sommes de séries


I.A.
1
Z n dt
I.A.1) Justifier que la série de terme général an = − converge.
n n−1 t
I.A.2) Montrer qu’il existe une constante réelle A telle que Hn = ln n + A + o(1). En déduire
+∞
que Hn ∼ ln n .
+∞

I.B. Soit r un entier naturel.


X Hn
Pour quelles valeurs de r la série est-elle convergente ?
n≥1
(n + 1)r
+∞
X Hn
Dans toute la suite on notera Sr = lorsque la série converge.
n=1
(n + 1)r

I.C.
I.C.1) Donner sans démonstration les développements en série entière des fonctions t 7→ ln(1 − t)
1
et t 7→ ainsi que leur rayon de convergence.
1−t
I.C.2) En déduire que la fonction
ln(1 − t)
t 7−→ −
1−t
est développable en série entière sur ]−1 ; 1[ et préciser son développement en série entière
à l’aide des réels Hn .

I.D. Pour tout couple d’entiers naturels (p, q) et pour tout ε ∈ ]0 ; 1[ , on note :
Z 1 Z 1
Ip,q = tp (ln t)q dt et ε
Ip,q = tp (ln t)q dt .
0 ε

I.D.1) Montrer que l’intégrale Ip,q existe pour tout couple d’entiers naturels (p, q).

I.D.2) Montrer que :

ε q ε εp+1 (ln ε)q


∀p ∈ N, ∀q ∈ N∗ , ∀ε ∈ ]0 ; 1[ , Ip,q =− Ip,q−1 − ·
p+1 p+1
I.D.3) En déduire que l’on a :
q
∀p ∈ N, ∀q ∈ N∗ , Ip,q = − Ip,q−1 .
p+1

I.D.4) En déduire une expression de Ip,q en fonction des entiers p et q .

1
I.E.
Soit r un entier naturel non nul et f une fonction développable en série entière sur ]−1 ; 1[ .
+∞
X X an
On suppose que pour tout x ∈ ]−1 ; 1[ , f (x) = an xn et que converge
n=0 n≥0
(n + 1)r
absolument.
Montrer que :
1 +∞
an
Z X
(ln t)r−1 f (t) dt = (−1)r−1 (r − 1)! ·
0 n=0
(n + 1)r

I.F.
I.F.1) Déduire des questions précédentes que pour tout entier r ≥ 2 :
+∞
Hn (−1)r 1 ln(1 − t)
X Z
Sr = = (ln t)r−1 dt .
n=1
(n + 1)r (r − 1)! 0 1−t

(−1)r 1 (ln t)r−2 (ln(1 − t))2


Z
I.F.2) Établir que l’on a alors Sr = dt .
2(r − 2)! 0 t

1 1 (ln t)2
Z
I.F.3) En déduire que S2 = dt , puis trouver la valeur de S2 en fonction de ζ(3).
2 0 1−t

II. La fonction β
II.A. La fonction Γ

II.A.1) Soit x > 0. Montrer que t 7→ tx−1 e−t est intégrable sur ]0 ; +∞[ .
Z +∞
Dans toute la suite, on notera Γ la fonction définie sur R∗+ par Γ(x) = tx−1 e−t dt .
0
On admettra que Γ est de classe C ∞ sur son ensemble de définition, à valeurs strictement
positives et qu’elle vérifie, pour tout réel x > 0, la relation Γ(x + 1) = xΓ(x).
Z +∞
II.A.2) Soient x et α deux réels strictement positifs. Justifier l’existence de tx−1 e−αt dt et
0
donner sa valeur en fonction de Γ(x) et αx .

II.B. La fonction β et son équation fonctionnelle


Z 1
Pour (x, y) ∈ (R∗+ )2 , on définit β(x, y) = tx−1 (1 − t)y−1 dt .
0

II.B.1) Justifier l’existence de β(x, y) pour x > 0 et y > 0.

II.B.2) Montrer que pour tous réels x > 0 et y > 0, β(x, y) = β(y, x).
x
II.B.3) Soient x > 0 et y > 0. Établir que β(x + 1, y) = β(x, y).
x+y
xy
II.B.4) En déduire que pour x > 0 et y > 0, β(x + 1, y + 1) = β(x, y).
(x + y)(x + y + 1)

II.C. Relation entre la fonction β et la fonction Γ


Γ(x)Γ(y)
On veut montrer que pour x > 0 et y > 0, β(x, y) = , relation qui sera notée (R).
Γ(x + y)
II.C.1) Expliquer pourquoi il suffit de montrer la relation (R) pour x > 1 et y > 1.
Dans toute la suite de cette question, on supposera que x > 1 et y > 1.

2
+∞ ux−1
Z
II.C.2) Montrer que β(x, y) = du .
0 (1 + u)x+y
u
On pourra utiliser le changement de variable t = ·
1+u
II.C.3) On note Fx,y la primitive sur R+ de t 7→ e−t tx+y−1 qui s’annule en 0. Montrer que :

∀ t ∈ R+ , Fx,y (t) ≤ Γ(x + y) .

+∞ ux−1
Z

II.C.4) Soit G(a) = x+y
Fx,y (1 + u)a du .
0 (1 + u)
Montrer que G est définie et continue sur R+ .

II.C.5) Montrer que lim G(a) = Γ(x + y)β(x, y).


a→+∞

II.C.6) Montrer que G est de classe C 1 sur tout segment [c ; d] inclus dans R∗+ , puis que G est
de classe C 1 sur R∗+ .

II.C.7) Exprimer pour a > 0, G′ (a) en fonction de Γ(x), e−a et ay−1 .

II.C.8) Déduire de ce qui précède la relation (R).

III. La fonction digamma


On définit la fonction ψ (appelée fonction digamma) sur R∗+ comme étant la dérivée de
x 7→ ln(Γ(x)).
Γ′ (x)
Pour tout réel x > 0, ψ(x) = ·
Γ(x)
1
III.A. Montrer que pour tout réel x > 0, ψ(x + 1) − ψ(x) = ·
x
III.B. Sens de variation de ψ
∂β
III.B.1) À partir de la relation (R), justifier que est définie sur (R∗+ )2 .
∂y
∂β 
Établir que pour tous réels x > 0 et y > 0, (x, y) = β(x, y) ψ(y) − ψ(x + y) .
∂y
III.B.2) Soit x > 0 fixé. Quel est le sens de variations sur R∗+ de la fonction y 7→ β(x, y) ?

III.B.3) Montrer que la fonction ψ est croissante sur R∗+ .

III.C. Une expression de ψ comme somme d’une série de fonctions

III.C.1) Montrer que pour tout réel x > −1 et pour tout entier n ≥ 1 :
n 
1 1
X 
ψ(1 + x) − ψ(1) = ψ(n + x + 1) − ψ(n + 1) + − ·
k=1
k k+x

III.C.2) Soit n un entier ≥ 2 et x un réel > −1. On pose p = E(x) + 1, où E(x) désigne la partie
entière de x.
Prouver que :
p+1
0 ≤ ψ(n + x + 1) − ψ(n) ≤ Hn+p − Hn−1 ≤ ·
n
III.C.3) En déduire que, pour tout réel x > −1,
+∞
X 1 1

ψ(1 + x) = ψ(1) + − ·
n=1
n n+x

3
III.D. Un développement en série entière
On note g la fonction définie sur [−1 ; +∞[ par :
+∞
X 1 1

g(x) = − ·
n=2
n n+x

III.D.1) Montrer que g est de classe C ∞ sur [−1 ; +∞[ .


Préciser notamment la valeur de g(k) (0) en fonction de ζ(k + 1) pour tout entier k ≥ 1.

III.D.2) Montrer que pour tout entier n et pour tout x ∈ ]−1 ; 1[


n
g(k) (0)
xk ≤ ζ(2) |x|n+1 .
X
g(x) −
k=0
k!

Montrer que g est développable en série entière sur [−1 ; 1] .

III.D.3) Prouver que pour tout x dans ]−1 ; 1[ ,


+∞
X
ψ(1 + x) = ψ(1) + (−1)n+1 ζ(n + 1)xn .
n=1

IV. Une expression de Sr en fonction de valeurs entières de ζ


∂2β
Dans cette partie, on note B la fonction définie sur R∗+ par B(x) = (x, 1).
∂y 2
IV.A. Une relation entre B et ψ
Justifier que B est définie sur R∗+ .
À l’aide de la relation trouvée au III.B., établir que pour tout réel x > 0 :
2
+ ψ ′ (1) − ψ ′ (1 + x) .

xB(x) = ψ(1 + x) − ψ(1)

En déduire que B est C ∞ sur R∗+ .

IV.B. Expression de Sr à l’aide de la fonction B


Z 1 2
IV.B.1) Montrer que pour tout réel x > 0, B(x) = ln(1 − t) tx−1 dt .
0

IV.B.2) Donner sans justification une expression, à l’aide d’une intégrale, de B (p) (x), pour tout
entier naturel p et tout réel x > 0.
(−1)r
IV.B.3) En déduire que pour tout entier r ≥ 2, Sr = lim B (r−2) (x) .
2(r − 2)! x→0+
IV.B.4) Retrouver alors la valeur de S2 déjà calculée au I.F.3.
2
+ ψ ′ (1) − ψ ′ (1 + x) .

IV.C. Soit ϕ la fonction définie sur ]−1 ; +∞[ par ϕ(x) = ψ(1 + x) − ψ(1)
IV.C.1) Monter que ϕ est C ∞ sur son ensemble de définition et donner pour tout entier naturel
n ≥ 2 la valeur de ϕ(n) (0) en fonction des dérivées successives de ψ au point 1.

IV.C.2) Conclure que, pour tout entier r ≥ 3,


r−2
X
2Sr = rζ(r + 1) − ζ(k + 1)ζ(r − k) .
k=1

• • • FIN • • •

4
Centrale, 2015, MP, II
(10 pages)

Partie I

I.A - Z n  
1 dt 1  n 1 n
I.A.1) Pour n > 2, on a an =− = − ln(t) n−1 = − ln
n  t n
n−1  n  n −1
1 1 1 1 1 1
= + ln 1 − = + − − 2 +o
n n n n 2n n2
donc an ∼ − 1 2 donc, par comparaison à une série de Riemann, ( an )n>2 converge .
P
n→∞ 2n
n n
 Puisque ∀k > 2, ak = 1 − ln(k) + ln(k − 1), par télescopage, 1 − ln(n) = H − 1 − ln(n)
P P
I.A.2) ak = n
k k=2 k=2
k
Pn ∞
P P
donc Hn = ln(n) + 1 + ak = ln(n) + 1 + ak + o(1) puisque la série ( an )n>2 converge.
k=2 k=2
Ainsi ∃ A ∈ R, Hn = ln(n) + A + o(1) .
+∞

 On en déduit directement Hn ∼ ln(n) .


n→∞

I.B - Si r 6 1, on a (n + 1)r Hn r = Hn −−−→ +∞ donc, par comparaison à la série de Riemann divergente


   (n + 1) n→∞
P 1 P Hn
, diverge.
(n + 1)r (n + 1)r
 
Si r > 1, prenons s tel que r > s > 1, on a Hn = Hn 1 = o 1 car
(n + 1)r (n + 1)r−s (n + 1)s (n + 1)s
Hn ln(n)
r−s ∼ r−s −−−→ 0 puisque r − s > 0. Par comparaison à la série de Riemann conver-
(n + 1) n→∞ (n + 1)
  n→∞

gente
P 1 ,
P H n converge.
(n + 1)s (n + 1)r
 
P Hn
On conclut que : converge si et seulement si r > 1 .
(n + 1)r n>1

I.C -
+∞ n
X t
I.C.1)  ∀t ∈] − 1, 1[, ln(1 − t) = − (rayon de convergence R = 1) .
n=1
n
+∞
1 X
 ∀t ∈] − 1, 1[, = tn (rayon de convergence R = 1) .
1 − t n=0

I.C.2) Le produit de Cauchy des deux séries ci-dessus a donc un rayon de convergence supérieur ou égal à 1 et
on a ! +∞ ! !
+∞ n +∞ X n
ln(1 − t) X t X
n
X 1 n
∀t ∈] − 1, 1[, =− t =− t .
1−t n=1
n n=0 n=1
k
k=1

+∞
ln(1 − t) ln(1 − t)
Hn tn .
P
Donc t 7→ 1−t est développable en série entière sur ] − 1, 1[ et ∀t ∈] − 1, 1[, 1−t = −
n=1
Centrale, 2015, MP, II 2/10

I.D -

I.D.1) Pour (p, q) ∈ N 2 , t 7→ tp (ln t)q est continue sur ]0, 1] et t tp (ln t)q = tp+1/2 (ln t)q −−−→ 0 car p + 12 > 0,
  t→0+

soit tp (ln t)q =+ o √1 donc t 7→ tp (ln t)q est intégrable sur ]0, 1] et donc Ip,q existe pour tout (p, q) ∈ N 2 .
0 t

p+1
I.D.2) t 7→ pt + 1 et t 7→ (ln t)q sont de classe C 1 sur le segment [ε, 1] donc par intégration par parties,

1 1
tp+1 tp+1 (ln t)q−1
 Z
ε
Ip,q = (ln t)q − q dt
p+1 ε ε p+1 t

q εp+1
soit ∀p ∈ N, ∀q ∈ N ∗ , ∀ε ∈]0, 1[, ε
Ip,q =− ε
Ip,q−1 − (ln ε)q .
p+1 p+1

ε ε
I.D.3) Puisque p > 0 et q > 1, selon [1], Ip,q−1 −−−→ Ip,q−1 et Ip,q −−−→ Ip,q . De plus, εp+1 (ln ε)q −−−→ 0
ε→0+ ε→0+ ε→0+
q
donc, à la limite quand ε → 0+ , la formule du [2] donne Ip,q = − Ip,q−1 .
p+1

     Z 1
q q−1 1 q q!
I.D.4) On a donc Ip,q = − − ··· − I0,q = (−1) I0,q avec I0,q = tp dt =
p+1 p+1 p+1 (p + 1)q+1 0
1 q!
donc Ip,q = (−1)q .
p+1 (p + 1)q+1

+∞
On a ∀t ∈]0, 1[, (ln t)r−1 f (t) = an tn (ln t)r−1 . Posons, pout t ∈]0, 1[, un (t) = an tn (ln t)r−1 . On a :
P
I.E -
n=0
• ∀n ∈ N, un est continue sur ]0, 1[ et, d’après [2], un est intégrable sur ]0, 1], donc sur ]0, 1[, car r ∈ N ∗ ;
un converge simplement sur ]0, 1[ vers t 7→ (ln t)r−1 f (t) qui est continue sur ]0, 1[;
P
• selon ci-dessus,
Z 1 Z 1
|an |
• ∀n ∈ N, un (t) dt = |an | tn (ln t)r−1 dt = (−1)r−1 |an |In,r−1 = (r − 1)! et, par
0 0 (n + 1)r
   Z 1 
P |an | P
hypothèse, r converge donc un (t) dt converge .
(n + 1) 0
Le théorème d’intégration terme à terme s’applique et, sachant
Z 1
an
un (t) dt = an In,r−1 = (−1)r−1 (r − 1)! ,
0 (n + 1)r
Z 1 +∞
X an
on obtient : (ln t)r−1 f (t) dt = (−1)r−1 (r − 1)! .
0 n=0
(n + 1)r

I.F -
ln(1 − t)
I.F.1) Pour r > 2, on applique [E] à f : t 7→ 1 − t qu’on sait depuis [C.3] être développable en série entière
 ] − 1, 1[ avec
sur   comme coefficient
 de tn dans ce développement, an = −Hn . C’est légitime puisque
P |an | P Hn
= est alors convergente d’après [B].
(n + 1)r (n + 1)r
Z 1 +∞
ln(1 − t) X −Hn
On obtient donc (ln t)r−1 dt = (−1)r−1 (r − 1)! soit
0 1−t n=0
(n + 1)r

+∞ 1
(−1)r ln(1 − t)
Z
∗ Hn X
∀r ∈ N \ {1}, Sr = = (ln t)r−1 dt .
n=0
(n + 1)r (r − 1)! 0 1−t
Centrale, 2015, MP, II 3/10

I.F.2) Pour [c, d] ⊂]0, 1[, on a, par intégration par parties,


" 2 #d 2
d
ln(1 − t) ln(1 − t) r − 1 d ln(1 − t) (ln t)r−2
Z Z
r−1 r−1
(ln t) dt = − (ln t) + dt
c 1−t 2 2 c t
c
2 2 2
ln(1 − c) (ln c)r−1 ln(1 − d) (ln d)r−1 r − 1 d ln(1 − t) (ln t)r−2
Z
= − + dt.
2 2 2 c t
2 2 2
Or ln(1−c) (ln c)r−1 ∼ + c2 (ln c)r−1 −−−→ 0, ln(1−d) (ln d)r−1 ∼ − ln(1−d) (d−1)r−1 −−−→ 0
c→0 c→0 + d→1 d→1−
2
ln(1 − t) (ln t)r−2
car r − 1 > 0. Enfin ω : t 7→ t est continue sur ]0, 1[, ω(t) ∼ + t (ln t)r−2 −−−→ 0
t→0 t→0+
√ r−2
2 r−3/2
donc ϕ est prolongeable par continuité en 0 , 1 − t ω(t) ∼ − (−1) ln(1 − t) (1 − t) −−−→ 0
  t→1 t→1−

(car r > 3/2) donc ω(t) = o √ 1 et donc ω est intégrable sur ]0, 1[. Tout ceci justifie un passage
t→1− 1−t
à la limite pour (c, d) → (0, 1) et donne, avec le résultat du [1], la formule

1
2
(−1)r ln(1 − t) (ln t)r−2
Z

∀r ∈ N \ {1}, Sr = dt .
2(r − 2)! 0 t

21
ln(1 − t)
Z
1
I.F.3)  En particulier, S2 = dt. Effectuons dans cette intégrale le changement de variable
2 0 t
1 1 (ln u)2
Z
C 1 et strictement décroissant u = 1 − t, on obtient S2 = du .
2 0 1−u
 Appliquons le résultat du [E] à r = 3 et f : t 7→ 1 −1 qui est bien développable en série entière sur ]−1, 1[
t    
n
P |an | P 1
avec comme coefficient de t dans son développement, an = 1 donc = ,
(n + 1)3 (n + 1)3
Z 1 +∞
(ln u)2 X 1
série de Riemann convergente. On a donc du = 2 = 2ζ(3) et donc S2 = ζ(3) .
0 1−u n=0
(n + 1)3

Partie II

II.A -
II.A.1) γ : t 7→ tx−1 e−t est continue sur ]0, +∞[, γ(t) ∼ + tx−1 = 1−x 1 avec 1 − x < 1 et t2 tx−1 e−t −−−→ 0
  t→0 t t→+∞
1
donc γ(t) = o 2 donc t 7→ t x−1 −t
e est intégrable sur ]0, +∞[ .
t→+∞ t

II.A.2) Par le changement de variable C 1 et strictement croissant u = αt dans l’intégrale définissant Γ, on a


Z +∞ Z +∞ Z +∞
x−1 −t x−1 −αu
Γ(x) = t e dt = (αu) e α du = α x
ux−1 e−αu du
0 0 0

Z +∞ Z +∞
Γ(x)
donc ux−1 e−αu du existe et ux−1 e−αu du = .
0 0 αx

II.B -
Centrale, 2015, MP, II 4/10

II.B.1) Pour x > 0 et y > 0, φ : t 7→ tx−1 (1 − t)y−1 est continue sur ]0, 1[, φ(t) ∼ tx−1 =1 avec
t→0+ t1−x
1 − x < 1 et φ(t) ∼ (1 − t)y−1 = 1 avec 1 − y < 1 donc ψ est intégrable sur ]0, 1[ et donc
t→1− (1 − t)1−y
Z 1
2
∀(x, y) ∈ R ∗+ , β(x, y) = tx−1 (1 − t)y−1 dt existe .
0

Z 0
1
II.B.2) Le changement de variable C et strictement décroissant u = 1−t donne β(x, y) = (1−u)x−1 uy−1 (−du)
2 1
soit ∀(x, y) ∈ R ∗+ , β(x, y) = β(y, x) .

II.B.3) Par intégration par parties avec u(t) = tx et v 0 (t) = (1 − t)y−1 , on a, puisque tous les termes existent,

1 1
x 1 x−1
 x
t (1 − t)y
Z Z
x y−1
β(x + 1, y) = t (1 − t) dt = − + t (1 − t)y dt
0 y 0 y 0
x 1 x−1 x 1  x−1
Z Z 
= t (1 − t)y−1 (1 − t) dt = t (1 − t)y−1 − tx (1 − t)y−1 dt
y 0 y 0
x 
= β(x, y) − β(x + 1, y)
y

x+y x ∗ 2
 x
donc y β(x + 1, y) = y β(x, y) et donc ∀(x, y) ∈ R + , β(x + 1, y) = x + y β(x, y) .

y
II.B.4) Selon [3&2], β(x + 1, y + 1) = x + x x x
y + 1 β(x, y + 1) = x + y + 1 β(y + 1, x) = x + y + 1 x + y β(y, x) =
x y ∗ 2
 xy
x + y + 1 x + y β(x, y) donc ∀(x, y) ∈ R + , β(x + 1, y + 1) = (x + y)(x + y + 1) β(x, y) .

II.C -
II.C.1) Si la relation (R) est vraie pour x > 1 et y > 1, on a, selon [B.4]

2 (x + y)(x + y + 1) (x + y)(x + y + 1) Γ(x + 1)Γ(y + 1)


∀(x, y) ∈ R ∗+ , β(x, y) = β(x + 1, y + 1) =
xy (R) xy Γ(x + y + 2)
(x + y)(x + y + 1) xΓ(x) yΓ(y) Γ(x)Γ(y)
= =
xy (x + y + 1)(x + y)Γ(x + y) Γ(x + y)

donc si (R) est vraie pour x > 1 et y > 1 alors elle est vérifiée pour tout x > 0 et y > 0 .

u , θ est C 1 sur ]0, +∞[ avec ∀u > 0, θ0 (u) =


II.C.2) Soit θ(u) = 1 + 1 donc θ est strictement croissante
u (1 + u)2
sur ]0, +∞[ et on a lim θ(u) = 0 et lim θ(u) = 1. Ainsi le changement de variable t = θ(u) dans
u→0+ u→+∞
β(x, y) donne
Z +∞  x−1  y−1
u 1 du
β(x, y) =
0 1+u 1+u (1 + u)2
+∞
ux−1
Z
2
soit ∀(x, y) ∈ R ∗+ , β(x, y) = du .
0 (1 + u)x+y

II.C.3) Puisque x + y − 1 > 0, t 7→ e−t tx+y−1 est continue sur R + donc la primitive de cette fonction qui s’annule
Z t
en 0 est Fx,y : t 7→ e−u ux+y−1 du et, comme ∀u ∈ [0, +∞[, e−u ux+y−1 > 0 et que Γ(x + y) existe, on
0
a bien ∀t ∈ R + , Fx,y (t) 6 Γ(x + y) .

II.C.4) Soit g : (a, u) 7−→ ux−1 


x+y Fx,y (1 + u)a . On a :
(1 + u)
Centrale, 2015, MP, II 5/10

• ∀u ∈ [0, +∞[, a 7→ g(a, u) est continue sur R + car Fx,y est de classe C 1 sur R + ;
• ∀a ∈ R + , u 7→ g(a, u) est continue (par morceaux) sur [0, +∞[ car x − 1 > 0 et Fx,y de classe C 1 sur
R+;
• ∀a ∈ R + , ∀u ∈ [0, +∞[, g(a, u) 6 ux−1 Γ(x + y) d’après [3] et u 7→ ux−1 Γ(x + y) est
(1 + u)x+y (1 + u)x+y
intégrable sur [0, +∞[ selon [2].
Le théorème de continuité des intégrales à paramètre donne que G est définie et continue sur R + .

II.C.5) On peut appliquer le théorème de convergence dominée pour a → +∞ car:


• ∀u ∈ [0, +∞[, a(1 + u) −−−→ +∞ donc ∀u ∈ [0, +∞[, g(a, u) −−−→ ux−1 Γ(x + y);
a→+∞ a→+∞ (1 + u)x+y
• u 7→ ux−1 Γ(x + y) est continue (par morceaux) sur [0, +∞[;
(1 + u)x+y
• on a la domination vue au [4]: ∀a ∈ R + , ∀u ∈ [0, +∞[, g(a, u) 6 ux−1 Γ(x + y) avec
(1 + u)x+y
u 7→ ux−1 Γ(x + y) intégrable sur [0, +∞[.
(1 + u)x+y
Z +∞
ux−1
Donc G(a) −−−→ Γ(x + y) du soit, avec [2], lim G(a) = Γ(x + y) β(x, y) .
a→+∞ 0 (1 + u)x+y a→+∞

II.C.6)  Avec la notation du [4] et [c, d] ⊂ R ∗+ :


• ∀a ∈ R + , u 7→ g(a, u) est continue (par morceaux) et intégrable sur [0, +∞[;
• ∀a ∈ R + , ∀u ∈ [0, +∞[,
∂g
(a, u) = ux−1 0
(1+u) Fx,y

(1+u)a = ux−1 (1+u) e−(1+u)a (1+
∂a (1 + u)x+y (1 + u)x+y
x+y−1
u)a = ax+y−1 e−(1+u)a ux−1 existe;
∂g
• ∀u ∈ [0, +∞[, a 7→ (a, u) est continue sur R + car x + y − 1 > 0;
∂a
∂g
• ∀a ∈ R + , u 7→ (a, u) est continue (par morceaux) sur [0, +∞[ car x − 1 > 0;
∂a
∂g
• ∀a ∈ [c, d], ∀u ∈ [0, +∞[, (a, u) 6 dx+y−1 e−(1+u)c ux−1 = dx+y−1 e−c e−c u ux−1 et u 7→ e−c u ux−1
∂a
est intégrable sur ]0, +∞[ selon [A.2] puisque c > 0.
Ainsi le théorème de dérivation des intégrales à paramètre donne que G est de classe C 1 sur [c, d] .
 Ceci étant valable pour tout [c, d] ⊂ R ∗+ , on a G de classe C 1 sur R ∗+ .

II.C.7) De plus, la formule de dérivation donne, dans ce cas,


Z +∞ Z +∞
∂g
∀a ∈ R ∗+ , G0 (a) = (a, u) du = ax+y−1 e−a e−a u ux−1 du
0 ∂a 0

soit, avec le résultat de [A.2], ∀a ∈ R ∗+ , G0 (a) = Γ(x) ay−1 e−a .

2
Z a Z a
0
II.C.8) On a donc ∀(a, a ) ∈ R ∗+ , G(a) − G(a ) = 0 0
G (t) dt = Γ(x) ty−1 e−t dt. Or G(a) −−−→ Γ(x +
a0 a0 a→+∞
y) β(x, y) d’après [5], G(a0 ) −− −→ G(0) car G est continue en 0 selon [4] ce qui donne G(a0 ) −− −→ 0
a0 →0
Z a a0 →0

car Fx,y (0) = 0. D’autre part, ty−1 e−t dt −−−→


0
Γ(y) donc, en passant à la limite, on obtient
a0 (a,a )→(+∞,0)
Γ(x) Γ(y)
Γ(x + y) β(x, y) − 0 = Γ(x) Γ(y), or Γ(x + y) 6= 0, donc ∀x > 1, ∀y > 1, β(x, y) = .
Γ(x + y)

Partie III

 
  Γ(x + 1) 1 .
III.A - ∀x > 0, ln Γ(x+1) −ln Γ(x) = ln = ln x donc, en dérivant, ∀x > 0, ψ(x + 1) − ψ(x) = x
Γ(x)
Centrale, 2015, MP, II 6/10

III.B -
III.B.1)  Selon les résultats admis au [II], Γ est de classe C ∞ sur ]0, +∞[ et ne s’y annule pas donc ∀x > 0, y 7→
Γ(x) Γ(y) 2 ∂β
= β(x, y) est de classe C ∞ sur ]0, +∞[. En particulier, ∀(x, y) ∈ R ∗+ , (x, y) existe .
Γ(x + y) ∂y
2 ∂β Γ(x) Γ0 (y) Γ(x) Γ(y) Γ0 (x + y)
 On a ∀(x, y) ∈ R ∗+ , (x, y) = − 2
∂y Γ(x + y) Γ(x + y) 
Γ(x) Γ(y) Γ (y) Γ0 (x + y)
 0
= −
Γ(x + y) Γ(y) Γ(x + y)
∗ 2 ∂β
 
donc ∀(x, y) ∈ R + , (x, y) = β(x, y) ψ(y) − ψ(x + y) .
∂y

0
III.B.2) Si y < y 0 , ∀t ∈]0, 1[, tx (1 − t)y > tx (1 − t)y donc β(x, y) > β(x, y 0 ). Ainsi y 7→ β(x, y) décroı̂t sur R ∗+ .

2 ∂β 2
III.B.3) En conséquence, ∀(x, y) ∈ R ∗+ , (x, y) 6 0. De plus, ∀(x, y) ∈ R ∗+ , β(x, y) > 0 donc, grâce à la
∂y
2
formule du [1], ∀(x, y) ∈ R ∗+ , ψ(y) 6 ψ(x + y) ce qui montre que ψ est croissante sur R ∗+ .

III.C -
n 
P 
III.C.1) Pour tout x > −1 et n > 1, par télescopage, ψ(x + n + 1) − ψ(x + 1) = ψ(x + k + 1) − ψ(x + k) =
k=1
n n
P 1 d’après [A]. En particulier, pour x = 0, ψ(n + 1) − ψ(1) = P 1 donc, en soustrayant membre
k=1
x+k k=1
k
à membre,
n  
X 1 1
ψ(n + 1) − ψ(1) − ψ(x + n + 1) + ψ(x + 1) = −
k x+k
k=1
n  
X 1 1
soit ∀x > −1, ∀n > 2, ψ(x + 1) − ψ(1) = ψ(x + n + 1) − ψ(n + 1) + − .
k x+k
k=1

III.C.2) On a 0 < x + 1 6 p + 1 donc, par croissance de ψ,


n+p
X   n+p
X1
0 6 ψ(x + n + 1) − ψ(n) 6 ψ(p + n + 1) − ψ(n) = ψ(k + 1) − ψ(k) =
k
k=n k=n

n+p
P 1 =H 1 6 1 , on obtient
et n+p − Hn−1 . De plus, puisque ∀k ∈ [[n, n + p]], n
k=n
k k

p+1
0 6 ψ(x + n + 1) − ψ(n) 6 Hn+p − Hn−1 6 .
n

P 1 
III.C.3) Pour x > −1, la série − 1 est convergente car 1 − 1 = x ∼ x . De plus,
k>1
k x+k k x+k k(k + x) k→+∞ k 2
p+1
s x > −1 étant fixé donc p = E(x) + 1 également, on a n −−−→ 0 donc l’inégalité du [2], donne
n→∞
ψ(x + n + 1) − ψ(n) −−−→ 0. En particulier, ψ(n + 1) − ψ(n) −−−→ 0. Or l’égalité du [1] donne
n→∞ n→∞

n  
  X 1 1
ψ(x + 1) = ψ(x + n + 1) − ψ(n) − ψ(n + 1) − ψ(n) +ψ(1) + −
k x+k
k=1
| {z } | {z }
−−−→0 −−−→0 | {z }
n→∞ n→∞
converge

+∞  
X 1 1
donc ∀x > −1, ψ(x + 1) = ψ(1) + − .
k x+k
k=1
Centrale, 2015, MP, II 7/10

III.D -
1 − 1 . On a :
III.D.1)  Posons, pour n > 2, vn (x) = n x+n
(k) (−1)k+1 k!
• ∀n ∈ N ∗ \{1}, vn est de classe C ∞ sur ]−n, +∞[ donc sur [−1, +∞[ avec ∀k > 1, vn (x) = ;
(x + n)k+1
• ∀x > −1, vn (x) ∼ x P
donc la série ( vn )n>2 converge simplement sur [−1, +∞[;
n→+∞ n2 P 
(k) [−1,+∞[ k! (k)
• Pour k > 1, vn ∞ = k+1 donc, puisque k + 1 > 1, la série vn converge
(n − 1) n>2
normalement donc uniformément sur [−1, +∞[.
Donc le théorème de dérivation terme à terme s’applique et g est de classe C ∞ sur [−1, +∞[ .
+∞
X (k) (−1)k+1 k!
 Et ∀x ∈ [−1, +∞[, ∀k ∈ N ∗ , g (k) (x) = vn(k) (x). Or vn (0) = donc g (k) (0) =
nk+1
n=2
+∞
(−1)k+1 k!
P 1 soit ∀k ∈ N ∗ , g (k) (0) = (−1)k+1 k! ζ(k + 1) − 1 .

k+1
n=2 n

III.D.2)  La formule de Taylor avec reste intégral à l’ordre n en 0 s’écrit, pour x ∈ [−1, +∞[,
n 1
g (k) (0) xn+1
X Z
g(x) − xk = (1 − t)n g (n+1) (t x) dt
k! n! 0
k=0

donc
n 1
g (k) (0) |x|n+1
X Z
∀x > −1, g(x) − xk 6 (1 − t)n g (n+1) (t x) dt.
k! n! 0
k=0

+∞ +∞
Or ∀t ∈ [0, 1], ∀x > −1, g (n+1) (t x) = (−1)n+2 (n + 1)! 1 P P 1
n+2 6 (n + 1)! n+2 car,
p=2 (p + t x) p=2 (p − 1)

pour tout p > 2, 0 < p − 1 6 p + t x et, d’autre part, ∀p > 2, 1 6 1 car n > 0. On
(p − 1)n+2 (p − 1)2
obtient donc :
n 1
g (k) (0) |x|n+1
X Z h i1
∀x > −1, g(x) − xk 6 (1 − t)n (n + 1)!ζ(2) dt = |x|n+1 ζ(2) −(1 − t)n+1
k! n! 0 0
k=0

n
X g (k) (0)
et donc ∀x > −1, g(x) − xk 6 ζ(2) |x|n+1 .
k!
k=0

 Mais, si x ∈] − 1, 1[, ζ(2) |x|n+1 −−−→ 0 donc l’inégalité ci-dessus implique que
n→+∞

n
X g (k) (0)
∀x ∈] − 1, 1[, xk −−−→ g(x)
k! n→+∞
k=0

et donc g est développable en série entière sur ] − 1, 1[ .

III.D.3) Avec l’égalité vue au [C.3], o,n obtient

+∞    
X 1 1 1
∀x ∈] − 1, 1[, ψ(x + 1) = ψ(1) + − = ψ(1) + 1 − + g(x)
k x+k x+1
k=1
+∞
X g (n) (0)
x
= ψ(1) + + xn
x + 1 n=0 n!
+∞
X +∞
X
(−1)n+1 xn + g(0) + (−1)n+1 ζ(n + 1) − 1 xn

= ψ(1) + selon [1]
n=1 n=1
Centrale, 2015, MP, II 8/10

+∞
X
ce qui donne bien ∀x ∈] − 1, 1[, ψ(x + 1) = ψ(1) + (−1)n+1 ζ(n + 1) xn .
n=1

Remarque: Une autre démonstration de cette formule est possible à l’aide d’une famille sommable.

Partie IV

IV.A -  Comme on l’a signalé au [III.B.1], pour tout x > 0, y 7→ β(x, y) est de classe C ∞ sur R ∗+ . Notamment
∂2β
∀x > 0, B(x) = (x, 1) existe .
∂y 2
2 ∂β
 Reprenons l’égalité vue au [III.B.1]: ∀(x, y) ∈ R ∗+ ,

(x, y) = β(x, y) ψ(y) − ψ(x + y) . On en
∂y
déduit :

2 ∂2β ∂β
∀(x, y) ∈ R ∗+ , 0 0
 
2 (x, y) = ∂y (x, y) ψ(y) − ψ(x + y) + β(x, y) ψ (y) − ψ (x + y)
∂y
2
= β(x, y) ψ(y) − ψ(x + y) + β(x, y) ψ 0 (y) − ψ 0 (x + y) .


Z 1
1 2
tx−1 dt = donc ∀x > 0, xB(x) = ψ(1) − ψ(x + 1) + ψ 0 (1) − ψ 0 (x + 1) .

Or β(x, 1) =
0 x
 Puisque Γ est C ∞ sur R ∗+ et ne s’y annule pas, ψ est C ∞ sur R ∗+ donc B est de classe C ∞ sur R ∗+ .

IV.B -
 
IV.B.1) Soit x > 0 fixé. Posons h(y, t) = tx−1 (1 − t)y−1 = tx−1 exp (y − 1) ln(1 − t) . On a :
• ∀y ∈ R ∗+ , t 7→ h(y, t) est continue (par morceaux) et intégrable sur ]0, 1[ d’après [II.B.1];
2
• ∀(y, t) ∈ R ∗+ ×]0, 1[, ∂h (y, t) = ln(1−t)tx−1 (1−t)y−1 et ∂ h2 (y, t) = ln(1−t) tx−1 (1−t)y−1 existent;
2
∂y ∂y
2
• pour tout t ∈]0, 1[, y 7→ ∂h (y, t) et y 7→ ∂ h2 (y, t) sont continues sur R ∗+ ;
∂y ∂y
• pour tout y ∈ R ∗+ , t 7→ ∂h (y, t) est continue sur ]0, 1[, ∂h (y, t) ∼ tx avec x > 0 et ∂h (y, t) =
  ∂y ∂y t→0+ ∂y t→1−

o √1 car (1 − t)y−1/2 ln(1 − t) −−−→ 0 donc t 7→7→ ∂h (y, t) est intégrable sur ]0, 1[;
1−t t→1− ∂y
2
• pour tout y ∈ R ∗+ , t 7→ ∂ h2 (y, t) est continue (par morceaux) sur ]0, 1[;
∂y
• soit c tel que 0 < c < 1, on a la domination pour y ∈ [c, +∞[ :

∂2h 2 x−1 2
∀t ∈]0, 1[, 2 (y, t) = ln(1 − t) t (1 − t)y−1 6 ln(1 − t) tx−1 (1 − t)c−1 = H(t)
∂y
 
avec H continue sur ]0, 1[, H(t) ∼ + tx+1 , H(t) = − o √1 donc H intégrable sur ]0, 1[.
t→0 t→1 1−t
Ainsi le théorème de dérivation des intégrales à paramètre s’applique et y 7→ β(x, y) est de classe C 2
sur [c, +∞[ et sa dérivée seconde est donnée par dérivation sous l’intégrale. Ceci donne, en y = 1,
Z 1
2
∀x > 0, B(x) = ln(1 − t) tx−1 dt .
0

Z 1 2 p
(p)
IV.B.2) De la même façon, on obtiendrait ∀p ∈ N, ∀x > 0, B (x) = ln(1 − t) ln(t) tx−1 dt .
0
Centrale, 2015, MP, II 9/10

1
2
(−1)r ln(1 − t) (ln t)r−2
Z
IV.B.3) Selon [I.F.2], ∀r > 2, Sr = dt. Mais, on a ∀r > 2, ∀x > 0, B (r−2) (x) =
2(r − 2)! 0 t
Z 1
2 r−2 x−1
ln(1 − t) ln(t) t dt et
0
2
2 r−2 x−1 ln(1 − t) (ln t)r−2
• ∀t ∈]0, 1[[, ln(1 − t) ln(t) t −−−→ t ;
x→0+
2
ln(1 − t) (ln t)r−2
• t 7→ t est continue (par morceaux) sur ]0, 1[;
2 r−2
2 r−2 x−1 ln(1 − t) ln t
• on a la domination ∀x ∈ R ∗+ , ∀t ∈]0, 1[[, ln(1 − t) ln(t) t 6 t et cette
fonction dominante est intégrable sur ]0, 1[ d’après [I.F.2]
En appliquant le théorème de convergence dominée pour x → 0+ , on obtient

1
2
ln(1 − t) (ln t)r−2
Z
(r−2)
B (x) −−−→ dt
+
x→0 0 t

(−1)r
et donc Sr = lim B (r−2) (x) .
2(r − 2)! x→0+

IV.B.4) Notamment, S2 = 1 lim+ B(x) et la formule du [A] jointe au développement trouvé au [III.3] donne
2 x→0

1h 2 i
B(x) = ψ(1) − ψ(x + 1) + ψ 0 (1) − ψ 0 (x + 1)
x
1h i
=+ ψ(1) − ψ(1) − ζ(2)x + o(x))2 + ζ(2) − ζ(2) + 2ζ(3)x + o(x)
x→0 x
= + 2ζ(3) + o(1)
x→0

ce qui redonne bien S2 = ζ(3) .

IV.C -

IV.C.1)  ψ est de classe C ∞ sur R ∗+ donc ϕ est de classe C ∞ sur ] − 1, +∞[ .


 La formule de Leibniz donne, pour tout x ∈] − 1, +∞ [ et tout n > 2,

n−1
X n
(n)
 (n)
ψ (k) (x+1)ψ (n−k) (x+1)+ψ (n) (x+1) ψ(x+1)−ψ(1) −ψ (n+1) (x+1)

ϕ (x) = ψ(x+1)−ψ(1) ψ (x+1)+
k
k=1

n−1
X 
n (k)
donc ∀n > 2, ϕ(n) (0) = ψ (1)ψ (n−k) (1) − ψ (n+1) (1) .
k
k=1

IV.C.2) Selon [A], B est C ∞ sur ]0, +∞[ et , selon [B.3], ∀p ∈ N, B (p) (x) −−−→
+
2(−1)p p!Sp+2 . Le théorème
x→0
de prolongement C ∞ donne que B se prolonge en une fonction Be C ∞ sur [0, +∞[ et telle que ∀p ∈
(p) p e admet donc un développement limité à l’ordre r − 2 en 0 donné par la
N, Be (0) = 2(−1) p!Sp+2 . B
formule de Taylor-Young:

r−2
X
2(−1)p Sp+2 xp + o xr−2 .

B(x)
e =
x→0
p=0
Centrale, 2015, MP, II 10/10

"r−1 # r−2
ϕ(x) 1 X ϕ(q) (0) q r−1
X ϕ(p+1) (0)
xp + o xr−2 car
 
Mais ∀x > 0, B(x)
e = B(x) = = x +o x =
x x→0+ x q=0 q! p=0
(p + 1)!
ϕ(p+1) (0)
ϕ(0) = 0. Par unicité du développement limité, on a donc ∀p ∈ [[0, r − 2]], 2(−1)p Sp+2 = .. En
(p + 1)!
ϕ(r−1) (0)
particulier, ∀r > 2, 2Sr = (−1)r . Et, pour r > 3, la formule du [1] donne
(r − 1)!
"r−2  #
(−1)r X r − 1 (k)

(r−1−k) (r)
2Sr = ψ (1)ψ (1) − ψ (1) .
(r − 1)! k
k=1

D’autre part, le développement en série entière trouvé au [III.D.3] donne ψ (n) (1) = (−1)n+1 n!ζ(n + 1)
pour tout n ∈ N ∗ . On a donc
"r−2  #
(−1)r X r − 1

k+1 r−k r+1
2Sr = (−1) k!ζ(k + 1) (−1) (r − 1 − k)!ζ(r − k) − (−1) r!ζ(r + 1)
(r − 1)! k
k=1
"r−2 #
(−1)r X (r − 1)! r+1 r+1
= (−1) k!(r − 1 − k)!ζ(k + 1)ζ(r − k) − (−1) r!ζ(r + 1)
(r − 1)! k!(r − 1 − k)!
k=1

r−2
X
soit 2Sr = rζ(r + 1) − ζ(k + 1)ζ(r − k) .
k=1

* * *
* *
*
Concours Centrale-Supélec
Mathématiques 1 - MP - 2015
4 heures - Calculatrices autorisées

Autour de la transformation de Radon


L’objectif de ce problème est l’étude d’un certain opérateur intégral agissant sur les fonctions du plan, appelé
transformation de Radon. On se propose d’établir une formule d’inversion et d’interpréter la transformation en
termes de fonctions invariantes sur un groupe de matrices. Enfin on étudiera une application du procédé dans
le domaine de l’imagerie médicale.
Notations
On note R le corps des nombres réels et M3 (R) l’algèbre des matrices carrées de taille 3 × 3 à coefficients dans
R. Le groupe multiplicatif des éléments inversibles de M3 (R) est noté GL3 (R) et son élément  I3 .
 neutre,
cos θ
Les éléments du plan vectoriel R2 seront notés en colonne, pour tout réel θ on notera ~uθ = .
sin θ
Le plan est muni de sa structure euclidienne canonique, donnée par le produit scalaire
   
x1 x
, 2 = x1 x2 + y1 y2
y1 y2

L’ensemble des matrices des rotations vectorielles


 planes
 est appelé groupe spécial orthogonal et noté SO(2),
cos θ − sin θ
son élément neutre, I2 . On écrira Rθ = .
sin θ cos θ

I Préliminaires géométriques
 
b1
A
Soit G le sous-ensemble de M3 (R) des matrices de la forme M (A, ~b) =  b2  où A est un élément
0 0 1
 
b
du groupe spécial orthogonal SO(2) et ~b = 1
est un vecteur quelconque du plan euclidien R2 .
b2

I.A - Isométries affines directes du plan euclidien


I.A.1) Déterminer un couple (A, ~b) dans SO(2) × R2 tel que l’on ait M (A, ~b) = I3 .
I.A.2) Soit (A, ~b) et (A0 , ~b0 ) dans SO(2) × R2 . Montrer que M (A, ~b)M (A0 , ~b0 ) = M (AA0 , A~b0 + ~b)
I.A.3) Montrer que les éléments de G sont inversibles et expliciter l’inverse de M (A, ~b).
I.A.4) Démontrer que G est un sous-groupe de GL3 (R) .
→ R2

G
I.A.5) L’application Φ : est-elle surjective ? Est-elle injective ?
M (A, b) 7→ ~b
~

I.B - Droites affines du plan


 
u1
Pour q ∈ R et ~u = vecteur unitaire de R2 , on note ∆(q, ~u) la droite affine du plan passant par le point
u2
(qu1 , qu2 ) et orthogonale à ~u .
 
~e1 + ~e2
I.B.1) Représenter graphiquement ∆(~0, ~e1 ) et ∆ 2, √ .
2
I.B.2) Déterminer une équation cartésienne de ∆(q, ~uθ ).

x(t) = q cos θ − t sin θ
I.B.3) Montrer qu’une paramétrisation de ∆(q, ~uθ ) est donnée par lorsque t parcourt
y(t) = q sin θ + t cos θ
R.
I.B.4) À quelle condition les droites ∆(q, ~u) et ∆(r, ~v ) sont-elles confondues ?

1
I.C - Action de G sur les droites

G → D
On note D l’ensemble des droites affines du plan et on considère l’application Ψ : .
M (A, ~b) 7→ ∆(hA~e1 , ~bi, A~e1 )
 
  1
I.C.1) Représenter Ψ M (A, ~b) dans le cas A = Rπ/6 et ~b = .
2
I.C.2) Déterminer Ψ M (I2 , ~0) .


I.C.3) Vérifier que Ψ(M (Rθ , q~uθ )) = ∆(q, ~uθ ) ; en déduire que Ψ est surjective.
I.C.4) Soit H l’ensemble des matrices M (A, ~b) de G telles que Ψ(M (A, ~b)) = ∆(~0, ~e1 ).
a) Décrire les éléments de H .
b) Montrer que H est un sous-groupe de G .
c) Montrer que pour tout g de G , et tout h de H , on a Ψ(gh) = Ψ(g).
Pour tout entier n, on note Bn l’ensemble des fonctions f de classe C 1 sur R2 à valeurs dans R telles que
(x, y) 7→ (x2 + y 2 )n f (x, y) est bornée sur R2 .
Si f est une fonction continue sur R2 on appelle transformée de Radon de f la fonction fˆ définie, là où c’est
possible, par
Z +∞
fˆ(q, θ) = f (q cos θ − t sin θ, q sin θ + t cos θ)dt
−∞

II Fonctions radicales
II.A - Étude d’un exemple
1
On considère, dans cette sous-partie seulement, la fonction f définie par : ∀(x, y) ∈ R2 , f (x, y) =
1 + x2 + y 2
II.A.1) Établir que f est dans B1 .
π
II.A.2) Montrer que fˆ est définie sur R2 avec fˆ(q, θ) = p .
1 + q2
Z 2π
1 R0 (q)
II.A.3) On pose R(q) = fˆ(q, θ) dθ. Démontrer que q 7→ est intégrable sur ]0, +∞[ et que
2π 0 q
+∞
R0 (q)
Z
1
f (0, 0) = − dq
π 0 q

On pourra, pour calculer cette dernière intégrale, procéder au changement de variable q = sh (u).
∂f
II.A.4) La fonction est-elle dans B2 ?
∂x

II.B - Fonctions radicales : cas général


On suppose dans le reste de cette partiepqu’il existe une fonction ϕ de R+ vers R, continue et intégrable sur
R+ , telle que : ∀(x, y) ∈ R2 , f (x, y) = ϕ( x2 + y 2 ).
Z 2π
1
II.B.1) Pour r ∈ R+ , calculer f¯(r) = f (r cos t, r sin t) dt.
2π 0
Z +∞
rϕ(r)
II.B.2) Justifier la convergence, pour tout réel q > 0, de p dr.
q r2 − q2
II.B.3) Démontrer que la transformée de Radon de f est définie sur R2 et que
Z +∞
rϕ(r)
∀q ∈ R+ , ∀θ ∈ R fˆ(q, θ) = 2 p dr.
q r2 − q2

2π +∞
rf¯(r)
Z Z
1
II.B.4) En déduire que ∀q ∈ R+ , fˆ(q, θ) dθ = 2 p dr.
2π 0 q r2 − q2

2
III Transformée de Radon d’une fonction de B1
On considère dans cette partie une fonction f appartenant à B1 et on rappelle que
Z +∞
fˆ(q, θ) = f (q cos θ − t sin θ, q sin θ + t cos θ)dt
−∞

III.A -
Vérifier que fˆ est définie sur R2 .

III.B -
Justifier que pour tout q et tout θ on a fˆ(−q, θ + π) = fˆ(q, θ).

III.C -
Z 2π
1
On pose encore f¯(r) = f (r cos t, r sin t) dt.
2π 0
III.C.1) Démontrer que f¯ est de classe C 1 sur R.
III.C.2) Démontrer que la fonction r 7→ r2 f¯(r) est bornée sur R.
∂f ∂f
III.C.3) Montrer que si on suppose de plus que et sont dans B2 , alors r 7→ r4 f¯0 (r) est bornée sur R.
∂x ∂y
Sous ces hypothèses, on peut démontrer en manipulant des intégrales doubles que la formule du II.B.4 reste
vraie. Nous admettrons donc dans la suite que
2π +∞
rf¯(r)
Z Z
1
∀q ∈ R+ , fˆ(q, θ) dθ = 2 p dr.
2π 0 q r2 − q2

IV Formule d’inversion
On souhaite retrouver la fonction f à partir de sa transformée fˆ. À cet effet on pose pour (x, y) ∈ R2 ,
Z 2π
1
Rx,y (q) = fˆ(x cos θ + y sin θ + q, θ) dθ
2π 0

+∞ 0
−1 Rx,y (q)
Z
L’objectif est de démontrer la formule d’inversion de Radon : ∀(x, y) ∈ R2 , f (x, y) = dq.
π 0 q

IV.A - Résultats préliminaires


Z +∞
dt π
IV.A.1) Justifier l’existence de l’intégrale √
et montrer que sa valeur est .
1
2
t t −1 2
IV.A.2) Soient ε et r fixés tels que 0 < ε < r. Avec le changement de variables q = r cos θ, établir que
Z r √
dq r 2 − ε2
=
r2 ε
p
2 r2 − q2
ε q

IV.B - Étude d’une fonction définie par une intégrale


Z +∞
th(qt)
Soit h une fonction de classe C 1 sur R+ . On suppose que r 7→ r2 h(r) est bornée et on pose H(q) = √ dt
1 t2 − 1
IV.B.1) Montrer que H est continue sur ]0, +∞[.
 
1
IV.B.2) Montrer qu’au voisinage de +∞ on a H(q) = O
q2
IV.B.3) Démontrer que si on suppose de plus que r 7→ r4 h0 (r) est bornée, alors la fonction H est de classe C 1 sur
]0, +∞[.

3
IV.C - Vers la formule d’inversion
∂f ∂f
On considère une fonction f de B1 dont les dérivées partielles et sont dans B2 .
∂x ∂y
rf¯(r)
Z 2π Z +∞
1
On pose, avec les notations de la partie III : ∀q ∈ R+ , F (q) = fˆ(q, θ)dθ = 2 p dr.
2π 0 q r2 − q2
 
1 1
IV.C.1) Justifier que F est de classe C sur ]0, +∞[ et qu’au voisinage de +∞ on a F (q) = O .
q
!
rf¯(r)
Z +∞ 0 Z +∞ Z +∞
F (q) F (ε) 1
IV.C.2) Démontrer : ∀ε > 0, dq = − +2 dr dq.
q2
p
ε q ε ε q r2 − q2
IV.C.3) On admet que l’on peut intervertir les deux intégrales ci dessus et donc que
! !
rf¯(r) rf¯(r)
Z +∞ Z +∞ Z +∞ Z r
1
∀ε > 0, dr dq = dq dr.
q2 q
p p
ε r2 − q2 ε ε q
2 r2 − q2
f¯(r)
Z +∞ 0 Z +∞
F (q)
En déduire que ∀ε > 0, dq = −2ε √ dr.
ε q ε r r 2 − ε2

IV.D - La formule d’inversion


∂f ∂f
IV.D.1) On considère une fonction f dans B1 telle que et sont dans B2 .
∂x ∂y
IV.D.2) Établir la formule d’inversion de Radon pour cette fonction f au point (x, y) = (0, 0).
IV.D.3) Les hypothèses faites sur f sont-elles nécessaires pour que la formule d’inversion de Radon soit vérifiée
au point (x, y) = (0, 0) ?
IV.D.4) Proposer une démarche pour obtenir la formule d’inversion de Radon en tout couple (x, y) à partir de la
formule en (0, 0).

V Interprétation et application à la radiographie


La transformation de Radon peut être introduite dans le cadre plus général de l’analyse sur les groupes. Le but
de cette partie est d’interpréter l’opérateur étudié plus haut en termes de fonctions invariantes sur G

V.A - Fonction invariante sur G


Si f est une fonction définie sur R2 , on note f ∗ la fonction f ◦ Φ, définie sur G par f ∗ (g) = f (Φ(g)) où
Φ : G → R2 est la fonction introduite dans la question I.A.5.
V.A.1) Démontrer que pour tout g dans G et r tel que Φ(r) = ~0 on a f ∗ (gr) = f ∗ (g).
On dit alors que f ∗ est invariante par les rotations du plan, vues comme des éléments de G .
V.A.2) On suppose à présent que f vérifie les hypothèses permettant de définir sa transformée de Radon et
on va démontrer que fˆ peut également être vue comme une fonction sur G , sujette à un autre type
d’invariance.
Démontrer que si deux droites ∆(q1 , ~uθ1 ) et ∆(q2 , ~uθ2 ) coïncident, alors fˆ(q1 , θ1 ) = fˆ(q2 , θ2 ).
Ce résultat permet de faire l’abus de notation fˆ(∆(q, ~uθ )) = fˆ(q, θ)) sans qu’il en résulte d’ambiguïté.
V.A.3) On définit à présent fˆ∗ sur G en composant fˆ par Ψ : on pose, pour tout g ∈ G , fˆ∗ (g) = fˆ(Ψ(g)).
Démontrer que fˆ∗ est H-invariante, c’est-à-dire que pour tous g ∈ G et h ∈ H, fˆ∗ (gh) = fˆ∗ (g) .

V.B - Reconstruction en radiographie


Une technique courante en imagerie médicale consiste à mesurer l’intensité d’un faisceau de rayons X avant et
après la traversée d’une certaine zone, l’objectif étant de déterminer la densité des tissus dans la zone. L’objectif
des dernières questions du problème est d’illustrer, dans un modèle de dimension deux, comment la formule
d’inversion de Radon peut être utilisée dans ce cadre.
On modélise la densité des tissus, exprimée dans des unités convenables, par une fonction inconnue f nulle en
dehors de la zone à étudier et dont on suppose qu’elle vérifie des hypothèses assurant l’existence de f .

4
En supposant que chaque faisceau de rayons X incident est porté par une droite affine ∆ , et en notant I son
intensité mesurée de part et d’autre de la zone visée, un raisonnement heuristique donne
  Z

ln = f
Is ∆

où le membre de droite désigne l’intégrale de f sur la droite ∆ dans un sens à préciser.


V.B.1) Proposer une définition rigoureuse du membre de droite de (V.1) dans le cas où ∆ = ∆(q, ~uθ ).
V.B.2) Expliquer comment la formule d’inversion de Radon permet en principe de connaître la densité des tissus
dans la zone radiographiée.

5
Concours Centrale-Supélec

Mathématiques 1 - MP - 2015

I Préliminaires géométriques
I.A - Isométries affines directes du plan euclidien
 
0
I.A.1) Il suffit de prendre A = I2 et ~b =
0
I.A.2) Résultat immédiat en faisant un produit par blocs.
I.A.3) A est inversible. En prenant A0 = A−1 et ~b0 = −A−1~b dans la formule de la question 2), on obtient
M (A, ~b).M (A0 , ~b0 ) = I3 .
Tout élément M (A, ~b) de G est donc inversible et M (A, ~b)−1 = M (A−1 , −A−1~b)
I.A.4) D’après A.3), G ⊂ GL3 (R).
D’après A.2), SO(2) étant stable pour le produit, G est stable pour le produit.
D’après A.3), SO(2) étant stable pour l’inversion, G est stable pour l’inversion.
G est donc un sous-groupe de GL3 (R)

I.A.5) ∀~b ∈ R2 , Φ(M (I2 , ~b)) = ~b donc Φ est surjective


∀A ∈ SO(2), Φ(M (A, ~0)) = ~0 donc Φ n’est pas injective

I.B - Droites affines du plan


   
0 0
I.B.1) ∆(0, ~e1 ) est la droite passant par et dirigée par . C’est l’axe Oy.
0 1
  √   
~e1 + ~e2 2 1
∆ 2, √ est la droite passant par √ et dirigée par
2 2 −1
   
q cos θ − sin θ
I.B.2) La droite ∆(q, ~uθ ) passe par et est dirigée par .
q sin θ cos θ
Elle a donc comme équation cartésienne : cos θ X + sin θ Y = q
   
q cos θ − sin θ
I.B.3) La droite ∆(q, ~uθ ) passe par et est dirigée par .
q sin θ cos θ

x(t) = q cos θ − t sin θ
Elle a donc comme équation paramétrique , pour t ∈ R
y(t) = q sin θ + t cos θ

I.B.4) La droite ∆(q, ~u) est orthogonale à ~u et la droite ∆(r, ~v ), est orthogonale à ~v . Si elles sont confondues,
alors ~u et ~v sont colinéaires et comme ils sont unitaires, ils sont égaux ou opposés. Soit θ ∈ R tel que
~u = ~uθ .
Si ~u = ~v , alors ∆(q, ~u) et ∆(r, ~v ) ont pour équation cartésienne cos θ X +sin θ Y = q et cos θ X +sin θ Y =
r. Si elles sont confondues alors q = r.
Si ~u = −~v , alors ~v = ~uθ+π et ∆(q, ~u) et ∆(r, ~v ) ont pour équation cartésienne cos θ X + sin θ Y = q et
− cos θ X − sin θ Y = r. Si elles sont confondues alors q = −r.
Réciproquement, si ~u = ~v et q = r ou si ~u = −~v et q = −r alors ∆(q, ~u) = ∆(r, ~v ).
Finalement, ∆(q, ~u) et ∆(r, ~v ) sont confondues si et seulement si ~u = ~v et q = r ou ~u = −~v et q = −r.

1
I.C - Action de G sur les droites
I.C.1) On remarque que Ψ(M (Rθ , ~b)) est la droite passant par hRθ ~e1 , ~biRθ ~e1 = h~uθ , ~bi~uθ qui est la projection
orthogonale de ~b sur Vect (~uθ ) et orthogonale à ~uθ , donc
Ψ(M (Rθ , ~b)) est la droite passant par ~b et orthogonale à ~uθ .
   
~ 1 ~ 1
Dans le cas A = Rπ/6 et b = , Ψ(M (A, b)) est la droite passant par et orthogonale à ~uπ/6 .
2 2
 
~ 0
I.C.2) Ψ(M (I2 , 0)) est la droite passant par et orthogonale à ~e1 . Ψ(M (I2 , ~0)) est l’axe Oy
0
I.C.3) Ψ(M (Rθ , q~uθ )) est la droite passant par q~uθ et orthogonale à ~uθ
On a donc bien Ψ(M (Rθ , q~uθ )) = ∆(q, ~uθ )
Toute droite affine de R2 est de la forme ∆(q, ~uθ ) en prenant ~uθ orthogonal à la droite et q la distance
de la droite à l’origine. L’application Ψ est donc surjective.

I.C.4) a) Soit A ∈ SO(2) et ~b ∈ R2 . Il existe θ ∈ R tel que A = Rθ .


M (A, ~b) est dans H si et seulement si la droite passant par b et orthogonale à uθ est égale à ∆(0, ~e1 ),
c’est à dire à l’axe Oy.
Donc M (A, ~b) est dans H si et seulement si A = ±I2 et ~b ∈ Oy.
   
1 0 0 −1 0 0
H est donc l’ensemble des matrices de la forme 0 1 x ou  0 −1 x avec x ∈ R
0 0 1 0 0 1
b) On vérifie facilement que H est une partie non vide de G, stable pour le produit et pour l’inverse ;
H est donc un sous groupe de G
 
cos θ − sin θ x
c) Soit g ∈ G et h ∈ H. Il existe θ ∈ R et (x, y) ∈ R2 tels que g =  sin θ cos θ y  et il existe z ∈ R
  0 0 1
t 0 0
et t ∈ {−1, 1} tels que h = 0 t z .
0 0 1
 
t cos θ −t sin θ −z sin θ + x
On a alors gh =  t sin θ t cos θ z cos θ + y .
0 0 1
 
−z sin θ + x
Ψ(gh) est donc la droite passant par et orthogonale à t~uθ et donc à ~uθ et Ψ(g) est la
  z cos θ + y
x
droite passant par et orthogonale à ~uθ .
y
     
x −z sin θ + x z sin θ
Or, − = est orthogonal à ~uθ , ces deux droites sont donc égales et
y z cos θ + y −z cos θ
pour tout g de G , et tout h de H , on a Ψ(gh) = Ψ(g)

II Fonctions radicales
II.A - Étude d’un exemple
x2 + y 2
II.A.1) ∀(x, y) ∈ R2 , (x2 + y 2 )f (x, y) = 6 1.
1 + x2 + y 2
f est de classe C 1 sur R2 , à valeurs dans R et (x, y) →
7 (x2 + y 2 )f (x, y) est bornée sur R2 donc
f est dans B1 .
1 1
II.A.2) ∀(q, θ) ∈ R2 , f (q cos θ − t sin θ, q sin θ + t cos θ) = 2 2
et t 7→ est intégrable sur R,
1+q +t 1 + q 2 + t2
donc fˆ est définie sur R2 .

2
Z +∞ Z +∞ √ dt i+∞
dt 1 1+q 2 1 h t π
fˆ(q, θ) = 2 2
=p t2
=p arctan( p ) =p .
−∞ 1+q +t 1 + q2 −∞ 1+ 1+q 2 1+q 2 2
1 + q −∞ 1 + q2
π
Donc fˆ(q, θ) = p .
1 + q2
Z 2π Z 2π
1 1 π π
II.A.3) ∀q ∈]0, +∞[, R(q) = fˆ(q, θ) dθ = p dθ = p .
2π 0 2π 0 1 + q2 1 + q2
R0 (q) π
On a donc =− .
q (1 + q 2 )3/2
R0 (q)
◦ q 7→ est continue sur ]0, +∞[
q
0
R (q)
◦ q 7→ est continue sur [0, 1] donc intégrable sur ]0, 1]
q
0
R0 (q)
 
R (q) 1 1
◦ = Oq→+∞ et q →
7 est intégrable sur [1, +∞[ donc q →
7 est intégrable sur 1, +∞[
q q2 q2 q
R0 (q)
et donc q 7→ est intégrable sur ]0, +∞[ .
q
1 +∞ R0 (q) 1 +∞
Z Z Z +∞
π 1
− dq = − − dq = dq.
π 0 q π 0 (1 + q 2 )3/2 0 (1 + q 2 )3/2
1
q 7→ est intégrable sur ]0, +∞[ et u 7→ sh u est bijective et de classe C 1 de ]0, +∞[ dans
(1 + q 2 )3/2
]0, +∞[, on peut donc procéder au changement de variable q = sh (u) (dq = ch u du).
Z +∞ Z +∞ Z +∞ Z +∞
1 1 1 1
dq = 2 ch u du = 2 ch u du = du
0 (1 + q 2 )3/2
0 (1 + sh (u)) 3/2
0 ( ch (u)) 3/2
0 ch 2 (u)
h i+∞
= th (u) =1
0
+∞
R0 (q)
Z
1
Or f (0, 0) = 1, on a donc − dq = f (0, 0)
π 0 q
∂f −2x
II.A.4) ∀(x, y) ∈ R2 ,(x, y) = .
∂x (1 + x2 + y 2 )2
∂f −2x
lim x4 (x, 0) = lim x4 = +∞
x→+∞ ∂x x→+∞ (1 + x2 )2
∂f ∂f
(x, y) 7→ (x2 + y 2 )2 (x, y) n’est donc pas bornée sur R2 et donc la fonction n’est pas dans B2 .
∂x ∂x

II.B - Fonctions radicales : cas général


Z 2π
1
II.B.1) ∀(r, t) ∈ R+ × [0, 2π], f (r cos t, r sin t) = ϕ(r) donc f (r cos t, r sin t) dt = ϕ(r) et f¯(r) = ϕ(r).
2π 0
II.B.2) Pour tout réel q > 0,
rϕ(r)
◦ r 7→ p est continue sur ]q, +∞[
r2 − q2
 
rϕ(r) 1 1
◦ Sur ]q, q + 1], p = Or→q √ et r →
7 √ est intégrable sur ]q, q + 1] donc
r2 − q2 r−q r−q
rϕ(r)
r 7→ p est intégrable sur ]q, q + 1].
r2 − q2
rϕ(r)
◦ Sur [q + 1, +∞[, p = Ox→+∞ (ϕ(r)) et r → 7 ϕ(r) est intégrable sur [q + 1, +∞[ donc
r2 − q2
rϕ(r)
r 7→ p est intégrable sur [q + 1, +∞[.
r2 − q2
Z +∞
rϕ(r) rϕ(r)
Finalement, r 7→ p est intégrable sur ]q, +∞[ et p dr converge.
r2 − q2 q r2 − q2

3
Z +∞
rϕ(r) p
II.B.3) Pour tout réel q > 0, p dr converge et r 7→ r2 − q 2 est C 1 et bijective de ]q, +∞[ dans
q r2 − q2
p r p
]0, +∞[. On peut donc faire le changement de variable t = r2 − q 2 , (dt = p dr , r = t2 + q 2 ),
r2 − q2
Z +∞ p Z +∞ Z +∞ p
rϕ(r)
et donc ϕ( t2 + q 2 ) dt converge et p dr = ϕ( t2 + q 2 ) dt
0 q r2 − q2 0
p
Or, ∀(θ, t) ∈ R × [0, 2π], f (q cos θ − t sin θ, q sin θ + t cos θ) = ϕ( t2 + q 2 )
Z +∞ Z +∞
rϕ(r)
et donc p dr = f (q cos θ − t sin θ, q sin θ + t cos θ) dt.
q r2 − q2 0
p
De même en faisant le changement de variable t = − r2 − q 2 , on montre que
Z +∞ Z 0
rϕ(r)
p dr = f (q cos θ − t sin θ, q sin θ + t cos θ) dt.
q r2 − q2 −∞
Z +∞
rϕ(r)
Et finalement, fˆ(q, θ) = 2 p dr.
q r2 − q2
II.B.4) En II.B.1), on a montré que f¯(r) = ϕ(r)
Z +∞
rϕ(r)
Dans la question précédente, on a montré que fˆ(q, θ) = 2 p dr et donc, que fˆ(q, θ) ne
q r2 − q2
dépend pas de θ.
2π +∞
rf¯(r)
Z Z
1
On en déduit : ∀q ∈ R+ , fˆ(q, θ) dθ = fˆ(q, θ) = 2 p dr.
2π 0 q r2 − q2

III Transformée de Radon d’une fonction de B1


III.A -
Soit (q, θ) ∈ R2 , f appartient à B1 donc f est de classe C 1 sur R2 et ((q cos θ−t sin θ)2 +(q sin θ+t cos θ)2 )f (q cos θ−
t sin θ, q sin θ + t cos θ) = (q 2 + t2 )f (q cos θ − t sin θ, q sin θ + t cos θ) est bornée pour t ∈ R.
On en déduit que :
◦ t 7→ f (q cos θ − t sin θ, q sin θ + t cos θ) est continue sur R.  
1 1
◦ Sur [1, +∞[, |f (q cos θ − t sin θ, q sin θ + t cos θ)| = Ot→+∞ 2 et t 7→ 2 est intégrable sur [1, +∞[,
t t
donc t 7→ f (q cos θ − t sin θ, q sin θ + t cos θ) est intégrable sur [1, +∞[.
◦ De même, t 7→ f (q cos θ − t sin θ, q sin θ + t cos θ) est intégrable sur ] − ∞, −1].
◦ Sur [−1, 1], t 7→ f (q cos θ − t sin θ, q sin θ + t cos θ) est continue, donc intégrable.
Finalement, t 7→ f (q cos θ − t sin θ, q sin θ + t cos θ) est intégrable sur R et donc fˆ est définie sur R2 .

III.B -
Pour tout q et tout
Z +∞θ,
fˆ(−q, θ + π) = f (−q cos(θ + π) − t sin(θ + π), −q sin(θ + π) + t cos(θ + π))dt
−∞
Z +∞
= f (q cos θ + t sin θ, q sin θ − t cos θ)dt
−∞
Z +∞
= f (q cos θ − u sin θ, q sin θ + u cos θ)du (changement de variable u = −t)
−∞
= fˆ(q, θ)
Donc pour tout q et tout θ on a fˆ(−q, θ + π) = fˆ(q, θ).

III.C -
III.C.1) On note h(r, t) = f (r cos t, r sin t). Soit a ∈ R+∗ , montrons que f¯ est de classe C 1 sur [−a, a].
◦ h est de classe C 1 sur [−a, a] × [0, 2π] et
∂h ∂f ∂f
∀(r, t) ∈ [−a, a] × [0, 2π], (r, t) = cos t (r cos t, r sin t) + sin t (r cos t, r sin t)
∂r ∂x ∂y

4
◦ ∀r ∈ [−a, a], t 7→ h(r, t) est continue, donc intégrable sur le segment [0, 2π]
∂h ∂h
◦ est continue sur le compact [−a, a] × [0, 2π], on note M = sup .
∂r [−a,a]×[0,2π] ∂r
t 7→ M est positive, continue et intégrable sur [0, 2π] et ∀(r, t) ∈ [−a, a] × [0, 2π], |h(r, t)| 6 M
Z
D’après le théorème de dérivation sous le signe , f¯ est de classe C 1 sur [−a, a] pour tout a ∈ R+∗ , et

donc f¯ est de classe C 1 sur R et


Z 2π  
1 ∂f ∂f
∀r ∈ R, f¯0 (r) = cos t (r cos t, r sin t) + sin t (r cos t, r sin t) dt.
2π 0 ∂x ∂y
III.C.2) f appartient à B1 , donc (x, y) 7→ (x2 +y 2 )f (x, y) est bornée sur R2 . Notons M = sup (x2 + y 2 )f (x, y) ,
(x,y)∈R2
alors, ∀(r, t) ∈ R2 , |r2 f (r cos t, r sin t)| 6 M , donc
Z 2π Z 2π
1 1
r2 f¯(r) = r2 f (r cos t, r sin t) dt 6 M dt = M .
2π 0 2π 0
Donc, la fonction r 7→ r2 f¯(r) est bornée sur R.
∂f ∂f ∂f ∂f
III.C.3) On suppose que et sont dans B2 , alors, (x, y) 7→ (x2 + y 2 )2 (x, y) et (x, y) 7→ (x2 + y 2 )2 (x, y)
∂x ∂y ∂x ∂y
sont bornées sur R2 .
∂f ∂f
soit M = sup (x2 + y 2 )2 (x, y) + sup (x2 + y 2 )2 (x, y) ,
(x,y)∈R2 ∂x (x,y)∈R2 ∂y
 
2 4 ∂f ∂f
alors, ∀(r, t) ∈ R , r (r cos t, r sin t) + (r cos t, r sin t) 6 M .
∂x ∂y
On en déduit que ∀r ∈ R,
r4 2π
Z  
4 ¯0 ∂f ∂f
r f (r) = cos t (r cos t, r sin t) + sin t (r cos t, r sin t) dt
2π 0 ∂x ∂y
r4 2π ∂f
Z  
∂f
6 (r cos t, r sin t) + (r cos t, r sin t) dt
2π 0 ∂x ∂y
6 M.
r 7→ r4 f¯0 (r) est donc bornée sur R.

IV Formule d’inversion
IV.A - Résultats préliminaires
√ t √
IV.A.1) À l’aide du changement de variable u = t2 − 1 (du = √ , t = u2 + 1), on montre que la fonction
 t2 − 
1  
1 1 1 π
t 7→ √ admet comme primitive t 7→ − arctan √ , de plus , lim − arctan √ =−
2
t t −1 2
t −1 t→1 2
t −1 2
et
  Z +∞
1 dt π
lim − arctan √ = 0. On en déduit que l’intégrale √ existe et vaut .
t→+∞ 2
t −1 1
2
t t −1 2
Z r
dq
IV.A.2) Soit ((ε, r) ∈ R2 tels que 0 < ε < r. On montre facilement que l’intégrale p converge et
2 r2 − q2
ε q
que θ 7→ r cos θ est une bijection de classe C 1 de [arccos rε , 0[ dans [ε, r[. On fait donc le changement de
variable q = r cos θ (dq = −r sin θ dθ) :
Z r Z 0 Z arccos rε
dq −r sin θ dθ 1 dθ
= = 2
(cos θ)2
p p
ε q
2 2
r −q 2
arccos r (r cos θ)
ε 2 2
r − (r cos θ) 2 r 0
ε
p ε 2

1 h i arccos r 1 ε 1 1 − (r) r 2 − ε2
= 2 tan θ = 2 tan(arccos ) = 2 ε =
r 0 r r r r r2 ε
Z r √
dq r 2 − ε2
On a donc = .
r2 ε
p
2 r2 − q2
ε q

5
IV.B - Étude d’une fonction définie par une intégrale
th(qt)
IV.B.1) On note u(t, q) = √ . Soit a ∈ R+∗ , montrons que H est continue sur [a, +∞[.
t2 − 1
On suppose que r 7→ r2 h(r) est bornée, soit M = sup |r2 h(r)|.
r∈R+
◦ u est continue sur ]1, +∞[×[a, +∞[
M
◦ Soit ϕ(r) = √ ; ϕ est positive, continue et d’après IV.A.1), intégrable sur ]1, +∞[ et ∀(t, q) ∈
ta2 t2 − 1
]1, +∞[×[a, +∞[, |u(t, q)| 6 ϕ(t)
D’après le théorème de continuité sous le signe somme, H est donc continue sur [a, +∞[ pour tout
a ∈ R+∗ et donc H est continue sur ]0, +∞[.
M
IV.B.2) En gardant les notations de la question précédente, on a : |h(tq)| 6 2 2 et donc
t q
Z +∞ Z +∞ Z +∞
th(qt) tM 1 dt 1 π
|H(q)| = √ dt 6 √ dt = 2 √ = 2
1
2
t −1 1
2 2 2
t q t −1 q 1 2
t t −1 q 2
 
1
On a donc au voisinage de +∞, H(q) = O .
q2
IV.B.3) On garde les notations de B.1).
On suppose que r 7→ r4 h0 (r) est bornée, soit N = sup |r4 h0 (r)|.
r∈R+
1
◦ u est de classe C sur ]1, +∞[×[a, +∞[
◦ ∀q ∈ [a, +∞[, t 7→ u(t, q) est intégrable sur ]1, +∞[
∂u t2 h0 (tq)
◦ (t, q) = √
∂q t2 − 1
N
Soit ϕ(r) = √ ; de même qu’en B.1), on montre que ϕ est positive, continue et intégrable
t a t2 − 1
2 4
∂u
sur ]1, +∞[ et ∀(t, q) ∈]1, +∞[×[a, +∞[, (t, q) 6 ϕ(t)
∂q
D’après le théorème de dérivation sous le signe somme, H est donc de classe C 1 sur [a, +∞[ pour tout
a ∈ R+∗ et donc H est de classe C 1 sur ]0, +∞[.

IV.C - Vers la formule d’inversion


+∞
rf¯(r)
Z
IV.C.1) F (q) = 2 p dr.
q r2 − q2
En faisant le changement de variable r = tq, pour q > 0, on obtient :
Z +∞ ¯
tf (tq)
F (q) = 2q √ dt.
1 t2 − 1
∂f ∂f
La fonction f est dans B1 et ses dérivées partielles et sont dans B2 , donc d’après III.C), f¯ est de
∂x ∂y
classe C 1 sur R et r 7→ r4 f¯0 (r) est bornée sur R.
On utilise alors IV.B.2) et IV.B.3) en remplaçant h et H par f¯ et F , on obtient que F est de classe C 1 sur ]0, +∞[
   
1 1
et qu’au voisinage de +∞ on a F (q) = qO et donc au voisinage de +∞ on a F (q) = O .
q2 q
Z +∞
F (q)
IV.C.2) Pour tout ε > 0, dq converge car F est continue sur ]0, +∞[ et qu’au voisinage de +∞ on a
  ε q2
1
F (q) = O
q
u = F (q) u0 = F 0 (q)
F étant de classe C 1 , on fait une intégration par parties : 0
v = q12 v = − 1q
Z +∞  +∞ Z +∞ 0 Z +∞ 0
F (q) F (q) F (q) F (ε) F (q)
2
dq = − + dq = + dq
ε q q ε ε q ε ε q

6
+∞ +∞ Z +∞
F 0 (q) F 0 (q)
Z Z
F (ε) F (q)
On en déduit la convergence de l’intégrale dq et dq = − + .
ε q ε q ε ε q2
!
+∞
rf¯(r) rf¯(r)
Z +∞ Z +∞ Z +∞
F 0 (q)
Z
F (ε) 1
Or F (q) = 2 dr et donc dq = − +2 dr dq.
q2
p p
q r2 − q2 ε q ε ε q r2 − q2

IV.C.3) En utilisant la question IV.C.2) et l’interversion des intégrales, on obtient :


!
rf¯(r)
Z +∞ 0 Z +∞ Z r
F (q) F (ε)
dq = − +2 p dq dr.
ε q ε ε ε q
2 r2 − q2
2 +∞ rf¯(r)
Z
F (ε)
Or = √ dr.
ε ε ε r 2 − ε2

rf¯(r)
Z r Z r
dq r 2 − ε2
et dq = rf¯(r) = rf¯(r) dr. (IV.A.2))
r2 ε
p p
2 r2 − q2 2 r2 − q2
ε q ε q

2 +∞ rf¯(r)
Z +∞ 0 Z +∞
r2 − ε2
Z
F (q) ¯
Donc dq = − √ dr + 2 rf (r) dr
q ε ε 2 2 r2 ε
ε
Z +∞ √ r −ε! ε
1 r 2 − ε2
= −2 rf¯(r) √ − dr
ε ε r 2 − ε2 r2 ε
rf¯(r) f¯(r)
Z +∞ Z +∞
r 2 − ε2
 
=2 √ 1− dr = −2ε √ dr.
ε ε r 2 − ε2 r2 ε r r2 − ε2
f¯(r)
Z +∞ 0 Z +∞
F (q)
Et donc, ∀ε > 0, dq = −2ε √ dr.
ε q ε r r 2 − ε2

IV.D - La formule d’inversion


+∞
f¯(r) 1 +∞ f¯(tε)
Z Z
IV.D.1) En faisant le changement de variable r = tε, on obtient √ dr = √ dt
ε r r 2 − ε2 ε 1 t t2 − 1
+∞ 0 +∞
f¯(tε)
Z Z
F (q)
et donc dq = −2 √ dt.
ε q 1 t t2 − 1
Z +∞ ¯
f (tε)
Calculons lim √ dt.
ε→0 1 t t2 − 1
f¯(tε)
◦ Pour tout ε > 0, t 7→ √ est continue sur ]1, +∞[.
t t2 − 1
f¯(tε) f (0, 0) f (0, 0)
◦ ∀t ∈]1, +∞[, lim √ = √ et t 7→ √ est continue sur ]1, +∞[.
ε→0 t t2 − 1 2
t t −1 t t2 − 1
◦ D’après la questions III.C.1, on sait que f¯ est continue sur R et donc bornée sur le segment [0, 1]
D’après la questions III.C.2, on sait que r 7→ r2 f¯(r) est bornée sur R et donc f¯ est bornée sur ]1, +∞[
M
f¯ est donc bornée sur R+ ; Notons M = sup |f¯|. Soit ϕ(t) = √ , ϕ est positive, continue et
R+ t t2 − 1
f¯(tε)
intégrable sur ]1, +∞[, et ∀(ε, t) ∈]0, +∞[×]1, +∞[, √ 6 ϕ(t).
t t2 − 1
D’après l’extension du théorème de convergence dominée au cas d’une famille à paramètre réel, on peut
Z +∞ ¯ Z +∞
f (tε) f (0, 0)
conclure que lim √ dt = √ dt.
ε→0 1 t t −12 t t2 − 1
1
Z +∞
dt π
En IV.A.1), on a montré √ = .
1
2
t t −1 2
Z +∞ 0 Z +∞ 0
F (q) F (q)
On a donc lim dq = −πf (0, 0), ce qui montre que l’intégrale dq existe et que
ε→0 ε q 0 q
Z +∞ 0
−1 F (q)
f (0, 0) = dq.
π 0 q
Or, R0,0 = F , on a donc montré la formule d’inversion de Radon pour f au point (x, y) = (0, 0) :
0
−1 +∞ R0,0 (q)
Z
f (0, 0) = dq
π 0 q

7
1
IV.D.2) Dans la partie II, on a étudié la fonction f définie par : ∀(x, y) ∈ R2 , f (x, y) = . On a montré
1 +Zx2 + y 2
+∞
1 R0 (q)
la formule d’inversion de Radon pour f au point (x, y) = (0, 0) : f (0, 0) = − dq et on a
π 0 q
∂f
montré que la fonction n’est pas dans B2 .
∂x
Les hypothèses faites sur f ne sont donc pas nécessaires pour que la formule d’inversion de Radon soit
vérifiée au point ((x, y) = (0, 0).
IV.D.3) Pour obtenir la formule d’inversion de Radon en un point (x0 , y0 ), on peut considérer la fonction (x, y) 7→
f (x + x0 , y + y0 ) et lui appliquer la formule d’inversion de Radon en (0, 0).

V Interprétation et application à la radiographie


V.A - Fonction invariante sur G
V.A.1) Soit g dans G et r tel que Φ(r) = ~0.
Il existe A ∈ SO(2) et ~b ∈ R2 tels que g = M (A, ~b) ; il existe B ∈ SO(2) tel que r = M (B, ~0).
On a alors gr = M (AB, ~b), et donc Φ(gr) = Φ(g) = ~b et f ∗ (gr) = f ∗ (g).
V.A.2) Supposons que les deux droites ∆(q1 , ~uθ1 ) et ∆(q2 , ~uθ2 ) coïncident. D’après la question I.B.4), on a donc
q1 = q2 et ~uθ1 = ~uθ2 ou bien q1 = −q2 et ~uθ1 = −~uθ2 c’est à dire q1 = q2 et θ1 ≡ θ2 [2π] ou bien q1 = −q2
et θ1 ≡ θ2 + π[2π].
De III.B, on déduit que fˆ(q1 , θ1 ) = fˆ(q2 , θ2 )

V.A.3) Soient g ∈ G et h ∈ H. Dans I.C.4)c), on a montré que Ψ(gh) = Ψ(g). On a donc fˆ∗ (gh) = fˆ∗ (g).

V.B - Reconstruction en radiographie


   
q cos θ − sin θ
V.B.1) ∆(q, ~uθ ) est la droite passant par et dirigée par .
q sin θ cos θ
Z Z +∞     Z +∞
q cos θ − sin θ
On peut définir f = f +t dt = f (q cos θ − t sin θ, q sin θ +
uθ )
∆(q,~ −∞ q sin θ cos θ −∞
t cos θ)dt = fˆ(q, θ).
Z
f = fˆ(q, θ).
uθ )
∆(q,~

V.B.2) L’intensité mesurée de part et d’autre de la zone visée nous donne la transformée de Radon fˆ de la
fonction f .
La formule d’inversion de Radon permet ensuite de calculer f en fonction de fˆ et donc de connaître la
densité des tissus dans la zone radiographiée.

8
CCP MP Maths 2 2014
Les calculatrices sont autorisées

Le sujet est composé de deux exercices et d’un problème, tous indépendants.

Partie I : EXERCICE 1
Soit les suites réelles (un ), (vn ) et (wn ) définies par :

 un+1 = un + 3vn
∀n ∈ N vn+1 = 3un + vn + 4wn et (u0 , v0 , w0 ) = (1, 0, 1).
wn+1 = 4vn + wn

I.1.
 
1 3 0
I.1.a Justifier sans calcul que la matrice A =  3 1 4  ∈ M3 (R) est diagonalisable.
0 4 1
I.1.b Diagonaliser la matrice A ∈ M3 (R).
I.1.c Déterminer la matrice An pour tout n ∈ N. On pourra utiliser la calculatrice.
I.2. Expliciter les termes un , vn et wn en fonction de n.

Partie II : EXERCICE 2
Soit n un entier supérieur à 2 et E un espace vectoriel sur R de dimension n. On appelle projecteur de E, tout
endomorphisme p de E vérifiant p ◦ p = p.
II.1. Soit p un projecteur de E.
II.1.a Démontrer que les sous-espaces vectoriels Ker(p) et Im(p) sont supplémentaires dans E.
II.1.b En déduire que la trace de p (notée Tr(p)) est égale au rang de p (noté rg(p)).
II.1.c Un endomorphisme u de E vérifiant Tr(u) = rg(u) est-il nécessairement un projecteur de E ?
II.2. Donner un exemple de deux matrices A et B de M3 (R) de rang 1 telles que A soit diagonalisable et B
ne soit pas diagonalisable. Justifier la réponse.
II.3. Soit u un endomorphisme de E de rang 1.
II.3.a Démontrer qu’il existe une base β = (e1 , · · · , en ) de E telle que la matrice Matβ (u) de u dans β
soit de la forme :
 
0 ··· 0 a1
 0 ··· 0 a2 
Matβ (u) =  .  ∈ Mn (R) où a1 , · · · , an sont n nombres réels.
 
.. ..
 .. . . 
0 ··· 0 an

II.3.b Démontrer que u est diagonalisable si, et seulement si, la trace de u est non nulle.
II.3.c On suppose que Tr(u) = rg(u) = 1. Démontrer que u est un projecteur.
 
1 1 −1
II.3.d Soit la matrice A =  1 1 −1  ∈ M3 (R). Démontrer que A est la matrice d’un projecteur
1 1 −1
de R3 dont on déterminera l’image et le noyau.

1
Partie III : PROBLEME
Notations et rappels
Soit n un entier supérieur à 1. On désigne par diag(α1 , · · · , αn ) la matrice diagonale de Mn (R) dont les
coefficients diagonaux sont les réels α1 , · · · , αn dans cet ordre. Si M ∈ Mn (R), on note tM sa transposée.
On munit l’espace vectoriel E = Rn du produit scalaire canonique noté h | i et de la norme euclidienne
k k associée. On note S(E) le sous-espace des endomorphismes symétriques de E, c’est-à-dire l’ensemble des
endomorphismes s de E vérifiant :

∀(x, y) ∈ E 2 , hs(x)|yi = hx|s(y)i.

Un endomorphisme symétrique s de E est dit symétrique positif (respectivement symétrique défini positif) si :

∀x ∈ E, hs(x)|xi ≥ 0 (respectivement ∀x ∈ E \ {0}, hs(x)|xi > 0).

Une matrice S de Mn (R) est dite symétrique positive (respectivement symétrique définie positive) si :
t t
∀X ∈ Mn,1 (R), XSX ≥ 0 (respectivement ∀X ∈ Mn,1 (R) \ {0}, XSX > 0).

On note Sn+ (R) (respectivement Sn++ (R)) l’ensemble des matrices symétriques positives (respectivement symé-
triques définies positives) de Mn (R).
On rappelle qu’un endomorphisme s de E est symétrique (respectivement symétrique positif, symétrique dé-
fini positif) si, et seulement si, sa matrice dans toute base orthonormée de E est symétrique (respectivement
symétrique positive, symétrique définie positive).
On admet que, pour tous réels positifs a1 , · · · , an ,
n
!1/n n
Y 1X
ai ≤ ai (inégalité arithmético-géométrique).
i=1
n i=1

Objectif du problème
On se donne une matrice S de Sn+ (R) (ou Sn++ (R)) et on étudie le maximum (ou minimum) de la forme linéaire
A 7→ Tr(AS) sur des ensembles de matrices.

Questions préliminaires
III.1.
III.1.a Enoncer (sans démonstration) le théorème de réduction des endomorphismes symétriques de l’es-
pace euclidien E et sa version relative aux matrices symétriques réelles.
III.1.b Toute matrice symétrique à coefficients complexes est-elle nécessairement diagonalisable ? On
pourra par exemple considérer la matrice de M2 (C) :
 
i 1
S= .
1 −i

III.2. Soit s ∈ S(E), de valeurs propres (réelles) λ1 , · · · , λn rangées dans l’ordre croissant :

λ1 ≤ λ2 ≤ · · · ≤ λn .

Soit β = (1 , · · · , n ) une base orthonormée de E telle que, pour tout i ∈ {1, · · · , n}, i est un vecteur
propre associé à la valeur propre λi . Pour tout vecteur x de E, on pose :

Rx (x) = hs(x)|xi.

III.2.a Exprimer Rs (x) à l’aide des λi et des coordonnées de x dans la base β.


III.2.b En déduire l’inclusion : Rs (S(0, 1)) ⊂ [λ1 , λn ] où S(0, 1) désigne la sphère unité de E.
III.3.
III.3.a On suppose dans cette question que s est symétrique positif (respectivement symétrique défini
positif). Démontrer que les valeurs propres de s sont toutes positives (respectivement strictement
positives).

2
III.3.b Soit S = (si,j ) ∈ Sn+ (R), de valeurs propres λ1 , · · · , λn rangées dans l’ordre croissant :

λ1 ≤ λ2 ≤ · · · ≤ λn .

On note s l’endomorphisme de E représenté par S dans la base canonique B = (e1 , · · · , en ). Exprimer


le terme général si,j de S comme un produit scalaire et démontrer que :

∀i ∈ {1, · · · , n} λ1 ≤ si,i ≤ λn .

Un maximum sur On (R)


On note In la matrice unité de Mn (R) et On (R) le groupe des matrices orthogonales de Mn (R).
III.4. Démontrer que l’application M 7→ tM M − In est continue de Mn (R) dans Mn (R).
III.5. Justifier que, si A = (ai,j ) est une matrice orthogonale, alors :

∀(i, j) ∈ {1, · · · , n}2 |ai,j | ≤ 1.

III.6. En déduire que le groupe orthogonal On (R) est une partie compacte de Mn (R).
III.7. Soit S ∈ Sn+ (R), de valeurs propres (positives) λ1 , · · · , λn . On pose ∆ = diag(λ1 , · · · , λn ).
Si A est une matrice orthogonale, on note T (A) le nombre réel T (A) = Tr(AS).
III.7.a Soit A ∈ On (R). Démontrer qu’il existe une matrice orthogonale B telle que :

T (A) = Tr(B∆).

III.7.b Démontrer que l’application T de On (R) dans R admet un maximum sur On (R)
que l’on notera t.
III.7.c Démontrer que, pour toute matrice orthogonale A de On (R), T (A) ≤ Tr(S), puis déterminer le
réel t.

Inégalité d’Hadamard
Soit S = (si,j ) ∈ Sn+ (R), de valeurs propres (réelles positives) λ1 , · · · , λn rangées dans l’ordre croissant :

0 ≤ λ1 ≤ λ2 ≤ · · · ≤ λn .

III.8. Démontrer l’inégalité valable pour tout S ∈ Sn+ (R) :


 n
1
det(S) ≤ Tr(S) (∗).
n

III.9. Soit α = (α1 , · · · , αn ) ∈ Rn , D = diag(α1 , · · · , αn ) et Sα = tDSD. Démontrer que Sα ∈ Sn+ (R) et


calculer Tr(Sα ).
III.10. Dans cette question, on suppose que les coefficients diagonaux si,i de S sont strictement positifs et,
1
pour 1 ≤ i ≤ n, on pose αi = √ . En utilisant l’inégalité (∗), démontrer que :
si,i
n
Y
det(S) ≤ si,i .
i=1

n
Y
III.11. Pour tout réel ε > 0, on pose Sε = S + εIn . Démontrer que det(Sε ) ≤ (si,i + ε), puis conclure que :
i=1

n
Y n
Y
λi ≤ si,i (inégalité d’Hadamard).
i=1 i=1

Application de l’inégalité d’Hadamard : détermination d’un minimum


Soit S ∈ Sn++ (R), de valeurs propres 0 < λ1 ≤ · · · ≤ λn , et ∆ = diag(λ1 , . . . , λn ). Soit Ω ∈ On (R) telle
que S = Ω∆ tΩ. On désigne par U l’ensemble des matrices de Sn++ (R) de déterminant égal à 1.

3
III.12. Démontrer que, pour tout A ∈ U, la matrice B = tΩAΩ est une matrice de U vérifiant :

Tr(AS) = Tr(B∆).

III.13. Démontrer que {Tr(AS) \ A ∈ U } = {Tr(B∆) \ B ∈ U }, puis que ces ensembles admettent une borne
inférieure que l’on notera m.
III.14. Démontrer que, si B = (bi,j ) ∈ U :

Tr(B∆) ≥ n(λ1 · · · λn )1/n (b1,1 · · · bn,n )1/n .

III.15. En déduire que, pour B = (bi,j ) ∈ U, Tr(B∆) ≥ n(det(S))1/n .


1
III.16. Pour tout entier k tel que 1 ≤ k ≤ n, on pose µk = (det(S))1/n et D = diag(µ1 , · · · , µn ).
λk
Déterminer le réel m.

Fin de l’énoncé

4
Un corrigé de l’épreuve
CCP MP Maths 2 2014

Partie I : EXERCICE 1
I.1.
I.1.a La matrice A est symétrique à coefficients réels donc elle est diagonalisable d’après le théorème
spectral.
I.1.b On calcule le polynôme caractéristique χA (t) = det(tI3 − A) : la calculatrice fournit
χA (t) = t3 − 3t2 − 22t + 24 = (t − 1)(t − 6)(t + 4), donc le spectre de A vaut Sp(A) = {1, 6, −4}.
Pour chacune des valeurs propres λ de A, on résout avec la calculatrice le système linéaire AX = λX,
où l’inconnue X ∈ R3 .    
1 0 0 −4 3 3
On en déduit que A = P DP −1 avec D =  0 6 0  et P =  0 5 −5 
0 0 −4 3 4 4
I.1.c Si An = P Dn P −1 , alors An+1 = (P Dn P −1 )(P DP −1 ) = P Dn+1 P −1 , or A0 = P D0 P −1 , donc
d’après le principe de récurrence, pour tout n ∈ N, An = P Dn P −1 .
 
−8 0 6
1
La calculatrice fournit P −1 =  3 5 4  (en normalisant les vecteurs colonnes de P , on
50
3 −5 4
aurait pu remplacer P par une matrice Q orthogonale, auquel cas Q−1 = tQ),
32 + 9(−4)n + 9 6n −15((−4)n − 6n ) 12(−2 + (−4)n + 6n )
 
1 
puis, An = P Dn P −1 = −15((−4)n − 6n ) 25((−4)n + 6n ) −20((−4)n − 6n ) .
50
12(−2 + (−4) + 6 ) −20((−4) − 6 ) 18 + 16((−4)n + 6n )
n n n n

 
un
I.2. D’après l’énoncé, pour tout n ∈ N, si l’on pose Xn =  vn , on a Xn+1 = AXn , donc par récurrence,
  wn
1
on montre que Xn = An X0 , où X0 =  0 .
1
Après simplifications, on obtient :
1 −7 1
un = (8 + 21((−4)n + 6n )), vn = ((−4)n − 6n ) et wn = (−3 + 14((−4)n + 6n )).
50 10 25

Partie II : EXERCICE 2
II.1.
II.1.a Lemme : Commençons par montrer que Im(p) = {x ∈ E/p(x) = x}.
En effet, si x ∈ Im(p), alors il existe y ∈ E tel que x = p(y), donc p(x) = p ◦ p(y) = p(y) = x.
Et réciproquement si x = p(x), alors x ∈ Im(p).
p est annulé par le polynôme X 2 − X = X(X − 1) et X et X − 1 sont premiers entre eux, donc
d’après le théorème de décomposition des noyaux,
E = Ker(X(X − 1))(p) = Ker(p) ⊕ Ker(p − IdE ) = Ker(p) ⊕ Im(p) d’après le lemme.
II.1.b Notons (e1 , . . . , er ) une base de Im(p), où r = rg(p) et (er+1 , . . . , en ) une base de Ker(p).
Alors e = (e1 , . . . , en ) est une base de E et d’après le lemme,
 la matrice de p dans la base e se
Ir 0r,n−r
décompose par blocs selon mat(p, e) = , où 0p,q désigne la matrice nulle à p
0n−r,r 0r,r
lignes et q colonnes.
Ainsi Tr(p) = Tr(mat(p, e)) = r = rg(p).

1
II.1.c Prenons f une base de E et considérons l’endomorphisme u de E défini par :
 
3 0
02,n−2
mat(u, f ) = M =  0 −1  (on a bien n ≥ 2).
0n−2,2 0n−2,n−2
Alors Tr(u) = 2 = rg(u), mais M 2 6= M , donc u n’est pas un projecteur. Ainsi un endomorphisme u
de E vérifiant Tr(u) = rg(u) n’est pas nécessairement un projecteur de E.
 
1 0 0
II.2. A =  0 0 0  est diagonale, donc diagonalisable, et son rang vaut 1.
0 0 0
 
0 1 0
Posons B =  0 0 0 . Ainsi B est aussi de rang 1.
0 0 0
Si B était diagonalisable, comme χB = X 3 , B serait semblable à la matrice nulle, donc on aurait B = 0
ce qui est faux. Ainsi B n’est pas diagonalisable.
II.3.
II.3.a D’après la formule du rang, dim(Ker(u)) = n − 1, donc il existe une base de Ker(u) de la forme
(e1 , · · · , en−1 ). On peut la compléter en une base β = (e1 , · · · , en ) de E.
X n
Pour tout i ∈ {1, . . . , n − 1}, u(ei ) = 0, donc en posant u(en ) = ai ei ,
i=1
 
0 ··· 0 a1
 0 ··· 0 a2 
on a bien Matβ (u) =  .
 
.. .. ..
 . . . 
0 ··· 0 an
II.3.b Supposons d’abord que Tr(u) = 0. Alors an = 0 et la matrice de u étant triangulaire supérieure,
χu (X) = X n , donc Sp(u) = {0}. Alors si u était diagonalisable, il existerait une base dans laquelle
la matrice de u serait nulle, ce qui est faux car rg(u) = 1. Ainsi u n’est pas diagonalisable.
Supposons maintenant que Tr(u) 6= 0. Alors an 6= 0 et χu (X) = X n−1 (X − an ), donc il existe un
vecteur propre fn associé à an . Les sous-espaces propres étant en somme directe, on sait alors que
(e1 , . . . , en−1 , fn ) est une base de vecteurs propres de E, donc u est diagonalisable.
II.3.c Avec les notations de la question précédente, an = 1 6= 0
 
0 ··· 0 0
 .. .. .. 
et mat(u, (e1 , . . . , en−1 , fn )) =  . . .  = M . On a M 2 = M , donc u est un projecteur.

 0 ··· 0 0 
0 ··· 0 1
 
1
II.3.d Les trois colonnes de A étant 2 à 2 colinéaires et non nulles, rg(A) = 1 et Im(A) = Vect  1 .
1
De plus Tr(A) = 1, donc d’après la question précédente, A est une matrice de projecteur.
Par
 la formule
  du  rang, dim(Ker(A)) = 2, 
oron vérifie
  que Aannule
 les deux vecteurs indépendants
1 1 1 1
 −1  et  0 , donc Ker(A) = Vect  −1  ,  0 .
0 1 0 1

Partie III : PROBLEME


Questions préliminaires
III.1.
III.1.a Soit s ∈ S(E). Selon le théorème spectral, il existe une base orthonormée de E constituée de
vecteurs propres de s : s est orthodiagonalisable.
Traduction matricielle : si S ∈ Sn (R), il existe une matrice P orthogonale et une matrice D diagonale
telles que S = P DP −1 = P D tP .

2
III.1.b χS (X) = X 2 − Tr(S)X + det(S) = X 2 , donc Sp(S) = {0} et, à nouveau, si S était diagonalisable,
elle serait semblable à la matrice nulle, donc elle serait nulle, ce qui est faux.
Ainsi S est symétrique à coefficients complexes sans être diagonalisable.
III.2.
Xn n
X n
X
III.2.a Notons x = xi i . Alors s(x) = λi xi i . Or la base β est orthonormée, donc Rs (x) = λi x2i .
i=1 i=1 i=1
n
X
III.2.b Supposons que x ∈ S(0, 1). Alors 1 = kxk2 = x2i .
i=1
n
X n
X n
X n
X
Ainsi, Rs (x) = λi x2i ≤ λn x2i = λn et Rs (x) = λi x2i ≥ λ1 x2i = λ1 .
i=1 i=1 i=1 i=1
On a bien montré que, pour tout x ∈ S(0, 1), Rs (x) ∈ [λ1 , λn ]
III.3.
III.3.a Supposons que s est symétrique défini positif.
Soit λ une valeur propre de s. Il existe un vecteur propre x : x est non nul et s(x) = λx.
Ainsi 0 < hs(x)|xi = λkxk2 et kxk > 0, donc λ > 0.
Si maintenant s est seulement symétrique positif, on a 0 ≤ λkxk2 donc λ ≥ 0.
III.3.b si,j est la i-ème coordonnée dans la base B du vecteur s(ej ), or B est orthonormée, car on utilise
le produit scalaire canonique de Rn , donc si,j = hei |s(ej )i.
En particulier, si,i = hei |s(ei )i, or ei est un vecteur unitaire, donc d’après la question 2.b,
si,i = Rs (ei ) ∈ [λ1 , λn ].

Un maximum sur On (R)


III.4. Les coefficients de tM M − In sont des fonctions polynomiales des coefficients de M , donc d’après le
cours, l’application M 7→ tM M − In est continue.
III.5. D’après le cours, les colonnes de A forment une base orthonormée de Rn , donc pour tout j ∈ {1, · · · , n},
Xn
a2i,j = 1, ce qui implique : pour tout i ∈ {1, · · · , n}, |ai,j | ≤ 1.
i=1
III.6. Si, pour tout M = (mi,j ) ∈ Mn (R), on pose kM k∞ = max |mi,j |, on définit d’après le cours
(i,j)∈{1,...,n}2
une norme sur Mn (R), pour laquelle On (R) est bornée d’après la question précédente. En dimension
finie, toutes les normes sont équivalentes, donc On (R) est encore bornée quelque soit la norme utilisée sur
Mn (R).
De plus, si l’on note f l’application M 7→ tM M − In de la question 4, alors On (R) = f −1 ({0n,n }), or le
singleton {0n,n } est un fermé et f est continue, donc On (R) est un fermé borné de Mn (R), or Mn (R) est
de dimension finie, donc On (R) est une partie compacte de Mn (R).
III.7.a D’après la question 1.a, il existe une matrice P orthogonale telle que S = P ∆P −1 = P ∆ tP .
Ainsi, T (A) = Tr([AP ∆]P −1 ) = Tr(P −1 [AP ∆]) = Tr(B∆) en posant B = P −1 AP .
A et P sont toutes deux orthogonales et On (R) est un groupe multiplicatif, donc B est orthogonale.
III.7.b On vérifie que, pour tout (C, D) ∈ Mn (R)2 et pour tout α ∈ R,
Tr((αC + D)S) = αTr(CS) + Tr(DS), donc l’application C 7→ Tr(CS) est linéaire de Mn (R) dans
R, or Mn (R) est de dimension finie, donc c’est une application continue. Sa restriction T sur On (R)
est donc aussi continue. Mais On (R) est compact, donc T admet un maximum sur On (R).
III.7.c Avec les notations de la question 7.a, T (A) = Tr(B∆), donc en convenant de noter Mi,j
n
X n X
X n
le (i, j)-ème coefficient d’une matrice M , T (A) = (B∆)i,i = Bi,j ∆j,i , mais ∆ est diagonale,
i=1 i=1 j=1
n
X
donc T (A) = λi Bi,i .
i=1
n
X
D’après la question 5, et les λi étant positifs, T (A) ≤ λi = Tr(S).
i=1
Ainsi t ≤ Tr(S), mais de plus Tr(S) = T (In ) et In est une matrice orthogonale, donc t = Tr(S).

3
Inégalité d’Hadamard
III.8. L’inégalité demandée est une conséquence de l’inégalité aritmético-géométrique, car on sait
Yn n
X
que det(S) = λi et Tr(S) = λi .
i=1 i=1
III.9. On identifiera Mn,1 (R) avec Rn .
Soit X ∈ Rn . tXSα X = t(DX)S(DX) ≥ 0 car DX ∈ Rn et car S est symétrique positive. Ceci montre
que Sα ∈ Sn+ (R).
n
X Xn X n
X
t t
Tr(Sα ) = ( DSD)i,i = [ D]i,j Sj,k Dk,i , mais D est diagonale, donc Tr(Sα ) = αi2 si,i .
i=1 i=1 (j,k)∈{1,...,n}2 i=1

III.10. On peut appliquer l’inégalité (∗) à la matrice Sα car elle est bien dans Sn+ (R),
n
!2 n
2
Y 1 1X 1
or det(Sα ) = det(D) det(S) = αi,i det(S) et Tr(Sα ) = si,i = 1,
i=1
n n i=1 si,i
n
!2 n
Y 1 Y
donc det(S) ≤ = si,i .
α
i=1 i,i i=1
III.11. Pour tout X ∈ Rn , tXSε X = tXSX + εkXk2 ≥ 0, donc Sε ∈ Sn+ (R). De plus d’après la question
3.b, pour tout i ∈ {1, . . . , n}, 0 ≤ λ1 ≤ si,i , donc si,i + ε > 0, ce qui permet d’appliquer l’inégalité de la
Yn
question précédente à Sε : pour tout ε > 0, det(Sε ) ≤ (si,i + ε).
i=1
De plus il existe P ∈ On (R) telle que S = P ∆P −1 , où ∆ = diag(λ1 , . . . , λn ), donc Sε = P (∆ + εIn )P −1 ,
Yn n
Y Yn
ce qui prouve que det(Sε ) = (λi + ε). Ainsi, pour tout ε > 0, (λi + ε) ≤ (si,i + ε) et on conclut
i=1 i=1 i=1
en faisant tendre ε vers 0.
Application de l’inégalité d’Hadamard : détermination d’un minimum
III.12. Soit X ∈ Rn \ {0}. tXBX = t(ΩX)A(ΩX) > 0, car A ∈ Sn++ (R) et ΩX ∈ Rn \ {0} (Ω est orthogonale,
donc elle est inversible). Ainsi B ∈ Sn++ (R).
De plus Ω est orthogonale, donc d’après le cours, |det(Ω)| = 1. Or, det(A) = 1,
donc det(B) = det(Ω)2 det(A) = 1 : on a prouvé que B ∈ U.
Tr(AS) = Tr([AΩ∆] tΩ) = Tr( tΩ[AΩ∆]) = Tr(B∆).
III.13. D’après la question précédente, {Tr(AS) \ A ∈ U} ⊂ {Tr(B∆) \ B ∈ U}.
Réciproquement, soit B ∈ U. On pose A = ΩB tΩ. En adaptant la démonstration de la question précédente,
on montre que A ∈ U et que Tr(AS) = Tr(B∆), donc {Tr(AS) \ A ∈ U} = {Tr(B∆) \ B ∈ U}.
Xn
Prenons x ∈ {Tr(B∆) \ B ∈ U}. Il existe B ∈ U telle que x = Tr(B∆) = λi Bi,i . Mais B ∈ Sn++ (R),
i=1
donc d’après 3.b, pour tout i ∈ {1, . . . , n}, Bi,i > 0. Ainsi x > 0. Ceci prouve que {Tr(B∆) \ B ∈ U} est
une partie non vide de R minorée par 0. Elle possède donc une borne inférieure.
III.14. Par application de l’inégalité arithmético-géométrique,
n n
!1/n
1 1X Y
on obtient Tr(B∆) = λi bi,i ≥ λi bi,i , ce qui fournit l’inégalité demandée.
n n i=1 i=1
n n
!1/n
Y Y
III.15. Soit B = (bi,j ) ∈ U. D’après la question 11, bi,i ≥ det(B) = 1, donc bi,i ≥ 1.
i=1 i=1
n
!1/n
Y
Ainsi, d’après la question précédente, Tr(B∆) ≥ n λi = n(det(S))1/n .
i=1
III.16. Ainsi n(det(S))1/n est un minorant de {Tr(B∆) \ B ∈ U }, or la borne inférieure est le plus grand des
minorants, donc m ≥ n(det(S))1/n .
 
x1 n
Pour tout X =  ...  ∈ Rn \ {0}, tXDX =
X
µi x2i > 0, donc D ∈ Sn++ (R).
 

xn i=1

4
n n
Y det(S) X
De plus det(D) = µi = = 1, donc D ∈ U. Or Tr(D∆) = µi λi = n(det(S))1/n , donc
i=1
λ1 · · · λn i=1
m = n(det(S))1/n .

Fin du corrigé

5
e3a PSI A (3 heures)
calculatrices interdites.

Questions d’application du cours.


Parmi les affirmations suivantes, indiquez sans justification (sauf en 2.d) celles que vous
jugez vraies et celles que vous jugez fausses.
Soient a, b deux réels. On note Ea,b l’espace vectoriel des suites réelles qui vérifient la relation

∀n ∈ N, un+2 = aun+1 + bun

Q.1.
a. Pour tous réels a et b, Ea,b contient au moins une suite géométrique non nulle.
b. Pour que Ea,b contienne deux suites géométriques linéairement indépendantes, il suffit que
a = −3 et b = 4.
c. Pour que Ea,b contienne deux suites géométriques linéairement indépendantes, il faut que a = −3
et b = 4.
d. Ea,b contient deux suites géométriques indépendantes quand a = 2 et b = −1.
e. La condition a2 + 4b ≥ 0 est une condition nécessaire pour qu’il existe dans Ea,b deux suites
géométriques indépendantes.
Q.2.
a. L’application f : u ∈ Ea,b 7→ (u0 , u1 ) ∈ R2 est une application linéaire toujours surjective mais
injective seulement si a2 + 4b < 0.
b. La condition a 6= 0 est une condition suffisante pour que g : u ∈ Ea,b 7→ (u0 , u2 ) ∈ R2 soit un
isomorphisme.
c. La dimension de Ea,b est égale à deux si et seulement si a2 + 4b ≥ 0.
d. Donner, en le justifiant soigneusement, une base de l’espace vectoriel Ea,b dans le cas où a = −1
et b = −1.
n
Q.3. On considère la série entière n≥1 an xn où an = nn! et on note R son rayon de convergence.
P
an+1
a. On a limn→+∞ an = 1 et on en déduit que R = 1.
b. On a limn→+∞ an+1
an > 1 et on en déduit que la série entière diverge pour toute valeur du réel
x.
c. On a R < 1/2.
P n sin(n)
Q.4. On considère la série entière n≥1 an x où an = n et on note r son rayon de convergence.
a. On a : ∀n ∈ N∗ , |an | ≤ 1
n et donc r ≤ 1.
b. On a r ≥ 1.
n−1
P
c. Le rayon de convergence de la série entière n≥1 sin(n)x vaut 1 et donc r = 1.
d. On a r ≥ 1/2.
 
 P+∞ sin(n) x−cos(1)
e. Pour tout x ∈ − 12 , 12 ,

n=1 n = arctan sin(1) .
 1 1  P+∞ sin(n)
f. Pour tout x ∈ − 2 , 2 , n=1 n = − ln |1 − xei |.
 1 1  P+∞ sin(n) 1
g. Pour tout x ∈ − 2 , 2 , n=1 n = 2 ln(1 − 2x cos(1) + x2 ).

1
Problème.
Dans tout le problème, E désigne l’espace vectoriel des suites réelles.
On pourra noter une suite u ∈ E sous la forme u = (u0 , u1 , u2 , . . . , un , . . . ) ou sous la forme u = (un ).
Une suite u de E est dite périodique de période p ∈ N∗ lorsqu’elle vérifie : ∀n ∈ N, un+p = un .

Partie A.
Soit l’ensemble S0 = {u ∈ E/ ∀n ∈ N, un+2 + un = 0}.
1. Soient les deux suites λ et µ définies par : ∀n ∈ N, λn = cos n π2 et µn = sin n π2 .
 

1.1 Vérifier que λ et µ sont des éléments de S0 .


1.2 Montrer que ces deux suites sont périodiques.
2. Montrer que S0 est un sous-espace vectoriel de E.
3. Donner une base de S0 et préciser sa dimension.
4. Soit u ∈ S0 non nulle.
4.1 La suite u est-elle convergente ?
P
4.2 La série un de terme général un est-elle convergente ?
4.3 Soit f la fonction de la variable réelle x donnée par f (x) = +∞ n
P
n=0 un x .
Donner l’ensemble de définition de f et une expression de f (x) à l’aide des fonctions usuelles
et des termes u0 et u1 .

Partie B.
Soit S = {u ∈ E/ ∃ a ∈ R, ∀n ∈ N, un+2 + un = 2a}, c’est à dire l’ensemble des suites réelles u pour
lesquelles il existe une constante réelle a telle que pout tout entier naturel n, un+2 + un = 2a.
1.1.1 On prend ∀n ∈ N, un = (−1)n . Vérifier que u ∈
/ S.
1.2 On prend ∀n ∈ N, un = (−1)E(n/2) où E(t) désigne la partie entière du réel t. Vérifier que
u ∈ S et préciser la valeur du réel a correspondant.
1.3 On prend un = 5. Vérifier que u ∈ S et préciser la valeur du réel a correspondant.
2. Vérifier que les suites constantes appartiennent à S.
3. Déterminer les suites géométriques appartenant à S.
4. Montrer que S est un sous-espace vectoriel de E.
5. A-t-on S ⊂ S0 ? S0 ⊂ S ?
u0 +u2
6. Soit ϕ : u ∈ S 7→ ϕ(u) = 2 . Montrer que ϕ est une forme linéaire sur S. Quel est son
noyau ?
7. Soit v ∈ E définie par ∀n ∈ N, vn = 1. Montrer que S = S0 ⊕ Vect(v) où Vect(v) est la droite
vectorielle engendrée par la suite v.
8. Soit u ∈ S. Déterminer alors pour tout entier naturel n, une expression de un en fonction de n.
9. Montrer que tout élément u ∈ S est une suite périodique de période 4.
10. Prouver que l’application θ : u ∈ S 7→ θ(u) = (u0 , u1 , u2 ) ∈ R3 est un isomorphisme d’espaces
vectoriels.
On note C = (I, J, K) la base de S obtenue comme image réciproque de la base canonique de
R3 par θ :
θ(I) = (1, 0, 0), θ(J) = (0, 1, 0), θ(K) = (0, 0, 1)
11. Expliciter les cinq premiers termes de chacune des suites I, J, K.
12. Soient k ∈ N∗ et Tk : u ∈ E 7→ Tk (u) = w définie par ∀n ∈ N, wn = ukn .

2
12.1 Vérifier que Tk est un endomorphisme de E.
12.2 Le sous-espace S est-il stable par T2 ?
12.3 Le sous-espace S est-il stable par T3 ?
12.4 Ecrire la matrice, dans la base C obtenue à la question 10, de l’endomorphisme τ3 induit
par T3 sur S.
12.5 L’endomorphisme τ3 de S est-il diagonalisable ?
12.6 Reconnaı̂tre alors la nature géométrique de τ3 .
13. Soient u ∈ S et h la fonction de la variable réelle x donnée par h(x) = +∞ n
P
n=0 un x . Exprimer
h(x) à l’aide des fonctions usuelles pour x ∈] − 1, 1[. Etudier les prolongements possibles en −1
et 1.

Partie C.
Soient p ∈ N, p ≥ 2 fixé et l’espace vectoriel Sp = {u ∈ E/ ∃a ∈ R, ∀n ∈ N, un+p + un = 2a}.
1. Montrer que tout élément de Sp est périodique de période 2p.
 
0 1 0 0 ... 0
 0 0 1 0 ... 0 
 
 .. .. . . .. .. 
 . . . . . 
 ∈ Mp+1 (R)
2. Soit F = 
 .. 
 0 0 . 1 0 
 
 −1 0 . . . . . . 0 2 
0 0 ... ... 0 1
2.1 Calculer le polynôme caractéristique de la matrice F .
2.2 Déterminer les valeurs propres de F .
2.3 F est-elle inversible ?
2.4 F est-elle diagonalisable dans Mp+1 (C) ? Dans Mp+1 (R) ?
3. Prouver que l’application δ définie par ∀u ∈ Sp , δ(u) = (u0 , u1 , . . . , up−1 , a) ∈ Rp+1 où a =
u0 +up
2 , est un isomorphisme d’espaces vectoriels. Quelle est la dimension de Sp ?
On note Cp l’image réciproque de la base canonique de Rp+1 par δ.
4. Soit ψ l’application définie par

ψ : u ∈ Sp 7→ ψ(u) = t telle que ∀n ∈ N, tn = un+1

4.1 Vérifier que ψ est un endomorphisme de Sp .


4.2 Sans nouveau calcul, préciser ψ 2p = ψ ◦ · · · ◦ ψ, composée 2p fois de l’application ψ.
4.3 Ecrire la matrice de ψ dans la base Cp de Sp .
4.4 ψ est-elle diagonalisable ?
4.5 Prouver que ψ est bijective et déterminer son inverse ψ −1 .

3
e3a PSI A
un corrigé.

Questions d’application du cours.


Aucune justification n’était demandée. Je donne cependant des contre-exemples dans le cas d’une
réponse fausse et des bribes de preuve dans le cas contraire.
Q.1. Une suite géométrique non nulle a pour terme général un = λq n avec q, λ ∈ R et λ non nul (q
peut, lui, être nul et la suite est alors nulle à partir du rang 1, son terme d’indice 0 valant λ).
Une telle suite est dans Ea,b si et seulement si (en divisant par λ 6= 0)

∀n ∈ N, q n (q 2 − aq − b) = 0

Si ceci a lieu alors q 2 − aq − b = 0 (prendre n = 0). Réciproquement, cette dernière condition


donne u ∈ Ea,b . La condition nécessaire et suffisante pour que u ∈ Ea,b est donc q 2 = aq + b.
a. FAUX. a = b = −1 donne un contre-exemple car aucun réel n’est racine de X 2 + X + 1.
b. VRAI. X 2 + 3X − 4 = (X + 4)(X − 1) et ((−4)n ) et (1) sont deux suites géométriques
indépendantes de E−3,4 .
c. FAUX. E0,1 contient les deux suites géométriques indépendantes (1) et ((−1)n ).
d. FAUX. X 2 − 2X + 1 = (X − 1)2 et les seules suites géométriques de E2,−1 sont les (λ1n ) c’est
à dire les suites constantes (qui forment un espace vectoriel de dimension 1).
e. VRAI. Si Ea,b contient deux suites géométriques indépendantes, leurs raisons sont différentes
(pour l’indépendance) et sont racines de X 2 − aX − b (pour l’appartenance). Ce polynôme DOIT
donc avoir deux racines réelles distinctes et on DOIT avoir a2 + 4b > 0. A fortiori, il FAUT que
a2 + 4b ≥ 0.
Q.2. Le cours indique que Ea,b est un espace vectoriel de dimension 2 et donne une méthode pratique
pour en déterminer une base. Ce sont les points abordés ici.
a. FAUX. f est bien linéaire. Elle est injective (si u0 = u1 = 0 alors tous les un sont nuls, par
récurrence). Elle est surjective car à partir d’un couple (α, β) on peut construire (par récurrence)
une suite u ∈ Ea,b telle que u0 = α et u1 = β. On peut ainsi dire que f est un ISOmorphisme et
que Ea,b est de dimension 2.
b. VRAI. g est toujours linéaire. Supposons a 6= 0 et soit u ∈ Ea,b telle que g(u) = (0, 0). On a
alors au1 = u2 − bu0 = 0 et donc (a 6= 0) u1 = 0. f étant injective, on en déduit que u = 0. g est
alors injective et, par argument de dimension, est un isomorphisme.
c. FAUX. On a vu que Ea,b est TOUJOURS de dimension 2.
d. Soit j = e2iπ/3 . La suite complexe (j n ) vérifie la relation de récurrence pour a = b = −1. En
passant aux parties réelle et imaginaire, on obtient que (cos(2nπ/3)) et (sin(2nπ/3)) vérifient
aussi la relation.√Comme ce sont des suites réelles, elles sont dans E−1,−1 . Leurs images par f sont
(1, −1/2) et (0, 3/2). Ces images étant indépendantes, les suites le sont. Par dimension/cardinal
elles forment une base de E−1,−1 .
Q.3. Pour étudier la série entière, on peut utiliser la définition (avec le Lemme d’Abel) ou la règle
n
de D’Alembert. Dans le premier cas, on remarque (formule de Stirling) que an ∼ √e2πn ; ainsi,
(an xn ) est bornée si |xe| < 1 et non bornée si |xe| > 1 et le rayon de convergence vaut R = 1/e.
xn+1 |
Avec la seconde méthode, on remarque que les ak sont non nuls et que pour x 6= 0, |an+1 |an xn | =
1 n n
 P
|x| 1 + n → e|x| ; si |xe| < 1, (an x ) converge absolument et si|xe| > 1, la même série
diverge grossièrement ce qui redonne R = 1/e.
a. FAUX. Le rapport tend vers e.

1
b. FAUX. C’est la déduction qui est erronée.
c. VRAI. e > 2 et donc R = 1/e < 1/2. Sans calculer le rayon de convergence, il suffirait de
montrer que (an /2n ) est non bornée pour conclure.
Q.4.
1
a. FAUX. bn = n! vérifie |bn | ≤ n1 pour tout n mais le rayon de convergence de la série entière
associée est infini et donc strictement plus grand que 1. On a effectivement |an | ≤ n1 et donc
(an xn ) est bornée si |x| ≤ 1. On peut en déduire que r ≥ 1.
b. VRAI. On vient de le justifier.
c. Vrai. (sin(n)xn−1 ) est bornée si |x| < 1 et non bornée si |x|P> 1 (car (sin(n)) n’est pas de
limite nulle, ce que l’on pourrait prouver par l’absurde) et (sin(n)xn−1 ) a donc un rayon
n
P
de convergence qui vaut 1. (an x ) étant la série entière primitive, elle a le même rayon de
convergence r = 1.
d. VRAI. 1 est bien supérieur à 1/2. Il nous suffit de remarquer que (an /2n ) est bornée pour
conclure que r ≥ 1/2.
e. FAUX. L’égalité est fausse en x = 0 car arctan(− cos(1)/ sin(1)) 6= 0.
n−1 qui est la partie imaginaire de in n−1
P P
f. FAUX. Pour |x| < 1, on a n≥1 sin(n)x n≥1 e x
ei
qui vaut (somme de série géométrique de raison xei ) 1−xe n−1 =
P
i . On a donc n≥1 sin(n)x
sin(1) x−cos(1)
x2 −2x cos(1)+1
. Par ailleurs, − ln(|1 − xei |) = − 12 ln(1 + x2 − 2x cos(1)) se dérive en − 1−2x cos(1)+x2
.
sin(1)
Ce terme n’étant pas égal à x2 −2x cos(1)+1
(par exemple en 0), l’égalité proposée est fausse (si
elle avait lieu, les dérivées devraient être égales).
g. FAUX. Sauf erreur de calcul de ma part, on conclut de manière similaire. La fonction proposée
ressemble plus à la somme de la série entière de terme général cos(n)
n (fais-je une erreur ?).

Problème.
Partie A.
1.1 Comme cos(x + π) = − cos(x) et sin(x + π) = − sin(x), on a immédiatement λn+2 = −λn et
µn+2 = −µn pour tout n. Ainsi, λ, µ ∈ S0 .
1.2 On en déduit que pour tout n, λn+4 = λn et µn+4 = µn . λ et µ sont donc périodiques de période
4.
2. S0 est non vide (il contient la suite nulle) et stable par combinaisons linéaires. Comme c’est un
sous-ensemble de E, c’est donc un sous-espace vectoriel de E.
3. Les éléments de S0 sont les suites récurrentes linéaires d’ordre 2 à coefficients constants d’équation
π
caractéristique r2 + 1 = 0. Les solutions de cette dernière étant e±i 2 , le cours indique que (λ, µ)
est une base de S0 et que cet espace est de dimension 2.
4.1 Soit u ∈ S0 . u est combinaison linéaire de λ et µ. En regradant les termes d’indice 0 et 1, on
trouve les coefficients de la combinaison et on trouve
 π  π
∀n ∈ N, un = u0 cos n + u1 sin n
2 2
u étant non nulle u0 ou u1 est non nul. Or, u4n = u0 , u4n+1 = u1 , u4n+2 = −u0 et u4n+3 = −u1 .
On trouve donc deux extraites qui convergent vers des limites différentes. La suite u ne converge
donc pas.
P
4.2 La série un est donc grossièrement divergente (le terme général n’est pas de limite nulle).

2
4.3 u est bornée et donc ∀x ∈ [−1, 1], (un xn ) est bornée. Par ailleurs si x > 1, l’une des extraites de
(|un xn |) est de limite infinie ((|u4n x4n |) ou (|u4n+1 x4n+1 |) selon que u0 6= 0 ou u1 6= 0) et (un xn )
n’est donc pas bornée. f est donc la somme d’une série entière de rayon de convergence égal à
1. Comme f (1) et f (−1) n’existent pas (divergence grossière de série), le domaine de définition
de f est ] − 1, 1[.
On remarque que
   
4n+3
X Xn Xn X n Xn
∀n ∈ N, ∀x ∈] − 1, 1[, uk xk = u0  x4k − x4k+2  + u1  x4k+1 − x4k+3 
k=0 p=0 p=0 p=0 p=0

On sait calculer les sommes (géométriques de raison x4 ) et un passage à la limite donne


(1 − x2 )(u0 + u1 x) u0 + u1 x
∀x ∈] − 1, 1[, f (x) = 4
=
1−x 1 + x2

Partie B.
1.1 Supposons, par l’absurde, que u ∈ S et notons a le réel associé. On a 2a = u2 + u0 = 2 et
2a = u1 + u3 = −2 ce qui amène une contradiction. Ainsi u ∈ / S.
1.2 On a ici u4n = u4n+1 = 1 et u4n+2 = u4n+3 = −1 pour tout n. On en déduit que un+2 + un = 0
pour tout n (par exemple en distinguant suivant la congruence modulo 4 de n). On a donc u ∈ S
et la constante correspondante est nulle.
1.3 On a ici un = un+2 = 10 pour tout n. On a donc u ∈ S et la constante correspondante est 5.
2. Le même calcul montre que toute suite constante est dans S avec une constante égale à u0 .
3. Soit u une suite géométrique. Il existe des réels q et λ tels que ∀n, un = λq n .
- Si u ∈ S alors u0 + u2 = u1 + u3 et donc λ(1 + q 2 ) = λq(1 + q 2 ). On en déduit que λ = 0 ou
q = 1. Dans les deux cas, u est constante.
- Réciproquement, les suites constantes sont dans S.
Les suites géométriques qui sont dans S sont exactement les suites constantes.
4. On sait déjà que S est non vide et inclus dans E. Si u et v sont dans S associées à des constantes
a et b et si λ ∈ R alors pour tout n, (u + λv)n+2 + (u + λv)n = a + λb. Ainsi u + λv ∈ S
et la constante associée est a + λb. S est donc aussi stable par combinaisons linéaires et c’est
finalement un sous-espace vectoriel de E.
5. De façon immédiate, on a S0 ⊂ S (est élément de S0 est dans S de constante associée nulle).
L’inclusion réciproque est fausse puisque la suite constante égale à 1 est dans S sans être dans
S0 .
6. On a immédiatement ϕ(λu + v) = λϕ(u) + ϕ(v) c’est à dire la linéarité de ϕ. Comme ϕ est à
valeur dans R, c’est une forme linéaire. On notera que c’est l’application qui à un élément de S
associe la constante correspondante de la définition.
Les éléments du noyau de ϕ sont les élément de S correspondant à une constante nulle et donc
ker(ϕ) = S0
7. v est dans S mais pas dans l’hyperplan ker(φ). C’est donc un vecteur qui engendre un supplémentaire
de cet hyperplan (les hyperplans sont les sous-espaces de codimension 1) :
S = S0 ⊕ Vect(v)
8. Soit u ∈ S et a = u0 +u
2 . u − av est alors un élément de S0 et est donc combinaison linéaire de
2

λ et de µ définies en partue A. Les termes d’indice 0 et 1 donnent les valeurs des constantes et
on obtient finalement
u0 + u2 u0 − u2  nπ  2u − u − u
1 0 2
 nπ 
∀n ∈ N, un = + cos + sin
2 2 2 2 2

3
9. Un élément de S est combinaison linéaire des trois suites v, λ, µ qui sont périodique de période
4. Tout élément de S est donc aussi périodique de période 4.
10. S est de dimension 3 (comme somme directe d’un espace de dimension 2 et d’un autre de
dimension 3). θ est immédiatement linéaire. De plus, si u ∈ ker(θ) alors les trois premiers termes
de u sont nuls. On a aussi u1 + u3 = u0 + u2 et donc u3 = 0. Par 4-périodicité, les un sont tous
nuls. θ est donc une application linéaire injective entre deux espaces de même dimension : c’est
un isomorphisme.
11. Si u ∈ S, u3 = u0 + u2 − u1 . On peut alors continuer par 4-périodicité
I = (1, 0, 0, 1, 1, 0, 0, 1, . . . ), J = (0, 1, 0, −1, 0, 1, 0, −1 . . . ), K = (0, 0, 1, 1, 0, 0, 1, 1 . . . )

12.1 La linéarité de Tk est immédiate ((u + λv)kn = ukn + λvkn est vrai pour tout n). Tk allant de E
dans E, c’est un endomorphisme de E.
12.2 w = T2 (I) = (1, 0, 1, 0, . . . ) n’est pas dans S (w0 + w2 = 0 6= w1 + w3 ). S n’est donc pas stable
par T2 .
12.3 T3 (I) = (1, 1, 0, 0, 1, 1, 0, 0, . . . ) = I + J, T3 (J) = (0, −1, 0, 1, 0, −1, 0, 1, · · · = −J et T3 (K) =
(0, 1, 1, 0, 0, 1, 1, 0, . . . ) = J + K. Les éléments d’une base de S étant envoyés dans S par l’appli-
cation linéaire T3 , S est stable par T3 .
12.4 Le calcul précédente donne  
1 0 0
Mat(T3 , C) =  1 −1 1 
0 0 1
12.5 I − K et I + J + K sont vecteurs propres indépendants de T3 associés à la valeur propre 1. J
est vecteur propre de T3 associé à la valeur propre −1. Les sous-espaces propres étant en somme
directe, on doit avoir les égalités
Sp(T3 ) = {1, −1}, E1 (T3 ) = Vect(I − K, I + J + K), E−1 (T3 ) = Vect(J)

12.6 T3 est la symétrie par rapport à Vect(I − K, I + J + K) de direction Vect(J).


13. Soit u ∈ S et a = u0 +u
2 . On a w = u − av ∈ S0 . On en déduit que (toutes les quantités écrites
2

existent)
+∞
X +∞
X
k
∀x ∈] − 1, 1[, h(x) = wk x + a xk
k=0 k=0
w0 +w1 x a
Avec la question 4.3 de la partie A, la première somme vaut 1+x2
. La seconde vaut 1−x . Avec
les valeurs de w0 et w1 on obtient donc
u0 + u2 (u0 − u2 ) + (2u1 − u0 − u2 )x
∀x ∈] − 1, 1[, h(x) = +
2(1 − x) 2(1 + x2 )
On imagine que l’énoncé veut ensuite parler de prolongement par continuité en 1 et −1, c’est à
dire qu’il faut voir si h admet une limite finie en 1 ou −1.
C’est toujours le cas en −1, la limite valant 3u0 −2u
4
1 +u2
.
Il y a une limite finie en 0 si et seulement si u0 + u2 = 0 c’est à dire u ∈ S0 et dans ce cas, la
limite vaut alors u0 +u
2 .
1

Partie C.
1. Il s’agit de généraliser le résultat vu en question 9 de la partie B (cas p = 2). Soit u ∈ Sp et a
la constante associée ; pour tout entier n, on a
un+p + un = a = un+2p + un+p
et ainsi un+2p = un , ce qui montre que u est 2p-périodique.

4
2.1 Un développement par rapport à la dernière ligne donne

−x 1 0 ... 0
.. ..
0 −x 1 . .
det(F − xIp+1 ) = (1 − x) .. .. .. ..
. . . . 0
.. ..
0 . . 1
−1 0 ... 0 −x

où le déterminant est de taille p. On développe ce dernier par rapport à la première colonne pour
obtenir
det(F − xIp+1 )) = (1 − x) (−x)p − (−1)p+1 = (−1)p+1 (x − 1)(xp + 1)


2.2 Les valeurs propres de F sont les racines de son polynôme caractéristique.
- Les valeurs propres complexes sont 1 et les racines p-ièmes de −1 et on a donc
(2k+1)π
i
SpC (F ) = {1} ∪ {e p / 0 ≤ k ≤ p − 1}

- Si p est pair, SpR (F ) = {1}. Si p est impair, SpR (F ) = {1, −1}.


2.3 0 n’étant pas valeur propre de F , F ∈ GLp+1 (R).
2.4 F possède p + 1 valeurs propres complexes et estde taille p + 1. Elle est donc C-diagonalisable à
sous-espaces propres de dimension 1 (car les sous-espaces propres sont en somme directe).
Il y a au plus deux sous-espaces propres réels qui sont au plus de dimension 1 (car la multiplicité
de chaque valeur propre est égale à 1). Ainsi, la somme des dimension des sous-espaces propres
réels de F est au plus 2 et donc différente de p + 1. F n’est pas diagonalisable dans R.
3. δ est immédiatement linéaire.
Si u ∈ ker(δ) alors u0 = · · · = up−1 = u0 +up = 0. La constante a associée à u vaut (u0 +up )/2 = 0
et on a donc ∀n, un+p = −un . On en déduit que up = · · · = u2p−1 = 0. u est 2p-périodique et
ses 2p premiers termes sont nuls. u est donc la suite nulle. δ est donc injective (noyau restreint
à la suite nulle).
Réciproquement, soit (a0 , . . . , ap−1 , x) ∈ Rp+1 . On définit u de manière récurrente par

∀i ∈ [0..p − 1], ui = ai ; ∀n ≥ p, un = 2x − un−p

Il est aisé de voir que cette suite est bien définie. Par définition, u ∈ Sp (associée à la constante
x) et δ(u) = (a0 , . . . , ap−1 , x). On a donc montré la surjectivité de δ.
δ est finalement un isomorphisme et

dim(Sp ) = dim(Rp+1 ) = p + 1

4.1 ψ est linéaire. Si u ∈ Sp (avec une constante a) et t = ψ(u) alors ∀n, tn +tn+p = un+1 +un+p+1 =
2a. On a donc t ∈ Sp (avec la même constante a).
ψ est donc un endomorphisme de Sp .
4.2 Les éléments de Sp étant 2p-périodiques, on a ψ 2p = IdSp .
4.3 Soit x = (x0 , . . . , xp ) ∈ Rp+1 ; on a (les termes non précisés n’ayant pas d’importance)

δ −1 (x) = (x0 , . . . , xp−1 , 2xp − x0 , 2xp − x1 , . . . )

ψ(δ −1 (x)) = (x1 , . . . , xp−1 , 2xp − x0 , 2xp − x1 , . . . )


δ(ψ(δ −1 (x))) = (x1 , . . . , xp−1 , 2xp − x0 , xp )
En notant (e0 , . . . , ep ) les éléments de la base canonique de Rp+1 et Ei = δ −1 (Ei ), on a donc

ψ(δ −1 (x)) = x1 E0 + · · · + xp−1 Ep−2 + (2xp − x0 )Ep−1 + xp Ep

5
La première colonne de la matrice cherchée est constituée des coefficients obtenus quand x = e0
c’est à dire x0 = 1 et x1 , . . . , xp = 0.
Plus généralement la colonne i de la matrice cherchée est constituée des coefficients obtenus
quand x = ei c’est à dire xk = δi,k .
Avec les formules obtenues, on obtient que la matrice cherchée est F .
4.4 F n’étant pas diagonalisable dans R, ψ n’est pas diagonalisable.
4.5 F étant inversible, ψ est un isomorphisme. χF annule ψ et donc ψ p+1 − ψ p + ψ − Id = 0. Ainsi

ψ −1 = ψ p − ψ p−1 + Id

On a donc ψ −1 (u) = w avec wn = un+p − un+p−1 + wn .

6
Tournez la page S.V.P.
Tournez la page S.V.P.
IN CHOISY – 12 1020 – D’après documents fournis
Corrigé de E3A 2012 PC math A

Première partie
 
2x − 2y − z
1. On calcule AXM = 19  2(2x − 2y − z)  puis t XM AXM = 19 (2x − 2y − z)(x + 2y − 2z). On a donc
−2(2x − 2y − z)
t
XM AXM = 0 ⇔ (2x − 2y − z)(x + 2y − 2z) = 0 ⇔ 2x − 2y − z = 0 ou x + 2y − 2z = 0.
S est donc la réunion de deux plans qui sont perpendiculaires puisque leurs vecteurs normaux de coor-
données (2, −2, −1) et (1, 2, −2) sont orthogonaux.
2. (a) On vérifie que les trois vecteurs sont orthogonaux deux à deux et de norme égale à 1.
       
1 0 2 0
A × 31  2  =  0  donc f (~e1 ) = 0. De même A × 13  1  =  0  donc f (~e2 ) = 0. Enfin
 −2  0
  2 0
2 9
A × 31  −2  = 27 1 
18  donc f (~e3 ) = ~e1 .
−1 −18
(b) La matrice de fdans la base 
C est donc égale à U . A est donc semblable à U avec pour matrice de
1 2 2
passage P = 13  2 1 −2 . On a bien U = P −1 AP avec P orthogonale puisque c’est la matrice
−2 2 −1
de passage de la base canonique à la base C orthonormale.
(c) La matrice U est triangulaire. Elle admet 0 comme valeur propre triple. Si elle était diagonalisable,
elle serait semblable à la matrice nulle et serait donc égale à 0. Ce n’est pas le cas donc A, semblable
à U , n’est pas diagonalisable.
(d) (i) XM = P XM 0 .
(ii) t XM AXM = t XM 0 t P AP XM 0 = t XM 0 P −1 AP XM 0 = t XM 0 U XM 0 = x0 z 0 puisque t P = P −1 . Le
point M de coordonnées (x0 , y 0 , z 0 ) dans R0 est donc dans S si et seulement si x0 z 0 = 0.
x0 = 0 est l’équation du plan (Oy 0 z 0 ) et z 0 = 0 celle du plan (Ox0 y 0 ). S est bien la réunion de
deux plans perpendiculaires.

3. (a) Le rang de f est égal au rang de U donc égal à 1. U 2 = 0 donc f ◦ f = 0.


(b) Si~v a pour coordonnées
 (x, y, z) dans labase canonique, alors f (~v ) a pour coordonnées AXM =
2x − 2y − z 1
1 
2(2x − 2y − z)  = 2x−2y−z  2  donc f (~v ) = 2x−2y−z ~e1 . On peut donc prendre par
9 9 3
−2(2x − 2y − z) −2
exemple: ~c = 31 ~e1 et ϕ(~v ) = 2x − 2y − z.

Deuxième partie

1. (a) vn+1 + 2wn+1 = − 12 (vn + 2wn ). La suite (vn + 2wn ) est donc une suite géométrique de raison − 12 .
Son premier terme vaut v0 + 2w0 = 0. On en déduit que pour tout n > 0: vn + 2wn = 0.
(b) un+1 + 3wn+1 = − 21 (vn + 2wn ) = 0. On a donc un + 3wn = 0 pour tout n > 1.
(c) Avec le (a) et le (b) on déduit pour n > 1: wn+1 = 41 (−3wn + 2wn − wn ) = − 21 wn . La suite (wn )n>1
est donc géométrique de raison − 12 et de premier terme w1 = 14 (−1 − 2 + 1) = − 21 . On en déduit
pour n > 1: wn = (− 12 )n−1 (− 12 ) = (− 12 )n .
D’où vn = −2(− 12 )n et un = −3(− 12 )n .
Les trois suites convergent donc vers 0.
 
−3 1 −1
2. (a) La définition des trois suites donne: M = 41  −2 0 −2 .
1 −1 −1
 
−3 − 4λ 1 −1 −3 − 4λ −4λ − 2 4λ + 2
(b) det(M − λI3 ) = det( 14  −2 −4λ −2 ) = 413 −2 −4λ − 2 0 par les
1 −1 −1 − 4λ 1 0 −4λ − 2
opérations C2 ← C2 + C1 et C3 ← C3 − C1 .

1
−4λ 0 0
1
Par l’opération L1 ← L1 − L2 + L3 on obtient det(M − λI3 ) = −2 −4λ − 2
64
0 =
1 0 −4λ − 2
λ
− 16 (4λ + 2)2 . Les valeurs propres de M sont 0 (simple) et − 12 (double).
 
−1 1 −1
M X = − 21 X ⇐⇒ 14  −2 2 −2  X = 0 ⇐⇒ x − y + z = 0. Le sous-espace propre associé à − 12
1 −1 1
est de dimension 2 et a pour base ~u1 = (1, 1, 0), ~u2 = (1, 0, −1).
 
−3 1 −1
M X = 0 ⇐⇒ 14  −2 0 −2  X = 0 ⇐⇒ z = −x et y = 2x. Le sous-espace propre associé à 0
1 −1 −1
est de dimension 1 et a pour base ~u3 = (1, 2, −1).
 1 
−2 0 0
M est donc diagonalisable, semblable à D =  0 − 12 0 , avec une matrice de passage égale à
  0 0 0
1 1 1
P = 1 0 2 .
0 −1 −1
 1 n   
(− 2 ) a a
(c) On en déduit Xn = M n X0 = P Dn P −1 X0 d’où Yn = P −1 Xn = Dn Y0 =  (− 12 )n b  si Y0 =  b .
0 c
Par suite Yn a pour limite le vecteur nul, Xn aussi et on retrouve que les suites (un ), (vn ), (wn )
convergent vers 0.

Troisième partie

1. f est une application de E dans E, linéaire puisque f (λ~x + µ~y ) = u(λ~x + µ~y )~a = λu(~x)~a + µu(~y )~a =
λf (~x) + µf (~y ).
Im(f ) = Vect(~a) car u n’étant pas l’application nulle, il existe ~x tel que u(~x) 6= 0. Comme ~a 6= 0, le rang
de f est égal à 1.
2. (a) f (~x) = 0 ⇐⇒ u(~x) = 0 ⇐⇒ ~x ∈ Ker(u). Comme u est une forme linéaire non nulle, dim(Ker(u)) =
n − 1 > 1. 0 est donc bien valeur propre de f , le sous-espace propre est Ker(u) de dimension n − 1.
(b) De f (~x) = λ~x on déduit ~x = λ1 u(~x)~a donc ~x est colinéaire à ~a. De f (~a) = u(~a)~a on déduit λ = u(~a).
(c) Premier cas: u(~a) = 0. f a pour seule valeur propre 0, le sous-espace propre associé est Ker(u) de
dimension n − 1. f n’est donc pas diagonalisable.
Deuxième cas: u(~a) 6= 0. f a deux valeurs propre, 0 et u(~a). Le sous-espace propre associé à 0 est
Ker(u) de dimension n − 1, celui associé à u(~a) est Vect(~a) de dimension 1. f est donc diagonalisable.
(d) f est diagonalisable si et seulement si u(~a) 6= 0.

3. (a) Montrons par récurrence sur p > 1 la propriété P (p): f p (~x) = u(~x)(u(~a))p−1~a. P (1) est vérifiée par
définition de f .
Supposons P (p) vraie. f p+1 (~x) = f (f p (~x)) = u(~x)(u(~a))p−1 f (~a) = u(~x)(u(~a))p~a puisque f (~a) =
u(~a)~a. Donc P (p + 1) est vraie.
(b) f est diagonalisable si et seulement si il existe un polynôme Q scindé à racines simples tel que
Q(f ) = 0.
(c) Si u(~a) = 0, f 2 (~x) = f (u(~x)~a) = u(~x)f (~a) = u(~x)u(~a)~a = 0. On a donc f 2 = 0. Si f était
diagonalisable, il existerait une base dans laquelle sa matrice serait diagonale D avec D2 = 0, donc
on aurait D = 0 et f serait nul, ce qui est exclu puisque f a pour rang 1. f n’est donc pas
diagonalisable.
(d) Si u(~a) 6= 0, on a d’après le (a) pour p = 2: f 2 (~x) = u(~x)u(~a)~a = u(~a)f (~x) pour tout vecteur ~x. On
en déduit que f 2 − u(~a)f = 0. Le polynôme Q = X 2 − u(~a)X = X(X − u(~a)) est scindé à racines
simples et vérifie Q(f ) = 0. f est donc diagonalisable.

4. Im(g) = Vect(~b) avec ~b 6= 0. On peut donc définir une application v de E dans R par g(~x) = v(~x)~b.
Montrons que v est linéaire. D’une part, g(λ~x + µ~y ) = v(λ~x + µ~y )~b. D’autre part, g(λ~x + µ~y ) =

2
λg(~x) + µg(~y ) = (λv(~x) + µv(~y ))~b. Puisque ~b 6= 0 on déduit que v(λ~x + µ~y ) = λv(~x) + µv(~y ). v est donc
une forme linéaire, non nulle (sinon on aurait g = 0).

5. g 2 (~x) = v(~x)g(~b) = v(~x)v(~b)~b. Si g 2 6= 0 on a donc v(~b) 6= 0. En utilisant le résultat du 2.(c) on déduit que
g est diagonalisable de valeurs propres 0 (d’ordre n − 1) et v(~b) d’ordre 1. On obtient la matrice diagonale
demandée avec α = v(~b).
6. (a) On peut compléter une famille libre de E par des vecteurs de E pour obtenir une base de E.
(b) Le vecteur g(~en ) est non nul et appartient à Ker(g) puisque g 2 = 0. Il forme donc une famille libre
que l’on peut compléter par des vecteurs ~e2 , ..., ~en−1 de Ker(g) pour obtenir une base de Ker(g) (par
le théorème de la base incomplète).
(c) Puisque le vecteur ~en n’est pas dans l’hyperplan Ker(g), la famille B = (g(~en ), ~e2 , ..., ~en ) est une
famille libre possédant n vecteurs; c’est donc une base de E. La matrice de g dans cette base est
bien celle qui est proposée.

7. Si deux matrices sont semblables alors elles ont la même trace.


Nous venons de montrer qu’une matrice de rang 1 est soit semblable à la matrice diagonale diag(0, ..., 0, α)
avec α 6= 0, soit semblable à la matrice élémentaire E1,n de trace nulle. Si deux matrices de rang 1 ont
la même trace α, alors soit α 6= 0 et les deux matrices sont semblables à diag(0, ..., 0, α), soit α = 0 et les
deux matrices sont semblables à E1,n . Dans les deux cas elles sont semblables entre elles.

Quatrième partie

u(~
x)
1. Si h(~x) = 0 alors ~x = a) ~
u(~ a ∈ Vect(~a) (on a supposé u(~a) 6= 0). Donc Ker(h) ⊂ Vect(~a).
De h(~a) = 0 on déduit que Vect(~a) ⊂ Ker(h). On a donc bien Ker(h) = Vect(~a).
On calcule u(h(~x)) = u(~a)u(~x) − u(~x)u(~a) = 0. Donc Im(h) ⊂ Ker(u). Réciproquement, si u(~x) = 0 alors
1
h(~x) = u(~a)x d’où ~x = u(~ x) ∈ Im(h). On a donc bien montré que Im(h) = Ker(u).
a) h(~

2. h(~x) = λ~x ⇐⇒ (u(~a) − λ)~x = u(~x)~a.


Si λ = u(~a) on obtient u(~x) = 0: λ = u(~a) est donc valeur propre de sous-espace propre associé égal à
Ker(u).
Si λ 6= u(~a) on obtient que ~x est colinéaire à ~a; comme h(~a) = 0, 0 est valeur propre de sous-espace
propre associé égal à Vect(~a). Si E est de dimension finie n, h est diagonalisable puisque la somme des
dimensions des sous-espaces propres est égale à n − 1 + 1 = n, dimension de E.

3. Montrons par récurrence sur p > 1 la propriété P (p): hp (~x) = (u(~a))p−1 h(~x).
P (1) est vérifiée car (u(~a))0 = 1.
Supposons P (p) vraie. hp+1 (~x) = h(hp (~x)) = (u(~a))p−1 h2 (~x). Or h2 (~x) = u(~a)h(~x) puisque h(~a) = 0. On
en déduit hp+1 (~x) = (u(~a))p h(~x). Donc P (p + 1) est vraie.
4. ||hp (~x)|| = |u(~a)|p−1 ||h(~x)|| a pour limite 0 quand p tend vers l’infini puisque |u(~a)| < 1.

5. (a) Nous allons appliquer le résultat du 4. en prenant pour E l’ensemble des applications continues de
[0, π] dans R muni de la norme ||f ||∞ = supt∈[0,π] |f (t)|, pour u la forme linéaire sur E définie par

u(f ) = 0 f (t)dt et pour ~a la fonction définie par a(t) = sin(3t).

On obtient u(a) = 0 sin(3t)dt = − 31 (cos(3π) − cos(0)) = 23 ; on a donc bien |u(a)| < 1. En posant
h(f ) = u(a)f − u(f )a on obtient fp+1 = h(fp ), donc fp = hp (f0 ) a sa norme qui tend vers 0, donc
pour tout t ∈ [0, π], fp (t) tend vers 0 quand p tend vers l’infini.
(b) Appliquons à nouveau le résultat du 4. en prenant E = R3 , pour u la forme linéaire définie par
u(x, y, z) = (x − y + z) et ~a = ( 41 , 12 , − 14 ).
On obtient u(~a) = − 12 donc on a donc bien |u(a)| < 1. L’application h est définie par h(x, y, z) =
− 21 (x, y, z)−(x−y +z)~a = (− 34 x+ 14 y − 41 z, − 12 x− 12 z, 14 x− 41 y − 14 z). On a donc (up+1 , vp+1 , wp+1 ) =
h(up , vp , wp ) et par suite (up , vp , wp ) = hp (u0 , v0 , w0 ) a sa norme qui tend vers 0 quand p tend vers
l’infini. On retrouve bien que les suites (un ), (vn ) et (wn ) convergent vers 0.

3
Mines-Ponts 2012. Option MP. Mathématiques I.
Corrigé pour serveur UPS par JL. Lamard (jean-louis.lamard@prepas.org)

A. Équations algébriques réciproques.


P
n P
n
1. Si P = ak X k il vient un (X) = an−k X k ce qui prouve que un est bien une application de Rn [X] dans
k=0 k=0 S
lui-même. Elle est S
clairement linéaire et involutive donc un est bien une symétrie de Rn [X] par rapport à P {0}
parallèlement à D {0}. 
P
n
2. Il résulte immédiatement de ce qui précède que P = ak X k appartient à P (resp. D) si et seulement ak = an−k
k=0
(resp ak = −an−k ) pour tout entier k entre 0 et Int(n/2). 
3. Soit R réciproque (donc non nul). Si R est constant alors R = a0 6= 0 et si R est de degré n > 1 alors a0 = ±an 6= 0
donc dans les deux cas 0 n’est pas racine de R. 
Soit désormais R un polynôme réciproque admettant une racine x. Alors R est forcément de degré n > 1 d’après
1 1 1
ce qui précède et R( ) = ± n R(x) = 0 donc est également racine de R. 
x x x
n 1
Soit R ∈ D de degré n. Il vient X R( ) = −R(X) donc R(1) = −R(1). Ainsi 1 est racine de tout élément de D.
X

1
Soit R ∈ P de degré 2n + 1. Il vient X 2n+1 R( ) = R(X) donc −R(−1) = R(−1). Ainsi −1 est racine de tout
X
élément de P de degré impair. 
1 1
4 • Supposons P et Q réciproques de degrés respectifs p et q. Il vient X p P ( ) = ε1 P (X) et X q Q( ) = ε2 Q(X)
X X
avec ε1 = ±1 suivant l’espèce de ces deux polynômes. Alors R est de degré n = p + q et :
1 1 1
X n R( ) = X p+q P ( )Q( ) = ε1 ε2 P (X)Q(X) = ε1 ε2 R(X) donc R est réciproque et son espèce est donnée par
X X X
ε1 ε2 .
• Supposons désormais R et par exemple P réciproques. On note toujours p et q les degrés de P et de Q, l’espèce de
1 1
R étant donnée par ε. Il vient X p+q P ( )Q( ) = εP (X)Q(X) d’une part et
X X
1 1 1 1 1
X p+q P ( )Q( ) = X p P ( )X q Q( ) = ε1 P (X)X q Q( ) d’autre part.
X X X X X
1 εP (X)Q(X)
Ainsi la fraction rationnelle X q Q( ) est-elle égale à = εε1 Q(X) ce qui prouve que Q est réciproque
X ε1 P (X)
d’espèce donnée par εε1 .
• En conclusion si R = P Q et si deux de ces trois polynômes sont réciproques alors le troisième l’est également et
soit ils sont tous trois de première espèce soit deux sont de seconde espèce et le troisième de première espèce. 
5. On note que X − 1 ∈ D donc si P ∈ P alors D = (X − 1)P ∈ D par la question précédente.
Réciproquement si D appartient à D alors 1 est racine (question 3.) donc il existe un unique polynôme P tel que
D = (X − 1)P et la question précédente implique que P ∈ P.
En conclusion D ∈ D si et seulement si il existe P ∈ P tel que D = (X − 1)P . 
6. Si P ∈ P est de degré impair alors 1 est racine et on prouve exactement comme ci-dessus :
Un polynôme P de degré impair est élément de P si et seulement si il existe D ∈ D tel que P = (X + 1)D 
7. L’unicité en cas d’existence est évidente car si P et Q sont deux polynômes convenant ils prennent la même valeur
en une infinité de réels.
Pour l’existence on raisonne par récurrence sur p.
1 1
Soit le prédicat Ep :hh Il existe un polynôme Pp de degré p tel que X p + = Pp (X + ) ii
Xp X
E0 et E1 sont vrais. Supposons Ek vrai jusqu’au rang p avec p > 1. Il vient alors :
1 1 1 1 1 1 1
X p+1 + p+1 = (X + )(X p + p ) − (X p−1 + p−1 ) = (X + )Pp (X + ) − Pp−1 (X + ) par hypothèse
X X X X X X X
de récurrence ce qui prouve bien que Ep+1 est vrai. 
8. Soit R réciproque n’admettant ni 1 ni -1 pour racine. Alors R est réciproque de première espèce et de degré 2n
pair par la question 3.
Comme R est réciproque de première espèce il existe d’après la question 7 un polynôme P de degré n tel que
1 1 1
R(X) = P (X + ). Donc x 6= 0 est racine de R si et seulement si P (x + ) = 0 
Xn X x

∼ Mines-Ponts-2012-maths1-corrige.TEX ∼
Ce polynôme n’est pas unique ni même son degré car par exemple le polynôme (X 2 + 1)P convient également. En
1
effet (X + )2 + 1 n’admet aucune racine réelle. 
X

B. Un problème de dénombrement.

9. Si u ∈ Si,j ou u ∈ Si,j on a évidemment 0 6 uk 6 j pour tout k de 1 à i. Donc ce sont deux ensembles finis. 
P
i+1 P
i
Si u ∈ Si+1,j on a uk = j donc uk 6 j puisque ui+1 ∈ N ce qui prouve que l’application proposée, notée ϕ,
k=0 k=0
P
i+1 P
i+1
est bien définie. Elle est injective car si ϕ(u) = ϕ(v) on a uk = vk pour k de 0 à i et comme uk = j = vk
k=0 k=0

P
i
on a également ui+1 = vi+1 ce qui prouve que u = v. Elle est surjective car si v ∈ Si,j en notant a = j − vk on
k=0
a ϕ(u) = v où u est l’élément de Si+1,j défini par uk = vk pour k de 0 à i et ui+1 = a. 
′ ′
10. Si,j+1 est clairement la réunion disjointe de Si,j+1 et de Si,j
de sorte que s′i,j+1
= si,j+1 + s′i,j 
Donc s′i+1,j+1 = si+1,j+1 + s′i+1,j en changeant i en i + 1. Or Si+1,j+1 est équipotent à Si,j+1

par la question 9. De
′ ′ ′
sorte que si+1,j+1 = si,j+1 + si+1,j 
11. Soit pour n > 2 le prédicat En :hh s′i,j = Cin−1 dès lors que i + j = n ii
• Comme i et j sont des entiers strictement positifs, seul le couple (i, j) = (1, 1) est tel que i + j = 2. L’unique

élément de S1,1 est le couple (1, 0). Donc s′1,1 = 1. Or on a aussi Cii+j−1 = C11 = 1 ce qui prouve que E2 est vrai.
• Supposons désormais Ek vrai jusqu’au rang n > 2 et soit un couple (i, j) ∈ N∗ × N∗ tel que i + j = n + 1.
′ ′
Premier cas : i = 1. Alors Si,j = S1,n est clairement constitué des suites (1, u1 ) avec 0 6 u1 6 n − 1 donc
′ i
si,j = n = Cn et En+1 est bien vrai.
Deuxième cas : j = 1. alors Si,j ′ ′
= Sn,1 ne comprend qu’un seul élément : (1, 0, 0, . . ., 0) et s′i,j = 1 = Cnn et En+1
est encore vrai.
Troisième cas : i > 2 et j > 2. Alors d’après la question 10 on a s′i,j = s′i−1,j + s′i,j−1 et par hypothèse de récurrence
s′i,j = Ci−1 i i
n−1 + Cn−1 = Cn et là encore En+1 est vrai.

En conclusion le prédicat est vrai pour tout entier n > 2. 


Par la question 9 on a si+1,j = s′i,j donc si,j = s′i−1,j = Ci−1
i+j−2 pour i > 2.
Par ailleurs s1,j est clairement égal à 1 et dans ce cas Ci+j−2 = C0j−1 = 1 également.
i−1

En conclusion si,j = Ci−1 ∗ ∗


i+j−2 pour tout (i, j) ∈ N × N . 

C. Polynôme caractéristique d’un produit de matrices.


 
12. Soit A inversible. Il vient det(AB − λIn ) = det A(BA − λIn )A−1 ) = det(BA − λIn ) ∀λ ∈ C
Ainsi ΦAB = ΦBA car la différence admet une infinité de racines. 
13. Supposons désormais ni A ni B inversible.
1
La suite (Ak ) avec Ak = A − In converge vers A et Ak est inversible pour k assez grand car comme 0 est valeur
k
1
propre de A, n’est pas racine de ΦA pour k > K0 .
k
On a det(Ak B − λIn ) = det(BAk − λIn ) pour k > K0 par la question précédente.
Or par continuité du produit matriciel (application bilinéaire sur un produit d’espaces de dimension finie) , la
suite (Ak B − λIn ) converge vers AB − λIn lorsque k → +∞. La continuité de la fonction déterminant (application
multilineaire sur un produit d’espaces de dimension finie) implique alors que det(Ak B −λIn ) −−−−−→ det(AB −λIn )
k→+∞
De même det(BAk − λIn ) −−−−−→ det(BA − λIn )
k→+∞
Par unicité de la limite on a ainsi det(AB − λIn ) = det(BA − λIn ) ∀λ ∈ C et donc ΦAB = ΦBA comme
précédemment. 

D. Étude spectrale de certaines matrices.


j−1
14. si,j = Ci−1
i+j−2 = Ci+j−2 = sj,i de sorte que S est symétrique réelle donc ortho-diagonalisable. 

Si n = 0 alors S = (s1,1 ) = (1) et ΦS = X − 1. √


 
1 1 1 3+ 5
Si n = 1 alors S = et ΦS = X 2 − 3X + 1. S est alors semblable à diag(λ, ) avec λ = .
1 2 λ 2

∼ Mines-Ponts-2012-maths1-corrige.TEX page 2 ∼
 
1 1 1
Si n = 2 alors S =  1 2 3  et ΦS = X 3 − 9X 2 + 9X − 1
1 3 6
1
15. t 7−→ P (t)Q(t)e−t est continue donc localement intégrable sur [0, +∞[ et P (t)Q(t)e−t = o( ) au voisinage de
t2
+∞ ce qui prouve que ψ est bien définie sur Rn [X]2 . Elle est clairement bilinéaire, symétrique et positive. En outre
ψ(P, P ) = 0 implique que P 2 (t)e−t = 0 pour t > 0 puisque t 7−→ P 2 (t)e−t est positive et continue. Ainsi P admet
une infinité de racines donc est le polynôme nul. Ce qui établit finalement que ψ est un produit scalaire. 
Z +∞
1 Γ(i + j + 1) (i + j)!
16. ψ(B1 , Bj ) = ti+j et d t = = = Cii+j = si+1,j+1 .
i!j! 0 i!j! i!j!
Ainsi S n’est autre que la matrice de ψ dans la base B ce qui établit que S est définie positive. 
Il en résulte que S est de rang n + 1. 

P
i
Par ailleurs par définition même de Si,j et Si,j on a clairement s′i,j = si,j ce qui se traduit matriciellement par
j=1
S ′ = S Te où Te est la matrice triangulaire supérieure inversible définie par e
ti,j = 1 pour j 6 i.
Donc rg S ′ = n + 1. 
(j)
17. La formule de dérivation de Leibniz montre que fi (t) = Pi,j (t)e−t où Pi,j est un polynôme de degré i et de
valuation max(0, i − j). (1).
Par croissances comparées on a donc fi,j (t) = o(tk ) quand t → +∞ pour tout entier k. (2) 
P
i i!
En particulier en explicitant la formule de Leibniz pour j = i il vient Li (t) = (−1)i+k tk de sorte
k=0 (i − k)!(k!)2
1
que cela définit un polynôme Li de degré i (coefficient dominant ) et la famille L est libre (échelonnée en degré)
i!
donc constitue une base de Rn [X] 
Z +∞
(−1)i (i)
Il vient ψ(Li , Bj ) = Ii,j avec Ii,j = tj fi (t) d t.
i!j! 0
(i−1) (i−1)
Si j = 0 et i > 1 on a Ii,0 = lim fi (t) − fi (0) = 0 − 0 = 0 d’après (1) et (2).
t→+∞
Si 1 6 j < i une intégration partie entre 0 et A suivie d’un passage à la limite montre (toujours grâce à (1) et (2))
Z +∞
(i−1)
que Ii,j = −j tj−1 fi (t) d t.
0 Z +∞
(i−j)
Par itération Ii,j = (−1)j j! fi (t) d t = 0 d’après (2) puisque i − j > 0.
0

Ainsi ψ(Li , Bj ) = 0 dès que j < i ce qui prouve que la base L est orthonormale car Li est orthogonal à
vect(B0 , B1 , . . ., Bi−1 ) = Ri−1 [X]
Compte-tenu de l’expression de Li ci-dessus et du fait que ψ(Li , X k ) = 0 pour k < i il vient
Z +∞ Z
Xi 1 i −t (−1)i +∞ i (i) (−1)i
ψ(Li , Li ) = ψ(Li , )= t Li (t)e d t = t f i (t) d t = Ii,i
i! i! 0 (i!)2 0 (i!)2
Z +∞
Or par un calcul ci-dessus on a Ii,i = (−1)i i! fi (t) d t = (−1)i i!Γ(i + 1) = (−1)i (i!)2 .
0
Ainsi ψ(Li , Li ) = 1 et la famille L est bien une base orthonormale de Rn [X]. 
j
P j
P
19. On a τ (X j ) = (X − 1)j = (−1)j−i Cij X i et τ −1 (X j ) = (X + 1)j = Cij X i . Donc
i=0 i=0
T est la matrice triangulaire supérieure telle que ui,j = (−1)i−j Ci−1
j−1 pour 1 6 i 6 j 6 n + 1 
i−1
U est la matrice triangulaire supérieure telle que ui,j = Cj−1 pour 1 6 i 6 j 6 n + 1 
j
P
L’expression de Li trouvée à la question 17 s’écrit également Lj (X) = (−1)j−i Cij Bi donc PB→L = T 
i=0

D’après la question 16 on a S = M(ψ, B). Or M(ψ, L) = In+1 donc In+1 = t T ST soit S = t U U . 


Or det(U ) = 1 car U est triangulaire supérieure avec uniquement des 1 sur sa diagonale. Ainsi det S = 1 
D’après la question 16 on a S ′ = S Te donc (puisque det Te = 1) on a det S ′ = 1. 
20. DU se déduit de U en multipliant chaque ligne i par (−1)i−1 et (DU )D se déduit de DU en multipliant chaque
colonne j par (−1)j−i de sorte que DU D se déduit de U en multipliant chaque terme de U par (−1)i+j .
Compte-tenu des expressions de T et U ci-dessus on a donc DU D = T = U −1 soit (DU )2 = In+1 . 

∼ Mines-Ponts-2012-maths1-corrige.TEX page 3 ∼
De S = t U U on tire alors S −1 = U −1t U −1 = (DU D)(t Dt U t D) = DU t U D car D = t D et D2 = In+1
ce qui prouve (puisque D = D−1 ) que S −1 est semblable à U t U . 
21. On a donc ΦU t U (X) = ΦS −1 (X) = det(XIn+1 − S −1 ) = det(XS − In+1 ) puisque det S −1 = 1.
1 1
Ainsi ΦU t U (X) = (−X)n+1 det( In+1 − S) = (−X)n+1 ΦS ( ).
X X
Mais par ailleurs par la question 13 on a ΦU t U (X) = Φt UU (X) = ΦS (X).
Ce qui prouve que ΦS est un polynôme réciproque de première espèce si n est impair et de seconde sinon. 

FIN

∼ Mines-Ponts-2012-maths1-corrige.TEX page 4 ∼

Vous aimerez peut-être aussi